Sie sind auf Seite 1von 118

of P111,592.48 and P157,791.48, or a total of P369,383.

96 which tax liabilities were paid by


petitioner ... . On November 17, 1955, an amended return was filed ... wherein intangible
personal properties with the value of P396,308.90 were claimed as exempted from taxes. On
November 23, 1955, respondent, pending investigation, issued another assessment for estate
and inheritance taxes in the amounts of P202,262.40 and P267,402.84, respectively, or a total
of P469,665.24 ... . In a letter dated January 11, 1956, respondent denied the request for
exemption on the ground that the law of Tangier is not reciprocal to Section 122 of the
National Internal Revenue Code. Hence, respondent demanded the payment of the sums of
P239,439.49 representing deficiency estate and inheritance taxes including ad valorem
penalties, surcharges, interests and compromise penalties ... . In a letter dated February 8,
1956, and received by respondent on the following day, petitioner requested for the
reconsideration of the decision denying the claim for tax exemption of the intangible personal
properties and the imposition of the 25% and 5% ad valorem penalties ... . However,
respondent denied request, in his letter dated May 5, 1956 ... and received by petitioner on
May 21, 1956. Respondent premised the denial on the grounds that there was no reciprocity
[with Tangier, which was moreover] a mere principality, not a foreign country. Consequently,
respondent demanded the payment of the sums of P73,851.21 and P88,023.74 respectively,
or a total of P161,874.95 as deficiency estate and inheritance taxes including surcharges,
interests and compromise penalties." 4
The matter was then elevated to the Court of Tax Appeals. As there was no dispute between
the parties regarding the values of the properties and the mathematical correctness of the
deficiency assessments, the principal question as noted dealt with the reciprocity aspect as
well as the insisting by the Collector of Internal Revenue that Tangier was not a foreign
country within the meaning of Section 122. In ruling against the contention of the Collector of
Internal Revenue, the appealed decision states: "In fine, we believe, and so hold, that the
expression "foreign country", used in the last proviso of Section 122 of the National Internal
Revenue Code, refers to a government of that foreign power which, although not an
international person in the sense of international law, does not impose transfer or death upon
intangible person properties of our citizens not residing therein, or whose law allows a similar
exemption from such taxes. It is, therefore, not necessary that Tangier should have been
recognized by our Government order to entitle the petitioner to the exemption benefits of the
proviso of Section 122 of our Tax. Code." 5
Hence appeal to this court by petitioner. The respective briefs of the parties duly submitted,
but as above indicated, instead of ruling definitely on the question, this Court, on May 30,
1962, resolve to inquire further into the question of reciprocity and sent back the case to the
Court of Tax Appeals for the motion of evidence thereon. The dispositive portion of such
resolution reads as follows: "While section 122 of the Philippine Tax Code aforequoted speaks
of 'intangible personal property' in both subdivisions (a) and (b); the alleged laws of Tangier
refer to 'bienes muebles situados en Tanger', 'bienes muebles radicantes en Tanger',
'movables' and 'movable property'. In order that this Court may be able to determine whether
the alleged laws of Tangier grant the reciprocal tax exemptions required by Section 122 of the
Tax Code, and without, for the time being, going into the merits of the issues raised by the
petitioner-appellant, the case is [remanded] to the Court of Tax Appeals for the reception of
evidence or proof on whether or not the words `bienes muebles', 'movables' and 'movable
properties as used in the Tangier laws, include or embrace 'intangible person property', as
used in the Tax Code." 6 In line with the above resolution, the Court of Tax Appeals admitted

G.R. No. L-13250 October 29, 1971


THE COLLECTOR OF INTERNAL REVENUE, petitioner,
vs.
ANTONIO CAMPOS RUEDA, respondent..
Assistant Solicitor General Jose P. Alejandro and Special Attorney Jose G. Azurin, (O.S.G.)
for petitioner.
Ramirez and Ortigas for respondent.
FERNANDO, J.:
The basic issue posed by petitioner Collector of Internal Revenue in this appeal from a
decision of the Court of Tax Appeals as to whether or not the requisites of statehood, or at
least so much thereof as may be necessary for the acquisition of an international personality,
must be satisfied for a "foreign country" to fall within the exemption of Section 122 of the
National Internal Revenue Code 1 is now ripe for adjudication. The Court of Tax Appeals
answered the question in the negative, and thus reversed the action taken by petitioner
Collector, who would hold respondent Antonio Campos Rueda, as administrator of the estate
of the late Estrella Soriano Vda. de Cerdeira, liable for the sum of P161,874.95 as deficiency
estate and inheritance taxes for the transfer of intangible personal properties in the
Philippines, the deceased, a Spanish national having been a resident of Tangier, Morocco
from 1931 up to the time of her death in 1955. In an earlier resolution promulgated May 30,
1962, this Court on the assumption that the need for resolving the principal question would be
obviated, referred the matter back to the Court of Tax Appeals to determine whether the
alleged law of Tangier did grant the reciprocal tax exemption required by the aforesaid Section
122. Then came an order from the Court of Tax Appeals submitting copies of legislation of
Tangier that would manifest that the element of reciprocity was not lacking. It was not until
July 29, 1969 that the case was deemed submitted for decision. When the petition for review
was filed on January 2, 1958, the basic issue raised was impressed with an element of
novelty. Four days thereafter, however, on January 6, 1958, it was held by this Court that the
aforesaid provision does not require that the "foreign country" possess an international
personality to come within its terms. 2 Accordingly, we have to affirm.
The decision of the Court of Tax Appeals, now under review, sets forth the background facts
as follows: "This is an appeal interposed by petitioner Antonio Campos Rueda as
administrator of the estate of the deceased Doa Maria de la Estrella Soriano Vda. de
Cerdeira, from the decision of the respondent Collector of Internal Revenue, assessing
against and demanding from the former the sum P161,874.95 as deficiency estate and
inheritance taxes, including interest and penalties, on the transfer of intangible personal
properties situated in the Philippines and belonging to said Maria de la Estrella Soriano Vda.
de Cerdeira. Maria de la Estrella Soriano Vda. de Cerdeira (Maria Cerdeira for short) is a
Spanish national, by reason of her marriage to a Spanish citizen and was a resident of
Tangier, Morocco from 1931 up to her death on January 2, 1955. At the time of her demise
she left, among others, intangible personal properties in the Philippines." 3 Then came this
portion: "On September 29, 1955, petitioner filed a provisional estate and inheritance tax
return on all the properties of the late Maria Cerdeira. On the same date, respondent, pending
investigation, issued an assessment for state and inheritance taxes in the respective amounts
1

evidence submitted by the administrator petitioner Antonio Campos Rueda, consisting of


exhibits of laws of Tangier to the effect that "the transfers by reason of death of movable
properties, corporeal or incorporeal, including furniture and personal effects as well as of
securities, bonds, shares, ..., were not subject, on that date and in said zone, to the payment
of any death tax, whatever might have been the nationality of the deceased or his heirs and
legatees." It was further noted in an order of such Court referring the matter back to us that
such were duly admitted in evidence during the hearing of the case on September 9, 1963.
Respondent presented no evidence." 7
The controlling legal provision as noted is a proviso in Section 122 of the National Internal
Revenue Code. It reads thus: "That no tax shall be collected under this Title in respect of
intangible personal property (a) if the decedent at the time of his death was a resident of a
foreign country which at the time of his death did not impose a transfer tax or death tax of any
character in respect of intangible person property of the Philippines not residing in that foreign
country, or (b) if the laws of the foreign country of which the decedent was a resident at the
time of his death allow a similar exemption from transfer taxes or death taxes of every
character in respect of intangible personal property owned by citizens of the Philippines not
residing in that foreign country." 8 The only obstacle therefore to a definitive ruling is whether
or not as vigorously insisted upon by petitioner the acquisition of internal personality is a
condition sine qua non to Tangier being considered a "foreign country". Deference to the De
Lara ruling, as was made clear in the opening paragraph of this opinion, calls for an
affirmance of the decision of the Court of Tax Appeals.
It does not admit of doubt that if a foreign country is to be identified with a state, it is required
in line with Pound's formulation that it be a politically organized sovereign community
independent of outside control bound by penalties of nationhood, legally supreme within its
territory, acting through a government functioning under a regime of
law. 9 It is thus a sovereign person with the people composing it viewed as an organized
corporate society under a government with the legal competence to exact obedience to its
commands. 10 It has been referred to as a body-politic organized by common consent for
mutual defense and mutual safety and to promote the general welfare. 11 Correctly has it been
described by Esmein as "the juridical personification of the nation." 12 This is to view it in the
light of its historical development. The stress is on its being a nation, its people occupying a
definite territory, politically organized, exercising by means of its government its sovereign will
over the individuals within it and maintaining its separate international personality. Laski could
speak of it then as a territorial society divided into government and subjects, claiming within its
allotted area a supremacy over all other institutions. 13 McIver similarly would point to the
power entrusted to its government to maintain within its territory the conditions of a legal order
and to enter into international relations. 14 With the latter requisite satisfied, international law
do not exact independence as a condition of statehood. So Hyde did opine. 15
Even on the assumption then that Tangier is bereft of international personality, petitioner has
not successfully made out a case. It bears repeating that four days after the filing of this
petition on January 6, 1958 in Collector of Internal Revenue v. De Lara, 16 it was specifically
held by us: "Considering the State of California as a foreign country in relation to section 122
of our Tax Code we believe and hold, as did the Tax Court, that the Ancilliary Administrator is
entitled the exemption from the inheritance tax on the intangible personal property found in the
Philippines." 17 There can be no doubt that California as a state in the American Union was in

the alleged requisite of international personality. Nonetheless, it was held to be a foreign


country within the meaning of Section 122 of the National Internal Revenue Code. 18
What is undeniable is that even prior to the De Lara ruling, this Court did commit itself to the
doctrine that even a tiny principality, that of Liechtenstein, hardly an international personality in
the sense, did fall under this exempt category. So it appears in an opinion of the Court by the
then Acting Chief Justicem Bengson who thereafter assumed that position in a permanent
capacity, in Kiene v. Collector of Internal Revenue. 19 As was therein noted: 'The Board found
from the documents submitted to it proof of the laws of Liechtenstein that said country
does not impose estate, inheritance and gift taxes on intangible property of Filipino citizens
not residing in that country. Wherefore, the Board declared that pursuant to the exemption
above established, no estate or inheritance taxes were collectible, Ludwig Kiene being a
resident of Liechtestein when he passed away." 20 Then came this definitive ruling: "The
Collector hereafter named the respondent cites decisions of the United States Supreme
Court and of this Court, holding that intangible personal property in the Philippines belonging
to a non-resident foreigner, who died outside of this country is subject to the estate tax, in
disregard of the principle 'mobilia sequuntur personam'. Such property is admittedly taxable
here. Without the proviso above quoted, the shares of stock owned here by the Ludwig Kiene
would be concededly subject to estate and inheritance taxes. Nevertheless our Congress
chose to make an exemption where conditions are such that demand reciprocity as in this
case. And the exemption must be honored." 21
WHEREFORE, the decision of the respondent Court of Tax Appeals of October 30, 1957 is
affirmed. Without pronouncement as to costs.
Concepcion, C.J., Makalintal, Zaldivar, Castro, Villamor and Makasiar, JJ., concur.
Reyes, J.B.L., J., concurs in the result.
Teehankee and Barredo, JJ., took no part.

G.R. No. 175888


February 11, 2009
SUZETTE NICOLAS y SOMBILON, Petitioner,
vs.
ALBERTO ROMULO, in his capacity as Secretary of Foreign Affairs; RAUL GONZALEZ,
in his capacity as Secretary of Justice; EDUARDO ERMITA, in his capacity as Executive
Secretary; RONALDO PUNO, in his capacity as Secretary of the Interior and Local
Government; SERGIO APOSTOL, in his capacity as Presidential Legal Counsel; and
L/CPL. DANIEL SMITH, Respondents.
x - - - - - - - - - - - - - - - - - - - - - - -x
G.R. No. 176051
February 11, 2009
JOVITO R. SALONGA, WIGBERTO E. TAADA, JOSE DE LA RAMA, EMILIO C.
CAPULONG, H. HARRY L. ROQUE, JR., FLORIN HILBAY, and BENJAMIN POZON,
Petitioners,
vs.
DANIEL SMITH, SECRETARY RAUL GONZALEZ, PRESIDENTIAL LEGAL COUNSEL
SERGIO APOSTOL, SECRETARY RONALDO PUNO, SECRETARY ALBERTO ROMULO,
The Special 16th Division of the COURT OF APPEALS, and all persons acting in their
capacity, Respondents.
x - - - - - - - - - - - - - - - - - - - - - - -x
G.R. No. 176222
February 11, 2009
BAGONG ALYANSANG MAKABAYAN (BAYAN), represented by Dr. Carol Araullo;
GABRIELA, represented by Emerenciana de Jesus; BAYAN MUNA, represented by Rep.
Satur Ocampo; GABRIELA WOMEN'S PARTY, represented by Rep. Liza Maza;
KILUSANG MAYO UNO (KMU), represented by Elmer Labog; KILUSANG MAGBUBUKID
NG PILIPINAS (KMP), represented by Willy Marbella; LEAGUE OF FILIPINO STUDENTS
(LFS), represented by Vencer Crisostomo; and THE PUBLIC INTEREST LAW CENTER,
represented by Atty. Rachel Pastores, Petitioners,
vs.
PRESIDENT GLORIA MACAPAGAL-ARROYO, in her capacity as concurrent Defense
Secretary, EXECUTIVE SECRETARY EDUARDO ERMITA, FOREIGN AFFAIRS
SECRETARY ALBERTO ROMULO, JUSTICE SECRETARY RAUL GONZALEZ, AND
INTERIOR AND LOCAL GOVERNMENT SECRETARY RONALDO PUNO, Respondents.
DECISION
AZCUNA, J.:
These are petitions for certiorari, etc. as special civil actions and/or for review of the Decision
of the Court of Appeals in Lance Corporal Daniel J. Smith v. Hon. Benjamin T. Pozon, et al., in
CA-G.R. SP No. 97212, dated January 2, 2007.
The facts are not disputed.
Respondent Lance Corporal (L/CPL) Daniel Smith is a member of the United States Armed
Forces. He was charged with the crime of rape committed against a Filipina, petitioner herein,
sometime on November 1, 2005, as follows:
The undersigned accused LCpl. Daniel Smith, Ssgt. Chad Brian Carpentier, Dominic
Duplantis, Keith Silkwood and Timoteo L. Soriano, Jr. of the crime of Rape under Article 266-

A of the Revised Penal Code, as amended by Republic Act 8353, upon a complaint under
oath filed by Suzette S. Nicolas, which is attached hereto and made an integral part hereof as
Annex "A," committed as follows:
"That on or about the First (1st) day of November 2005, inside the Subic Bay Freeport Zone,
Olongapo City and within the jurisdiction of this Honorable Court, the above-named accuseds
(sic), being then members of the United States Marine Corps, except Timoteo L. Soriano, Jr.,
conspiring, confederating together and mutually helping one another, with lewd design and by
means of force, threat and intimidation, with abuse of superior strength and taking advantage
of the intoxication of the victim, did then and there willfully, unlawfully and feloniously sexually
abuse and have sexual intercourse with or carnal knowledge of one Suzette S. Nicolas, a 22year old unmarried woman inside a Starex Van with Plate No. WKF-162, owned by Starways
Travel and Tours, with Office address at 8900 P. Victor St., Guadalupe, Makati City, and
driven by accused Timoteo L. Soriano, Jr., against the will and consent of the said Suzette S.
Nicolas, to her damage and prejudice.
CONTRARY TO LAW."1
Pursuant to the Visiting Forces Agreement (VFA) between the Republic of the Philippines and
the United States, entered into on February 10, 1998, the United States, at its request, was
granted custody of defendant Smith pending the proceedings.
During the trial, which was transferred from the Regional Trial Court (RTC) of Zambales to the
RTC of Makati for security reasons, the United States Government faithfully complied with its
undertaking to bring defendant Smith to the trial court every time his presence was required.
On December 4, 2006, the RTC of Makati, following the end of the trial, rendered its Decision,
finding defendant Smith guilty, thus:
WHEREFORE, premises considered, for failure of the prosecution to adduce sufficient
evidence against accused S/SGT. CHAD BRIAN CARPENTER, L/CPL. KEITH SILKWOOD
AND L/CPL. DOMINIC DUPLANTIS, all of the US Marine Corps assigned at the USS Essex,
are hereby ACQUITTED to the crime charged.
The prosecution having presented sufficient evidence against accused L/CPL. DANIEL J.
SMITH, also of the US Marine Corps at the USS Essex, this Court hereby finds him GUILTY
BEYOND REASONABLE DOUBT of the crime of RAPE defined under Article 266-A,
paragraph 1 (a) of the Revised Penal Code, as amended by R.A. 8353, and, in accordance
with Article 266-B, first paragraph thereof, hereby sentences him to suffer the penalty of
reclusion perpetua together with the accessory penalties provided for under Article 41 of the
same Code.
Pursuant to Article V, paragraph No. 10, of the Visiting Forces Agreement entered into by the
Philippines and the United States, accused L/CPL. DANIEL J. SMITH shall serve his sentence
in the facilities that shall, thereafter, be agreed upon by appropriate Philippine and United
States authorities. Pending agreement on such facilities, accused L/CPL. DANIEL J. SMITH is
hereby temporarily committed to the Makati City Jail.
Accused L/CPL. DANIEL J. SMITH is further sentenced to indemnify complainant SUZETTE
S. NICOLAS in the amount of P50,000.00 as compensatory damages plus P50,000.00 as
moral damages.
SO ORDERED.2
As a result, the Makati court ordered Smith detained at the Makati jail until further orders.
3

On December 29, 2006, however, defendant Smith was taken out of the Makati jail by a
contingent of Philippine law enforcement agents, purportedly acting under orders of the
Department of the Interior and Local Government, and brought to a facility for detention under
the control of the United States government, provided for under new agreements between the
Philippines and the United States, referred to as the Romulo-Kenney Agreement of December
19, 2006 which states:
The Government of the Republic of the Philippines and the Government of the United States
of America agree that, in accordance with the Visiting Forces Agreement signed between our
two nations, Lance Corporal Daniel J. Smith, United States Marine Corps, be returned to U.S.
military custody at the U.S. Embassy in Manila.
(Sgd.) Kristie A. Kenney
Representative of the United States
of America

(Sgd.) Alberto G. Romulo


Representative of the Republic
of the Philippines

DATE: 12-19-06

DATE: December 19, 2006

Sec. 25. After the expiration in 1991 of the Agreement between the Philippines and the United
States of America concerning Military Bases, foreign military bases, troops, or facilities shall
not be allowed in the Philippines except under a treaty duly concurred in by the Senate and,
when the Congress so requires, ratified by a majority of the votes cast by the people in a
national referendum held for that purpose, and recognized as a treaty by the other contracting
State.
The reason for this provision lies in history and the Philippine experience in regard to the
United States military bases in the country.
It will be recalled that under the Philippine Bill of 1902, which laid the basis for the Philippine
Commonwealth and, eventually, for the recognition of independence, the United States
agreed to cede to the Philippines all the territory it acquired from Spain under the Treaty of
Paris, plus a few islands later added to its realm, except certain naval ports and/or military
bases and facilities, which the United States retained for itself.
This is noteworthy, because what this means is that Clark and Subic and the other places in
the Philippines covered by the RP-US Military Bases Agreement of 1947 were not Philippine
territory, as they were excluded from the cession and retained by the US.
Accordingly, the Philippines had no jurisdiction over these bases except to the extent allowed
by the United States. Furthermore, the RP-US Military Bases Agreement was never advised
for ratification by the United States Senate, a disparity in treatment, because the Philippines
regarded it as a treaty and had it concurred in by our Senate.
Subsequently, the United States agreed to turn over these bases to the Philippines; and with
the expiration of the RP-US Military Bases Agreement in 1991, the territory covered by these
bases were finally ceded to the Philippines.
To prevent a recurrence of this experience, the provision in question was adopted in the 1987
Constitution.
The provision is thus designed to ensure that any agreement allowing the presence of foreign
military bases, troops or facilities in Philippine territory shall be equally binding on the
Philippines and the foreign sovereign State involved. The idea is to prevent a recurrence of
the situation in which the terms and conditions governing the presence of foreign armed
forces in our territory were binding upon us but not upon the foreign State.
Applying the provision to the situation involved in these cases, the question is whether or not
the presence of US Armed Forces in Philippine territory pursuant to the VFA is allowed "under
a treaty duly concurred in by the Senate xxx and recognized as a treaty by the other
contracting State."
This Court finds that it is, for two reasons.
First, as held in Bayan v. Zamora,5 the VFA was duly concurred in by the Philippine Senate
and has been recognized as a treaty by the United States as attested and certified by the duly
authorized representative of the United States government.
The fact that the VFA was not submitted for advice and consent of the United States Senate
does not detract from its status as a binding international agreement or treaty recognized by
the said State. For this is a matter of internal United States law. Notice can be taken of the
internationally known practice by the United States of submitting to its Senate for advice and
consent agreements that are policymaking in nature, whereas those that carry out or further

and the Romulo-Kenney Agreement of December 22, 2006 which states:


The Department of Foreign Affairs of the Republic of the Philippines and the Embassy of the
United States of America agree that, in accordance with the Visiting Forces Agreement signed
between the two nations, upon transfer of Lance Corporal Daniel J. Smith, United States
Marine Corps, from the Makati City Jail, he will be detained at the first floor, Rowe (JUSMAG)
Building, U.S. Embassy Compound in a room of approximately 10 x 12 square feet. He will be
guarded round the clock by U.S. military personnel. The Philippine police and jail authorities,
under the direct supervision of the Philippine Department of Interior and Local Government
(DILG) will have access to the place of detention to ensure the United States is in compliance
with the terms of the VFA.
The matter was brought before the Court of Appeals which decided on January 2, 2007, as
follows:
WHEREFORE, all the foregoing considered, we resolved to DISMISS the petition for having
become moot.3
Hence, the present actions.
The petitions were heard on oral arguments on September 19, 2008, after which the parties
submitted their memoranda.
Petitioners contend that the Philippines should have custody of defendant L/CPL Smith
because, first of all, the VFA is void and unconstitutional.
This issue had been raised before, and this Court resolved in favor of the constitutionality of
the VFA. This was in Bayan v. Zamora,4 brought by Bayan, one of petitioners in the present
cases.
Against the barriers of res judicata vis--vis Bayan, and stare decisis vis--vis all the parties,
the reversal of the previous ruling is sought on the ground that the issue is of primordial
importance, involving the sovereignty of the Republic, as well as a specific mandate of the
Constitution.
The provision of the Constitution is Art. XVIII, Sec. 25 which states:
4

implement these policymaking agreements are merely submitted to Congress, under the
provisions of the so-called CaseZablocki Act, within sixty days from ratification.6
The second reason has to do with the relation between the VFA and the RP-US Mutual
Defense Treaty of August 30, 1951. This earlier agreement was signed and duly ratified with
the concurrence of both the Philippine Senate and the United States Senate.
The RP-US Mutual Defense Treaty states:7
MUTUAL DEFENSE TREATY BETWEEN THE REPUBLIC OF THE PHILIPPINES AND THE
UNITED STATES OF AMERICA. Signed at Washington, August 30, 1951.
The Parties of this Treaty
Reaffirming their faith in the purposes and principles of the Charter of the United Nations and
their desire to live in peace with all peoples and all governments, and desiring to strengthen
the fabric of peace in the Pacific area.
Recalling with mutual pride the historic relationship which brought their two peoples together
in a common bond of sympathy and mutual ideals to fight side-by-side against imperialist
aggression during the last war.
Desiring to declare publicly and formally their sense of unity and their common determination
to defend themselves against external armed attack, so that no potential aggressor could be
under the illusion that either of them stands alone in the Pacific area.
Desiring further to strengthen their present efforts for collective defense for the preservation of
peace and security pending the development of a more comprehensive system of regional
security in the Pacific area.
Agreeing that nothing in this present instrument shall be considered or interpreted as in any
way or sense altering or diminishing any existing agreements or understandings between the
Republic of the Philippines and the United States of America.
Have agreed as follows:
Article I. The parties undertake, as set forth in the Charter of the United Nations, to settle any
international disputes in which they may be involved by peaceful means in such a manner that
international peace and security and justice are not endangered and to refrain in their
international relation from the threat or use of force in any manner inconsistent with the
purposes of the United Nations.
Article II. In order more effectively to achieve the objective of this Treaty, the Parties
separately and jointly by self-help and mutual aid will maintain and develop their individual and
collective capacity to resist armed attack.
Article III. The Parties, through their Foreign Ministers or their deputies, will consult together
from time to time regarding the implementation of this Treaty and whenever in the opinion of
either of them the territorial integrity, political independence or security of either of the Parties
is threatened by external armed attack in the Pacific.
Article IV. Each Party recognizes that an armed attack in the Pacific area on either of the
parties would be dangerous to its own peace and safety and declares that it would act to meet
the common dangers in accordance with its constitutional processes.
Any such armed attack and all measures taken as a result thereof shall be immediately
reported to the Security Council of the United Nations. Such measures shall be terminated

when the Security Council has taken the measures necessary to restore and maintain
international peace and security.
Article V. For the purpose of Article IV, an armed attack on either of the Parties is deemed to
include an armed attack on the metropolitan territory of either of the Parties, or on the island
territories under its jurisdiction in the Pacific Ocean, its armed forces, public vessels or aircraft
in the Pacific.
Article VI. This Treaty does not affect and shall not be interpreted as affecting in any way the
rights and obligations of the Parties under the Charter of the United Nations or the
responsibility of the United Nations for the maintenance of international peace and security.
Article VII. This Treaty shall be ratified by the Republic of the Philippines and the United
Nations of America in accordance with their respective constitutional processes and will come
into force when instruments of ratification thereof have been exchanged by them at Manila.
Article VIII. This Treaty shall remain in force indefinitely. Either Party may terminate it one year
after notice has been given to the other party.
In withness whereof the undersigned Plenipotentiaries have signed this Treaty.
Done in duplicate at Washington this thirtieth day of August, 1951.
For the Republic of the Philippines:
(Sgd.) Carlos P. Romulo
(Sgd.) Joaquin M. Elizalde
(Sgd.) Vicente J. Francisco
(Sgd.) Diosdado Macapagal
For the United States of America:
(Sgd.) Dean Acheson
(Sgd.) John Foster Dulles
(Sgd.) Tom Connally
(Sgd.) Alexander Wiley8
Clearly, therefore, joint RP-US military exercises for the purpose of developing the capability
to resist an armed attack fall squarely under the provisions of the RP-US Mutual Defense
Treaty. The VFA, which is the instrument agreed upon to provide for the joint RP-US military
exercises, is simply an implementing agreement to the main RP-US Military Defense Treaty.
The Preamble of the VFA states:
The Government of the United States of America and the Government of the Republic of the
Philippines,
Reaffirming their faith in the purposes and principles of the Charter of the United Nations and
their desire to strengthen international and regional security in the Pacific area;
Reaffirming their obligations under the Mutual Defense Treaty of August 30, 1951;
Noting that from time to time elements of the United States armed forces may visit the
Republic of the Philippines;
Considering that cooperation between the United States and the Republic of the Philippines
promotes their common security interests;
5

Recognizing the desirability of defining the treatment of United States personnel visiting the
Republic of the Philippines;
Have agreed as follows:9
Accordingly, as an implementing agreement of the RP-US Mutual Defense Treaty, it was not
necessary to submit the VFA to the US Senate for advice and consent, but merely to the US
Congress under the CaseZablocki Act within 60 days of its ratification. It is for this reason
that the US has certified that it recognizes the VFA as a binding international agreement, i.e.,
a treaty, and this substantially complies with the requirements of Art. XVIII, Sec. 25 of our
Constitution.10
The provision of Art. XVIII, Sec. 25 of the Constitution, is complied with by virtue of the fact
that the presence of the US Armed Forces through the VFA is a presence "allowed under" the
RP-US Mutual Defense Treaty. Since the RP-US Mutual Defense Treaty itself has been
ratified and concurred in by both the Philippine Senate and the US Senate, there is no
violation of the Constitutional provision resulting from such presence.
The VFA being a valid and binding agreement, the parties are required as a matter of
international law to abide by its terms and provisions.
The VFA provides that in cases of offenses committed by the members of the US Armed
Forces in the Philippines, the following rules apply:
Article V
Criminal Jurisdiction
xxx
6. The custody of any United States personnel over whom the Philippines is to exercise
jurisdiction shall immediately reside with United States military authorities, if they so request,
from the commission of the offense until completion of all judicial proceedings. United States
military authorities shall, upon formal notification by the Philippine authorities and without
delay, make such personnel available to those authorities in time for any investigative or
judicial proceedings relating to the offense with which the person has been charged. In
extraordinary cases, the Philippine Government shall present its position to the United States
Government regarding custody, which the United States Government shall take into full
account. In the event Philippine judicial proceedings are not completed within one year, the
United States shall be relieved of any obligations under this paragraph. The one year period
will not include the time necessary to appeal. Also, the one year period will not include any
time during which scheduled trial procedures are delayed because United States authorities,
after timely notification by Philippine authorities to arrange for the presence of the accused,
fail to do so.
Petitioners contend that these undertakings violate another provision of the Constitution,
namely, that providing for the exclusive power of this Court to adopt rules of procedure for all
courts in the Philippines (Art. VIII, Sec. 5[5]). They argue that to allow the transfer of custody
of an accused to a foreign power is to provide for a different rule of procedure for that
accused, which also violates the equal protection clause of the Constitution (Art. III, Sec. 1.).
Again, this Court finds no violation of the Constitution.

The equal protection clause is not violated, because there is a substantial basis for a different
treatment of a member of a foreign military armed forces allowed to enter our territory and all
other accused.11
The rule in international law is that a foreign armed forces allowed to enter ones territory is
immune from local jurisdiction, except to the extent agreed upon. The Status of Forces
Agreements involving foreign military units around the world vary in terms and conditions,
according to the situation of the parties involved, and reflect their bargaining power. But the
principle remains, i.e., the receiving State can exercise jurisdiction over the forces of the
sending State only to the extent agreed upon by the parties.12
As a result, the situation involved is not one in which the power of this Court to adopt rules of
procedure is curtailed or violated, but rather one in which, as is normally encountered around
the world, the laws (including rules of procedure) of one State do not extend or apply except
to the extent agreed upon to subjects of another State due to the recognition of
extraterritorial immunity given to such bodies as visiting foreign armed forces.
Nothing in the Constitution prohibits such agreements recognizing immunity from jurisdiction
or some aspects of jurisdiction (such as custody), in relation to long-recognized subjects of
such immunity like Heads of State, diplomats and members of the armed forces contingents of
a foreign State allowed to enter another States territory. On the contrary, the Constitution
states that the Philippines adopts the generally accepted principles of international law as part
of the law of the land. (Art. II, Sec. 2).
Applying, however, the provisions of VFA, the Court finds that there is a different treatment
when it comes to detention as against custody. The moment the accused has to be detained,
e.g., after conviction, the rule that governs is the following provision of the VFA:
Article V
Criminal Jurisdiction
xxx
Sec. 10. The confinement or detention by Philippine authorities of United States personnel
shall be carried out in facilities agreed on by appropriate Philippines and United States
authorities. United States personnel serving sentences in the Philippines shall have the right
to visits and material assistance.
It is clear that the parties to the VFA recognized the difference between custody during the
trial and detention after conviction, because they provided for a specific arrangement to cover
detention. And this specific arrangement clearly states not only that the detention shall be
carried out in facilities agreed on by authorities of both parties, but also that the detention shall
be "by Philippine authorities." Therefore, the Romulo-Kenney Agreements of December 19
and 22, 2006, which are agreements on the detention of the accused in the United States
Embassy, are not in accord with the VFA itself because such detention is not "by Philippine
authorities."
Respondents should therefore comply with the VFA and negotiate with representatives of the
United States towards an agreement on detention facilities under Philippine authorities as
mandated by Art. V, Sec. 10 of the VFA.
Next, the Court addresses the recent decision of the United States Supreme Court in Medellin
v. Texas ( 552 US ___ No. 06-984, March 25, 2008), which held that treaties entered into by
6

the United States are not automatically part of their domestic law unless these treaties are
self-executing or there is an implementing legislation to make them enforceable.1avvphi1
On February 3, 2009, the Court issued a Resolution, thus:
"G.R. No. 175888 (Suzette Nicolas y Sombilon v. Alberto Romulo, et al.); G.R. No. 176051
(Jovito R. Salonga, et al. v. Daniel Smith, et al.); and G.R. No. 176222 (Bagong Alyansang
Makabayan [BAYAN], et al. v. President Gloria Macapagal-Arroyo, et al.).
The parties, including the Solicitor General, are required to submit within three (3) days a
Comment/Manifestation on the following points:
1. What is the implication on the RP-US Visiting Forces Agreement of the recent US
Supreme Court decision in Jose Ernesto Medellin v. Texas, dated March 25, 2008,
to the effect that treaty stipulations that are not self-executory can only be enforced
pursuant to legislation to carry them into effect; and that, while treaties may
comprise international commitments, they are not domestic law unless Congress
has enacted implementing statutes or the treaty itself conveys an intention that it be
"self-executory" and is ratified on these terms?
2. Whether the VFA is enforceable in the US as domestic law, either because it is
self-executory or because there exists legislation to implement it.
3. Whether the RP-US Mutual Defense Treaty of August 30, 1951 was concurred in
by the US Senate and, if so, is there proof of the US Senate advice and consent
resolution? Peralta, J., no part."
After deliberation, the Court holds, on these points, as follows:
First, the VFA is a self-executing Agreement, as that term is defined in Medellin itself, because
the parties intend its provisions to be enforceable, precisely because the Agreement is
intended to carry out obligations and undertakings under the RP-US Mutual Defense Treaty.
As a matter of fact, the VFA has been implemented and executed, with the US faithfully
complying with its obligation to produce L/CPL Smith before the court during the trial.
Secondly, the VFA is covered by implementing legislation, namely, the Case-Zablocki Act,
USC Sec. 112(b), inasmuch as it is the very purpose and intent of the US Congress that
executive agreements registered under this Act within 60 days from their ratification be
immediately implemented. The parties to these present cases do not question the fact that the
VFA has been registered under the Case-Zablocki Act.1avvphi1
In sum, therefore, the VFA differs from the Vienna Convention on Consular Relations and the
Avena decision of the International Court of Justice (ICJ), subject matter of the Medellin
decision. The Convention and the ICJ decision are not self-executing and are not registrable
under the Case-Zablocki Act, and thus lack legislative implementing authority.
Finally, the RP-US Mutual Defense Treaty was advised and consented to by the US Senate
on March 20, 1952, as reflected in the US Congressional Record, 82nd Congress, Second
Session, Vol. 98 Part 2, pp. 2594-2595.
The framers of the Constitution were aware that the application of international law in
domestic courts varies from country to country.
As Ward N. Ferdinandusse states in his Treatise, DIRECT APPLICATION OF
INTERNATIONAL CRIMINAL LAW IN NATIONAL COURTS, some countries require
legislation whereas others do not.

It was not the intention of the framers of the 1987 Constitution, in adopting Article XVIII, Sec.
25, to require the other contracting State to convert their system to achieve alignment and
parity with ours. It was simply required that the treaty be recognized as a treaty by the other
contracting State. With that, it becomes for both parties a binding international obligation and
the enforcement of that obligation is left to the normal recourse and processes under
international law.
Furthermore, as held by the US Supreme Court in Weinberger v. Rossi,13 an executive
agreement is a "treaty" within the meaning of that word in international law and constitutes
enforceable domestic law vis--vis the United States. Thus, the US Supreme Court in
Weinberger enforced the provisions of the executive agreement granting preferential
employment to Filipinos in the US Bases here.
Accordingly, there are three types of treaties in the American system:
1. Art. II, Sec. 2 treaties These are advised and consented to by the US Senate in
accordance with Art. II, Sec. 2 of the US Constitution.
2. ExecutiveCongressional Agreements: These are joint agreements of the
President and Congress and need not be submitted to the Senate.
3. Sole Executive Agreements. These are agreements entered into by the
President. They are to be submitted to Congress within sixty (60) days of ratification
under the provisions of the Case-Zablocki Act, after which they are recognized by
the Congress and may be implemented.
As regards the implementation of the RP-US Mutual Defense Treaty, military aid or assistance
has been given under it and this can only be done through implementing legislation. The VFA
itself is another form of implementation of its provisions.
WHEREFORE, the petitions are PARTLY GRANTED, and the Court of Appeals Decision in
CA-G.R. SP No. 97212 dated January 2, 2007 is MODIFIED. The Visiting Forces Agreement
(VFA) between the Republic of the Philippines and the United States, entered into on
February 10, 1998, is UPHELD as constitutional, but the Romulo-Kenney Agreements of
December 19 and 22, 2006 are DECLARED not in accordance with the VFA, and respondent
Secretary of Foreign Affairs is hereby ordered to forthwith negotiate with the United States
representatives for the appropriate agreement on detention facilities under Philippine
authorities as provided in Art. V, Sec. 10 of the VFA, pending which the status quo shall be
maintained until further orders by this Court.
The Court of Appeals is hereby directed to resolve without delay the related matters pending
therein, namely, the petition for contempt and the appeal of L/CPL Daniel Smith from the
judgment of conviction.
No costs.
SO ORDERED.

G.R. No. L-10280


September 30, 1963
QUA CHEE GAN, JAMES UY, DANIEL DY alias DEE PAC, CHAN TIONG YU, CUA CHU
TIAN, CHUA LIM PAO alias JOSE CHUA and BASILIO KING, petitioners-appellants,
vs.
THE DEPORTATION BOARD, respondent-appellee.
Sabido and Sabido Law Offices and Ramon T. Oben for petitioners-appellants.
Solicitor General for respondent-appellee.

assisting the Central Bank official in the investigation of the purchase of $130,000.00 from the
Clark Air Force Base, wherein said petitioner was involved.
After due trial, the court rendered a decision on January 18, 1956, upholding the validity
of the delegation by the president to the Deportation Board of his power to conduct
investigations for the purpose of determining whether the stay of an alien in this country would
be injurious to the security, welfare and interest of the State. The court, likewise, sustained the
power of the deportation Board to issue warrant of arrest and fix bonds for the alien's
temporary release pending investigation of charges against him, on the theory that the power
to arrest and fix the amount of the bond of the arrested alien is essential to and complement
the power to deport aliens pursuant to Section 69 of the Revised Administrative Code.
Consequently, the petitioners instituted the present appeal. .
It may be pointed out at the outset that after they were provisionally released on bail,
but before the charges filed against them were actually investigated, petitioners-appellant
raised the question of jurisdiction of the Deportation Board, first before said body, then in the
Court of First Instance, and now before us. Petitioners-appellants contest the power of the
President to deport aliens and, consequently, the delegation to the Deportation Board of the
ancillary power to investigate, on the ground that such power is vested in the Legislature. In
other words, it is claimed, for the power to deport to be exercised, there must be a legislation
authorizing the same.
Under Commonwealth Act No. 613 (Immigration Act of 1940), the Commissioner of
Immigration was empowered to effect the arrest and expulsion of an alien, after previous
determination by the Board of Commissioners of the existence of ground or grounds therefor
(Sec- 37). With the enactment of this law, however, the legislature did not intend to delimit or
concentrate the exercise of the power to deport on the Immigration Commissioner alone,
because in its Section 52, it provides:.
SEC. 52. This Act is in substitution for and supersedes all previous laws
relating to the entry of aliens into the Philippines, and their exclusion, deportation,
and repatriation therefrom, with the exception of section sixty-nine of Act Numbered
Twenty-seven hundred and eleven which shall continue in force and effect: ..."
(Comm. Act No. 613).
Section 69 of Act No. 2711 (Revised Administrative Code) referred to above
reads:.
SEC. 69 Deportation of subject to foreign power. A subject of a foreign
power residing in the Philippines shall not be deported, expelled, or excluded from
said Islands or repatriated to his own country by the President of the Philippines
except upon prior investigation, conducted by said Executive or his authorized
agent, of the ground upon which Such action is contemplated. In such case the
person concerned shall be informed of the charge or charges against him and he
shall be allowed not less than these days for the preparation of his defense. He shall
also have the right to be heard by himself or counsel, to produce witnesses in his
own behalf, and to cross-examine the opposing witnesses."
While it may really be contended that the aforequoted provision did not expressly confer
on the President the authority to deport undesirable aliens, unlike the express grant to the
Commissioner of Immigration under Commonwealth Act No. 613, but merely lays down the
procedure to be observed should there be deportation proceedings, the fact that such a

BARRERA, J.:
This is an appeal from the decision of the Court of First Instance of Manila (in Sp. Proc.
No. 20037) denying the petition for writs of habeas corpus and/or prohibition, certiorari, and
mandamus filed by Qua Chee Gan, James Uy, Daniel Dy alias Dee Pac, Chan Tiong Yu,
Chua Chu Tian, Chua Lim Pao alias Jose Chua, and Basilio King. The facts of the case,
briefly stated, are as follows:.
On May 12, 1952, Special Prosecutor Emilio L. Galang charged the above-named
petitioners before the Deportation Board, with having purchased U.S. dollars in the total sum
of $130,000.00, without the necessary license from the Central Bank of the Philippines, and of
having clandestinely remitted the same to Hongkong and petitioners, Qua Chee Gan, Chua
Lim Pao alias Jose Chua, and Basilio King, with having attempted to bribe officers of the
Philippine and United States Governments (Antonio Laforteza, Chief of the Intelligence
Division of the Central Bank, and Capt. A. P. Charak of the OSI, U.S. Air Force) in order to
evade prosecution for said unauthorized purchase of U.S. dollars.1
Following the filing of said deportation charges, a warrant for the arrest of said aliens
was issued by the presiding member of the Deportation Board. Upon their filing surety bond
for P10,000.00 and cash bond for P10,000.00, herein petitioners-appellants were provisionally
set at liberty.
On September 22, 1952, petitioners-appellants filed a joint motion to dismiss the
charges presented against them in the Deportation Board for the reason, among others, that
the same do not constitute legal ground for deportation of aliens from this country, and that
said Board has no jurisdiction to entertain such charges. This motion to dismiss having been
denied by order of the Board of February 9, 1953, petitioners-appellants filed in this Court a
petition for habeas corpus and/or prohibition, which petition was given due course in our
resolution of July 7, 1953, but made returnable to the Court of First Instance of Manila (G.R.
No. L-6783). The case was docketed in the lower court as Special Proceeding No. 20037.
At the instance of petitioners and upon their filing a bond for P5,000.00 each, a writ of
preliminary injunction was issued by the lower court, restraining the respondent Deportation
Board from hearing Deportation charges No. R-425 against petitioners, pending final
termination of the habeas corpus and/or prohibition proceedings.
On July 29, 1953, the respondent Board filed its answer to the original petition,
maintaining among others, that the Deportation Board, as an agent of the President, has
jurisdiction over the charges filed against petitioners and the authority to order their arrest; and
that, while petitioner Qua Chee Gan was acquitted of the offense of attempted bribery of a
public official, he was found in the same decision of the trial court that he did actually offer
money to an officer of the United States Air Force in order that the latter may abstain from
8

procedure was provided for before the President can deport an alien-which provision was
expressly declared exempted from the repealing effect of the Immigration Act of 1940-is a
clear indication of the recognition, and inferentially a ratification, by the legislature of the
existence of such power in the Executive. And the, exercise of this power by the chief
Executive has been sanctioned by this Court in several decisions.2
Under the present and existing laws, therefore, deportation of an undesirable alien may
be effected in two ways: by order of the President, after due investigation, pursuant to Section
69 of the Revised Administrative Code, and by the Commissioner of Immigration, upon
recommendation by the Board of Commissioners, under Section 37 of Commonwealth Act No.
613.
Petitioners contend, however, that even granting that the President is invested with
power to deport, still he may do so only upon the grounds enumerated in Commonwealth Act
No. 613, as amended, and on no other, as it would be unreasonable and undemocratic to hold
that an alien may be deported upon an unstated or undefined ground depending merely on
the unlimited discretion of the Chief Executive. This contention is not without merit,
considering that whenever the legislature believes a certain act or conduct to be a just cause
for deportation, it invariably enacts a law to that effect. Thus, in a number of amendatory acts,
grounds have been added to those originally contained in Section 37 of Commonwealth Act
No. 613, as justifying deportation of an alien, as well as other laws which provide deportation
as part of the penalty imposed on aliens committing violation thereof.
Be this as it may, the charges against the herein petitioners constitute in effect an act of
profiteering, hoarding or blackmarketing of U.S. dollars, in violation of the Central Bank
regulations an economic sabotage which is a ground for deportation under the
provisions of Republic Act 503 amending Section 37 of the Philippine Immigration Act of 1940.
The President may therefore order the deportation of these petitioners if after investigation
they are shown to have committed the act charged.
There seems to be no doubt that the President's power of investigation may be
delegated. This is clear from a reading of Section 69 of the Revised Administrative Code
which provides for a "prior investigation, conducted by said Executive (the President) or his
authorized agent." The first executive order on the subject was that of Governor General
Frank Murphy (No. 494, July 26, 1934), constituting a board to take action on complaints
against foreigners, to conduct investigations and thereafter make recommendations. By virtue
of Executive Order No. 33 dated May 29, 1936, President Quezon created the Deportation
Board primarily to receive complaints against aliens charged to be undesirable, to conduct
investigation pursuant to Section 69 of the Revised Administrative Code and the rules and
regulations therein provided, and make the corresponding recommendation. 3 Since then, the
Deportation Board has been conducting the investigation as the authorized agent of the
President.
This gives rise to the question regarding the extent of the power of the President to
conduct investigation, i.e., whether such authority carries with it the power to order the arrest
of the alien complained of, since the Administrative Code is silent on the matter, and if it does,
whether the same may be delegated to the respondent Deportation Board.1awphl.nt
Let it be noted that Section 69 of the Revised Administrative Code, unlike
Commonwealth Act No. 613 wherein the Commissioner of Immigration was specifically
granted authority, among others, to make arrests, fails to provide the President with like

specific power to be exercised in connection with such investigation. It must be for this reason
that President Roxas for the first time, saw it necessary to issue his Executive Order No. 69,
dated July 29, 1947, providing
For the purpose of insuring the appearance of aliens charged before the
Deportation Board created under Executive Order No. 37, dated January 4, 1947,
and facilitating the execution of the order of deportation whenever the President
decides the case against the respondent. I, Manuel Roxas, President of the
Philippines, by virtue of the powers vested in me by law, do hereby order that all
respondents in deportation proceedings shall file a bond with the Commissioner of
Immigration in such amount and containing such conditions as he may prescribe. .
xxx
xxx
xxx
Note that the executive order only required the filing of a bond to secure appearance of
the alien under investigation. It did not authorize the arrest of the respondent.
It was only on January 5, 1951, when President Quirino reorganized the Deportation
Board by virtue of his Executive Order No. 398, that the Board was authorized motu proprio or
upon the filing of formal charges by the Special Prosecutor of the Board, to issue the warrant
for the arrest of the alien complained of and to hold him under detention during the
investigation unless he files a bond for his provisional release in such amount and under such
conditions as may be prescribed by the Chairman of the Board.
As has been pointed out elsewhere, Section 69 of the Revised Administrative Code,
upon whose authority the President's power to deport is predicated, does not provide for the
exercise of the power to arrest. But the Solicitor General argues that the law could not have
denied to the Chief Executive acts which are absolutely necessary to carry into effect the
power of deportation granted him, such as the authority to order the arrest of the foreigner
charged as undesirable.
In this connection, it must be remembered that the right of an individual to be secure in
his person is guaranteed by the Constitution in the following language:.
3. The right of the People to be secure in their persons, houses, papers and
effects against unreasonable searches and seizures shall not be violated, and no
warrants shall issue but upon probable cause, to be determined by the judge after
examination under oath or affirmation of the complainant and the witnesses he may
produce, and particularly describing the place to be searched, and the persons or
things to be seized." (Sec 1, Art. III, Bill of Rights, Philippine Constitution).
As observed by the late Justice Laurel in his concurring opinion in the case of
Rodriguez, et al. v. Villamiel, et al. (65 Phil. 230, 239), this provision is not the same as that
contained in the Jones Law wherein this guarantee is placed among the rights of the accused.
Under our Constitution, the same is declared a popular right of the people and, of course,
indisputably it equally applies to both citizens and foreigners in this country. Furthermore, a
notable innovation in this guarantee is found in our Constitution in that it specifically provides
that the probable cause upon which a warrant of arrest may be issued, must be determined by
the judge after examination under oath, etc., of the complainant and the witnesses he may
produce. This requirement "to be determined by the judge" is not found in the Fourth
Amendment of the U.S. Constitution, in the Philippine Bill or in the Jones Act, all of which do
not specify who will determine the existence of a probable cause. Hence, under their
9

provisions, any public officer may be authorized by the Legislature to make such
determination, and thereafter issue the warrant of arrest. Under the express terms of our
Constitution, it is, therefore, even doubtful whether the arrest of an individual may be ordered
by any authority other than the judge if the purpose is merely to determine the existence of a
probable cause, leading to an administrative investigation. The Constitution does not
distinguish between warrants in a criminal case and administrative warrants in administrative
proceedings. And, if one suspected of having committed a crime is entitled to a determination
of the probable cause against him, by a judge, why should one suspected of a violation of an
administrative nature deserve less guarantee? Of course it is different if the order of arrest is
issued to carry out a final finding of a violation, either by an executive or legislative officer or
agency duly authorized for the purpose, as then the warrant is not that mentioned in the
Constitution which is issuable only on probable cause. Such, for example, would be a warrant
of arrest to carry out a final order of deportation, or to effect compliance of an order of
contempt.
The contention of the Solicitor General that the arrest of a foreigner is necessary to
carry into effect the power of deportation is valid only when, as already stated, there is already
an order of deportation. To carry out the order of deportation, the President obviously has the
power to order the arrest of the deportee. But, certainly, during the investigation, it is not
indispensable that the alien be arrested. It is enough, as was true before the executive order
of President Quirino, that a bond be required to insure the appearance of the alien during the
investigation, as was authorized in the executive order of President Roxas. Be that as it may,
it is not imperative for us to rule, in this proceeding - and nothing herein said is intended to so
decide on whether or not the President himself can order the arrest of a foreigner for
purposes of investigation only, and before a definitive order of deportation has been issued.
We are merely called upon to resolve herein whether, conceding without deciding that the
President can personally order the arrest of the alien complained of, such power can be
delegated by him to the Deportation Board.
Unquestionably, the exercise of the power to order the arrest of an individual demands
the exercise of discretion by the one issuing the same, to determine whether under specific
circumstances, the curtailment of the liberty of such person is warranted. The fact that the
Constitution itself, as well as the statute relied upon, prescribe the manner by which the
warrant may be issued, conveys the intent to make the issuance of such warrant dependent
upon conditions the determination of the existence of which requires the use of discretion by
the person issuing the same. In other words, the discretion of whether a warrant of arrest shall
issue or not is personal to the one upon whom the authority devolves. And authorities are to
the effect that while ministerial duties may be delegated, official functions requiring the
exercise of discretion and judgment, may not be so delegated. Indeed, an implied grant of
power, considering that no express authority was granted by the law on the matter under
discussion, that would serve the curtailment or limitation on the fundamental right of a person,
such as his security to life and liberty, must be viewed with caution, if we are to give meaning
to the guarantee contained in the Constitution. If this is so, then guarantee a delegation of that
implied power, nebulous as it is, must be rejected as inimical to the liberty of the people. The
guarantees of human rights and freedom can not be made to rest precariously on such a
shaky foundation.
We are not unaware of the statements made by this Court in the case of Tan Sin v.
Deportation Board (G.R. No. L-11511, Nov. 28,1958). It may be stated, however, that the

power of arrest was not squarely raised in that proceeding, but only as a consequence of
therein petitioner's proposition that the President had no inherent power to deport and that the
charges filed against him did not constitute ground for deportation. .
IN VIEW OF THE FOREGOING, Executive Order No. 398, series of 1951, insofar as it
empowers the Deportation Board to issue warrant of arrest upon the filing of formal charges
against an alien or aliens and to fix bond and prescribe the conditions for the temporary
release of said aliens, is declared illegal. As a consequence, the order of arrest issued by the
respondent Deportation Board is declared null and void and the bonds filed pursuant to such
order of arrest, decreed cancelled. With the foregoing modification, the decision appealed
from is hereby affirmed. No costs. So ordered.
Padilla, Bautista Angelo, Labrador, Concepcion, Paredes, Dizon, Regala and Makalintal, JJ.,
concur.
Bengzon, C.J., reserved his vote.
Reyes, J.B.L., J., took no part.

10

G.R. Nos. 95122-23 May 31, 1991


BOARD OF COMMISSIONERS (COMMISSION ON IMMIGRATION AND DEPORTATION),
BOARD OF SPECIAL INQUIRY, COMMISSIONER ANDREA D. DOMINGO, ASSOCIATE
COMMISSIONER JORGE V. SARMIENTO, ACTING ASSOCIATE COMMISSIONER
REGINO R. SANTIAGO, MEMBERS OF THE BOARD OF SPECIAL INQUIRY,
ESTANISLAO CANTA, LEO MAGAHOM and BENJAMIN KALAW, petitioners,
vs.
HON. JOSELITO DELA ROSA, Presiding Judge, RTC Manila, Branch 29, WILLIAM T.
GATCHALIAN, respondents.
BOARD OF COMMISSIONERS (COMMISSION ON IMMIGRATION AND DEPORTATION),
BOARD OF SPECIAL INQUIRY, COMMISSIONER ANDREA D. DOMINGO, ASSOCIATE
COMMISSIONER JORGE V. SARMIENTO, ACTING ASSOCIATE COMMISSIONER
REGINO R. SANTIAGO, MEMBERS OF THE BOARD OF SPECIAL INQUIRY,
ESTANISLAO CANTA, LEO MAGAHOM and BENJAMIN KALAW, petitioners,
vs.
HON. TERESITA DIZON CAPULONG, Presiding Judge, RTC Branch 172, Valenzuela,
Metro Manila, DEE HUA T. GATCHALIAN, SHERWING T. GATCHALIAN, KENNETH T.
GATCHALIAN, REXLON T. GATCHALIAN, and WESLIE T. GATCHALIAN, respondents.
G.R. Nos. 95612-13 May 31, 1991
WILLIAM T. GATCHALIAN, petitioner,
vs.
BOARD OF COMMISSIONERS (COMMISSION ON IMMIGRATION AND DEPORTATION),
et al., respondents.
The Solicitor General for petitioners.
edesma, Saludo & Associates for respondent William Gatchalian.
Cervo and Tanay Law Office for respondent T.D. Capulong, D.H.T. Gatchalian, et al.

petition and petitioners' comment as answer to the counter-petition and gives due course to
the petitions.
There is no dispute as to the following facts:
On July 12, 1960, Santiago Gatchalian, grandfather of William Gatchalian, was recognized by
the Bureau of Immigration as a native born Filipino citizen following the citizenship of his
natural mother, Marciana Gatchalian (Annex "1", counter-petition). Before the Citizenship
Evaluation Board, Santiago Gatchalian testified that he has five (5) children with his wife Chu
Gim Tee, namely: Jose Gatchalian, Gloria Gatchalian, Francisco Gatchalian, Elena
Gatchalian and Benjamin Gatchalian (Annex "2", counter-petition).
On June 27, 1961, William Gatchalian, then a twelve-year old minor, arrived in Manila from
Hongkong together with Gloria, Francisco, and Johnson, all surnamed Gatchalian. They had
with them Certificates of Registration and Identity issued by the Philippine Consulate in
Hongkong based on a cablegram bearing the signature of the then Secretary of Foreign
Affairs, Felixberto Serrano, and sought admission as Filipino citizens. Gloria and Francisco
are the daughter and son, respectively, of Santiago Gatchalian; while William and Johnson
are the sons of Francisco.
After investigation, the Board of Special Inquiry No. 1 rendered a decision dated July 6, 1961,
admitting William Gatchalian and his companions as Filipino citizens (Annex "C", petition). As
a consequence thereof, William Gatchalian was issued Identification Certificate No. 16135 by
the immigration authorities on August 16, 1961 (Annex "D", petition).
On January 24, 1962, the then Secretary of Justice issued Memorandum No. 9 setting aside
all decisions purporting to have been rendered by the Board of Commissioners on appeal or
on review motu proprio of decisions of the Board of Special Inquiry. The same memorandum
directed the Board of Commissioners to review all cases where entry was allowed on the
ground that the entrant was a Philippine citizen. Among those cases was that of William and
others.
On July 6, 1962, the new Board of Commissioners, after a review motu proprio of the
proceedings had in the Board of Special Inquiry, reversed the decision of the latter and
ordered the exclusion of, among others, respondent Gatchalian (Annex "E", petition). A
warrant of exclusion also dated July 6, 1962 was issued alleging that "the decision of the
Board of Commissioners dated July 6, 1962 . . . has now become final and executory (Annex
"F", petition).
The actual date of rendition of said decision by the Board of Commissioners (whether on July
6, 1962 or July 20, 1962) became the subject of controversy in the 1967 case of Arocha vs.
Vivo (21 SCRA 532) wherein this Court sustained the validity of the decision of the new Board
of Commissioners having been promulgated on July 6, 1962, or within the reglementary
period for review.
Sometime in 1973, respondent Gatchalian, as well as the others covered by the July 6, 1962
warrant of exclusion, filed a motion for re-hearing with the Board of Special Inquiry where the
deportion case against them was assigned.
On March 14, 1973, the Board of Special Inquiry recommended to the then Acting
Commissioner Victor Nituda the reversal of the July 6, 1962 decision of the then Board of
Commissioners and the recall of the warrants of arrest issued therein (Annex "5", counterpetition).

BIDIN, J.:p
This is a petition for certiorari and prohibition filed by the Solicitor General seeking 1) to set
aside the Resolution/Temporary Restraining Order dated September 7, 1990, issued by
respondent Judge de la Rosa in Civil Case No. 90-54214 which denied petitioners' motion to
dismiss and restrained petitioners from commencing or continuing with any of the proceedings
which would lead to the deportation of respondent William Gatchalian, docketed as D.C. No.
90-523, as well as the Order of respondent Judge Capulong dated September 6, 1990 in Civil
Case No. 3431-V-90 which likewise enjoined petitioners from proceeding with the deportation
charges against respondent Gatchalian, and 2) to prohibit respondent judges from further
acting in the aforesaid civil cases.
On October 23, 1990, respondent Gatchalian filed his Comment with Counter-Petition,
docketed as G.R. Nos. 96512-13, alleging lack of jurisdiction on the part of respondent Board
of Commissioners, et al., over his person with prayer that he be declared a Filipino citizen, or
in the alternative, to remand the case to the trial court for further proceedings.
On December 13, 1990, petitioners filed their comment to respondent Gatchalian's counterpetition. The Court considers the comment filed by respondent Gatchalian as answer to the
11

On March 15, 1973, Acting Commissioner Nituda issued an order reaffirming the July 6, 1961
decision of the Board of Special Inquiry thereby admitting respondent Gatchalian as a Filipino
citizen and recalled the warrant of arrest issued against him (Annex "6", counter-petition).
On June 7, 1990, the acting director of the National Bureau of Investigation wrote the
Secretary of Justice recommending that respondent Gatchalian along with the other
applicants covered by the warrant of exclusion dated July 6, 1962 be charged with violation of
Sec. 37 (a), pars. 1 and 2, in relation to Secs. 45 (c), and (d) and (e) of Commonwealth Act
No. 613, as amended, also known as the Immigration Act of 1940 (Annex "G", petition).
On August 1, 1990, the Secretary of Justice indorsed the recommendation of the NBI to the
Commissioner of Immigration for investigation and immediate action (Annex "20", counterpetition).
On August 15, 1990, petitioner Commissioner Domingo of the Commission of Immigration and
Deportation * issued a mission order commanding the arrest of respondent William Gatchalian
(Annex "18", counter-petition). The latter appeared before Commissioner Domingo on August
20, 1990 and was released on the same day upon posting P200,000.00 cash bond.
On August 29, 1990, William Gatchalian filed a petition for certiorari and prohibition with
injunction before the Regional Trial Court of Manila, Br. 29, presided by respondent Judge
dela Rosa, docketed as Civil Case No. 90-54214.
On September 4, 1990, petitioners filed a motion to dismiss Civil Case No. 90-54214 alleging
that respondent judge has no jurisdiction over the Board of Commissioners and/or the Board
of Special Inquiry. Nonetheless, respondent judge dela Rosa issued the assailed order dated
September 7, 1990, denying the motion to dismiss.
Meanwhile, on September 6, 1990, respondent Gatchalian's wife and minor children filed
before the Regional Trial Court of Valenzuela, Metro Manila, Br. 172, presided by respondent
judge Capulong Civil Case No. 3431-V-90 for injunction with writ of preliminary injunction. The
complaint alleged, among others, that petitioners acted without or in excess of jurisdiction in
the institution of deportation proceedings against William. On the same day, respondent
Capulong issued the questioned temporary restraining order restraining petitioners from
continuing with the deportation proceedings against William Gatchalian.
The petition is anchored on the following propositions: 1) respondent judges have no
jurisdiction over petitioners (Board of Commissioners, et al.,) and the subject matter of the
case, appellate jurisdiction being vested by BP 129 with the Court of Appeals; 2) assuming
respondent judges have jurisdiction, they acted with grave abuse of discretion in preempting
petitioners in the exercise of the authority and jurisdiction to hear and determine the
deportation case against respondent Gatchalian, and in the process determine also his
citizenship; 3) respondent judge dela Rosa gravely abused his discretion in ruling that the
issues raised in the deportation proceedings are beyond the competence and jurisdiction of
petitioners, thereby disregarding the cases of Arocha vs. Vivo and Vivo vs. Arca (supra),
which put finality to the July 6, 1962 decision of the Board of Commissioners that respondent
Gatchalian is a Chinese citizen; and 4) respondent judge Capulong should have dismissed
Civil Case No. 3431-V-90 for forum-shopping.
In his counter-petition, William Gatchalian alleges among others that: 1) assuming that the
evidence on record is not sufficient to declare him a Filipino citizen, petitioners have no
jurisdiction to proceed with the deportation case until the courts shall have finally resolved the

question of his citizenship; 2) petitioners can no longer judiciously and fairly resolve the
question of respondent's citizenship in the deportation case because of their bias, prejudgment and prejudice against him; and 3) the ground for which he is sought to be deported
has already prescribed.
For purposes of uniformity, the parties herein will be referred to in the order the petitions were
filed.
Petitioners argue that under Sec. 9 (3) of BP 129, it is the Court of Appeals which has
exclusive appellate jurisdiction over all final judgments or orders of quasi-judicial agencies,
boards or commissions, such as the Board of Commissioners and the Board of Special
Inquiry.
Respondent, on the other hand, contends that petitioners are not quasi-judicial agencies and
are not in equal rank with Regional Trial Courts.
Under Sec. 21 (1) of Batas Pambansa Blg. 129, the Regional Trial Courts have concurrent
jurisdiction with this Court and the Court of Appeals to issue "writs of certiorari, prohibition,
mandamus, quo warranto, habeas corpus and injunction which may be enforced in any part of
their respective regions, . . ." Thus, the RTCs are vested with the power to determine whether
or not there has been a grave abuse of discretion on the part of any branch or instrumentality
of the government.
It is true that under Sec. 9 (3) of Batas Pambansa Blg. 129, the Court of Appeals is vested
with
(3) Exclusive appellate jurisdiction over all final judgments, decisions,
resolutions, order, or awards of Regional Trial Courts and quasi-judicial
agencies, instrumentalities, board or commission, except those falling
within the appellate jurisdiction of the Supreme Court in accordance with
the Constitution, the provisions of this Act, and of sub-paragraph (1) of the
third paragraph of and sub-paragraph (4) of the fourth paragraph of
Section 17 of the Judiciary Act of 1948.
It does not provide, however, that said exclusive appellate jurisdiction of the Court of Appeals
extends to all quasi-judicial agencies. The quasi-judicial bodies whose decisions are
exclusively appealable to the Court of Appeals are those which under the law, Republic Act
No. 5434, or their enabling acts, are specifically appealable to the Court of Appeals
(Presidential Anti-Dollar Salting Task Force vs. Court of Appeals, 171 SCRA 348 [1989];
Lupangco vs. Court of Appeals, 160 SCRA 848 [1988]). Thus, under Republic Act No. 5434, it
is specifically provided that the decisions of the Land Registration Commission (LRC), the
Social Security Commission (SSC), Civil Aeronautics Board (CAB), the Patent Office and the
Agricultural Invention Board are appealable to the Court of Appeals.
In the Presidential Anti-Dollar Salting Task Force (supra), this Court clarified the matter when
We ruled:
Under our Resolution dated January 11, 1983:
. . . The appeals to the Intermediate Appellate Court
(now Court of Appeals) from quasi-judicial bodies shall
continue to be governed by the provisions of Republic
Act No. 5434 insofar as the same is not inconsistent
with the provisions of B.P. Blg. 129.
12

The pertinent provisions of Republic Act No. 5434 are as follows:


Sec. 1. Appeals from specified agencies. Any
provision of existing law or Rules of Court to the
contrary notwithstanding, parties aggrieved by a final
ruling, award, order, or decision, or judgment of the
Court of Agrarian Relations; the Secretary of Labor
under Section 7 of Republic Act Numbered Six
hundred and two, also known as the "Minimum Wage
Law"; the Department of Labor under Section 23 of
Republic Act Numbered Eight hundred seventy-five,
also known as the "Industrial Peace Act"; the Land
Registration Commission; the Social Security
Commission; the Civil Aeronautics Board; the Patent
Office and the Agricultural Inventions Board, may
appeal therefrom to the Court of Appeals, within the
period and in the manner herein provided, whether the
appeal involves questions of fact, mixed questions of
fact and law, or questions of law, or all three kinds of
questions. From final judgments or decisions of the
Court of Appeals, the aggrieved party may appeal by
certiorari to the Supreme Court as provided under
Rule 45 of the Rules of Court.
Because of subsequent amendments, including the abolition of various
special courts, jurisdiction over quasi-judicial bodies has to be,
consequently, determined by the corresponding amendatory statutes.
Under the Labor Code, decisions and awards of the National Labor
Relations Commission are final and executory, but, nevertheless,
reviewable by this Court through a petition for certiorari and not by way of
appeal.
Under the Property Registration Decree, decision of the Commission of
Land Registration, en consulta, are appealable to the Court of Appeals.
The decisions of the Securities and Exchange Commission are likewise
appealable to the Appellate Court, and so are decisions of the Social
Security Commission.
As a rule, where legislation provides for an appeal from decisions of
certain administrative bodies to the Court of Appeals, it means that such
bodies are co-equal with the Regional Trial Courts, in terms of rank and
stature, and logically, beyond the control of the latter. (Emphasis supplied)
There are quasi-judicial agencies, as the National Labor Relations Commissions, whose
decisions are directly appealable to this Court. It is only when a specific law, as Republic Act
No. 5434, provides appeal from certain bodies or commissions to the Court of Appeals as the
Land Registration Commission (LRC), Securities and Exchange Commission (SEC) and
others, that the said commissions or boards may be considered co-equal with the RTCs in
terms of rank, stature and are logically beyond the control of the latter.

However, the Bureau of Immigration (or CID) is not among those quasi-judicial agencies
specified by law whose decisions, orders, and resolutions are directly appealable to the Court
of Appeals. In fact, its decisions are subject to judicial review in accordance with Sec. 25,
Chapter 4, Book VII of the 1987 Administrative Code, which provides as follows:
Sec. 25. Judicial Review.(1) Agency decisions shall be subject to
judicial review in accordance with this chapter and applicable laws.
xxx xxx xxx
(6) The review proceeding shall be filed in the court specified in the statute
or, in the absence thereof, in any court of competent jurisdiction in
accordance with the provisions on venue of the Rules of Court.
Said provision of the Administrative Code, which is subsequent to B.P. Blg. 129 and which
thus modifies the latter, provides that the decision of an agency like the Bureau of Immigration
should be subject to review by the court specified by the statute or in the absence thereof, it is
subject to review by any court of competent jurisdiction in accordance with the provisions on
venue of the Rules of Court.
B.P. Blg. 129 did not intend to raise all quasi-judicial bodies to the same level or rank of the
RTC except those specifically provided for under the law as aforestated. As the Bureau of
Immigration is not of equal rank as the RTC, its decisions may be appealable to, and may be
reviewed through a special civil action for certiorari by, the RTC (Sec. 21, (1) BP 129).
True, it is beyond cavil that the Bureau of Immigration has the exclusive authority and
jurisdiction to try and hear cases against an alleged alien, and in the process, determine also
their citizenship (Lao Gi vs. Court of Appeals, 180 SCRA 756 [1989]). And a mere claim of
citizenship cannot operate to divest the Board of Commissioners of its jurisdiction in
deportation proceedings (Miranda vs. Deportation Board, 94 Phil. 531 [1954]).
However, the rule enunciated in the above-cases admits of an exception, at least insofar as
deportation proceedings are concerned. Thus, what if the claim to citizenship of the alleged
deportee is satisfactory? Should the deportation proceedings be allowed to continue or should
the question of citizenship be ventilated in a judicial proceeding? In Chua Hiong vs.
Deportation Board (96 Phil. 665 [1955]), this Court answered the question in the affirmative,
and We quote:
When the evidence submitted by a respondent is conclusive of his
citizenship, the right to immediate review should also be recognized and
the courts should promptly enjoin the deportation proceedings. A citizen is
entitled to live in peace, without molestation from any official or authority,
and if he is disturbed by a deportation proceeding, he has the
unquestionable right to resort to the courts for his protection, either by a
writ of habeas corpus or of prohibition, on the legal ground that the Board
lacks jurisdiction. If he is a citizen and evidence thereof is satisfactory,
there is no sense nor justice in allowing the deportation proceedings to
continue, granting him the remedy only after the Board has finished its
investigation of his undesirability.
. . . And if the right (to peace) is precious and valuable at all, it must also
be protected on time, to prevent undue harassment at the hands of illmeaning or misinformed administrative officials. Of what use is this much
13

boasted right to peace and liberty if it can be availed of only after the
Deportation Board has unjustly trampled upon it, besmirching the citizen's
name before the bar of public opinion? (Emphasis supplied)
The doctrine of primary jurisdiction of petitioners Board of Commissioners over deportation
proceedings is, therefore, not without exception (Calacday vs. Vivo, 33 SCRA 413 [1970];
Vivo vs. Montesa, 24 SCRA 155 [1967]). Judicial intervention, however, should be granted
only in cases where the "claim of citizenship is so substantial that there are reasonable
grounds to believe that the claim is correct. In other words, the remedy should be allowed only
on sound discretion of a competent court in a proper proceeding (Chua Hiong vs. Deportation
Board, supra; Co. vs. Deportation Board, 78 SCRA 107 [1977]). It appearing from the records
that respondent's claim of citizenship is substantial, as We shall show later, judicial
intervention should be allowed.
In the case at bar, the competent court which could properly take cognizance of the
proceedings instituted by respondent Gatchalian would nonetheless be the Regional Trial
Court and not the Court of Appeals in view of Sec. 21 (1), BP 129, which confers upon the
former jurisdiction over actions for prohibition concurrently with the Court of Appeals and the
Supreme Court and in line with the pronouncements of this Court in Chua Hiong and Co
cases.
Ordinarily, the case would then be remanded to the Regional Trial Court. But not in the case
at bar. Considering the voluminous pleadings submitted by the parties and the evidence
presented, We deem it proper to decide the controversy right at this instance. And this course
of action is not without precedent for "it is a cherished rule of procedure for this Court to
always strive to settle the entire controversy in a single proceeding leaving no root or branch
to bear the seeds of future litigation. No useful purpose will be served if this case is remanded
to the trial court only to have its decision raised again to the Court of Appeals and from there
to this Court" (Marquez vs. Marquez, 73 Phil. 74; Keramic Industries, Inc. vs. Guerrero, 61
SCRA 265 [1974]) Alger Electric, Inc. vs. Court of Appeals (135 SCRA 37 [1985]), citing
Gayos vs. Gayos (67 SCRA 146 [1975]).
In Lianga Bay Logging Co., Inc. vs. Court of Appeals (157 SCRA 357 [1988]), We also stated:
Remand of the case to the lower court for further reception of evidence is
not necessary where the court is in a position to resolve the dispute based
on the records before it. On many occasions, the Court, in the public
interest and the expeditious administration of justice, has resolved actions
on the merits instead of remanding them to the trial court for further
proceedings, such as where the ends of justice would not be subserved
by the remand of the case or when public interest demands an early
disposition of the case or where the trial court had already received all the
evidence of the parties (Quisumbing vs. CA, 112 SCRA 703; Francisco, et
al., vs. The City of Davao, et al., supra; Republic vs. Security Credit &
Acceptance Corp., et al., 19 SCRA 58; Samal vs. CA, supra; Republic vs.
Central Surety & Insurance Co., 25 SCRA 641).
Likewise in Tejones vs. Gironella (159 SCRA 100 [1988]), We said:
Sound practice seeks to accommodate the theory which avoids waste of
time, effort and expense, both to the parties and the government, not to
speak of delay in the disposal of the case (cf. Fernandez vs. Garcia, 92

Phil. 592, 297). A marked characterstic of our judicial set-up is that where
the dictates of justice so demand . . . the Supreme Court should act, and
act with finality (Li Siu Liat vs. Republic, 21 SCRA 1039, 1046, citing
Samal vs. CA, 99 Phil. 230 and US vs. Gimenez, 34 Phil. 74.) (Beautifont,
Inc. vs. Court of appeals, et al., Jan. 29, 1988; See also Labo vs.
Commission on Elections, 176 SCRA 1 [1989]).
Respondent Gatchalian has adduced evidence not only before the Regional Trial Court but
also before Us in the form of public documents attached to his pleadings. On the other hand,
Special Prosecutor Renato Mabolo in his Manifestation (dated September 6, 1990; Rollo, p.
298, counter-petition) before the Bureau of Immigration already stated that there is no longer a
need to adduce evidence in support of the deportation charges against respondent. In
addition, petitioners invoke that this Court's decision in Arocha vs. Vivo and Vivo vs. Arca
(supra), has already settled respondent's alienage. Hence, the need for a judicial
determination of respondent's citizenship specially so where the latter is not seeking
admission, but is already in the Philippines (for the past thirty [30] years) and is being expelled
(Chua Hiong vs. Deportation Board, supra).
According to petitioners, respondent's alienage has been conclusively settled by this Court in
the Arocha and Vivo cases, We disagree. It must be noted that in said cases, the sole issue
resolved therein was the actual date of rendition of the July 6, 1962 decision of the then board
of Commissioners, i.e., whether the decision was rendered on July 6, 1962 or on July 20,
1962 it appearing that the figure (date) "20" was erased and over it was superimposed the
figure "6" thereby making the decision fall within the one-year reglementary period from July 6,
1961 within which the decision may be reviewed. This Court did not squarely pass upon any
question of citizenship, much less that of respondent's who was not a party in the aforesaid
cases. The said cases originated from a petition for a writ of habeas corpus filed on July 21,
1965 by Macario Arocha in behalf of Pedro Gatchalian. Well settled is the rule that a person
not party to a case cannot be bound by a decision rendered therein.
Neither can it be argued that the Board of Commissioners' decision (dated July 6, 1962)
finding respondent's claim to Philippine citizenship not satisfactorily proved, constitute res
judicata. For one thing, said decision did not make any categorical statement that respondent
Gatchalian is a Chinese. Secondly, the doctrine of res judicata does not apply to questions of
citizenship (Labo vs. Commission on Elections (supra); citing Soria vs. Commissioner of
Immigration, 37 SCRA 213; Lee vs. Commissioner of Immigration, 42 SCRA 561 [1971]; Sia
Reyes vs. Deportation Board, 122 SCRA 478 [1983]).
In Moy Ya Lim vs. Commissioner of Immigration (41 SCRA 292 [1971]) and in Lee vs.
Commissioner of Immigration (supra), this Court declared that:
(e)verytime the citizenship of a person is material or indispensable in a
judicial or administrative case, whatever the corresponding court or
administrative authority decides therein as to such citizenship is generally
not considered as res adjudicata, hence it has to be threshed out again
and again as the occasion may demand.
An exception to the above rule was laid by this Court in Burca vs. Republic (51 SCRA 248
[1973]), viz:
We declare it to be a sound rule that where the citizenship of a party in a
case is definitely resolved by a court or by an administrative agency, as a
14

material issue in the controversy, after a full-blown hearing with the active
participation of the Solicitor General or his authorized representative, and
this finding or the citizenship of the party is affirmed by this Court, the
decision on the matter shall constitute conclusive proof of such party's
citizenship in any other case or proceeding. But it is made clear that in no
instance will a decision on the question of citizenship in such cases be
considered conclusive or binding in any other case or proceeding, unless
obtained in accordance with the procedure herein stated.
Thus, in order that the doctrine of res judicata may be applied in cases of citizenship, the
following must be present: 1) a person's citizenship must be raised as a material issue in a
controversy where said person is a party; 2) the Solicitor General or his authorized
representative took active part in the resolution thereof, and 3) the finding or citizenship is
affirmed by this Court.
Gauged by the foregoing, We find the pre-conditions set forth in Burca inexistent in the Arocha
and Vivo cases relied upon by petitioners. Indeed, respondent William Gatchalian was not
even a party in said cases.
Coming now to the contention of petitioners that the arrest of respondent follows as a matter
of consequence based on the warrant of exclusion issued on July 6, 1962, coupled with the
Arocha and Vivo cases (Rollo, pp. 33), the Court finds the same devoid of merit.
Sec. 37 (a) of Commonwealth Act No. 613, as amended, otherwise known as the Immigration
Act of 1940, reads:
Sec. 37. (a) The following aliens shall be arrested upon the warrant of the
Commissioner of Immigration or of any other officer designated by him for
the purpose and deported upon the warrant of the Commissioner of
Immigration after a determination by the Board of Commissioner of the
existence of the ground for deportation as charged against the alien.
(Emphasis supplied)
From a perusal of the above provision, it is clear that in matters of implementing the
Immigration Act insofar as deportation of aliens are concerned, the Commissioner of
Immigration may issue warrants of arrest only after a determination by the Board of
Commissioners of the existence of the ground for deportation as charged against the alien. In
other words, a warrant of arrest issued by the Commissioner of Immigration, to be valid, must
be for the sole purpose of executing a final order of deportation. A warrant of arrest issued by
the Commissioner of Immigration for purposes of investigation only, as in the case at bar, is
null and void for being unconstitutional (Ang Ngo Chiong vs. Galang, 67 SCRA 338 [1975]
citing Po Siok Pin vs. Vivo, 62 SCRA 363 [1975]; Vivo vs. Montesa, 24 SCRA 155; Morano vs.
Vivo, 20 SCRA 562; Qua Chee Gan vs. Deportation Board, 9 SCRA 27 [1963]; Ng Hua To vs.
Galang, 10 SCRA 411; see also Santos vs. Commissioner of Immigration, 74 SCRA 96
[1976]).
As We held in Qua Chee Gan vs. Deportation Board (supra), "(t)he constitution does not
distinguish warrants between a criminal case and administrative proceedings. And if one
suspected of having committed a crime is entitled to a determination of the probable cause
against him, by a judge, why should one suspected of a violation of an administrative nature
deserve less guarantee?" It is not indispensable that the alleged alien be arrested for
purposes of investigation. If the purpose of the issuance of the warrant of arrest is to

determine the existence of probable cause, surely, it cannot pass the test of constitutionality
for only judges can issue the same (Sec. 2, Art. III, Constitution).
A reading of the mission order/warrant of arrest (dated August 15, 1990; Rollo, p. 183,
counter-petition) issued by the Commissioner of Immigration, clearly indicates that the same
was issued only for purposes of investigation of the suspects, William Gatchalian included.
Paragraphs 1 and 3 of the mission order directs the Intelligence Agents/Officers to:
xxx xxx xxx
1. Make a warrantless arrest under the Rules of Criminal Procedure, Rule
113, Sec. 5, for violation of the Immigration Act, Sec. 37, para. a; Secs. 45
and 46 Administrative Code;
xxx xxx xxx
3. Deliver the suspect to the Intelligence Division and immediately conduct
custodial interrogation, after warning the suspect that he has a right to
remain silent and a right to counsel; . . .
Hence, petitioners' argument that the arrest of respondent was based, ostensibly, on the July
6, 1962 warrant of exclusion has obviously no leg to stand on. The mission order/warrant of
arrest made no mention that the same was issued pursuant to a final order of deportation or
warrant of exclusion.
But there is one more thing that militates against petitioners' cause. As records indicate, which
petitioners conveniently omitted to state either in their petition or comment to the counterpetition of respondent, respondent Gatchalian, along with others previously covered by the
1962 warrant of exclusion, filed a motion for re-hearing before the Board of Special Inquiry
(BSI) sometime in 1973.
On March 14, 1973, the Board of Special Inquiry, after giving due course to the motion for rehearing, submitted a memorandum to the then Acting Commissioner Victor Nituda (Annex "5",
counter-petition) recommending 1 the reconsideration of the July 6, 1962 decision of the then
Board of Commissioners which reversed the July 6, 1961 decision of the then Board of
Special Inquiry No. 1 and 2 the lifting of the warrants of arrest issued against applicants. The
memorandum inferred that the "very basis of the Board of Commissioners in reversing the
decision of the Board of Special Inquiry was due to a forged cablegram by the then Secretary
of Foreign Affairs, . . ., which was dispatched to the Philippine Consulate in Hong Kong
authorizing the registration of applicants as P.I. citizens." The Board of Special Inquiry
concluded that "(i)f at all, the cablegram only led to the issuance of their Certificate(s) of
Identity which took the place of a passport for their authorized travel to the Philippines. It being
so, even if the applicants could have entered illegally, the mere fact that they are citizens of
the Philippines entitles them to remain in the country."
On March 15, 1973, then Acting Commissioner Nituda issued an Order (Annex "6", counterpetition) which affirmed the Board of Special Inquiry No. 1 decision dated July 6, 1961
admitting respondent Gatchalian and others as Filipino citizens; recalled the July 6, 1962
warrant of arrest and revalidated their Identification Certificates.
The above order admitting respondent as a Filipino citizen is the last official act of the
government on the basis of which respondent William Gatchalian continually exercised the
rights of a Filipino citizen to the present. Consequently, the presumption of citizenship lies in
favor of respondent William Gatchalian.
15

There should be no question that Santiago Gatchalian, grandfather of William Gatchalian, is a


Filipino citizen. As a matter of fact, in the very order of the BOC of July 6, 1962, which
reversed the July 6, 1961 BSI order, it is an accepted fact that Santiago Gatchalian is a
Filipino. The opening paragraph of said order states:
The claim to Filipino citizenship of abovenamed applicants is based on the
citizenship of one Santiago Gatchalian whose Philippine citizenship was
recognized by the Bureau of Immigration in an Order dated July 12, 1960.
(Annex "37", Comment with Counter-Petition).
Nonetheless, in said order it was found that the applicants therein have not satisfactorily
proven that they are the children and/or grandchildren of Santiago Gatchalian. The status of
Santiago Gatchalian as a Filipino was reiterated in Arocha and Arca (supra) where advertence
is made to the "applicants being the descendants of one Santiago Gatchalian, a Filipino." (at
p. 539).
In the sworn statement of Santiago Gatchalian before the Philippine Consul in Hongkong in
1961 (Annex "1" to the Comment of petitioners to Counter-Petition), he reiterated his status as
a Philippine citizen being the illegitimate child of Pablo Pacheco and Marciana Gatchalian, the
latter being a Filipino; that he was born in Manila on July 25, 1905; and that he was issued
Philippine Passport No. 28160 (PA-No. A91196) on November 18, 1960 by the Department of
Foreign Affairs in Manila. In his affidavit of January 23, 1961 (Annex "5", counter-petition),
Santiago reiterated his claim of Philippine citizenship as a consequence of his petition for
cancellation of his alien registry which was granted on February 18, 1960 in C.E.B. No. 3660L; and that on July 20, 1960, he was recognized by the Bureau of Immigration as a Filipino
and was issued Certificate No. 1-2123.
The dissenting opinions of my esteemed brethrens, Messrs. Justices F.P. Feliciano and H.G.
Davide, Jr., proposing to re-open the question of citizenship of Santiago Gatchalian at this
stage of the case, where it is not even put in issue, is quite much to late. As stated above, the
records of the Bureau of Immigration show that as of July 20, 1960, Santiago Gatchalian had
been declared to be a Filipino citizen. It is a final decision that forecloses a re-opening of the
same 30 years later. Petitioners do not even question Santiago Gatchalian's Philippine
citizenship. It is the citizenship of respondent William Gatchalian that is in issue and
addressed for determination of the Court in this case.
Furthermore, petitioners' position is not enhanced by the fact that respondent's arrest came
twenty-eight (28) years after the alleged cause of deportation arose. Section 37 (b) of the
Immigration Act states that deportation "shall not be effected . . . unless the arrest in the
deportation proceedings is made within five (5) years after the cause of deportation arises." In
Lam Shee vs. Bengzon (93 Phil. 1065 [1953]), We laid down the consequences of such
inaction, thus:
There is however an important circumstance which places this case
beyond the reach of the resultant consequence of the fraudulent act
committed by the mother of the minor when she admitted that she gained
entrance into the Philippines by making use of the name of a Chinese
resident merchant other than that of her lawful husband, and that is, that
the mother can no longer be the subject of deportation proceedings for the
simple reason that more than 5 years had elapsed from the date of her
admission. Note that the above irregularity was divulged by the mother

herself, who in a gesture of sincerity, made an spontaneous admission


before the immigration officials in the investigation conducted in
connection with the landing of the minor on September 24, 1947, and not
through any effort on the part of the immigration authorities. And
considering this frank admission, plus the fact that the mother was found
to be married to another Chinese resident merchant, now deceased, who
owned a restaurant in the Philippines valued at P15,000 and which gives
a net profit of P500 a month, the immigration officials then must have
considered the irregularity not serious enough when, inspire of that
finding, they decided to land said minor "as a properly documented
preference quota immigrant" (Exhibit D). We cannot therefore but wonder
why two years later the immigration officials would reverse their attitude
and would take steps to institute deportation proceedings against the
minor.
Under the circumstances obtaining in this case, we believe that much as
the attitude of the mother would be condemned for having made use of an
improper means to gain entrance into the Philippines and acquire
permanent residence there, it is now too late, not to say unchristian, to
deport the minor after having allowed the mother to remain even illegally
to the extent of validating her residence by inaction, thus allowing the
period of prescription to set in and to elapse in her favor. To permit his
deportation at this late hour would be to condemn him to live separately
from his mother through no fault of his thereby leaving him to a life of
insecurity resulting from lack of support and protection of his family. This
inaction or oversight on the part of immigration officials has created an
anomalous situation which, for reasons of equity, should be resolved in
favor of the minor herein involved. (Emphasis supplied)
In the case at bar, petitioners' alleged cause of action and deportation against herein
respondent arose in 1962. However, the warrant of arrest of respondent was issued by
Commissioner Domingo only on August 15, 1990 28 long years after. It is clear that
petitioners' cause of action has already prescribed and by their inaction could not now be
validly enforced by petitioners against respondent William Gatchalian. Furthermore, the
warrant of exclusion dated July 6, 1962 was already recalled and the Identification certificate
of respondent, among others, was revalidated on March 15, 1973 by the then Acting
Commissioner Nituda.
It is also proposed in the dissenting opinions of Messrs. Justices Feliciano and Davide, Jr.,
that the BOC decision dated July 6, 1962 and the warrant of exclusion which was found to be
valid in Arocha should be applicable to respondent William Gatchalian even if the latter was
not a party to said case. They also opined that under Sec. 37 (b) of the Immigration Act, the
five (5) years limitation is applicable only where the deportation is sought to be effected under
clauses of Sec. 37 (b) other than clauses 2, 7, 8, 11 and 12 and that no period of limitation is
applicable in deportations under clauses 2, 7, 8, 11 and 12.
The Court disagrees. Under Sec. 39 of the Immigration Act, it is reiterated that such
deportation proceedings should be instituted within five (5) years. Section 45 of the same Act
provides penal sanctions for violations of the offenses therein enumerated with a fine of "not
16

more than P1,000.00 and imprisonment for not more than two (2) years and deportation if he
is an alien." Thus:
Penal Provisions
Sec. 45. Any individual who
(a) When applying for an immigration document personates another
individual, or falsely appears in the name of deceased individual, or
evades the immigration laws by appearing under an assumed name;
fictitious name; or
(b) Issues or otherwise disposes of an immigration document, to any
person not authorized by law to receive such document; or
(c) Obtains, accepts or uses any immigration document, knowing it to be
false; or
(d) Being an alien, enters the Philippines without inspection and admission
by the immigration officials, or obtains entry into the Philippines by wilful,
false, or misleading representation or wilful concealment of a material fact;
or
(e) Being an alien shall for any fraudulent purpose represent himself to be
a Philippine citizen in order to evade any requirement of the immigration
laws; or
(f) In any immigration matter shall knowingly make under oath any false
statement or representations; or
(g) Being an alien, shall depart from the Philippines without first securing
an immigration clearance certificates required by section twenty-two of
this Act; or
(h) Attempts or conspires with another to commit any of the foregoing
acts, shall be guilty of an offense, and upon conviction thereof, shall be
fined not more than one thousand pesos, and imprisoned for not more
than two years, and deported if he is an alien. (Emphasis supplied)
Such offenses punishable by correctional penalty prescribe in 10 years (Art. 90, Revised
Penal Code); correctional penalties also prescribe in 10 years (Art. 92, Revised Penal Code).
It must be noted, however, that under Sec. 1, Act No. 3326 [1926], as amended, (Prescription
for Violations Penalized by Special Acts and Municipal Ordinances) "violations penalized by
special acts shall, unless otherwise provided in such acts, prescribe in accordance with the
following rules: . . .c) after eight years for those punished by imprisonment for two years or
more, but less than six years; . . ."
Consequently, no prosecution and consequent deportation for violation of the offenses
enumerated in the Immigration Act can be initiated beyond the eight-year prescriptive period,
the Immigration Act being a special legislation.
The Court, therefore, holds that the period of effecting deportation of an alien after entry or a
warrant of exclusion based on a final order of the BSI or BOC are not imprescriptible. The law
itself provides for a period of prescription. Prescription of the crime is forfeiture or loss of the
rights of the State to prosecute the offender after the lapse of a certain time, while prescription

of the penalty is the loss or forfeiture by the government of the right to execute the final
sentence after the lapse of a certain time (Padilla, Criminal Law, Vol. 1, 1974, at p. 855).
"Although a deportation proceeding does not partake of the nature of a criminal action,
however, considering that it is a harsh and extraordinary administrative proceeding affecting
the freedom and liberty of a person, the constitutional right of such person to due process
should not be denied. Thus, the provisions of the Rules of Court of the Philippines particularly
on criminal procedure are applicable to deportation proceedings." (Lao Gi vs. Court of
Appeals, supra). Under Sec. 6, Rule 39 of the Rules of Court, a final judgment may not be
executed after the lapse of five (5) years from the date of its entry or from the date it becomes
final and executory. Thereafter, it may be enforced only by a separate action subject to the
statute of limitations. Under Art. 1144 (3) of the Civil Code, an action based on judgment must
be brought within 10 years from the time the right of action accrues.
In relation to Sec. 37 (b) of the Immigration Act, the rule, therefore, is:
1. Deportation or exclusion proceedings should be initiated within five (5) years after the
cause of deportation or exclusion arises when effected under any other clauses other than
clauses 2, 7, 8, 11 and 12 and of paragraph (a) of Sec. 37 of the Immigration Act; and
2. When deportation or exclusion is effected under clauses 2, 7, 8, 11 and 12 of paragraph (a)
of Sec. 37, the prescriptive period of the deportation or exclusion proceedings is eight (8)
years.
In the case at bar, it took petitioners 28 years since the BOC decision was rendered on July 6,
1962 before they commenced deportation or exclusion proceedings against respondent
William Gatchalian in 1990. Undoubtedly, petitioners' cause of action has already prescribed.
Neither may an action to revive and/or enforce the decision dated July 6, 1962 be instituted
after ten (10) years (Art. 1144 [3], Civil Code).
Since his admission as a Filipino citizen in 1961, respondent William Gatchalian has
continuously resided in the Philippines. He married Ting Dee Hua on July 1, 1973 (Annex "8",
counter-petition) with whom he has four (4) minor children. The marriage contract shows that
said respondent is a Filipino (Annex "8"). He holds passports and earlier passports as a
Filipino (Annexes "9", "10" & "11", counter-petition). He is a registered voter of Valenzuela,
Metro Manila where he has long resided and exercised his right of suffrage (Annex 12,
counter-petition). He engaged in business in the Philippines since 1973 and is the
director/officer of the International Polymer Corp. and Ropeman International Corp. as a
Filipino (Annexes, "13" & "14", counter-petition). He is a taxpayer. Respondent claims that the
companies he runs and in which he has a controlling investment provides livelihood to 4,000
employees and approximately 25,000 dependents. He continuously enjoyed the status of
Filipino citizenship and discharged his responsibility as such until petitioners initiated the
deportation proceedings against him.
"The power to deport an alien is an act of the State. It is an act by or under the authority of the
sovereign power. It is a police measure against undesirable aliens whose presence in the
country is found to be injurious to the public good and domestic tranquility of the people" (Lao
Gi vs. Court of Appeals, supra). How could one who has helped the economy of the country
by providing employment to some 4,000 people be considered undesirable and be summarily
deported when the government, in its concerted drive to attract foreign investors, grants
Special Resident Visa to any alien who invest at least US$50,000.00 in the country? Even
assuming arguendo that respondent is an alien, his deportation under the circumstances is
17

unjust and unfair, if not downright illegal. The action taken by petitioners in the case at bar is
diametrically opposed to settled government policy.
Petitioners, on the other hand, claim that respondent is an alien. In support of their position,
petitioners point out that Santiago Gatchalian's marriage with Chu Gim Tee in China as well
as the marriage of Francisco (father of William) Gatchalian to Ong Chiu Kiok, likewise in
China, were not supported by any evidence other than their own self-serving testimony nor
was there any showing what the laws of China were. It is the postulate advanced by
petitioners that for the said marriages to be valid in this country, it should have been shown
that they were valid by the laws of China wherein the same were contracted. There being
none, petitioners conclude that the aforesaid marriages cannot be considered valid. Hence,
Santiago's children, including Francisco, followed the citizenship of their mother, having been
born outside of a valid marriage. Similarly, the validity of the Francisco's marriage not having
been demonstrated, William and Johnson followed the citizenship of their mother, a Chinese
national.
After a careful consideration of petitioner's argument, We find that it cannot be sustained.
In Miciano vs. Brimo (50 Phil. 867 [1924]; Lim and Lim vs. Collector of Customs, 36 Phil. 472;
Yam Ka Lim vs. Collector of Customs, 30 Phil. 46 [1915]), this Court held that in the absence
of evidence to the contrary, foreign laws on a particular subject are presumed to be the same
as those of the Philippines. In the case at bar, there being no proof of Chinese law relating to
marriage, there arises the presumption that it is the same as that of Philippine law.
The lack of proof of Chinese law on the matter cannot be blamed on Santiago Gatchalian
much more on respondent William Gatchalian who was then a twelve-year old minor. The fact
is, as records indicate, Santiago was not pressed by the Citizenship Investigation Board to
prove the laws of China relating to marriage, having been content with the testimony of
Santiago that the Marriage Certificate was lost or destroyed during the Japanese occupation
of China. Neither was Francisco Gatchalian's testimony subjected to the same scrutiny by the
Board of Special Inquiry. Nevertheless, the testimonies of Santiago Gatchalian and Francisco
Gatchalian before the Philippine consular and immigration authorities regarding their
marriages, birth and relationship to each other are not self-serving but are admissible in
evidence as statements or declarations regarding family reputation or tradition in matters of
pedigree (Sec. 34, Rule 130). Furtheremore, this salutary rule of evidence finds support in
substantive law. Thus, Art. 267 of the Civil Code provides:
Art. 267. In the absence of a record of birth, authentic document, final
judgment or possession of status, legitimate filiation may be proved by
any other means allowed by the Rules of Court and special laws. (See
also Art. 172 of the Family Code)
Consequently, the testimonies/affidavits of Santiago Gatchalian and Francisco Gatchalian
aforementioned are not self-serving but are competent proof of filiation (Art. 172 [2], Family
Code).
Philippine law, following the lex loci celebrationis, adheres to the rule that a marriage formally
valid where celebrated is valid everywhere. Referring to marriages contracted abroad, Art. 71
of the Civil Code (now Art. 26 of the Family Code) provides that "(a)ll marriages performed
outside of the Philippines in accordance with the laws in force in the country where they were
performed, and valid there as such, shall also be valid in this country . . ." And any doubt as to
the validity of the matrimonial unity and the extent as to how far the validity of such marriage

may be extended to the consequences of the coverture is answered by Art. 220 of the Civil
Code in this manner: "In case of doubt, all presumptions favor the solidarity of the family.
Thus, every intendment of law or facts leans toward the validity of marriage, the indissolubility
of the marriage bonds, the legitimacy of children, the community of property during marriage,
the authority of parents over their children, and the validity of defense for any member of the
family in case of unlawful aggression." (Emphasis supplied). Bearing in mind the "processual
presumption" enunciated in Miciano and other cases, he who asserts that the marriage is not
valid under our law bears the burden of proof to present the foreign law.
Having declared the assailed marriages as valid, respondent William Gatchalian follows the
citizenship of his father Francisco, a Filipino, as a legitimate child of the latter. Francisco, in
turn is likewise a Filipino being the legitimate child of Santiago Gatchalian who (the latter) is
admittedly a Filipino citizen whose Philippine citizenship was recognized by the Bureau of
Immigration in an order dated July 12, 1960.
Finally, respondent William Gatchalian belongs to the class of Filipino citizens contemplated
under Sec. 1, Article IV of the Constitution, which provides:
Sec. 1. The following are citizens of the Philippines:
(1) Those who are citizens of the Philippines at the time of the adoption of
this Constitution. . . .
This forecloses any further question about the Philippine citizenship of respondent William
Gatchalian.
The Court is not unaware of Woong Woo Yiu vs. Vivo (13 SCRA 552 [1965]) relied upon by
petitioners. The ruling arrived thereat, however, cannot apply in the case at bar for the simple
reason that the parties therein testified to have been married in China by a village leader,
which undoubtedly is not among those authorized to solemnize marriage as provided in Art.
56 of the Civil Code (now Art. 7, Family Code).
Premises considered, the Court deems it unnecessary to resolve the other issues raised by
the parties.
WHEREFORE, G.R. Nos. 95122-23 is DISMISSED for lack of merit; G.R. Nos. 95612-13 is
hereby GRANTED and respondent William Gatchalian is declared a Filipino citizen.
Petitioners are hereby permanently enjoined from continuing with the deportation proceedings
docketed as DC No. 90-523 for lack of jurisdiction over respondent Gatchalian, he being a
Filipino citizen; Civil Cases No. 90-54214 and 3431-V-90 pending before respondent judges
are likewise DISMISSED. Without pronouncement as to costs.
SO ORDERED.
Gutierrez, Jr., Gancayco, Sarmiento, Grio-Aquino and Medialdea, JJ., concur.
Fernan, C.J., and Narvasa, J., concur in the result.

18

G.R. No. 161434


March 3, 2004
MARIA JEANETTE C. TECSON and FELIX B. DESIDERIO, JR., petitioners,
vs.
The COMMISSION ON ELECTIONS, RONALD ALLAN KELLY POE (a.k.a. FERNANDO
POE, JR.) and VICTORINO X. FORNIER, respondents.
x-----------------------------x
G.R. No. 161634
March 3, 2004
ZOILO ANTONIO VELEZ, petitioner,
vs.
RONALD ALLAN KELLEY POE, a.k.a. FERNANDO POE, JR., respondent.
x-----------------------------x
G. R. No. 161824
March 3, 2004
VICTORINO X. FORNIER, petitioner,
vs.
HON. COMMISSION ON ELECTIONS and RONALD ALLAN KELLEY POE, ALSO KNOWN
AS FERNANDO POE JR., respondents.
DECISION
VITUG, J.:
Citizenship is a treasured right conferred on those whom the state believes are
deserving of the privilege. It is a "precious heritage, as well as an inestimable
acquisition,"1 that cannot be taken lightly by anyone - either by those who enjoy it or by
those who dispute it.
Before the Court are three consolidated cases, all of which raise a single question of profound
importance to the nation. The issue of citizenship is brought up to challenge the qualifications
of a presidential candidate to hold the highest office of the land. Our people are waiting for the
judgment of the Court with bated breath. Is Fernando Poe, Jr., the hero of silver screen, and
now one of the main contenders for the presidency, a natural-born Filipino or is he not?
The moment of introspection takes us face to face with Spanish and American colonial roots
and reminds us of the rich heritage of civil law and common law traditions, the fusion resulting
in a hybrid of laws and jurisprudence that could be no less than distinctly Filipino.
Antecedent Case Settings
On 31 December 2003, respondent Ronald Allan Kelly Poe, also known as Fernando Poe, Jr.
(hereinafter "FPJ"), filed his certificate of candidacy for the position of President of the
Republic of the Philippines under the Koalisyon ng Nagkakaisang Pilipino (KNP) Party, in the
forthcoming national elections. In his certificate of candidacy, FPJ, representing himself to be
a natural-born citizen of the Philippines, stated his name to be "Fernando Jr.," or "Ronald
Allan" Poe, his date of birth to be 20 August 1939 and his place of birth to be Manila.
Victorino X. Fornier, petitioner in G.R. No. 161824, entitled "Victorino X. Fornier, Petitioner,
versus Hon. Commission on Elections and Ronald Allan Kelley Poe, also known as Fernando
Poe, Jr., Respondents," initiated, on 09 January 2004, a petition docketed SPA No. 04-003
before the Commission on Elections ("COMELEC") to disqualify FPJ and to deny due course
or to cancel his certificate of candidacy upon the thesis that FPJ made a material
misrepresentation in his certificate of candidacy by claiming to be a natural-born Filipino

citizen when in truth, according to Fornier, his parents were foreigners; his mother, Bessie
Kelley Poe, was an American, and his father, Allan Poe, was a Spanish national, being the
son of Lorenzo Pou, a Spanish subject. Granting, petitioner asseverated, that Allan F. Poe
was a Filipino citizen, he could not have transmitted his Filipino citizenship to FPJ, the latter
being an illegitimate child of an alien mother. Petitioner based the allegation of the illegitimate
birth of respondent on two assertions - first, Allan F. Poe contracted a prior marriage to a
certain Paulita Gomez before his marriage to Bessie Kelley and, second, even if no such prior
marriage had existed, Allan F. Poe, married Bessie Kelly only a year after the birth of
respondent.
In the hearing before the Third Division of the COMELEC on 19 January 2004, petitioner, in
support of his claim, presented several documentary exhibits - 1) a copy of the certificate of
birth of FPJ, 2) a certified photocopy of an affidavit executed in Spanish by Paulita Poe y
Gomez attesting to her having filed a case for bigamy and concubinage against the father of
respondent, Allan F. Poe, after discovering his bigamous relationship with Bessie Kelley, 3) an
English translation of the affidavit aforesaid, 4) a certified photocopy of the certificate of birth
of Allan F. Poe, 5) a certification issued by the Director of the Records Management and
Archives Office, attesting to the fact that there was no record in the National Archives that a
Lorenzo Poe or Lorenzo Pou resided or entered the Philippines before 1907, and 6) a
certification from the Officer-In-Charge of the Archives Division of the National Archives to the
effect that no available information could be found in the files of the National Archives
regarding the birth of Allan F. Poe.
On his part, respondent, presented twenty-two documentary pieces of evidence, the more
significant ones being - a) a certification issued by Estrella M. Domingo of the Archives
Division of the National Archives that there appeared to be no available information regarding
the birth of Allan F. Poe in the registry of births for San Carlos, Pangasinan, b) a certification
issued by the Officer-In-Charge of the Archives Division of the National Archives that no
available information about the marriage of Allan F. Poe and Paulita Gomez could be found, c)
a certificate of birth of Ronald Allan Poe, d) Original Certificate of Title No. P-2247 of the
Registry of Deeds for the Province of Pangasinan, in the name of Lorenzo Pou, e) copies of
Tax Declaration No. 20844, No. 20643, No. 23477 and No. 23478 in the name of Lorenzo
Pou, f) a copy of the certificate of death of Lorenzo Pou, g) a copy of the purported marriage
contract between Fernando Pou and Bessie Kelley, and h) a certification issued by the City
Civil Registrar of San Carlos City, Pangasinan, stating that the records of birth in the said
office during the period of from 1900 until May 1946 were totally destroyed during World War
II.
On 23 January 2004, the COMELEC dismissed SPA No. 04-003 for lack of merit. Three days
later, or on 26 January 2004, Fornier filed his motion for reconsideration. The motion was
denied on 06 February 2004 by the COMELEC en banc. On 10 February 2004, petitioner
assailed the decision of the COMELEC before this Court conformably with Rule 64, in relation
to Rule 65, of the Revised Rules of Civil Procedure. The petition, docketed G. R. No. 161824,
likewise prayed for a temporary restraining order, a writ of preliminary injunction or any other
resolution that would stay the finality and/or execution of the COMELEC resolutions.
The other petitions, later consolidated with G. R. No. 161824, would include G. R. No.
161434, entitled "Maria Jeanette C. Tecson, and Felix B. Desiderio, Jr., vs. The Commission
on Elections, Ronald Allan Kelley Poe (a.k.a. Fernando Poe, Jr.), and Victorino X. Fornier,"
and the other, docketed G. R. No. 161634, entitled "Zoilo Antonio G. Velez, vs. Ronald Allan
19

Kelley Poe, a.k.a. Fernando Poe, Jr.," both challenging the jurisdiction of the COMELEC and
asserting that, under Article VII, Section 4, paragraph 7, of the 1987 Constitution, only the
Supreme Court had original and exclusive jurisdiction to resolve the basic issue on the case.
Jurisdiction of the Court
In G. R. No. 161824
In seeking the disqualification of the candidacy of FPJ and to have the COMELEC deny due
course to or cancel FPJs certificate of candidacy for alleged misrepresentation of a material
fact (i.e., that FPJ was a natural-born citizen) before the COMELEC, petitioner Fornier invoked
Section 78 of the Omnibus Election Code
"Section 78. Petition to deny due course to or cancel a certificate of candidacy. --- A
verified petition seeking to deny due course or to cancel a certificate of candidacy
may be filed by any person exclusively on the ground that any material
representation contained therein as required under Section 74 hereof is false"
in consonance with the general powers of COMELEC expressed in Section 52 of the Omnibus
Election Code "Section 52. Powers and functions of the Commission on Elections. In addition to
the powers and functions conferred upon it by the Constitution, the Commission
shall have exclusive charge of the enforcement and administration of all laws
relative to the conduct of elections for the purpose of ensuring free, orderly and
honest elections" and in relation to Article 69 of the Omnibus Election Code which would authorize
"any interested party" to file a verified petition to deny or cancel the certificate of
candidacy of any nuisance candidate.
Decisions of the COMELEC on disqualification cases may be reviewed by the Supreme Court
per Rule 642 in an action for certiorari under Rule 653 of the Revised Rules of Civil Procedure.
Section 7, Article IX, of the 1987 Constitution also reads
"Each Commission shall decide by a majority vote of all its Members any case or
matter brought before it within sixty days from the date of its submission for decision
or resolution. A case or matter is deemed submitted for decision or resolution upon
the filing of the last pleading, brief, or memorandum, required by the rules of the
Commission or by the Commission itself. Unless otherwise provided by this
Constitution or by law, any decision, order, or ruling of each Commission may be
brought to the Supreme Court on certiorari by the aggrieved party within thirty days
from receipt of a copy thereof."
Additionally, Section 1, Article VIII, of the same Constitution provides that judicial power is
vested in one Supreme Court and in such lower courts as may be established by law which
power "includes the duty of the courts of justice to settle actual controversies involving rights
which are legally demandable and enforceable, and to determine whether or not there has
been a grave abuse of discretion amounting to lack or excess of jurisdiction on the part of any
branch or instrumentality of the Government."
It is sufficiently clear that the petition brought up in G. R. No. 161824 was aptly elevated to,
and could well be taken cognizance of by, this Court. A contrary view could be a gross denial
to our people of their fundamental right to be fully informed, and to make a proper choice, on
who could or should be elected to occupy the highest government post in the land.

In G. R. No. 161434 and G. R. No. 161634


Petitioners Tecson, et al., in G. R. No. 161434, and Velez, in G. R. No. 161634, invoke the
provisions of Article VII, Section 4, paragraph 7, of the 1987 Constitution in assailing the
jurisdiction of the COMELEC when it took cognizance of SPA No. 04-003 and in urging the
Supreme Court to instead take on the petitions they directly instituted before it. The
Constitutional provision cited reads:
"The Supreme Court, sitting en banc, shall be the sole judge of all contests relating
to the election, returns, and qualifications of the President or Vice-President, and
may promulgate its rules for the purpose."
The provision is an innovation of the 1987 Constitution. The omission in the 1935 and the
1973 Constitution to designate any tribunal to be the sole judge of presidential and vicepresidential contests, has constrained this Court to declare, in Lopez vs. Roxas,4 as "not
(being) justiciable" controversies or disputes involving contests on the elections, returns and
qualifications of the President or Vice-President. The constitutional lapse prompted Congress,
on 21 June 1957, to enact Republic Act No. 1793, "An Act Constituting an Independent
Presidential Electoral Tribunal to Try, Hear and Decide Protests Contesting the Election of the
President-Elect and the Vice-President-Elect of the Philippines and Providing for the Manner
of Hearing the Same." Republic Act 1793 designated the Chief Justice and the Associate
Justices of the Supreme Court to be the members of the tribunal. Although the subsequent
adoption of the parliamentary form of government under the 1973 Constitution might have
implicitly affected Republic Act No. 1793, the statutory set-up, nonetheless, would now be
deemed revived under the present Section 4, paragraph 7, of the 1987 Constitution.
Ordinary usage would characterize a "contest" in reference to a post-election scenario.
Election contests consist of either an election protest or a quo warranto which, although two
distinct remedies, would have one objective in view, i.e., to dislodge the winning candidate
from office. A perusal of the phraseology in Rule 12, Rule 13, and Rule 14 of the "Rules of the
Presidential Electoral Tribunal," promulgated by the Supreme Court en banc on 18 April 1992,
would support this premise "Rule 12. Jurisdiction. - The Tribunal shall be the sole judge of all contests relating
to the election, returns, and qualifications of the President or Vice-President of the
Philippines.
"Rule 13. How Initiated. - An election contest is initiated by the filing of an election
protest or a petition for quo warranto against the President or Vice-President. An
election protest shall not include a petition for quo warranto. A petition for quo
warranto shall not include an election protest.
"Rule 14. Election Protest. - Only the registered candidate for President or for VicePresident of the Philippines who received the second or third highest number of
votes may contest the election of the President or the Vice-President, as the case
may be, by filing a verified petition with the Clerk of the Presidential Electoral
Tribunal within thirty (30) days after the proclamation of the winner."
The rules categorically speak of the jurisdiction of the tribunal over contests relating to the
election, returns and qualifications of the "President" or "Vice-President", of the Philippines,
and not of "candidates" for President or Vice-President. A quo warranto proceeding is
generally defined as being an action against a person who usurps, intrudes into, or unlawfully
20

holds or exercises a public office.5 In such context, the election contest can only contemplate
a post-election scenario. In Rule 14, only a registered candidate who would have received
either the second or third highest number of votes could file an election protest. This rule
again presupposes a post-election scenario.
It is fair to conclude that the jurisdiction of the Supreme Court, defined by Section 4,
paragraph 7, of the 1987 Constitution, would not include cases directly brought before it,
questioning the qualifications of a candidate for the presidency or vice-presidency before the
elections are held.
Accordingly, G. R. No. 161434, entitled "Maria Jeanette C. Tecson, et al., vs. Commission on
Elections et al.," and G. R. No. 161634, entitled "Zoilo Antonio Velez vs. Ronald Allan Kelley
Poe a.k.a. Fernando Poe, Jr." would have to be dismissed for want of jurisdiction.
The Citizenship Issue
Now, to the basic issue; it should be helpful to first give a brief historical background on the
concept of citizenship.
Perhaps, the earliest understanding of citizenship was that given by Aristotle, who, sometime
in 384 to 322 B.C., described the "citizen" to refer to a man who shared in the administration
of justice and in the holding of an office.6 Aristotle saw its significance if only to determine the
constituency of the "State," which he described as being composed of such persons who
would be adequate in number to achieve a self-sufficient existence.7 The concept grew to
include one who would both govern and be governed, for which qualifications like autonomy,
judgment and loyalty could be expected. Citizenship was seen to deal with rights and
entitlements, on the one hand, and with concomitant obligations, on the other.8 In its ideal
setting, a citizen was active in public life and fundamentally willing to submit his private
interests to the general interest of society.
The concept of citizenship had undergone changes over the centuries. In the 18th century, the
concept was limited, by and large, to civil citizenship, which established the rights necessary
for individual freedom, such as rights to property, personal liberty and justice.9 Its meaning
expanded during the 19th century to include political citizenship, which encompassed the right
to participate in the exercise of political power.10 The 20th century saw the next stage of the
development of social citizenship, which laid emphasis on the right of the citizen to economic
well-being and social security.11 The idea of citizenship has gained expression in the modern
welfare state as it so developed in Western Europe. An ongoing and final stage of
development, in keeping with the rapidly shrinking global village, might well be the
internationalization of citizenship.12
The Local Setting - from Spanish Times to the Present
There was no such term as "Philippine citizens" during the Spanish regime but "subjects of
Spain" or "Spanish subjects."13 In church records, the natives were called 'indios', denoting a
low regard for the inhabitants of the archipelago. Spanish laws on citizenship became highly
codified during the 19th century but their sheer number made it difficult to point to one
comprehensive law. Not all of these citizenship laws of Spain however, were made to apply to
the Philippine Islands except for those explicitly extended by Royal Decrees.14
Spanish laws on citizenship were traced back to the Novisima Recopilacion, promulgated in
Spain on 16 July 1805 but as to whether the law was extended to the Philippines remained to
be the subject of differing views among experts;15 however, three royal decrees were

undisputably made applicable to Spaniards in the Philippines - the Order de la Regencia of 14


August 1841,16 the Royal Decree of 23 August 1868 specifically defining the political status of
children born in the Philippine Islands,17 and finally, the Ley Extranjera de Ultramar of 04 July
1870, which was expressly made applicable to the Philippines by the Royal Decree of 13 July
1870.18
The Spanish Constitution of 1876 was never extended to the Philippine Islands because of the
express mandate of its Article 89, according to which the provisions of the Ultramar among
which this country was included, would be governed by special laws.19
It was only the Civil Code of Spain, made effective in this jurisdiction on 18 December 1889,
which came out with the first categorical enumeration of who were Spanish citizens. "(a) Persons born in Spanish territory,
"(b) Children of a Spanish father or mother, even if they were born outside of Spain,
"(c) Foreigners who have obtained naturalization papers,
"(d) Those who, without such papers, may have become domiciled inhabitants of
any town of the Monarchy."20
The year 1898 was another turning point in Philippine history. Already in the state of decline
as a superpower, Spain was forced to so cede her sole colony in the East to an upcoming
world power, the United States. An accepted principle of international law dictated that a
change in sovereignty, while resulting in an abrogation of all political laws then in force, would
have no effect on civil laws, which would remain virtually intact.
The Treaty of Paris was entered into on 10 December 1898 between Spain and the United
States.21 Under Article IX of the treaty, the civil rights and political status of the native
inhabitants of the territories ceded to the United States would be determined by its Congress "Spanish subjects, natives of the Peninsula, residing in the territory over which
Spain by the present treaty relinquishes or cedes her sovereignty may remain in
such territory or may remove therefrom, retaining in either event all their rights of
property, including the right to sell or dispose of such property or of its proceeds;
and they shall also have the right to carry on their industry, commerce, and
professions, being subject in respect thereof to such laws as are applicable to
foreigners. In case they remain in the territory they may preserve their allegiance to
the Crown of Spain by making, before a court of record, within a year from the date
of the exchange of ratifications of this treaty, a declaration of their decision to
preserve such allegiance; in default of which declaration they shall be held to have
renounced it and to have adopted the nationality of the territory in which they reside.
Thus
"The civil rights and political status of the native inhabitants of the territories hereby
ceded to the United States shall be determined by the Congress."22
Upon the ratification of the treaty, and pending legislation by the United States Congress on
the subject, the native inhabitants of the Philippines ceased to be Spanish subjects. Although
they did not become American citizens, they, however, also ceased to be "aliens" under
American laws and were thus issued passports describing them to be citizens of the
Philippines entitled to the protection of the United States.

21

The term "citizens of the Philippine Islands" appeared for the first time in the Philippine Bill of
1902, also commonly referred to as the Philippine Organic Act of 1902, the first
comprehensive legislation of the Congress of the United States on the Philippines ".... that all inhabitants of the Philippine Islands continuing to reside therein, who
were Spanish subjects on the 11th day of April, 1891, and then resided in said
Islands, and their children born subsequent thereto, shall be deemed and held to be
citizens of the Philippine Islands and as such entitled to the protection of the United
States, except such as shall have elected to preserve their allegiance to the Crown
of Spain in accordance with the provisions of the treaty of peace between the United
States and Spain, signed at Paris, December tenth eighteen hundred and ninety
eight."23
Under the organic act, a "citizen of the Philippines" was one who was an inhabitant of the
Philippines, and a Spanish subject on the 11th day of April 1899. The term "inhabitant" was
taken to include 1) a native-born inhabitant, 2) an inhabitant who was a native of Peninsular
Spain, and 3) an inhabitant who obtained Spanish papers on or before 11 April 1899.24
Controversy arose on to the status of children born in the Philippines from 11 April 1899 to 01
July 1902, during which period no citizenship law was extant in the Philippines. Weight was
given to the view, articulated in jurisprudential writing at the time, that the common law
principle of jus soli, otherwise also known as the principle of territoriality, operative in the
United States and England, governed those born in the Philippine Archipelago within that
period.25 More about this later.
In 23 March 1912, the Congress of the United States made the following amendment to the
Philippine Bill of 1902 "Provided, That the Philippine Legislature is hereby authorized to provide by law for
the acquisition of Philippine citizenship by those natives of the Philippine Islands
who do not come within the foregoing provisions, the natives of other insular
possession of the United States, and such other persons residing in the Philippine
Islands who would become citizens of the United States, under the laws of the
United States, if residing therein."26
With the adoption of the Philippine Bill of 1902, the concept of "Philippine citizens" had for the
first time crystallized. The word "Filipino" was used by William H. Taft, the first Civil Governor
General in the Philippines when he initially made mention of it in his slogan, "The Philippines
for the Filipinos." In 1916, the Philippine Autonomy Act, also known as the Jones Law restated
virtually the provisions of the Philippine Bill of 1902, as so amended by the Act of Congress in
1912 "That all inhabitants of the Philippine Islands who were Spanish subjects on the
eleventh day of April, eighteen hundred and ninety-nine, and then resided in said
Islands, and their children born subsequently thereto, shall be deemed and held to
be citizens of the Philippine Islands, except such as shall have elected to preserve
their allegiance to the Crown of Spain in accordance with the provisions of the treaty
of peace between the United States and Spain, signed at Paris December tenth,
eighteen hundred and ninety-eight and except such others as have since become
citizens of some other country; Provided, That the Philippine Legislature, herein
provided for, is hereby authorized to provide for the acquisition of Philippine
citizenship by those natives of the Philippine Islands who do not come within the

foregoing provisions, the natives of the insular possessions of the United States,
and such other persons residing in the Philippine Islands who are citizens of the
United States, or who could become citizens of the United States under the laws of
the United States, if residing therein."
Under the Jones Law, a native-born inhabitant of the Philippines was deemed to be a citizen
of the Philippines as of 11 April 1899 if he was 1) a subject of Spain on 11 April 1899, 2)
residing in the Philippines on said date, and, 3) since that date, not a citizen of some other
country.
While there was, at one brief time, divergent views on whether or not jus soli was a mode of
acquiring citizenship, the 1935 Constitution brought to an end to any such link with common
law, by adopting, once and for all, jus sanguinis or blood relationship as being the basis of
Filipino citizenship "Section 1, Article III, 1935 Constitution. The following are citizens of the Philippines
"(1) Those who are citizens of the Philippine Islands at the time of the adoption of
this Constitution
"(2) Those born in the Philippines Islands of foreign parents who, before the
adoption of this Constitution, had been elected to public office in the Philippine
Islands.
"(3) Those whose fathers are citizens of the Philippines.
"(4) Those whose mothers are citizens of the Philippines and upon reaching the age
of majority, elect Philippine citizenship.
"(5) Those who are naturalized in accordance with law."
Subsection (4), Article III, of the 1935 Constitution, taken together with existing civil law
provisions at the time, which provided that women would automatically lose their Filipino
citizenship and acquire that of their foreign husbands, resulted in discriminatory situations that
effectively incapacitated the women from transmitting their Filipino citizenship to their
legitimate children and required illegitimate children of Filipino mothers to still elect Filipino
citizenship upon reaching the age of majority. Seeking to correct this anomaly, as well as fully
cognizant of the newly found status of Filipino women as equals to men, the framers of the
1973 Constitution crafted the provisions of the new Constitution on citizenship to reflect such
concerns "Section 1, Article III, 1973 Constitution - The following are citizens of the
Philippines:
"(1) Those who are citizens of the Philippines at the time of the adoption of this
Constitution.
"(2) Those whose fathers or mothers are citizens of the Philippines.
"(3) Those who elect Philippine citizenship pursuant to the provisions of the
Constitution of nineteen hundred and thirty-five.
"(4) Those who are naturalized in accordance with law."
For good measure, Section 2 of the same article also further provided that

22

"A female citizen of the Philippines who marries an alien retains her Philippine
citizenship, unless by her act or omission she is deemed, under the law to have
renounced her citizenship."
The 1987 Constitution generally adopted the provisions of the 1973 Constitution, except for
subsection (3) thereof that aimed to correct the irregular situation generated by the
questionable proviso in the 1935 Constitution.
Section I, Article IV, 1987 Constitution now provides:
"The following are citizens of the Philippines:
"(1) Those who are citizens of the Philippines at the time of the adoption
of this Constitution.
"(2) Those whose fathers or mothers are citizens of the Philippines.
"(3) Those born before January 17, 1973 of Filipino mothers, who elect
Philippine citizenship upon reaching the age of majority; and
"(4) Those who are naturalized in accordance with law."
The Case Of FPJ
Section 2, Article VII, of the 1987 Constitution expresses:
"No person may be elected President unless he is a natural-born citizen of the
Philippines, a registered voter, able to read and write, at least forty years of age on
the day of the election, and a resident of the Philippines for at least ten years
immediately preceding such election."
The term "natural-born citizens," is defined to include "those who are citizens of the
Philippines from birth without having to perform any act to acquire or perfect their Philippine
citizenship."27
The date, month and year of birth of FPJ appeared to be 20 August 1939 during the regime of
the 1935 Constitution. Through its history, four modes of acquiring citizenship - naturalization,
jus soli, res judicata and jus sanguinis28 had been in vogue. Only two, i.e., jus soli and jus
sanguinis, could qualify a person to being a "natural-born" citizen of the Philippines. Jus soli,
per Roa vs. Collector of Customs29 (1912), did not last long. With the adoption of the 1935
Constitution and the reversal of Roa in Tan Chong vs. Secretary of Labor30 (1947), jus
sanguinis or blood relationship would now become the primary basis of citizenship by birth.
Documentary evidence adduced by petitioner would tend to indicate that the earliest
established direct ascendant of FPJ was his paternal grandfather Lorenzo Pou, married to
Marta Reyes, the father of Allan F. Poe. While the record of birth of Lorenzo Pou had not been
presented in evidence, his death certificate, however, identified him to be a Filipino, a resident
of San Carlos, Pangasinan, and 84 years old at the time of his death on 11 September 1954.
The certificate of birth of the father of FPJ, Allan F. Poe, showed that he was born on 17 May
1915 to an Espaol father, Lorenzo Pou, and a mestiza Espaol mother, Marta Reyes.
Introduced by petitioner was an "uncertified" copy of a supposed certificate of the alleged
marriage of Allan F. Poe and Paulita Gomez on 05 July 1936. The marriage certificate of Allan
F. Poe and Bessie Kelley reflected the date of their marriage to be on 16 September 1940. In
the same certificate, Allan F. Poe was stated to be twenty-five years old, unmarried, and a
Filipino citizen, and Bessie Kelley to be twenty-two years old, unmarried, and an American
citizen. The birth certificate of FPJ, would disclose that he was born on 20 August 1939 to

Allan F. Poe, a Filipino, twenty-four years old, married to Bessie Kelly, an American citizen,
twenty-one years old and married.
Considering the reservations made by the parties on the veracity of some of the entries on the
birth certificate of respondent and the marriage certificate of his parents, the only conclusions
that could be drawn with some degree of certainty from the documents would be that 1. The parents of FPJ were Allan F. Poe and Bessie Kelley;
2. FPJ was born to them on 20 August 1939;
3. Allan F. Poe and Bessie Kelley were married to each other on 16 September,
1940;
4. The father of Allan F. Poe was Lorenzo Poe; and
5. At the time of his death on 11 September 1954, Lorenzo Poe was 84 years old.
Would the above facts be sufficient or insufficient to establish the fact that FPJ is a naturalborn Filipino citizen? The marriage certificate of Allan F. Poe and Bessie Kelley, the birth
certificate of FPJ, and the death certificate of Lorenzo Pou are documents of public record in
the custody of a public officer. The documents have been submitted in evidence by both
contending parties during the proceedings before the COMELEC.
The birth certificate of FPJ was marked Exhibit "A" for petitioner and Exhibit "3" for
respondent. The marriage certificate of Allan F. Poe to Bessie Kelley was submitted as Exhibit
"21" for respondent. The death certificate of Lorenzo Pou was submitted by respondent as his
Exhibit "5." While the last two documents were submitted in evidence for respondent, the
admissibility thereof, particularly in reference to the facts which they purported to show, i.e.,
the marriage certificate in relation to the date of marriage of Allan F. Poe to Bessie Kelley and
the death certificate relative to the death of Lorenzo Pou on 11 September 1954 in San
Carlos, Pangasinan, were all admitted by petitioner, who had utilized those material
statements in his argument. All three documents were certified true copies of the originals.
Section 3, Rule 130, Rules of Court states that "Original document must be produced; exceptions. - When the subject of inquiry is
the contents of a document, no evidence shall be admissible other than the original
document itself, except in the following cases:
"x x x
xxx
xxx
"(d) When the original is a public record in the custody of a public office or is
recorded in a public office."
Being public documents, the death certificate of Lorenzo Pou, the marriage certificate of Allan
F. Poe and Bessie Kelly, and the birth certificate of FPJ, constitute prima facie proof of their
contents. Section 44, Rule 130, of the Rules of Court provides:
"Entries in official records. Entries in official records made in the performance of his
duty by a public officer of the Philippines, or by a person in the performance of a
duty specially enjoined by law, are prima facie evidence of the facts therein stated."
The trustworthiness of public documents and the value given to the entries made therein could
be grounded on 1) the sense of official duty in the preparation of the statement made, 2) the
penalty which is usually affixed to a breach of that duty, 3) the routine and disinterested origin
of most such statements, and 4) the publicity of record which makes more likely the prior
exposure of such errors as might have occurred.31
23

The death certificate of Lorenzo Pou would indicate that he died on 11 September 1954, at
the age of 84 years, in San Carlos, Pangasinan. It could thus be assumed that Lorenzo Pou
was born sometime in the year 1870 when the Philippines was still a colony of Spain.
Petitioner would argue that Lorenzo Pou was not in the Philippines during the crucial period of
from 1898 to 1902 considering that there was no existing record about such fact in the
Records Management and Archives Office. Petitioner, however, likewise failed to show that
Lorenzo Pou was at any other place during the same period. In his death certificate, the
residence of Lorenzo Pou was stated to be San Carlos, Pangasinan. In the absence of any
evidence to the contrary, it should be sound to conclude, or at least to presume, that the place
of residence of a person at the time of his death was also his residence before death. It would
be extremely doubtful if the Records Management and Archives Office would have had
complete records of all residents of the Philippines from 1898 to 1902.
Proof of Paternity and Filiation
Under Civil Law.
Petitioner submits, in any case, that in establishing filiation (relationship or civil status of the
child to the father [or mother]) or paternity (relationship or civil status of the father to the child)
of an illegitimate child, FPJ evidently being an illegitimate son according to petitioner, the
mandatory rules under civil law must be used.
Under the Civil Code of Spain, which was in force in the Philippines from 08 December 1889
up until the day prior to 30 August 1950 when the Civil Code of the Philippines took effect,
acknowledgment was required to establish filiation or paternity. Acknowledgment was either
judicial (compulsory) or voluntary. Judicial or compulsory acknowledgment was possible only
if done during the lifetime of the putative parent; voluntary acknowledgment could only be had
in a record of birth, a will, or a public document.32 Complementary to the new code was Act
No. 3753 or the Civil Registry Law expressing in Section 5 thereof, that "In case of an illegitimate child, the birth certificate shall be signed and sworn to
jointly by the parents of the infant or only by the mother if the father refuses. In the
latter case, it shall not be permissible to state or reveal in the document the name of
the father who refuses to acknowledge the child, or to give therein any information
by which such father could be identified."
In order that the birth certificate could then be utilized to prove voluntary acknowledgment of
filiation or paternity, the certificate was required to be signed or sworn to by the father. The
failure of such requirement rendered the same useless as being an authoritative document of
recognition.33 In Mendoza vs. Mella,34 the Court ruled "Since Rodolfo was born in 1935, after the registry law was enacted, the question
here really is whether or not his birth certificate (Exhibit 1), which is merely a
certified copy of the registry record, may be relied upon as sufficient proof of his
having been voluntarily recognized. No such reliance, in our judgment, may be
placed upon it. While it contains the names of both parents, there is no showing that
they signed the original, let alone swore to its contents as required in Section 5 of
Act No. 3753. For all that might have happened, it was not even they or either of
them who furnished the data to be entered in the civil register. Petitioners say that in
any event the birth certificate is in the nature of a public document wherein voluntary
recognition of a natural child may also be made, according to the same Article 131.

True enough, but in such a case, there must be a clear statement in the document
that the parent recognizes the child as his or her own."
In the birth certificate of respondent FPJ, presented by both parties, nowhere in the document
was the signature of Allan F. Poe found. There being no will apparently executed, or at least
shown to have been executed, by decedent Allan F. Poe, the only other proof of voluntary
recognition remained to be "some other public document." In Pareja vs. Pareja,35 this Court
defined what could constitute such a document as proof of voluntary acknowledgment:
"Under the Spanish Civil Code there are two classes of public documents, those
executed by private individuals which must be authenticated by notaries, and those
issued by competent public officials by reason of their office. The public document
pointed out in Article 131 as one of the means by which recognition may be made
belongs to the first class."
Let us leave it at that for the moment.
The 1950 Civil Code categorized the acknowledgment or recognition of illegitimate children
into voluntary, legal or compulsory. Voluntary recognition was required to be expressedly
made in a record of birth, a will, a statement before a court of record or in any authentic
writing. Legal acknowledgment took place in favor of full blood brothers and sisters of an
illegitimate child who was recognized or judicially declared as natural. Compulsory
acknowledgment could be demanded generally in cases when the child had in his favor any
evidence to prove filiation. Unlike an action to claim legitimacy which would last during the
lifetime of the child, and might pass exceptionally to the heirs of the child, an action to claim
acknowledgment, however, could only be brought during the lifetime of the presumed parent.
Amicus Curiae Ruben F. Balane defined, during the oral argument, "authentic writing," so as
to be an authentic writing for purposes of voluntary recognition, simply as being a genuine or
indubitable writing of the father. The term would include a public instrument (one duly
acknowledged before a notary public or other competent official) or a private writing admitted
by the father to be his.
The Family Code has further liberalized the rules; Article 172, Article 173, and Article 175
provide:
"Art. 172. The filiation of legitimate children is established by any of the following:
"(1) The record of birth appearing in the civil register or a final judgment; or
"(2) An admission of legitimate filiation in a public document or a private handwritten
instrument and signed by the parent concerned.
"In the absence of the foregoing evidence, the legitimate filiation shall be proved by:
"(1) The open and continuous possession of the status of a legitimate child; or
"(2) Any other means allowed by the Rules of Court and special laws.
"Art. 173. The action to claim legitimacy may be brought by the child during his or
her lifetime and shall be transmitted to the heirs should the child die during minority
or in a state of insanity. In these cases, the heirs shall have a period of five years
within which to institute the action.
"The action already commenced by the child shall survive notwithstanding the death
of either or both of the parties.
"x x x
xxx
x x x.
24

"Art. 175. Illegitimate children may establish their illegitimate filiation in the same
way and on the same, evidence as legitimate children.
"The action must be brought within the same period specified in Article 173, except
when the action is based on the second paragraph of Article 172, in which case the
action may be brought during the lifetime of the alleged parent."
The provisions of the Family Code are retroactively applied; Article 256 of the code reads:
"Art. 256. This Code shall have retroactive effect insofar as it does not prejudice or
impair vested or acquired rights in accordance with the Civil Code or other laws."
Thus, in Vda. de Sy-Quia vs. Court of Appeals,36 the Court has ruled:
"We hold that whether Jose was a voluntarily recognized natural child should be
decided under Article 278 of the Civil Code of the Philippines. Article 2260 of that
Code provides that 'the voluntary recognition of a natural child shall take place
according to this Code, even if the child was born before the effectivity of this body
of laws' or before August 30, 1950. Hence, Article 278 may be given retroactive
effect."
It should be apparent that the growing trend to liberalize the acknowledgment or recognition of
illegitimate children is an attempt to break away from the traditional idea of keeping well apart
legitimate and non-legitimate relationships within the family in favor of the greater interest and
welfare of the child. The provisions are intended to merely govern the private and personal
affairs of the family. There is little, if any, to indicate that the legitimate or illegitimate civil
status of the individual would also affect his political rights or, in general, his relationship to the
State. While, indeed, provisions on "citizenship" could be found in the Civil Code, such
provisions must be taken in the context of private relations, the domain of civil law; particularly
"Civil Law is that branch of law which has for its double purpose the organization of
the family and the regulation of property. It has thus [been] defined as the mass of
precepts which determine and regulate the relations of assistance, authority and
obedience among members of a family, and those which exist among members of a
society for the protection of private interests."37
In Yaez de Barnuevo vs. Fuster,38 the Court has held:
"In accordance with Article 9 of the Civil Code of Spain, x x x the laws relating to
family rights and duties, or to the status, condition and legal capacity of persons,
govern Spaniards although they reside in a foreign country; that, in consequence,
'all questions of a civil nature, such as those dealing with the validity or nullity of the
matrimonial bond, the domicile of the husband and wife, their support, as between
them, the separation of their properties, the rules governing property, marital
authority, division of conjugal property, the classification of their property, legal
causes for divorce, the extent of the latter, the authority to decree it, and, in general,
the civil effects of marriage and divorce upon the persons and properties of the
spouses, are questions that are governed exclusively by the national law of the
husband and wife."
The relevance of "citizenship" or "nationality" to Civil Law is best exemplified in Article 15 of
the Civil Code, stating that -

"Laws relating to family rights and duties, or to the status, condition and legal
capacity of persons are binding upon citizens of the Philippines, even though living
abroad" that explains the need to incorporate in the code a reiteration of the Constitutional provisions
on citizenship. Similarly, citizenship is significant in civil relationships found in different parts of
the Civil Code,39 such as on successional rights and family relations.40 In adoption, for
instance, an adopted child would be considered the child of his adoptive parents and
accorded the same rights as their legitimate child but such legal fiction extended only to define
his rights under civil law41 and not his political status.
Civil law provisions point to an obvious bias against illegitimacy. This discriminatory attitude
may be traced to the Spanish family and property laws, which, while defining proprietary and
successional rights of members of the family, provided distinctions in the rights of legitimate
and illegitimate children. In the monarchial set-up of old Spain, the distribution and inheritance
of titles and wealth were strictly according to bloodlines and the concern to keep these
bloodlines uncontaminated by foreign blood was paramount.
These distinctions between legitimacy and illegitimacy were codified in the Spanish Civil
Code, and the invidious discrimination survived when the Spanish Civil Code became the
primary source of our own Civil Code. Such distinction, however, remains and should remain
only in the sphere of civil law and not unduly impede or impinge on the domain of political law.
The proof of filiation or paternity for purposes of determining his citizenship status should thus
be deemed independent from and not inextricably tied up with that prescribed for civil law
purposes. The Civil Code or Family Code provisions on proof of filiation or paternity, although
good law, do not have preclusive effects on matters alien to personal and family relations. The
ordinary rules on evidence could well and should govern. For instance, the matter about
pedigree is not necessarily precluded from being applicable by the Civil Code or Family Code
provisions.
Section 39, Rule 130, of the Rules of Court provides "Act or Declaration about pedigree. The act or declaration of a person deceased, or
unable to testify, in respect to the pedigree of another person related to him by birth
or marriage, may be received in evidence where it occurred before the controversy,
and the relationship between the two persons is shown by evidence other than such
act or declaration. The word `pedigree includes relationship, family genealogy, birth,
marriage, death, the dates when and the places where these facts occurred, and the
names of the relatives. It embraces also facts of family history intimately connected
with pedigree."
For the above rule to apply, it would be necessary that (a) the declarant is already dead or
unable to testify, (b) the pedigree of a person must be at issue, (c) the declarant must be a
relative of the person whose pedigree is in question, (d) declaration must be made before the
controversy has occurred, and (e) the relationship between the declarant and the person
whose pedigree is in question must be shown by evidence other than such act or declaration.
Thus, the duly notarized declaration made by Ruby Kelley Mangahas, sister of Bessie Kelley
Poe submitted as Exhibit 20 before the COMELEC, might be accepted to prove the acts of
Allan F. Poe, recognizing his own paternal relationship with FPJ, i.e, living together with
Bessie Kelley and his children (including respondent FPJ) in one house, and as one family 25

"I, Ruby Kelley Mangahas, of legal age and sound mind, presently residing in
Stockton, California, U.S.A., after being sworn in accordance with law do hereby
declare that:
"1. I am the sister of the late Bessie Kelley Poe.
"2. Bessie Kelley Poe was the wife of Fernando Poe, Sr.
"3. Fernando and Bessie Poe had a son by the name of Ronald Allan Poe, more
popularly known in the Philippines as `Fernando Poe, Jr., or `FPJ.
"4. Ronald Allan Poe `FPJ was born on August 20, 1939 at St. Luke's Hospital,
Magdalena Street, Manila.
"x x x
xxx
xxx
"7. Fernando Poe Sr., and my sister Bessie, met and became engaged while they
were students at the University of the Philippines in 1936. I was also introduced to
Fernando Poe, Sr., by my sister that same year.
"8. Fernando Poe, Sr., and my sister Bessie had their first child in 1938.
"9. Fernando Poe, Sr., my sister Bessie and their first three children, Elizabeth,
Ronald, Allan and Fernando II, and myself lived together with our mother at our
family's house on Dakota St. (now Jorge Bocobo St.), Malate until the liberation of
Manila in 1945, except for some months between 1943-1944.
"10. Fernando Poe, Sr., and my sister, Bessie, were blessed with four (4) more
children after Ronald Allan Poe.
"x x x
xxx
xxx
"18. I am executing this Declaration to attest to the fact that my nephew, Ronald
Allan Poe is a natural born Filipino, and that he is the legitimate child of Fernando
Poe, Sr.
"Done in City of Stockton, California, U.S.A., this 12th day of January 2004.
Ruby Kelley Mangahas Declarant DNA Testing
In case proof of filiation or paternity would be unlikely to satisfactorily establish or would be
difficult to obtain, DNA testing, which examines genetic codes obtained from body cells of the
illegitimate child and any physical residue of the long dead parent could be resorted to. A
positive match would clear up filiation or paternity. In Tijing vs. Court of Appeals,42 this Court
has acknowledged the strong weight of DNA testing "Parentage will still be resolved using conventional methods unless we adopt the modern and
scientific ways available. Fortunately, we have now the facility and expertise in using DNA test
for identification and parentage testing. The University of the Philippines Natural Science
Research Institute (UP-NSRI) DNA Analysis Laboratory has now the capability to conduct
DNA typing using short tandem repeat (STR) analysis. The analysis is based on the fact that
the DNA of a child/person has two (2) copies, one copy from the mother and the other from
the father. The DNA from the mother, the alleged father and the child are analyzed to
establish parentage. Of course, being a novel scientific technique, the use of DNA test as
evidence is still open to challenge. Eventually, as the appropriate case comes, courts should
not hesitate to rule on the admissibility of DNA evidence. For it was said, that courts should
apply the results of science when competently obtained in aid of situations presented, since to
reject said result is to deny progress."

Petitioners Argument For Jurisprudential Conclusiveness


Petitioner would have it that even if Allan F. Poe were a Filipino citizen, he could not have
transmitted his citizenship to respondent FPJ, the latter being an illegitimate child. According
to petitioner, prior to his marriage to Bessie Kelley, Allan F. Poe, on July 5, 1936, contracted
marriage with a certain Paulita Gomez, making his subsequent marriage to Bessie Kelley
bigamous and respondent FPJ an illegitimate child. The veracity of the supposed certificate of
marriage between Allan F. Poe and Paulita Gomez could be most doubtful at best. But the
documentary evidence introduced by no less than respondent himself, consisting of a birth
certificate of respondent and a marriage certificate of his parents showed that FPJ was born
on 20 August 1939 to a Filipino father and an American mother who were married to each
other a year later, or on 16 September 1940. Birth to unmarried parents would make FPJ an
illegitimate child. Petitioner contended that as an illegitimate child, FPJ so followed the
citizenship of his mother, Bessie Kelley, an American citizen, basing his stand on the ruling of
this Court in Morano vs. Vivo,43 citing Chiongbian vs. de Leo44 and Serra vs. Republic.45
On the above score, the disquisition made by amicus curiae Joaquin G. Bernas, SJ, is most
convincing; he states "We must analyze these cases and ask what the lis mota was in each of them. If the
pronouncement of the Court on jus sanguinis was on the lis mota, the
pronouncement would be a decision constituting doctrine under the rule of stare
decisis. But if the pronouncement was irrelevant to the lis mota, the pronouncement
would not be a decision but a mere obiter dictum which did not establish doctrine. I
therefore invite the Court to look closely into these cases.
"First, Morano vs. Vivo. The case was not about an illegitimate child of a Filipino
father. It was about a stepson of a Filipino, a stepson who was the child of a
Chinese mother and a Chinese father. The issue was whether the stepson followed
the naturalization of the stepfather. Nothing about jus sanguinis there. The stepson
did not have the blood of the naturalized stepfather.
"Second, Chiongbian vs. de Leon. This case was not about the illegitimate son of a
Filipino father. It was about a legitimate son of a father who had become Filipino by
election to public office before the 1935 Constitution pursuant to Article IV, Section
1(2) of the 1935 Constitution. No one was illegitimate here.
"Third, Serra vs. Republic. The case was not about the illegitimate son of a Filipino
father. Serra was an illegitimate child of a Chinese father and a Filipino mother. The
issue was whether one who was already a Filipino because of his mother who still
needed to be naturalized. There is nothing there about invidious jus sanguinis.
"Finally, Paa vs. Chan.46 This is a more complicated case. The case was about the
citizenship of Quintin Chan who was the son of Leoncio Chan. Quintin Chan claimed
that his father, Leoncio, was the illegitimate son of a Chinese father and a Filipino
mother. Quintin therefore argued that he got his citizenship from Leoncio, his father.
But the Supreme Court said that there was no valid proof that Leoncio was in fact
the son of a Filipina mother. The Court therefore concluded that Leoncio was not
Filipino. If Leoncio was not Filipino, neither was his son Quintin. Quintin therefore
was not only not a natural-born Filipino but was not even a Filipino.

26

"The Court should have stopped there. But instead it followed with an obiter dictum.
The Court said obiter that even if Leoncio, Quintin's father, were Filipino, Quintin
would not be Filipino because Quintin was illegitimate. This statement about Quintin,
based on a contrary to fact assumption, was absolutely unnecessary for the case. x
x x It was obiter dictum, pure and simple, simply repeating the obiter dictum in
Morano vs. Vivo.
"x x x
xxx
xxx
"Aside from the fact that such a pronouncement would have no textual foundation in
the Constitution, it would also violate the equal protection clause of the Constitution
not once but twice. First, it would make an illegitimate distinction between a
legitimate child and an illegitimate child, and second, it would make an illegitimate
distinction between the illegitimate child of a Filipino father and the illegitimate child
of a Filipino mother.
"The doctrine on constitutionally allowable distinctions was established long ago by
People vs. Cayat.47 I would grant that the distinction between legitimate children and
illegitimate children rests on real differences. x x x But real differences alone do not
justify invidious distinction. Real differences may justify distinction for one purpose
but not for another purpose.
"x x x What is the relevance of legitimacy or illegitimacy to elective public service?
What possible state interest can there be for disqualifying an illegitimate child from
becoming a public officer. It was not the fault of the child that his parents had illicit
liaison. Why deprive the child of the fullness of political rights for no fault of his own?
To disqualify an illegitimate child from holding an important public office is to punish
him for the indiscretion of his parents. There is neither justice nor rationality in that.
And if there is neither justice nor rationality in the distinction, then the distinction
transgresses the equal protection clause and must be reprobated."
The other amici curiae, Mr. Justice Vicente Mendoza (a former member of this Court),
Professor Ruben Balane and Dean Martin Magallona, at bottom, have expressed similar
views. The thesis of petitioner, unfortunately hinging solely on pure obiter dicta, should indeed
fail.
Where jurisprudence regarded an illegitimate child as taking after the citizenship of its mother,
it did so for the benefit the child. It was to ensure a Filipino nationality for the illegitimate child
of an alien father in line with the assumption that the mother had custody, would exercise
parental authority and had the duty to support her illegitimate child. It was to help the child, not
to prejudice or discriminate against him.
The fact of the matter perhaps the most significant consideration is that the 1935
Constitution, the fundamental law prevailing on the day, month and year of birth of respondent
FPJ, can never be more explicit than it is. Providing neither conditions nor distinctions, the
Constitution states that among the citizens of the Philippines are "those whose fathers are
citizens of the Philippines." There utterly is no cogent justification to prescribe conditions or
distinctions where there clearly are none provided.
In Sum
(1) The Court, in the exercise of its power of judicial review, possesses jurisdiction
over the petition in G. R. No. 161824, filed under Rule 64, in relation to Rule 65, of

the Revised Rules of Civil Procedure. G.R. No. 161824 assails the resolution of the
COMELEC for alleged grave abuse of discretion in dismissing, for lack of merit, the
petition in SPA No. 04-003 which has prayed for the disqualification of respondent
FPJ from running for the position of President in the 10th May 2004 national
elections on the contention that FPJ has committed material representation in his
certificate of candidacy by representing himself to be a natural-born citizen of the
Philippines.
(2) The Court must dismiss, for lack of jurisdiction and prematurity, the petitions in
G. R. No. 161434 and No. 161634 both having been directly elevated to this Court
in the latters capacity as the only tribunal to resolve a presidential and vicepresidential election contest under the Constitution. Evidently, the primary
jurisdiction of the Court can directly be invoked only after, not before, the elections
are held.
(3) In ascertaining, in G.R. No. 161824, whether grave abuse of discretion has been
committed by the COMELEC, it is necessary to take on the matter of whether or not
respondent FPJ is a natural-born citizen, which, in turn, depended on whether or not
the father of respondent, Allan F. Poe, would have himself been a Filipino citizen
and, in the affirmative, whether or not the alleged illegitimacy of respondent
prevents him from taking after the Filipino citizenship of his putative father. Any
conclusion on the Filipino citizenship of Lorenzo Pou could only be drawn from the
presumption that having died in 1954 at 84 years old, Lorenzo would have been
born sometime in the year 1870, when the Philippines was under Spanish rule, and
that San Carlos, Pangasinan, his place of residence upon his death in 1954, in the
absence of any other evidence, could have well been his place of residence before
death, such that Lorenzo Pou would have benefited from the "en masse
Filipinization" that the Philippine Bill had effected in 1902. That citizenship (of
Lorenzo Pou), if acquired, would thereby extend to his son, Allan F. Poe, father of
respondent FPJ. The 1935 Constitution, during which regime respondent FPJ has
seen first light, confers citizenship to all persons whose fathers are Filipino citizens
regardless of whether such children are legitimate or illegitimate.
(4) But while the totality of the evidence may not establish conclusively that
respondent FPJ is a natural-born citizen of the Philippines, the evidence on hand
still would preponderate in his favor enough to hold that he cannot be held guilty of
having made a material misrepresentation in his certificate of candidacy in violation
of Section 78, in relation to Section 74, of the Omnibus Election Code. Petitioner
has utterly failed to substantiate his case before the Court, notwithstanding the
ample opportunity given to the parties to present their position and evidence, and to
prove whether or not there has been material misrepresentation, which, as so ruled
in Romualdez-Marcos vs. COMELEC,48 must not only be material, but also
deliberate and willful.
WHEREFORE, the Court RESOLVES to DISMISS
1. G. R. No. 161434, entitled "Maria Jeanette C. Tecson and Felix B. Desiderio, Jr.,
Petitioners, versus Commission on Elections, Ronald Allan Kelley Poe (a.k.a.
"Fernando Poe, Jr.,) and Victorino X. Fornier, Respondents," and G. R. No. 161634,

27

entitled "Zoilo Antonio Velez, Petitioner, versus Ronald Allan Kelley Poe, a.k.a.
Fernando Poe, Jr., Respondent," for want of jurisdiction.
2. G. R. No. 161824, entitled "Victorino X. Fornier, Petitioner, versus Hon.
Commission on Elections and Ronald Allan Kelley Poe, also known as Fernando
Poe, Jr.," for failure to show grave abuse of discretion on the part of respondent
Commission on Elections in dismissing the petition in SPA No. 04-003.
No Costs.
SO ORDERED.

28

G.R. No. 167569


September 4, 2009
CARLOS T. GO, SR., Petitioner,
vs.
LUIS T. RAMOS, Respondent.
x - - - - - - - - - - - - - - - - - - - - - - -x
G.R. No. 167570
JIMMY T. GO, Petitioner,
vs.
LUIS T. RAMOS, Respondent.
x - - - - - - - - - - - - - - - - - - - - - - -x
G.R. No. 171946
HON. ALIPIO F. FERNANDEZ, JR., in his capacity as the Commissioner of the BUREAU
OF IMMIGRATION; ATTY. FAISAL HUSSIN and ANSARI M. MACAAYAN, in their
capacity as Intelligence Officers of the BUREAU OF IMMIGRATION, Petitioners,
vs.
JIMMY T. GO a.k.a. JAIME T. GAISANO, Respondent.
DECISION
QUISUMBING, J.:
Before us are three petitions. G.R. Nos. 167569 and 167570 are petitions for review on
certiorari to set aside the October 25, 2004 Decision1 and February 16, 2005 Resolution2 of
the Court of Appeals in CA-G.R. SP No. 85143 that affirmed the Decision3 dated January 6,
2004 and Order4 dated May 3, 2004 of the Regional Trial Court (RTC) of Pasig City, Branch
167 in SCA No. 2218 upholding the preparation and filing of deportation charges against
Jimmy T. Go, the corresponding Charge Sheet5 dated July 3, 2001, and the deportation
proceedings thereunder conducted.
On the other hand, G.R. No. 171946, also a petition for review on certiorari, seeks to set aside
the December 8, 2005 Decision6 and March 13, 2006 Resolution7 of the appellate court in CAG.R. SP No. 88277.
Considering that the three cases arose from the same factual milieu, the Court resolved to
consolidate G.R. Nos. 167570 and 167569 with G.R. No. 171946 per Resolution8 dated
February 26, 2007.
These petitions stemmed from the complaint-affidavit9 for deportation initiated by Luis T.
Ramos before the Bureau of Immigration and Deportation (now Bureau of Immigration)
against Jimmy T. Go alleging that the latter is an illegal and undesirable alien. Luis alleged
that while Jimmy represents himself as a Filipino citizen, Jimmys personal circumstances and
other records indicate that he is not so. To prove his contention, Luis presented the birth
certificate of Jimmy, issued by the Office of the Civil Registrar of Iloilo City, which indicated
Jimmys citizenship as "FChinese." Luis argued that although it appears from Jimmys birth
certificate that his parents, Carlos and Rosario Tan, are Filipinos, the document seems to be
tampered, because only the citizenship of Carlos appears to be handwritten while all the other
entries were typewritten. He also averred that in September 1989 or thereabout, Jimmy,
through stealth, machination and scheming managed to cover up his true citizenship, and with

the use of falsified documents and untruthful declarations, was able to procure a Philippine
passport from the Department of Foreign Affairs.
Jimmy refuted the allegations in his counter-affidavit,10 averring that the complaint for
deportation initiated by Luis was merely a harassment case designed to oust him of his rightful
share in their business dealings. Jimmy maintained that there is no truth to the allegation that
he is an alien, and insisted that he is a natural-born Filipino. Jimmy alleged that his father
Carlos, who was the son of a Chinese father and Filipina mother, elected Philippine
citizenship in accordance with Article IV, Section 1, paragraph 411 of the 1935 Constitution and
Commonwealth Act No. 62512 (Com. Act No. 625), as evidenced by his having taken the Oath
of Allegiance on July 11, 1950 and having executed an Affidavit of Election of Philippine
citizenship on July 12, 1950. Although the said oath and affidavit were registered only on
September 11, 1956, the reason behind such late registration was sufficiently explained in an
affidavit. Jimmy added that he had even voted in the 1952 and 1955 elections.13 He denied
that his father arrived in the Philippines as an undocumented alien, alleging that his father has
no record of arrival in this country as alleged in the complaint-affidavit precisely because his
father was born and raised in the Philippines, and in fact, speaks fluent Ilonggo and Tagalog. 14
With regard to the erroneous entry in his birth certificate that he is "FChinese," he maintained
that such was not of his own doing, but may be attributed to the employees of the Local Civil
Registrars Office who might have relied on his Chinese-sounding surname when making the
said entry. He asserted that the said office has control over his birth certificate; thus, if his
fathers citizenship appears to be handwritten, it may have been changed when the
employees of that office realized that his father has already taken his oath as a Filipino.15 As
regards the entry in his siblings certificates of birth, particularly Juliet Go and Carlos Go, Jr.,
that their father is Chinese, Jimmy averred that the entry was erroneous because it was made
without prior consultation with his father.16
In a Resolution17 dated February 14, 2001, Associate Commissioner Linda L. MalenabHornilla dismissed the complaint for deportation against Jimmy. Associate Commissioner
Hornilla affirmed the findings of the National Bureau of Investigation tasked to investigate the
case that Jimmys father elected Filipino citizenship in accordance with the provisions of the
1935 Philippine Constitution. By operation of law, therefore, the citizenship of Carlos was
transmitted to Jimmy, making him a Filipino as well.
On March 8, 2001,18 the Board of Commissioners (Board) reversed said dismissal, holding
that Carlos election of Philippine citizenship was made out of time. Finding Jimmys claim to
Philippine citizenship in serious doubt by reason of his fathers questionable election thereof,
the Board directed the preparation and filing of the appropriate deportation charges against
Jimmy.
On July 3, 2001, the corresponding Charge Sheet was filed against Jimmy, charging him of
violating Section 37(a)(9)19 in relation to Section 45(c)20 of Com. Act No. 613, otherwise known
as The Philippine Immigration Act of 1940,21 as amended, committed as follows:
xxxx
1. That Respondent was born on October 25, 1952 in Iloilo City, as evidenced by a
copy of his birth certificate wherein his citizenship was recorded as "Chinese";
2. That Respondent through some stealth machinations was able to subsequently
cover up his true and actual citizenship as Chinese and illegally acquired a
29

Philippine Passport under the name JAIME T. GAISANO, with the use of falsified
documents and untruthful declarations, in violation of the above-cited provisions of
the Immigration Act[;]
3. That [R]espondent being an alien, has formally and officially represent[ed] and
introduce[d] himself as a citizen of the Philippines, for fraudulent purposes and in
order to evade any requirements of the immigration laws, also in violation of said
law.
CONTRARY TO LAW.22
On November 9, 2001, Carlos and Jimmy filed a petition for certiorari and prohibition23 with
application for injunctive reliefs before the RTC of Pasig City, Branch 167, docketed as SCA
No. 2218, seeking to annul and set aside the March 8, 2001 Resolution of the Board of
Commissioners, the Charge Sheet, and the proceedings had therein. In essence, they
challenged the jurisdiction of the Board to continue with the deportation proceedings.
In the interim, the Board issued a Decision24 dated April 17, 2002, in BSI-D.C. No. ADD-01117, ordering the apprehension and deportation of Jimmy. The dispositive portion of the
decision reads:
WHEREFORE, in view of the foregoing, the Board of Commissioners hereby Orders the
apprehension of respondent JIMMY T. GO @ JAIME T. GAISANO and that he be then
deported to CHINA of which he is a citizen, without prejudice, however, to the continuation of
any and all criminal and other proceedings that are pending in court or before the prosecution
arm of the Philippine Government, if any. And that upon expulsion, he is thereby ordered
barred from entry into the Philippines.
SO ORDERED.25
In view of the said Decision, Carlos and Jimmy filed on June 13, 2002 a supplemental petition
for certiorari and prohibition26 before the trial court and reiterated their application for injunctive
reliefs. The trial court issued a writ of preliminary prohibitory injunction pending litigation on
the main issue, enjoining the Bureau from enforcing the April 17, 2002 Decision.27 Later,
however, the trial court dissolved the writ in a Decision28 dated January 6, 2004 as a
consequence of the dismissal of the petition.
Carlos and Jimmy moved for reconsideration. But their motion was likewise denied.29
Following the dismissal of the petition in SCA No. 2218, the Board issued a warrant of
deportation30 which led to the apprehension of Jimmy. Jimmy commenced a petition for
habeas corpus, but the same was eventually dismissed by reason of his provisional release
on bail.31
Carlos and Jimmy then questioned the Decision in SCA No. 2218 as well as the Resolution
denying their motion for reconsideration by way of a petition for certiorari before the Court of
Appeals, docketed as CA-G.R. SP No. 85143. They imputed grave abuse of discretion by the
trial court for passing upon their citizenship, claiming that what they asked for in their petition
was merely the nullification of the March 8, 2001 Resolution and the charge sheet.
The appellate tribunal dismissed the petition.32 It did not find merit in their argument that the
issue of citizenship should proceed only before the proper court in an independent action, and
that neither the Bureau nor the Board has jurisdiction over individuals who were born in the
Philippines and have exercised the rights of Filipino citizens. The appellate tribunal also
rejected their claim that they enjoy the presumption of being Filipino citizens.

The Court of Appeals held that the Board has the exclusive authority and jurisdiction to try and
hear cases against an alleged alien, and in the process, determine their citizenship.
The appellate court agreed with the trial court that the principle of jus soli was never extended
to the Philippines; hence, could not be made a ground to ones claim of Philippine citizenship.
Like the trial court, the appellate tribunal found that Carlos failed to elect Philippine citizenship
within the reasonable period of three years upon reaching the age of majority. Furthermore, it
held that the belated submission to the local civil registry of the affidavit of election and oath of
allegiance in September 1956 was defective because the affidavit of election was executed
after the oath of allegiance, and the delay of several years before their filing with the proper
office was not satisfactorily explained.
The course of action taken by the trial court was also approved by the appellate tribunal. The
Court of Appeals stated that the trial court necessarily had to rule on the substantial and legal
bases warranting the deportation proceeding in order to determine whether the Board acted
without or in excess of jurisdiction, or with grave abuse of discretion. Moreover, the appellate
court found that due process was properly observed in the proceedings before the Board,
contrary to the claim of Jimmy.
Unfazed with the said ruling, they moved for reconsideration. Their motion having been
denied,33 Carlos and Jimmy each filed a petition for review on certiorari before this Court,
respectively docketed as G.R. Nos. 167569 and 167570.
Meanwhile, in view of the dismissal of CA-G.R. SP. No. 85143, Bureau of Immigration
Commissioner Alipio F. Fernandez, Jr. issued Warrant of Deportation No. AFF-04-00334 dated
November 16, 2004 to carry out the April 17, 2002 Decision in BSI-D.C. No. ADD-01-117. This
resulted in the apprehension and detention of Jimmy at the Bureau of Immigration Bicutan
Detention Center, pending his deportation to China.35
On account of his detention, Jimmy once again filed a petition for habeas corpus36 before the
RTC of Pasig City, Branch 167, docketed as SP. Proc. No. 11507 assailing his apprehension
and detention despite the pendency of his appeal and his release on recognizance.
In an Order37 dated December 6, 2004, the trial court dismissed the said petition ruling that the
remedy of habeas corpus cannot be availed of to obtain an order of release once a
deportation order has already been issued by the Bureau. Jimmy moved for reconsideration of
the Order, but this was also denied by the trial court in an Order38 dated December 28, 2004.
Jimmy assailed the Orders of the trial court in a petition for certiorari and prohibition before the
appellate court, docketed as CA-G.R. No. 88277. The Court of Appeals granted the petition
and enjoined the deportation of Jimmy until the issue of his citizenship is settled with finality by
the court. The Court of Appeals held as follows:
xxxx
the issuance of a warrant to arrest and deport the petitioner without any proof whatsoever of
his violation of the bail conditions [that he was previously granted] is arbitrary, inequitable and
unjust, for the policies governing the grant of his bail should likewise apply in the cancellation
of the said bail. Although a deportation proceeding does not partake of the nature of a criminal
action, yet considering that it is such a harsh and extraordinary administrative proceeding
affecting the freedom and liberty of a person who all his life has always lived in the
Philippines, where he has established his family and business interests, one who appears to
be not completely devoid of any claim to Filipino citizenship, being the son of a Filipina, whose
30

father is alleged to also have elected to be a Filipino, the constitutional right of such person to
due process cannot be peremptorily dismissed or ignored altogether, and indeed should not
be denied. If it later turns out that the petitioner is a Filipino after all, then the overly eager
Immigration authorities would have expelled and relegated to statelessness one who might in
fact be a Filipino by blood.
xxxx
WHEREFORE, in view of the foregoing, the petition with reference to the Warrant of
Deportation issued by the BID is hereby GRANTED. The Bureau of Immigration and
Deportation, through Commissioner Alipio F. Fernandez, Jr., Atty. Faizal Hussin and Ansari
Maca Ayan, and any of their deputized agents, are ENJOINED from deporting petitioner
Jimmy T. Go, a.k.a. Jaime T. Gaisano, until the issue of petitioners citizenship is finally settled
by the courts of justice.
SO ORDERED.39
Their motion for reconsideration40 having been denied on March 13, 2006, Hon. Alipio
Fernandez, in his capacity as the Commissioner of the Bureau of Immigration, and Atty. Faisal
Hussin and Ansari M. Macaayan, in their capacity as Intelligence Officers of the Bureau of
Immigration, are before this Court as petitioners in G.R. No. 171946.
The parties have raised the following grounds for their respective petitions:
G.R. No. 167569
I.
THE PROCEEDINGS HAD BEFORE THE BUREAU OF IMMIGRATION AND
DEPORTATION (B.I.D.) ARE NULL AND VOID FOR ITS FAILURE TO IMPLEAD AN
INDISPENSABLE PARTY IN THE PERSON OF PETITIONER CARLOS GO, SR.
II.
GIVEN THE SUBSTANTIAL EVIDENCE TO PROVE HEREIN PETITIONER CARLOS GO
SR.S FILIPINO CITIZENSHIP, A FULL BLOWN TRIAL UNDER THE MORE RIGID RULES
OF EVIDENCE PRESCRIBED IN COURT PROCEEDINGS SHOULD HAVE BEEN
CONDUCTED TO DETERMINE HIS FILIPINO CITIZENSHIP AND NOT THROUGH MERE
"SUMMARY PROCEEDINGS" SUCH AS THE ONE HAD BEFORE THE B.I.D. AS WELL AS
IN THE COURT A QUO.
III.
A FILIPINO CITIZEN IS NOT REQUIRED TO ELECT PHILIPPINE CITIZENSHIP.
IV.
ASSUMING CARLOS GO, SR. STILL NEEDS TO ELECT PHILIPPINE CITIZENSHIP, HE
HAD COMPLIED WITH ALL THE REQUIREMENTS OF COM. ACT NO. 625.
V.
PETITIONER CARLOS GO, SR. ENJOYS THE "PRESUMPTION OF CITIZENSHIP."
VI.
RESPONDENTS "CAUSE OF ACTION" HAD LONG PRESCRIBED.41
G.R. No. 167570
I.

THE PROCEEDINGS HAD BEFORE THE BUREAU OF IMMIGRATION AND


DEPORTATION (B.I.D.) ARE NULL AND VOID FOR ITS FAILURE TO IMPLEAD AN
INDISPENSABLE PARTY IN THE PERSON OF PETITIONERS FATHER, CARLOS GO, SR.
II.
THE DEPORTATION PROCEEDINGS BEFORE THE B.I.D. ARE NULL AND VOID FOR ITS
FAILURE TO OBSERVE DUE PROCESS.
III.
THE B.I.D.S CAUSE OF ACTION AGAINST HEREIN PETITIONER JIMMY T. GO HAD
ALREADY PRESCRIBED.
IV.
GIVEN THE SUBSTANTIAL EVIDENCE TO PROVE HEREIN PETITIONERS FILIPINO
CITIZENSHIP, A FULL BLOWN TRIAL UNDER THE MORE RIGID RULES OF EVIDENCE
PRESCRIBED IN COURT PROCEEDINGS SHOULD HAVE BEEN CONDUCTED TO
DETERMINE HIS FILIPINO CITIZENSHIP AND NOT THROUGH MERE "SUMMARY
PROCEEDINGS" SUCH AS THE ONE HAD BEFORE THE B.I.D.42
G.R. No. 171946
THE COURT OF APPEALS ERRED ON A QUESTION OF LAW IN ENJOINING
RESPONDENTS DEPORTATION.43
Succinctly stated, the issues for our resolution are: (a) whether the cause of action of the
Bureau against Carlos and Jimmy had prescribed; (b) whether the deportation proceedings
are null and void for failure to implead Carlos as an indispensable party therein; (c) whether
the evidence adduced by Carlos and Jimmy to prove their claim to Philippine citizenship is
substantial and sufficient to oust the Board of its jurisdiction from continuing with the
deportation proceedings in order to give way to a formal judicial action to pass upon the issue
of alienage; (d) whether due process was properly observed in the proceedings before the
Board; and (e) whether the petition for habeas corpus should be dismissed.
The arguments raised by Carlos and Jimmy in their respective petitions are merely a rehash
of the arguments they adduced before the appellate tribunal and the trial court. Once again,
they raised the same argument of prescription. As to Carlos, it is his position that being
recognized by the government to have acquired Philippine citizenship, evidenced by the
Certificate of Election issued to him on September 11, 1956, his citizenship could no longer be
questioned at this late date. As for Jimmy, he contends that the Boards cause of action to
deport him has prescribed for the simple reason that his arrest was not made within five (5)
years from the time the cause of action arose, which according to him commenced in 1989
when he was alleged to have illegally acquired a Philippine passport.
In any event, they argue that the deportation proceeding should be nullified altogether for
failure to implead Carlos as an indispensable party therein. Jimmy posits that the deportation
case against him was made to depend upon the citizenship of his father, Carlos, in that the
Board found justification to order his deportation by declaring that his father is a Chinese
citizen even though the latter was never made a party in the deportation proceedings. They
argue that the Board could not simply strip Carlos of his citizenship just so they could question
the citizenship of Jimmy. To do so without affording Carlos the opportunity to adduce evidence
to prove his claim to Philippine citizenship would be the height of injustice. For failing to
accord him the requisite due process, the whole proceeding should perforce be stuck down.
31

While they concede that the Board has jurisdiction to hear cases against an alleged alien, they
insist that judicial intervention may be resorted to when the claim to citizenship is so
substantial that there are reasonable grounds to believe that the claim is correct, like in this
case. Their claim to Philippine citizenship, they said, is clearly shown by the fact that they
were born, had been raised and had lived in this country all their lives; they speak fluent
Tagalog and Ilonggo; they engage in businesses reserved solely for Filipinos; they exercise
their right to suffrage; they enjoy the rights and privileges accorded only to citizens; and they
have no record of any Alien Certificate of Registration. More importantly, they contend that
they were validly issued Philippine passports. They further posit that the judicial intervention
required is not merely a judicial review of the proceedings below, but a full-blown, adversarial,
trial-type proceedings where the rules of evidence are strictly observed.
Considering that his citizenship affects that of his son, Carlos opted to present controverting
arguments to sustain his claim to Philippine citizenship, notwithstanding the fact that
according to him, he was never impleaded in the deportation proceedings.
Carlos takes exception to the ruling of the appellate court that the doctrine of jus soli failed to
accord him Philippine citizenship for the reason that the same was never extended to the
Philippines. He insists that if his Philippine citizenship is not recognized by said doctrine, it is
nonetheless recognized by the laws enforced prior to the 1935 Constitution, particularly the
Philippine Bill of 190244 and the Philippine Autonomy Act of August 29, 1916 (Jones Law of
1916).45
According to Carlos, the Philippine Bill of 1902 and the Jones Law of 1916 deemed all
inhabitants of the Philippine Islands as well as their children born after the passage of said
laws to be citizens of the Philippines. Because his father, Go Yin An, was a resident of the
Philippines at the time of the passage of the Jones Law of 1916, he (Carlos) undoubtedly
acquired his fathers citizenship. Article IV, first paragraph, of the 1935 Constitution therefore
applies to him. Said constitutional provision reads:
ARTICLE IV. Citizenship
SECTION 1. The following are citizens of the Philippines:
(1) Those who are citizens of the Philippine Islands at the time of the adoption of this
Constitution.
xxxx
Even assuming that his father remained as a Chinese, Carlos also claims that he followed the
citizenship of his Filipina mother, being an illegitimate son, and that he even validly elected
Philippine citizenship when he complied with all the requirements of Com. Act No. 625. He
submits that what is being disputed is not whether he complied with Com. Act No. 625, but
rather, the timeliness of his compliance. He stresses that the 3-year compliance period
following the interpretation given by Cuenco v. Secretary of Justice46 to Article IV, Section 1(4)
of the 1935 Constitution and Com. Act No. 625 when election must be made, is not an
inflexible rule. He reasoned that the same decision held that such period may be extended
under certain circumstances, as when the person concerned has always considered himself a
Filipino, like in his case.47
We deny the appeal of Carlos and Jimmy for lack of merit.
Carlos and Jimmys claim that the cause of action of the Bureau has prescribed is untenable.
Cases involving issues on citizenship are sui generis. Once the citizenship of an individual is

put into question, it necessarily has to be threshed out and decided upon. In the case of
Frivaldo v. Commission on Elections,48 we said that decisions declaring the acquisition or
denial of citizenship cannot govern a persons future status with finality. This is because a
person may subsequently reacquire, or for that matter, lose his citizenship under any of the
modes recognized by law for the purpose.49 Indeed, if the issue of ones citizenship, after it
has been passed upon by the courts, leaves it still open to future adjudication, then there is
more reason why the government should not be precluded from questioning ones claim to
Philippine citizenship, especially so when the same has never been threshed out by any
tribunal.
Jimmys invocation of prescription also does not persuade us. Section 37 (b) of Com. Act No.
613 states:
Section 37.
xxxx
(b) Deportation may be effected under clauses 2, 7, 8, 11 and 12 of this section at any time
after entry, but shall not be effected under any other clause unless the arrest in the
deportation proceedings is made within five years after the cause of deportation arises.
xxxx
As shown in the Charge Sheet, Jimmy was charged for violation of Section 37(a)(9),50 in
relation to Section 45(e)51 of Com. Act No. 613. From the foregoing provision, his deportation
may be effected only if his arrest is made within 5 years from the time the cause for
deportation arose. The court a quo is correct when it ruled that the 5-year period should be
counted only from July 18, 2000, the time when Luis filed his complaint for deportation. It is
the legal possibility of bringing the action which determines the starting point for the
computation of the period of prescription.52 Additionally, Section 2 of Act No. 3326,53 as
amended, entitled "An Act to Establish Periods of Prescription for Violations Penalized by
Special Acts and Municipal Ordinances and to Provide When Prescription Shall Begin to
Run," provides:
Sec. 2. Prescription shall begin to run from the day of the commission of the violation of the
law, and if the same be not known at the time, from the discovery thereof and the institution of
judicial proceedings for its investigation and punishment.
xxxx
The counting could not logically start in 1989 when his passport was issued because the
government was unaware that he was not a Filipino citizen. Had the government been aware
at such time that he was not a Filipino citizen or there were certain anomalies attending his
application for such passport, it would have denied his application.
As to the issue of whether Carlos is an indispensable party, we reiterate that an indispensable
party is a party in interest without whom no final determination can be had of an action, and
who shall be joined either as plaintiff or defendant.54 To be indispensable, a person must first
be a real party in interest, that is, one who stands to be benefited or injured by the judgment of
the suit, or the party entitled to the avails of the suit.55 Carlos clearly is not an indispensable
party as he does not stand to be benefited or injured by the judgment of the suit. What is
sought is the deportation of Jimmy on the ground that he is an alien. Hence, the principal
issue that will be decided on is the propriety of his deportation. To recall, Jimmy claims that he
is a Filipino under Section 1(3),56 Article IV of the 1935 Constitution because Carlos, his
32

father, is allegedly a citizen.57 Since his citizenship hinges on that of his fathers, it becomes
necessary to pass upon the citizenship of the latter. However, whatever will be the findings as
to Carlos citizenship will in no way prejudice him.
Citizenship proceedings, as aforestated, are a class of its own, in that, unlike other cases, res
judicata does not obtain as a matter of course. In a long line of decisions, this Court said that
every time the citizenship of a person is material or indispensable in a judicial or
administrative case, whatever the corresponding court or administrative authority decides
therein as to such citizenship is generally not considered as res judicata; hence, it has to be
threshed out again and again as the occasion may demand.58 Res judicata may be applied in
cases of citizenship only if the following concur:
1. a persons citizenship must be raised as a material issue in a controversy where
said person is a party;
2. the Solicitor General or his authorized representative took active part in the
resolution thereof; and
3. the finding or citizenship is affirmed by this Court.59
In the event that the citizenship of Carlos will be questioned, or his deportation sought, the
same has to be ascertained once again as the decision which will be rendered hereinafter
shall have no preclusive effect upon his citizenship. As neither injury nor benefit will redound
upon Carlos, he cannot be said to be an indispensable party in this case.
There can be no question that the Board has the authority to hear and determine the
deportation case against a deportee and in the process determine also the question of
citizenship raised by him.60 However, this Court, following American jurisprudence, laid down
the exception to the primary jurisdiction enjoyed by the deportation board in the case of Chua
Hiong v. Deportation Board61 wherein we stressed that judicial determination is permitted in
cases when the courts themselves believe that there is substantial evidence supporting the
claim of citizenship, so substantial that there are reasonable grounds for the belief that the
claim is correct.62 Moreover, when the evidence submitted by a deportee is conclusive of his
citizenship, the right to immediate review should also be recognized and the courts shall
promptly enjoin the deportation proceedings.63
While we are mindful that resort to the courts may be had, the same should be allowed only in
the sound discretion of a competent court in proper proceedings.64 After all, the Boards
jurisdiction is not divested by the mere claim of citizenship.65 Moreover, a deportee who claims
to be a citizen and not therefore subject to deportation has the right to have his citizenship
reviewed by the courts, after the deportation proceedings.66 The decision of the Board on the
question is, of course, not final but subject to review by the courts.671avvphi1
After a careful evaluation of the evidence, the appellate court was not convinced that the
same was sufficient to oust the Board of its jurisdiction to continue with the deportation
proceedings considering that what were presented particularly the birth certificates of Jimmy,
as well as those of his siblings, Juliet Go and Carlos Go, Jr. indicate that they are Chinese
citizens. Furthermore, like the Board, it found the election of Carlos of Philippine citizenship,
which was offered as additional proof of his claim, irregular as it was not made on time.
We find no cogent reason to overturn the above findings of the appellate tribunal. The
question of whether substantial evidence had been presented to allow immediate recourse to
the regular courts is a question of fact which is beyond this Courts power of review for it is not

a trier of facts.68 None of the exceptions69 in which this Court may resolve factual issues has
been shown to exist in this case. Even if we evaluate their arguments and the evidence they
presented once again, the same conclusion will still be reached.
One of the arguments raised to sustain Carlos claim to Philippine citizenship is the doctrine of
jus soli, or the doctrine or principle of citizenship by place of birth. To recall, both the trial court
and the Court of Appeals ruled that the doctrine of jus soli was never extended to the
Philippines. We agree. The doctrine of jus soli was for a time the prevailing rule in the
acquisition of ones citizenship.70 However, the Supreme Court abandoned the principle of jus
soli in the case of Tan Chong v. Secretary of Labor.71 Since then, said doctrine only benefited
those who were individually declared to be citizens of the Philippines by a final court decision
on the mistaken application of jus soli.72
Neither will the Philippine Bill of 190273 nor the Jones Law of 191674 make Carlos a citizen of
the Philippines. His bare claim that his father, Go Yin An, was a resident of the Philippines at
the time of the passage of the said laws, without any supporting evidence whatsoever will not
suffice.
It is a settled rule that only legitimate children follow the citizenship of the father and that
illegitimate children are under the parental authority of the mother and follow her nationality.75
Moreover, we have also ruled that an illegitimate child of a Filipina need not perform any act to
confer upon him all the rights and privileges attached to citizens of the Philippines; he
automatically becomes a citizen himself.76 However, it is our considered view that absent any
evidence proving that Carlos is indeed an illegitimate son of a Filipina, the aforestated
established rule could not be applied to him.
As to the question of whether the election of Philippine citizenship conferred on Carlos Filipino
citizenship, we find that the appellate court correctly found that it did not.
Com. Act No. 625 which was enacted pursuant to Section 1(4), Article IV of the 1935
Constitution, prescribes the procedure that should be followed in order to make a valid
election of Philippine citizenship. Under Section 1 thereof, legitimate children born of Filipino
mothers may elect Philippine citizenship by expressing such intention "in a statement to be
signed and sworn to by the party concerned before any officer authorized to administer oaths,
and shall be filed with the nearest civil registry. The said party shall accompany the aforesaid
statement with the oath of allegiance to the Constitution and the Government of the
Philippines."77
However, the 1935 Constitution and Com. Act No. 625 did not prescribe a time period within
which the election of Philippine citizenship should be made. The 1935 Charter only provides
that the election should be made "upon reaching the age of majority." The age of majority then
commenced upon reaching 21 years. In the opinions of the then Secretary of Justice on cases
involving the validity of election of Philippine citizenship, this dilemma was resolved by basing
the time period on the decisions of this Court prior to the effectivity of the 1935 Constitution. In
these decisions, the proper period for electing Philippine citizenship was, in turn, based on the
pronouncements of the Department of State of the United States Government to the effect
that the election should be made within a "reasonable time" after attaining the age of majority.
The phrase "reasonable time" has been interpreted to mean that the election should be made
within three (3) years from reaching the age of majority.78
It is true that we said that the 3-year period for electing Philippine citizenship may be extended
as when the person has always regarded himself as a Filipino. Be that as it may, it is our
33

considered view that not a single circumstance was sufficiently shown meriting the extension
of the 3-year period. The fact that Carlos exercised his right of suffrage in 1952 and 1955
does not demonstrate such belief, considering that the acts were done after he elected
Philippine citizenship. On the other hand, the mere fact that he was able to vote does not
validate his irregular election of Philippine citizenship. At most, his registration as a voter
indicates his desire to exercise a right appertaining exclusively to Filipino citizens but does not
alter his real citizenship, which, in this jurisdiction, is determined by blood (jus sanguinis). The
exercise of the rights and privileges granted only to Filipinos is not conclusive proof of
citizenship, because a person may misrepresent himself to be a Filipino and thus enjoy the
rights and privileges of citizens of this country.79
It is incumbent upon one who claims Philippine citizenship to prove to the satisfaction of the
court that he is really a Filipino. No presumption can be indulged in favor of the claimant of
Philippine citizenship, and any doubt regarding citizenship must be resolved in favor of the
state.80
As Carlos and Jimmy neither showed conclusive proof of their citizenship nor presented
substantial proof of the same, we have no choice but to sustain the Boards jurisdiction over
the deportation proceedings. This is not to say that we are ruling that they are not Filipinos, for
that is not what we are called upon to do. This Court necessarily has to pass upon the issue of
citizenship only to determine whether the proceedings may be enjoined in order to give way to
a judicial determination of the same. And we are of the opinion that said proceedings should
not be enjoined.
In our considered view, the allegation of Jimmy that due process was not observed in the
deportation proceedings must likewise fail.
Deportation proceedings are administrative in character, summary in nature, and need not be
conducted strictly in accordance with the rules of ordinary court proceedings.81 The essence of
due process is simply an opportunity to be heard, or as applied to administrative proceedings,
an opportunity to explain ones side or an opportunity to seek reconsideration of the action or
ruling complained of.82 As long as the parties are given the opportunity to be heard before
judgment is rendered, the demands of due process are sufficiently met.83 Although Jimmy was
not furnished with a copy of the subject Resolution and Charge Sheet as alleged by him, the
trial court found that he was given ample opportunity to explain his side and present
controverting evidence, thus:
x x x It must be stressed that after receiving the Order dated September 11, 2001 signed by
BSI Chief Ronaldo P. Ledesma on October 4, 2001, petitioner Jimmy T. Go admitted that
when his representative went to the B.I.D. to inquire about the said Order, the latter chanced
upon the Resolution dated February 14, 2001 and March 8, 2001 as well as the Charge Sheet
dated July 3, 2001. Hence on October 5, 2001, he filed a "Motion for Extension of Time to File
Memorandum" and as such, was allowed by Ronaldo P. Ledesma an extension of ten (10)
days to submit his required memorandum. x x x84
This circumstance satisfies the demands of administrative due process.
As regards the petition in G.R. No. 171946, petitioners contend that the appellate tribunal
erred in enjoining Jimmys deportation.85
Petitioners question the remedy availed of by Jimmy. They argue that the existence of the
remedy of an ordinary appeal proscribes the filing of the petition for certiorari as was done in

this case. They point out that the appeal period in habeas corpus cases is only 48 hours,
compared to a special civil action under Rule 65 of the Rules of Court which is 60 days. This
clearly shows that an ordinary appeal is the more plain, speedy and adequate remedy; hence,
it must be the one availed of.86 Since the decision of the trial court was not properly appealed,
the same may be said to have attained finality, and may no longer be disturbed.87
They maintain that the dismissal of the petition for habeas corpus by the trial court was
proper. A petition for habeas corpus has for its purpose only the determination of whether or
not there is a lawful ground for Jimmys apprehension and continued detention. They urge that
the decision of the Board dated April 17, 2002 that ordered Jimmys deportation has already
attained finality by reason of the belated appeal taken by Jimmy from the said decision on
April 2, 2004 before the Office of the President, or after almost two years from the time the
decision was rendered. Said decision of the Board, they insist, is the lawful ground that
sanctions Jimmys apprehension and detention.88
Petitioners in G.R. No. 171946 also argue that Jimmy cannot rely on the bail on recognizance
he was previously granted to question his subsequent apprehension and detention. Under the
Philippine Immigration Act of 1940, the power to grant bail can only be exercised while the
alien is still under investigation, and not when the order of deportation had already been
issued by the Board.89 Hence, the bail granted was irregular as it has no legal basis.
Furthermore, they said the petition for habeas corpus necessarily has to be dismissed
because the same is no longer proper once the applicant thereof has been charged before the
Board, which is the case with Jimmy.90 Nonetheless, they claim that the habeas corpus case
is rendered moot and academic as Jimmy is no longer being detained.91
On the other hand, Jimmy counters that the instant petition for certiorari and prohibition is the
most appropriate, speedy and adequate remedy in spite of the availability of ordinary appeal
considering that what is involved in this case is his cherished liberty. Grave abuse of
discretion on the part of the petitioners in ordering his arrest and detention, he argues, all the
more justifies the avails of the extraordinary writ.92 Contrary to the petitioners stand, Jimmy
argues that the April 17, 2002 Decision of the Board has not attained finality owing to the
availability of various remedies, one of which is an appeal, and in fact is actually void because
it was rendered without due process.93 He also insists that the bail issued to him is valid and
effective until the final determination of his citizenship before the proper courts.94 Moreover, he
maintains that the petition for habeas corpus was proper since its object is to inquire into the
legality of ones detention, and if found illegal, to order the release of the detainee.95 As in his
petition in G.R. No. 167570, Jimmy also contends that the proceedings before the Board is
void for failure to implead therein his father, and that he should have been given a full blown
trial before a regular court where he can prove his citizenship.96
Considering the arguments and contentions of the parties, we find the petition in G.R. No.
171946 meritorious.a1f
We have held in a litany of cases that the extraordinary remedies of certiorari, prohibition and
mandamus are available only when there is no appeal or any plain, speedy and adequate
remedy in the ordinary course of law. The writ of certiorari does not lie where an appeal may
be taken or where another adequate remedy is available for the correction of the error.97
The petitioners correctly argue that appeal should have been the remedy availed of as it is
more plain, speedy and adequate. The 48-hour appeal period demonstrates the adequacy of
34

such remedy in that no unnecessary time will be wasted before the decision will be reevaluated.
A petition for the issuance of a writ of habeas corpus is a special proceeding governed by
Rule 102 of the Revised Rules of Court. The objective of the writ is to determine whether the
confinement or detention is valid or lawful. If it is, the writ cannot be issued. What is to be
inquired into is the legality of a persons detention as of, at the earliest, the filing of the
application for the writ of habeas corpus, for even if the detention is at its inception illegal, it
may, by reason of some supervening events, such as the instances mentioned in Section 4 98
of Rule 102, be no longer illegal at the time of the filing of the application.99
Once a person detained is duly charged in court, he may no longer question his detention
through a petition for issuance of a writ of habeas corpus. His remedy would be to quash the
information and/or the warrant of arrest duly issued. The writ of habeas corpus should not be
allowed after the party sought to be released had been charged before any court. The term
"court" in this context includes quasi-judicial bodies of governmental agencies authorized to
order the persons confinement, like the Deportation Board of the Bureau of Immigration.100
Likewise, the cancellation of his bail cannot be assailed via a petition for habeas corpus.
When an alien is detained by the Bureau of Immigration for deportation pursuant to an order
of deportation by the Deportation Board, the Regional Trial Courts have no power to release
such alien on bail even in habeas corpus proceedings because there is no law authorizing
it.101
Given that Jimmy has been duly charged before the Board, and in fact ordered arrested
pending his deportation, coupled by this Courts pronouncement that the Board was not
ousted of its jurisdiction to continue with the deportation proceedings, the petition for habeas
corpus is rendered moot and academic. This being so, we find it unnecessary to touch on the
other arguments advanced by respondents regarding the same subject.
WHEREFORE, the petitions in G.R. Nos. 167569 and 167570 are DENIED. The Decision
dated October 25, 2004 and Resolution dated February 16, 2005 of the Court of Appeals in
CA-G.R. SP No. 85143 are AFFIRMED. The petition in G.R. No. 171946 is hereby
GRANTED. The Decision dated December 8, 2005 and Resolution dated March 13, 2006 of
the Court of Appeals in CA-G.R. SP No. 88277 are REVERSED and SET ASIDE. The
December 6, 2004 and December 28, 2004 Orders of the Regional Trial Court of Pasig City,
Branch 167 are hereby REINSTATED.
No pronouncement as to costs.
SO ORDERED.

35

G.R. No. 169958


March 5, 2010
DEPARTMENT OF JUSTICE SECRETARY RAUL M. GONZALEZ,BUREAU OF
IMMIGRATION COMMISSIONER and BOARD OF COMMISSIONERS CHAIRMAN ALIPIO
F. FERNANDEZ, JR., IMMIGRATION ASSOCIATE COMMISSIONERS and BOARD OF
COMMISSIONERS MEMBERS ARTHEL B. CARONONGAN, TEODORO B. DELARMENTE,
JOSE D.L. CABOCHAN, and FRANKLIN Z. LITTUA, Petitioners,
vs.
MICHAEL ALFIO PENNISI, Respondent.
DECISION
CARPIO, J.:
The Case
Before the Court is a petition for review1 assailing the 30 September 2005 Decision2 of the
Court of Appeals in CA-G.R. SP No. 87271.
The Antecedent Facts
The facts, gathered from the Court of Appeals decision, are as follows:
Michael Alfio Pennisi (respondent) was born on 13 March 1975 in Queensland, Australia to
Alfio Pennisi, an Australian national, and Anita T. Quintos (Quintos), allegedly a Filipino
citizen. In March 1999, respondent filed a petition for recognition as Filipino citizen before the
Bureau of Immigration (BI). Respondent submitted the following documents before the BI:
1. Certified photocopy of the certificate of birth of Quintos, and a certification issued by the
Local Civil Registrar of San Antonio, Nueva Ecija stating that Quintos was born on 14 August
1949 of Filipino parents, Felipe M. Quintos and Celina G. Tomeda, in Panabingan, San
Antonio, Nueva Ecija;
2. Certified true copy of the certificate of marriage of respondents parents dated 9 January
1971, indicating the Philippines as Quintos birthplace;
3. Certified true copy of Quintos Australian certificate of registration of alien, indicating her
nationality as Filipino;
4. Certified true copy of respondents birth certificate stating that he was born on 13 March
1975 and indicating the Philippines as his mothers birthplace; and
5. Certified true copy of the letter dated 14 July 1999 of the Australian Department of
Immigration and Multicultural Affairs, stating that as of 14 July 1999, Quintos has not been
granted Australian citizenship.
On 17 February 2000, BI Associate Commissioner Alan Roullo Yap issued an order granting
respondents petition for recognition as Filipino citizen. In a 2nd Indorsement dated 28
February 2000, the Secretary of the Department of Justice (DOJ) disapproved the order.
However, upon respondents submission of additional documents, BI Commissioner Rufus B.
Rodriguez granted the order as per Recognition Order No. 206679 dated 3 March 2000 which
states:
Finding the grounds cited in the instant petition for recognition as a citizen of the Philippines
filed on behalf of the applicant to be well-founded and meritorious, we hereby authorize the
recognition of MICHAEL ALFIO PENNISI as a citizen of the Philippines pursuant to Article III[,]
Section 1, para. 2 of the 1973 Constitution.

Henceforth, applicant shall be entitled to all the rights and privileges appurtenant thereto.
Once this Order is affirmed by the Secretary of Justice and upon payment of the
corresponding fees, he/she shall be issued an identification Certificate which shall indicate
prominently thereon the date of affirmation.
An Exit Clearance Certificate (ECC) fee shall also be assessed against the applicant
whenever he/she departs for abroad using a foreign passport or travel documents.
Give the applicant a copy of this Order.
SO ORDERED.3
In a 2nd Indorsement dated 8 March 2000, the DOJ affirmed Recognition Order No. 206679,
as follows:
Respectfully returned to the Commissioner of Immigration, Manila, the within records relating
to the request for reconsideration of this Departments 2nd Indorsement dated February 28,
2000, which disapproved the Order of that Office dated February 17, 2000 granting the
petition for recognition as a Filipino citizen of MICHAEL ALFIO PENNISI.
The additional documents submitted (duly authenticated Certificate of Birth of the petitioner
and Certificate of Marriage of his parents), together with the original records, satisfactorily
establish that petitioner was born in Queensland, Australia, on March 13, 1975, the legitimate
issue of the spouses Anita T. Quintos, a natural-born Filipino citizen, and Alfio Pennisi, an
Australian national, and may, therefore, be deemed a citizen of the Philippines pursuant to
Section 1(2), Article III of the 1973 Constitution, in relation to Section 1(2), Article IV of the
present Constitution.
Wherefore, the instant request for reconsideration is hereby granted and the above-mentioned
Order of that Office dated February 17, 2000 granting the petition for recognition as a Filipino
citizen of Michael Alfio Pennisi is now AFFIRMED.
This supersedes our aforesaid 2nd Indorsement dated February 28, 2000 on the same subject
matter.4
Thereafter, respondent was drafted and played for the Red Bull, a professional basketball
team in the Philippine Basketball Association (PBA).
On 7 August 2003, the Senate Committees on Games, Amusement and Sports and on
Constitutional Amendments (Senate Committees) jointly submitted Committee Report No.
2565 (Committee Report) recommending, among other things, that (1) the BI conduct
summary deportation proceedings against several Filipino-foreign PBA players, including
respondent; and (2) the DOJ Secretary conduct an immediate review of all orders of
recognition. Respondent was included in the list on the basis of the following findings of the
Senate Committees:
F. Michael Alfio Pennisi was able to present before the BI and the committees, the documents
required in granting recognition of Philippine citizenship, particularly the birth certificate of his
Filipino mother, Anita Tomeda Quintos;
However, a verification on the authenticity of the above documents reveals highly suspicious
circumstances.
His alleged mother and other relatives, specifically the parents of the former, namely: Felipe
M. Quintos and Celina G. Tomeda, who were mentioned in his application for recognition of

36

Philippine citizenship in the BI, are not known and have never existed in Panabingan, San
Antonio, Nueva Ecija.1avvphi1
According to the affidavits executed by Barangay Captain Ramon Soliman and Barangay
Treasurer Condrado P. Peralta of the abovementioned place, there are no Quintoses or
Tomedas that have lived or have resided in the said barangay.
Both barangay officials further claimed that even in their census or master list of voters, the
family names of Quintos or Tomedas do not exist.
His mothers certificate of birth in the civil registrar of San Antonio, Nueva Ecija was issued on
the basis of an application for late registration, which is ten (10) years after the date of birth.
Thereafter, the DOJ issued Department Order No. 412 dated 21 September 2004 creating a
special committee, with Chief State Counsel Ricardo V. Paras as Chairperson, to investigate
the citizenship of Filipino-foreign players in the PBA. The special committee required
respondent to submit a position paper in connection with the investigation. On 18 October
2004, the DOJ issued a resolution revoking respondents certificate of recognition and
directing the BI to begin summary deportation proceedings against respondent and other
Filipino-foreign PBA players.
On 20 October 2004, respondent and Davonn Harp (Harp), another Filipino-foreign PBA
player, filed a petition for prohibition with an application for temporary restraining order and
preliminary injunction before the Regional Trial Court of Pasig City, Branch 268 (trial court), to
enjoin the DOJ and BI from instituting summary deportation proceedings against them. On
even date, respondent received a letter from the BI directing him to submit, within five days
from notice, a memorandum in connection with the deportation proceedings being conducted
against him. Respondent submitted his memorandum on 25 October 2004.
In a hearing before the trial court on the same date, the Office of the Solicitor General,
representing the DOJ and BI, manifested that respondent would not be subjected to summary
deportation and that he would be given an opportunity to present evidence of his Filipino
citizenship in a full-blown trial on the merits. However, in a Summary Deportation6 Order dated
26 October 2004, the BI directed the deportation of several Filipino-foreign PBA players,
including respondent. Respondent and Harp withdrew their petition before the trial court
without prejudice, which the trial court granted in its order of 4 November 2004. Respondent
filed a petition for review, with an application for temporary restraining order and preliminary
injunction, before the Court of Appeals.
The Decision of the Court of Appeals
In its 30 September 2005 Decision, the Court of Appeals granted the petition.
The Court of Appeals noted that respondents citizenship was previously recognized by the BI
and DOJ and it was only after four years that the BI and DOJ reversed themselves in view of
the finding in the Committee Report. The Court of Appeals ruled that the "highly suspicious
circumstances" stated in the Committee Report referred to the affidavits of Barangay Captain
Ramon Soliman (Soliman) and Barangay Treasurer Condrado P. Peralta (Peralta) that there
were no Quintoses or Tomedas in the birthplace of respondents mother and that no such
surnames appeared in the census or master list of voters. The Court of Appeals ruled that
apart from the affidavits, no other evidence was presented to prove that Quintos was not a
Filipino citizen or that her birth certificate was false or fraudulently obtained. The Court of
Appeals ruled that respondents documentary evidence before the BI and DOJ have more

probative value and must prevail over the allegations of Soliman and Peralta. The Court of
Appeals further noted that among the documents presented by respondent were
authenticated documents issued by the Commonwealth of Australia attesting that Quintos
consistently presented herself to be a Filipino citizen. The Court of Appeals ruled that the
authenticity of the documents issued by the Australian government was never questioned nor
put in issue. The Court of Appeals further ruled that the fact that the Quintoses and Tomedas
were not included in the census or master list of voters did not automatically render Quintos
birth certificate invalid. The Court of Appeals ruled that unless a public document is declared
invalid by competent authority, it should be presumed valid and binding for all intents and
purposes.
The dispositive portion of the Court of Appeals Decision reads:
WHEREFORE, the instant petition is GRANTED. The assailed resolution of the Department of
Justice dated October 18, 2004 and summary deportation order of the Bureau of Immigration
dated October 26, 2004 are hereby ANNULLED and SET ASIDE.
SO ORDERED.7
Hence, the petition before this Court.
The Issue
Petitioners raise this sole issue in their Memorandum:8
Whether the Court of Appeals committed a reversible error in finding that respondent is a
Filipino citizen.
Petitioners allege that respondents petition was filed out of time. Petitioners further allege that
respondents voluntary departure from the Philippines had rendered the petition moot. Finally,
petitioners allege that the cancellation of respondents certificate of recognition as a Filipino
citizen and the issuance of the deportation order against him are valid.
The Ruling of this Court
The petition has no merit.
Late Filing of Petition
Petitioners allege that the petition filed before the Court of Appeals should have been
dismissed for late filing. Petitioners allege that respondent only had 15 days from 19 October
2004, the date of receipt of the 18 October 2004 DOJ Resolution, within which to file a petition
for review before the Court of Appeals. However, respondent filed his petition only on 4
November 2004, or one day beyond the reglementary period for filing the petition for review.
Petitioners allege that when the petition was filed, the 18 October 2004 DOJ Resolution had
already lapsed into finality.
We do not agree.
A one-day delay does not justify the appeals dismissal where no element of intent to delay
the administration of justice could be attributed to the petitioner.9 The Court has ruled:
The general rule is that the perfection of an appeal in the manner and within the period
prescribed by law is, not only mandatory, but jurisdictional, and failure to conform to the rules
will render the judgment sought to be reviewed final and unappealable. By way of exception,
unintended lapses are disregarded so as to give due course to appeals filed beyond the
reglementary period on the basis of strong and compelling reasons, such as serving the ends
of justice and preventing a grave miscarriage thereof. The purpose behind the limitation of the
37

period of appeal is to avoid an unreasonable delay in the administration of justice and to put
an end to controversies.10
Respondent had a valid excuse for the late filing of the petition before the Court of Appeals. It
is not disputed that there was a pending petition for prohibition before the trial court. Before
filing the petition for review before the Court of Appeals, respondent had to withdraw the
petition for prohibition before the trial court. The trial court granted the withdrawal of the
petition only on 4 November 2004, the date of filing of the petition for review before the Court
of Appeals. Under the circumstances, we find the one-day delay in filing the petition for review
excusable.
We reiterate:
Rules of procedure are merely tools designed to facilitate the attainment of justice. If the
application of the Rules would tend to frustrate rather than to promote justice, it is always
within our power to suspend the rules or except a particular case from their operation. Law
and jurisprudence grant to courts the prerogative to relax compliance with the procedural
rules, even the most mandatory in character, mindful of the duty to reconcile the need to put
an end to litigation speedily and the parties right to an opportunity to be heard.11
Hence, we sustain the Court of Appeals in accepting the petition for review although it was
filed one-day late.
Mootness of the Petition
Petitioners allege that the petition had been rendered moot because respondent already left
the country.
Petitioners cited Lewin v. The Deportation Board12 where the Court ruled:
x x x. Even if the deportation case is to proceed and even if this Court will decide this appeal
on the merits, there would be no practical value or effect of such action upon Lewin, because
he has already left the country. Consequently, the issues involved herein have become moot
and academic.13
However, we agree with respondent that the factual circumstances in Lewin are different from
the case before us. In Lewin, petitioner was an alien who entered the country as a temporary
visitor, to stay for only 50 days. He prolonged his stay by securing several extensions. Before
his last extension expired, he voluntarily left the country, upon filing a bond, without any
assurance from the Deportation Board that he would be admitted to the country upon his
return. The Court found that he did not return to the country, and at the time he was living in
another country. The Court ruled that Lewins voluntary departure from the country, his long
absence, and his status when he entered the country as a temporary visitor rendered
academic the question of his deportation as an undesirable alien.
In this case, respondent, prior to his deportation, was recognized as a Filipino citizen. He
manifested his intent to return to the country because his Filipino wife and children are
residing in the Philippines. The filing of the petitions before the Court of Appeals and before
this Court showed his intention to prove his Filipino lineage and citizenship, as well as the
error committed by petitioners in causing his deportation from the country. He was precisely
questioning the DOJs revocation of his certificate of recognition and his summary deportation
by the BI.
Therefore, we rule that respondents deportation did not render the present case moot.

Validity of the Cancellation of Respondents


Certificate of Recognition and the Issuance of Deportation Order by the BID
Petitioners allege that the DOJ adduced substantial evidence warranting the revocation of
respondents certificate of recognition and the filing of the deportation proceedings against
him. Petitioners likewise allege that the certificate of recognition did not attain finality as
claimed by respondent.
We agree with petitioners that the issuance of certificate of recognition to respondent has not
attained finality. In Go v. Ramos,14 the Court ruled that citizenship proceedings are a class of
its own and can be threshed out again and again as the occasion may demand. Res judicata
may be applied in cases of citizenship only if the following concur:
1. a persons citizenship must be raised as a material issue in a controversy where said
person is a party;
the Solicitor General or his authorized representative took active part in the resolution thereof;
and
the finding or citizenship is affirmed by this Court.15
However, the courts are not precluded from reviewing the findings of the BI. Judicial review is
permitted if the courts believe that there is substantial evidence supporting the claim of
citizenship, so substantial that there are reasonable grounds for the belief that the claim is
correct.16 When the evidence submitted by a deportee is conclusive of his citizenship, the right
to immediate review should be recognized and the courts should promptly enjoin the
deportation proceedings.17 Courts may review the actions of the administrative offices
authorized to deport aliens and reverse their rulings when there is no evidence to sustain the
rulings.18
In this case, we sustain the Court of Appeals that the evidence presented before the BI and
the DOJ, i.e., (1) certified photocopy of the certificate of birth of Quintos, and a certification
issued by the Local Civil Registrar of San Antonio, Nueva Ecija stating that Quintos was born
on 14 August 1949 of Filipino parents, Felipe M. Quintos and Celina G. Tomeda, in
Panabingan, San Antonio, Nueva Ecija; (2) certified true copy of the certificate of marriage of
respondents parents dated 9 January 1971, indicating the Philippines as Quintos birthplace;
(3) certified true copy of Quintos Australian certificate of registration of alien, indicating her
nationality as Filipino; (4) certified true copy of respondents birth certificate stating that he
was born on 13 March 1975 and indicating the Philippines as his mothers birthplace; and (5)
certified true copy of the letter dated 14 July 1999 of the Australian Department of Immigration
and Multicultural Affairs, stating that as of 14 July 1999, Quintos has not been granted
Australian citizenship, have more probative value and must prevail over the statements of
Soliman and Peralta before the Senate Committees. The Committee Report on respondent
stated:
F. Michael Alfio Pennisi was able to present before the BI and the committees, the
documents required in granting recognition of Philippine citizenship, particularly the birth
certificate of his Filipino mother, Anita Tomeda Quintos.
However, a verification of the authenticity of the above documents reveals highly suspicious
circumstances.
His alleged mother and other relatives, specifically the parents of the former, namely: Felipe
M. Quintos and Celina G. Tomeda, who were mentioned in his application for recognition of
38

Philippine citizenship in the BI, are not known and have never existed in Panabingan, San
Antonio, Nueva Ecija.
According to the affidavits executed by Barangay Captain Ramon Soliman and Barangay
Treasurer Condrado P. Peralta of the abovementioned place, there are no Quintoses or
Tomedas that have lived or have resided in the said barangay.
Both barangay officials further claimed that even in the census or master list of voters, the
family names of Quintos or Tomedas do not exist.
His mother's certificate of birth in the civil registrar of San Antonio, Nueva Ecija was issued on
the basis of an application for late registration, which is ten (10) years after the date of birth. 19
The memorandum20 of the DOJ special committee also cited only the affidavits of Soliman and
Peralta and then concluded that the evidence presented before the Senate Committees had
overcome the presumption that the entries in the certificate of live birth of Quintos are prima
facie evidence of the facts stated therein.21
We agree with the Court of Appeals that while the affidavits of Soliman and Peralta might
have cast doubt on the validity of Quintos certificate of live birth, such certificate remains valid
unless declared invalid by competent authority. The rule stands that "(d)ocuments consisting
of entries in public records made in the performance of a duty by a public officer are prima
facie evidence of the facts stated therein. x x x."22
We further sustain the Court of Appeals that there could be reasons why the Quintoses and
Tomedas were not included in the census, such as they could have been mere transients in
the place. As for their absence in the masters list of voters, they could have failed to register
themselves as voters. The late registration of Quintos certificate of live birth was made 10
years after her birth and not anytime near the filing of respondents petition for recognition as
Filipino citizen. As such, it could not be presumed that the certificates late filing was meant to
use it fraudulently. Finally, the Australian Department of Immigration and Multicultural Affairs
itself attested that as of 14 July 1999, Quintos has not been granted Australian citizenship.
Respondent submitted a certified true copy of Quintos Australian certificate of registration of
alien, indicating her nationality as Filipino. These pieces of evidence should prevail over the
affidavits submitted by Soliman and Peralta to the Senate Committees.
WHEREFORE, we DENY the petition. We AFFIRM the 30 September 2005 Decision of the
Court of Appeals in CA-G.R. SP No. 87271.
SO ORDERED.

39

G.R. Nos. 192147 & 192149


August 23, 2011
RENALD F. VILANDO, Petitioner,
vs.
HOUSE OF REPRESENTATIVES ELECTORAL TRIBUNAL, JOCELYN SY LIMKAICHONG
AND HON. SPEAKER PROSPERO NOGRALES, Respondents.
DECISION
MENDOZA, J.:
This is a petition for certiorari under Rule 65 of the Revised Rules of Court assailing the March
24, 2010 Decision1 of the House of Representatives Electoral Tribunal (HRET) dismissing the
petitions for quo warranto and declaring private respondent Jocelyn Sy Limkaichong
(Limkaichong) not disqualified as Member of the House of Representatives representing the
First District of Negros Oriental and its Resolution2 dated May 17, 2010, denying the motion
for reconsideration.
In the May 14, 2007 elections, Limkaichong filed her certificate of candidacy for the position of
Representative of the First District of Negros Oriental. She won over the other contender,
Olivia Paras.
On May 25, 2007, she was proclaimed as Representative by the Provincial Board of
Canvassers on the basis of Comelec Resolution No. 80623 issued on May 18, 2007.
On July 23, 2007, she assumed office as Member of the House of Representatives.
Meanwhile, petitions involving either the disqualification or the proclamation of Limkaichong
were filed before the Commission on Elections (COMELEC) which reached the Court.
The petitions, which questioned her citizenship, were filed against Limkaichong by her
detractors: Louis Biraogo (G.R. No. 179120);4 Olivia Paras (G.R. Nos. 179132-33);5 and
Renald F. Vilando (G.R. Nos. 179240-41).6 These three (3) petitions were consolidated with
the petition for certiorari filed by Limkaichong (G.R. Nos. 178831-32) assailing the Joint
Resolution issued by the COMELEC which resolved the disqualification cases against her.
On April 1, 2009, the Court granted the aforesaid petition of Limkaichong, reversed the Joint
Resolution of the Comelec, dismissed the three (3) other petitions, and directed the petitioners
to seek relief before the HRET by way of a petition for Quo Warranto.
On April 21, 2009 and May 27, 2009, petitioner Renald F. Vilando (Vilando), as taxpayer; and
Jacinto Paras, as registered voter of the congressional district concerned, filed separate
petitions for Quo Warranto against Limkaichong before the HRET. These petitions were
consolidated by the HRET as they both challenged the eligibility of one and the same
respondent. Petitioners asserted that Limkaichong was a Chinese citizen and ineligible for the
office she was elected and proclaimed. They alleged that she was born to a father (Julio Sy),
whose naturalization had not attained finality, and to a mother who acquired the Chinese
citizenship of Julio Sy from the time of her marriage to the latter. Also, they invoked the
jurisdiction of the HRET for a determination of Limkaichongs citizenship, which necessarily
included an inquiry into the validity of the naturalization certificate of Julio Sy.
For her defense, Limkaichong maintained that she is a natural-born Filipino citizen. She
averred that the acquisition of Philippine citizenship by her father was regular and in order and
had already attained the status of res judicata. Further, she claimed that the validity of such
citizenship could not be assailed through a collateral attack.

On March 24, 2010, the HRET dismissed both petitions and declared Limkaichong not
disqualified as Member of the House of Representatives. Pertinent portions of the HRET
decision reads:
By and large, petitioners failed to satisfy the quantum of proof to sustain their theory that
respondent is not a natural-born Filipino citizen and therefore not qualified as Representative
of the First District, Negros Oriental. This being so, their petitions must fail.
WHEREFORE, the Tribunal DISMISSES the instant petition for lack of merit and declares that
respondent Jocelyn Sy Limkaichong is not disqualified as Member of the House of
Representatives representing the First District, Negros Oriental.
As soon as the Decision becomes final and executory, notice of copies thereof shall be sent to
the President of the Philippines, the House of Representatives through the Speaker, the
Commission on Audit through the Chairman, pursuant to Rule 96 of the 2004 Rules of the
House of Representatives Electoral Tribunal. Let a copy of this Decision be furnished the
Chairman, Commission on Elections, for his information and appropriate action.
SO ORDERED.7
The petitioners sought reconsideration of the aforesaid decision, but it was denied by the
HRET in its Resolution dated May 17, 2010.
Hence, this petition for certiorari filed by Vilando anchored on the following
GROUNDS:
THE ONE-SIDED RESOLUTION OF THE SUBJECT PETITION FOR QUO WARRANTO
AND THE UTTER FAILURE OF THE HRET TO DISQUALIFY LIMKAICHONG AS MEMBER
OF THE HOUSE OF REPRESENTATIVES DESPITE MANIFEST EVIDENCE THAT SHE IS
NOT A NATURAL-BORN FILIPINO CITIZEN IS WHIMSICAL, CAPRICIOUS AND
ARBITRARY BECAUSE:
1. THE PETITION FOR QUO WARRANTO DOES NOT OPERATE AS A
COLLATERAL ATTACK ON THE CITIZENSHIP OF LIMKAICHONGS FATHER
FOR THE REASON THAT HER FATHERS CERTIFICATE OF NATURALIZATION
IS OF NO FORCE AND EFFECT FROM THE VERY BEGINNING, HENCE, THERE
IS ACTUALLY NOTHING BEING ATTACKED OR ASSAILED BY THE SAME.
2. LIMKAICHONG CANNOT DERIVE PHILIPPINE CITIZENSHIP FROM HER
MOTHER GIVEN THAT AT THE TIME OF HER BIRTH, HER MOTHER IS NOT
ALREADY A FILIPINO CITIZEN AS A RESULT OF HER MARRIAGE TO HER
FATHER AS PROVIDED FOR UNDER SECTION 1 (7) OF COMMONWEALTH
ACT NO. 63 IN RELATION TO ARTICLE 2 (1) CHAPTER II OF THE CHINESE
REVISED NATIONALITY LAW OF FEBRUARY 5, 1959.
3. HAVING THE PLENARY, ABSOLUTE AND EXCLUSIVE JURISDICTION TO
DETERMINE, AMONG OTHERS, THE QUALIFICATIONS OF MEMBERS OF THE
HOUSE OF REPRESENTATIVES, THE HRET CAN LOOK INTO THE ELIGIBILITY
OF LIMKAICHONG EVEN IF, AS AN INCIDENT THERETO, IT WOULD MEAN
LOOKING INTO THE VALIDITY OF THE CERTIFICATE OF NATURALIZATION.8
It should be noted that Limkaichongs term of office as Representative of the First District of
Negros Oriental from June 30, 2007 to June 30, 2010 already expired. As such, the issue
questioning her eligibility to hold office has been rendered moot and academic by the
40

expiration of her term. Whatever judgment is reached, the same can no longer have any
practical legal effect or, in the nature of things, can no longer be enforced.9 Thus, the petition
may be dismissed for being moot and academic.
Moreover, there was the conduct of the 2010 elections, a supervening event, in a sense, has
also rendered this case moot and academic. A moot and academic case is one that ceases to
present a justiciable controversy by virtue of supervening events, so that a declaration thereon
would be of no practical value. As a rule, courts decline jurisdiction over such case, or dismiss
it on ground of mootness. 10
Citizenship, being a continuing requirement for Members of the House of Representatives,
however, may be questioned at anytime.11 For this reason, the Court deems it appropriate to
resolve the petition on the merits. This position finds support in the rule that courts will decide
a question, otherwise moot and academic, if it is "capable of repetition, yet evading review."12
The question on Limkaichongs citizenship is likely to recur if she would run again, as she did
run, for public office, hence, capable of repetition.
In any case, the Court is of the view that the HRET committed no grave abuse of discretion in
finding that Limkaichong is not disqualified to sit as Member of the House of Representatives.
Vilandos argument, that the quo warranto petition does not operate as a collateral attack on
the citizenship of Limkaichongs father as the certificate of naturalization is null and void from
the beginning, is devoid of merit.
In this petition, Vilando seeks to disqualify Limkaichong on the ground that she is a Chinese
citizen. To prove his point, he makes reference to the alleged nullity of the grant of
naturalization of Limkaichongs father which, however, is not allowed as it would constitute a
collateral attack on the citizenship of the father. In our jurisdiction, an attack on a person's
citizenship may only be done through a direct action for its nullity.13
The proper proceeding to assail the citizenship of Limkaichongs father should be in
accordance with Section 18 of Commonwealth Act No. 473. As held in Limkaichong v.
Comelec,14 thus:
As early as the case of Queto v. Catolico,15 where the Court of First Instance judge motu
propio and not in the proper denaturalization proceedings called to court various grantees of
certificates of naturalization (who had already taken their oaths of allegiance) and cancelled
their certificates of naturalization due to procedural infirmities, the Court held that:
x x x It may be true that, as alleged by said respondents, that the proceedings for
naturalization were tainted with certain infirmities, fatal or otherwise, but that is beside the
point in this case. The jurisdiction of the court to inquire into and rule upon such infirmities
must be properly invoked in accordance with the procedure laid down by law. Such procedure
is the cancellation of the naturalization certificate. [Section 1(5), Commonwealth Act No. 63],
in the manner fixed in Section 18 of Commonwealth Act No. 473, hereinbefore quoted,
namely, "upon motion made in the proper proceedings by the Solicitor General or his
representatives, or by the proper provincial fiscal." In other words, the initiative must come
from these officers, presumably after previous investigation in each particular case.
Clearly, under law and jurisprudence, it is the State, through its representatives designated by
statute, that may question the illegally or invalidly procured certificate of naturalization in the
appropriate denaturalization proceedings. It is plainly not a matter that may be raised by
private persons in an election case involving the naturalized citizens descendant.

Vilando asserts that as an incident in determining the eligibility of Limkaichong, the HRET,
having the plenary, absolute and exclusive jurisdiction to determine her qualifications, can
pass upon the efficacy of the certificate of naturalization.
True, the HRET has jurisdiction over quo warranto petitions, specifically over cases
challenging ineligibility on the ground of lack of citizenship. No less than the 1987 Constitution
vests the HRET the authority to be the sole judge of all contests relating to the election,
returns and qualifications of its Members. This constitutional power is likewise echoed in the
2004 Rules of the HRET. Rule 14 thereof restates this duty, thus:
Rule 14. Jurisdiction. The Tribunal is the sole judge of all contests relating to the election,
returns, and qualifications of the Members of the House of Representatives.
Time and again, this Court has acknowledged this sole and exclusive jurisdiction of the
HRET.16 The power granted to HRET by the Constitution is intended to be as complete and
unimpaired as if it had remained originally in the legislature.17 Such power is regarded as full,
clear and complete and excludes the exercise of any authority on the part of this Court that
would in any wise restrict it or curtail it or even affect the same.18
Such power of the HRET, no matter how complete and exclusive, does not carry with it the
authority to delve into the legality of the judgment of naturalization in the pursuit of
disqualifying Limkaichong. To rule otherwise would operate as a collateral attack on the
citizenship of the father which, as already stated, is not permissible. The HRET properly
resolved the issue with the following ratiocination:
xxx We note that Jocelyn C. Limkaichong, not the father Julio Ong Sy, is the respondent in
the present case. The Tribunal may not dwell on deliberating on the validity of naturalization of
the father if only to pursue the end of declaring the daughter as disqualified to hold office.
Unfortunately, much as the Tribunal wants to resolve said issue, it cannot do so because its
jurisdiction is limited to the qualification of the proclaimed respondent Limkaichong, being a
sitting Member of the Congress.
Evidently, there is no basis to oblige the Tribunal to reopen the naturalization proceedings for
a determination of the citizenship of the ascendant of respondent. A petition for quo warranto
is not a means to achieve that purpose. To rule on this issue in this quo warranto proceeding
will not only be a clear grave abuse of discretion amounting to a lack or excess of jurisdiction,
but also a blatant violation of due process on the part of the persons who will be affected or
who are not parties in this case.19
Thus, the Office of the Solicitor General (OSG) wrote that "a collateral attack against a
judgment is generally not allowed, unless the judgment is void upon its face or its nullity is
apparent by virtue of its own recitals."20 Under the present situation, there is no evidence to
show that the judgment is void on its face:
As to the Order of the CFI, Negros Oriental dated July 9, 1957 and September 21, 1959 that
were offered in evidence, far from proving an invalid oath of allegiance and certificate of
naturalization, being public records, they do in fact constitute legitimate source of authority for
the conferment of status of the father of respondent as naturalized Filipino. Absent any
contrary declaration by a competent court, the Tribunal presumes the validity of the CFI
Orders of July 9, 1957 and September 21, 1959, and the resulting documentations of Julio
Sys acquisition of Filipino citizenship by naturalization as valid and of legal effect. The oath of

41

allegiance and certificate of naturalization are themselves proofs of the actual conferment of
naturalization.21
The HRET, therefore, correctly relied on the presumption of validity of the July 9, 1957 and
September 21, 1959 Orders of the Court of First Instance (CFI) Negros Oriental, which
granted the petition and declared Julio Sy a naturalized Filipino absent any evidence to the
contrary.
Records disclose that Limkaichong was born in Dumaguete City on November 9, 1959. The
governing law is the citizenship provision of the 1935 Constitution, the pertinent portion
thereof, reads:
Article IV
Section 1. The following are citizens of the Philippines:
xxx
(3) Those whose fathers are citizens of the Philippines.
(4) Those whose mothers are citizens of the Philippines and, upon reaching the age of
majority, elect Philippine citizenship.
xxx
Indubitably, with Limkaichongs father having been conferred the status as a naturalized
Filipino, it follows that she is a Filipino citizen born to a Filipino father.
Even on the assumption that the naturalization proceedings and the subsequent issuance of
certificate of naturalization were invalid, Limkaichong can still be considered a natural-born
Filipino citizen having been born to a Filipino mother and having impliedly elected Filipino
citizenship when she reached majority age. The HRET is, thus, correct in declaring that
Limkaichong is a natural-born Filipino citizen:
Respondent Limkaichong falls under the category of those persons whose fathers are citizens
of the Philippines. (Section 1(3), Article IV, 1935 Constitution) It matters not whether the father
acquired citizenship by birth or by naturalization. Therefore, following the line of transmission
through the father under the 1935 Constitution, the respondent has satisfactorily complied with
the requirement for candidacy and for holding office, as she is a natural-born Filipino citizen.
Likewise, the citizenship of respondent Limkaichong finds support in paragraph 4, Section 1,
Article IV of the 1935 Constitution.
Having failed to prove that Anesia Sy lost her Philippine citizenship, respondent can be
considered a natural born citizen of the Philippines, having been born to a mother who was a
natural-born Filipina at the time of marriage, and because respondent was able to elect
citizenship informally when she reached majority age. Respondent participated in the
barangay elections as a young voter in 1976, accomplished voters affidavit as of 1984, and
ran as a candidate and was elected as Mayor of La Libertad, Negros Oriental in 2004. These
are positive acts of election of Philippine citizenship. The case of In re: Florencio Mallare,
elucidates how election of citizenship is manifested in actions indubitably showing a definite
choice. We note that respondent had informally elected citizenship after January 17, 1973
during which time the 1973 Constitution considered as citizens of the Philippines all those who
elect citizenship in accordance with the 1935 Constitution. The 1987 Constitution provisions,
i.e., Section 1(3), Article [IV] and Section 2, Article [IV] were enacted to correct the anomalous
situation where one born of a Filipino father and an alien mother was automatically accorded

the status of a natural-born citizen, while one born of a Filipino mother and an alien father
would still have to elect Philippine citizenship yet if so elected, was not conferred natural-born
status. It was the intention of the framers of the 1987 Constitution to treat equally those born
before the 1973 Constitution and who elected Philippine citizenship upon reaching the age of
majority either before or after the effectivity of the 1973 Constitution. Thus, those who would
elect Philippine citizenship under par. 3, Section 1, Article [IV] of the 1987 Constitution are
now, under Section 2, Article [IV] thereof also natural-born Filipinos. The following are the
pertinent provisions of the 1987 Constitution:
Article IV
Section 1. The following are citizens of the Philippines:
(1) Those who are citizens of the Philippines at the time of the adoption of this
Constitution;
(2) Those whose fathers or mothers are citizens of the Philippines;
(3) Those born before January 17, 1973, of Filipino mothers, who elect Philippine
citizenship upon reaching the age of majority; and
(4) Those who are naturalized in accordance with law.
Section 2. Natural-born citizens are those who are citizens of the Philippines from birth without
having to perform any act to acquire or perfect their Philippine citizenship. Those who elect
Philippine citizenship in accordance with paragraph (3), Section 1 hereof shall be deemed
natural-born citizens.22
Vilandos assertion that Limkaichong cannot derive Philippine citizenship from her mother
because the latter became a Chinese citizen when she married Julio Sy, as provided for under
Section 1 (7) of Commonwealth Act No. 63 in relation to Article 2 (1) Chapter II of the Chinese
Revised Nationality Law of February 5, 1959, must likewise fail.
As aptly pointed out by the HRET, Vilando was not able to offer in evidence a duly certified
true copy of the alleged Chinese Revised Law of Nationality to prove that Limkaichongs
mother indeed lost her Philippine citizenship. Verily, Vilando failed to establish his case
through competent and admissible evidence to warrant a reversal of the HRET ruling.
Also, an application for an alien certificate of registration (ACR) is not an indubitable proof of
forfeiture of Philippine citizenship. It is well to quote the ruling of the HRET on this matter, to
wit:
An alien certificate of registration is issued to an individual who declares that he is not a
Filipino citizen. It is obtained only when applied for. It is in a form prescribed by the agency
and contains a declaration by the applicant of his or her personal information, a photograph,
and physical details that identify the applicant. It bears no indication of basis for foreign
citizenship, nor proof of change to foreign citizenship. It certifies that a person named therein
has applied for registration and fingerprinting and that such person was issued a certificate of
registration under the Alien Registration Act of 1950 or other special law. It is only evidence of
registration.
Unlike birth certificates registered pursuant to Act 3753 (The Civil Register Law), and much
less like other public records referred to under Section 23, Rule 132, an alien certificate of
registration is not a public document that would be prima facie evidence of the truth of facts
contained therein. On its face, it only certifies that the applicant had submitted himself or
42

herself to registration. Therefore, there is no presumption of alienage of the declarant. This is


especially so where the declarant has in fact been a natural-born Filipino all along and never
lost his or her status as such.23 1avvphi1
Thus, obtaining an ACR by Limkaichongs mother was not tantamount to a repudiation of her
original citizenship. Neither did it result in an acquisition of alien citizenship. In a string of
decisions, this Court has consistently held that an application for, and the holding of, an alien
certificate of registration is not an act constituting renunciation of Philippine citizenship. 24 For
renunciation to effectively result in the loss of citizenship, the same must be express.25 Such
express renunciation is lacking in this case.
Accordingly, Limkaichongs mother, being a Filipino citizen, can transmit her citizenship to her
daughter.
Well-settled is the principle that the judgments of the HRET are beyond judicial interference.
The only instance where this Court may intervene in the exercise of its so-called extraordinary
jurisdiction is upon a determination that the decision or resolution of the HRET was rendered
without or in excess of its jurisdiction, or with grave abuse of discretion or upon a clear
showing of such arbitrary and improvident use of its power to constitute a denial of due
process of law, or upon a demonstration of a very clear unmitigated error, manifestly
constituting such grave abuse of discretion that there has to be a remedy for such abuse. 26 In
this case, there is no showing of any such arbitrariness or improvidence. The HRET acted well
within the sphere of its power when it dismissed the quo warranto petition.
In fine, this Court finds sufficient basis to sustain the ruling of the HRET which resolved the
issue of citizenship in favor of Limkaichong.
WHEREFORE, the petition is DENIED. Accordingly, the Court affirms the March 24, 2010
Decision of the HRET declaring that Limkaichong is not disqualified as Member of the House
of Representatives representing the First District, Negros Oriental.
SO ORDERED.

43

citizenship" 1 in strict compliance with the provisions of Commonwealth Act No. 625 entitled
"An Act Providing for the Manner in which the Option to Elect Philippine Citizenship shall be
Declared by a Person Whose Mother is a Filipino Citizen." The OSG adds that "(w)hat he
acquired at best was only an inchoate Philippine citizenship which he could perfect by election
upon reaching the age of majority." 2 In this regard, the OSG clarifies that "two (2) conditions
must concur in order that the election of Philippine citizenship may be effective, namely: (a)
the mother of the person making the election must be a citizen of the Philippines; and (b) said
election must be made upon reaching the age of majority." 3 The OSG then explains the
meaning of the phrase "upon reaching the age of majority:"
The clause "upon reaching the age of majority" has been construed to
mean a reasonable time after reaching the age of majority which had been
interpreted by the Secretary of Justice to be three (3) years (VELAYO,
supra at p. 51 citing Op., Sec. of Justice No. 70, s. 1940, Feb. 27, 1940).
Said period may be extended under certain circumstances, as when a
(sic) person concerned has always considered himself a Filipino (ibid.,
citing Op. Nos. 355 and 422, s. 1955; 3, 12, 46, 86 and 97, s. 1953). But
in Cuenco, it was held that an election done after over seven (7) years
was not made within a reasonable time.
In conclusion, the OSG points out that Ching has not formally elected Philippine citizenship
and, if ever he does, it would already be beyond the "reasonable time" allowed by present
jurisprudence. However, due to the peculiar circumstances surrounding Ching's case, the
OSG recommends the relaxation of the standing rule on the construction of the phrase
"reasonable period" and the allowance of Ching to elect Philippine citizenship in accordance
with C.A. No. 625 prior to taking his oath as a member of the Philippine Bar.1wphi1.nt
On 27 July 1999, Ching filed a Manifestation, attaching therewith his Affidavit of Election of
Philippine Citizenship and his Oath of Allegiance, both dated 15 July 1999. In his
Manifestation, Ching states:
1. I have always considered myself as a Filipino;
2. I was registered as a Filipino and consistently declared myself as one in
my school records and other official documents;
3. I am practicing a profession (Certified Public Accountant) reserved for
Filipino citizens;
4. I participated in electoral process[es] since the time I was eligible to
vote;
5. I had served the people of Tubao, La Union as a member of the
Sangguniang Bayan from 1992 to 1995;
6. I elected Philippine citizenship on July 15, 1999 in accordance with
Commonwealth Act No. 625;
7. My election was expressed in a statement signed and sworn to by me
before a notary public;
8. I accompanied my election of Philippine citizenship with the oath of
allegiance to the Constitution and the Government of the Philippines;

BAR MATTER No. 914 October 1, 1999


RE: APPLICATION FOR ADMISSION TO THE PHILIPPINE BAR,
vs.
VICENTE D. CHING, applicant.
RESOLUTION
KAPUNAN, J.:
Can a legitimate child born under the 1935 Constitution of a Filipino mother and an alien
father validly elect Philippine citizenship fourteen (14) years after he has reached the age of
majority? This is the question sought to be resolved in the present case involving the
application for admission to the Philippine Bar of Vicente D. Ching.
The facts of this case are as follows:
Vicente D. Ching, the legitimate son of the spouses Tat Ching, a Chinese citizen, and Prescila
A. Dulay, a Filipino, was horn in Francia West, Tubao, La Union on 11 April 1964. Since his
birth, Ching has resided in the Philippines.
On 17 July 1998, Ching, after having completed a Bachelor of Laws course at the St. Louis
University in Baguio City, filed an application to take the 1998 Bar Examinations. In a
Resolution of this Court, dated 1 September 1998, he was allowed to take the Bar
Examinations, subject to the condition that he must submit to the Court proof of his Philippine
citizenship.
In compliance with the above resolution, Ching submitted on 18 November 1998, the following
documents:
1. Certification, dated 9 June 1986, issued by the Board of Accountancy of
the Professional Regulations Commission showing that Ching is a certified
public accountant;
2. Voter Certification, dated 14 June 1997, issued by Elizabeth B. Cerezo,
Election Officer of the Commission on Elections (COMELEC) in Tubao La
Union showing that Ching is a registered voter of the said place; and
3. Certification, dated 12 October 1998, also issued by Elizabeth B.
Cerezo, showing that Ching was elected as a member of the Sangguniang
Bayan of Tubao, La Union during the 12 May 1992 synchronized
elections.
On 5 April 1999, the results of the 1998 Bar Examinations were released and Ching was one
of the successful Bar examinees. The oath-taking of the successful Bar examinees was
scheduled on 5 May 1999. However, because of the questionable status of Ching's
citizenship, he was not allowed to take his oath. Pursuant to the resolution of this Court, dated
20 April 1999, he was required to submit further proof of his citizenship. In the same
resolution, the Office of the Solicitor General (OSG) was required to file a comment on Ching's
petition for admission to the bar and on the documents evidencing his Philippine citizenship.
The OSG filed its comment on 8 July 1999, stating that Ching, being the "legitimate child of a
Chinese father and a Filipino mother born under the 1935 Constitution was a Chinese citizen
and continued to be so, unless upon reaching the age of majority he elected Philippine
44

9. I filed my election of Philippine citizenship and my oath of allegiance to


(sic) the Civil Registrar of Tubao La Union, and
10. I paid the amount of TEN PESOS (Ps. 10.00) as filing fees.
Since Ching has already elected Philippine citizenship on 15 July 1999, the question raised is
whether he has elected Philippine citizenship within a "reasonable time." In the affirmative,
whether his citizenship by election retroacted to the time he took the bar examination.
When Ching was born in 1964, the governing charter was the 1935 Constitution. Under Article
IV, Section 1(3) of the 1935 Constitution, the citizenship of a legitimate child born of a Filipino
mother and an alien father followed the citizenship of the father, unless, upon reaching the
age of majority, the child elected Philippine citizenship. 4 This right to elect Philippine
citizenship was recognized in the 1973 Constitution when it provided that "(t)hose who elect
Philippine citizenship pursuant to the provisions of the Constitution of nineteen hundred and
thirty-five" are citizens of the Philippines. 5 Likewise, this recognition by the 1973 Constitution
was carried over to the 1987 Constitution which states that "(t)hose born before January 17,
1973 of Filipino mothers, who elect Philippine citizenship upon reaching the age of majority"
are Philippine citizens. 6 It should be noted, however, that the 1973 and 1987 Constitutional
provisions on the election of Philippine citizenship should not be understood as having a
curative effect on any irregularity in the acquisition of citizenship for those covered by the
1935 Constitution. 7 If the citizenship of a person was subject to challenge under the old
charter, it remains subject to challenge under the new charter even if the judicial challenge
had not been commenced before the effectivity of the new Constitution. 8
C.A. No. 625 which was enacted pursuant to Section 1(3), Article IV of the 1935 Constitution,
prescribes the procedure that should be followed in order to make a valid election of Philippine
citizenship. Under Section 1 thereof, legitimate children born of Filipino mothers may elect
Philippine citizenship by expressing such intention "in a statement to be signed and sworn to
by the party concerned before any officer authorized to administer oaths, and shall be filed
with the nearest civil registry. The said party shall accompany the aforesaid statement with the
oath of allegiance to the Constitution and the Government of the Philippines."
However, the 1935 Constitution and C.A. No. 625 did not prescribe a time period within which
the election of Philippine citizenship should be made. The 1935 Charter only provides that the
election should be made "upon reaching the age of majority." The age of majority then
commenced upon reaching twenty-one (21) years. 9 In the opinions of the Secretary of Justice
on cases involving the validity of election of Philippine citizenship, this dilemma was resolved
by basing the time period on the decisions of this Court prior to the effectivity of the 1935
Constitution. In these decisions, the proper period for electing Philippine citizenship was, in
turn, based on the pronouncements of the Department of State of the United States
Government to the effect that the election should be made within a "reasonable time" after
attaining the age of majority. 10 The phrase "reasonable time" has been interpreted to mean
that the election should be made within three (3) years from reaching the age of
majority. 11 However, we held in Cuenco vs. Secretary of Justice, 12 that the three (3) year
period is not an inflexible rule. We said:
It is true that this clause has been construed to mean a reasonable period
after reaching the age of majority, and that the Secretary of Justice has
ruled that three (3) years is the reasonable time to elect Philippine
citizenship under the constitutional provision adverted to above, which

period may be extended under certain circumstances, as when the person


concerned has always considered himself a Filipino. 13
However, we cautioned in Cuenco that the extension of the option to elect Philippine
citizenship is not indefinite:
Regardless of the foregoing, petitioner was born on February 16, 1923.
He became of age on February 16, 1944. His election of citizenship was
made on May 15, 1951, when he was over twenty-eight (28) years of age,
or over seven (7) years after he had reached the age of majority. It is clear
that said election has not been made "upon reaching the age of majority."
14

In the present case, Ching, having been born on 11 April 1964, was already thirty-five (35)
years old when he complied with the requirements of C.A. No. 625 on 15 June 1999, or over
fourteen (14) years after he had reached the age of majority. Based on the interpretation of
the phrase "upon reaching the age of majority," Ching's election was clearly beyond, by any
reasonable yardstick, the allowable period within which to exercise the privilege. It should be
stated, in this connection, that the special circumstances invoked by Ching, i.e., his
continuous and uninterrupted stay in the Philippines and his being a certified public
accountant, a registered voter and a former elected public official, cannot vest in him
Philippine citizenship as the law specifically lays down the requirements for acquisition of
Philippine citizenship by election.
Definitely, the so-called special circumstances cannot constitute what Ching erroneously
labels as informal election of citizenship. Ching cannot find a refuge in the case of In re:
Florencio Mallare, 15 the pertinent portion of which reads:
And even assuming arguendo that Ana Mallare were (sic) legally married
to an alien, Esteban's exercise of the right of suffrage when he came of
age, constitutes a positive act of election of Philippine citizenship. It has
been established that Esteban Mallare was a registered voter as of April
14, 1928, and that as early as 1925 (when he was about 22 years old),
Esteban was already participating in the elections and campaigning for
certain candidate[s]. These acts are sufficient to show his preference for
Philippine citizenship. 16
Ching's reliance on Mallare is misplaced. The facts and circumstances obtaining therein are
very different from those in the present case, thus, negating its applicability. First, Esteban
Mallare was born before the effectivity of the 1935 Constitution and the enactment of C.A. No.
625. Hence, the requirements and procedures prescribed under the 1935 Constitution and
C.A. No. 625 for electing Philippine citizenship would not be applicable to him. Second, the
ruling in Mallare was an obiter since, as correctly pointed out by the OSG, it was not
necessary for Esteban Mallare to elect Philippine citizenship because he was already a
Filipino, he being a natural child of a Filipino mother. In this regard, the Court stated:
Esteban Mallare, natural child of Ana Mallare, a Filipina, is therefore
himself a Filipino, and no other act would be necessary to confer on him
all the rights and privileges attached to Philippine citizenship (U.S. vs. Ong
Tianse, 29 Phil. 332; Santos Co vs. Government of the Philippine Islands,
42 Phil. 543, Serra vs. Republic, L-4223, May 12, 1952, Sy Quimsuan vs.
Republic, L-4693, Feb. 16, 1953; Pitallano vs. Republic, L-5111, June 28,
45

1954). Neither could any act be taken on the erroneous belief that he is a
non-filipino divest him of the citizenship privileges to which he is rightfully
entitled. 17
The ruling in Mallare was reiterated and further elaborated in Co vs. Electoral Tribunal of the
House of Representatives, 18 where we held:
We have jurisprudence that defines "election" as both a formal and an
informal process.
In the case of In re: Florencio Mallare (59 SCRA 45 [1974]), the Court held
that the exercise of the right of suffrage and the participation in election
exercises constitute a positive act of election of Philippine citizenship. In
the exact pronouncement of the Court, we held:
Esteban's exercise of the right of suffrage when he
came of age constitutes a positive act of Philippine
citizenship. (p. 52: emphasis supplied)
The private respondent did more than merely exercise his right of suffrage. He has
established his life here in the Philippines.
For those in the peculiar situation of the respondent who cannot be
excepted to have elected Philippine citizenship as they were already
citizens, we apply the In Re Mallare rule.
xxx xxx xxx
The filing of sworn statement or formal declaration is a requirement for
those who still have to elect citizenship. For those already Filipinos when
the time to elect came up, there are acts of deliberate choice which cannot
be less binding. Entering a profession open only to Filipinos, serving in
public office where citizenship is a qualification, voting during election
time, running for public office, and other categorical acts of similar nature
are themselves formal manifestations for these persons.
An election of Philippine citizenship presupposes that the person electing
is an alien. Or his status is doubtful because he is a national of two
countries. There is no doubt in this case about Mr. Ong's being a Filipino
when he turned twenty-one (21).
We repeat that any election of Philippine citizenship on the part of the
private respondent would not only have been superfluous but it would also
have resulted in an absurdity. How can a Filipino citizen elect Philippine
citizenship? 19
The Court, like the OSG, is sympathetic with the plight of Ching. However, even if we consider
the special circumstances in the life of Ching like his having lived in the Philippines all his life
and his consistent belief that he is a Filipino, controlling statutes and jurisprudence constrain
us to disagree with the recommendation of the OSG. Consequently, we hold that Ching failed
to validly elect Philippine citizenship. The span of fourteen (14) years that lapsed from the time
he reached the age of majority until he finally expressed his intention to elect Philippine
citizenship is clearly way beyond the contemplation of the requirement of electing "upon
reaching the age of majority." Moreover, Ching has offered no reason why he delayed his
election of Philippine citizenship. The prescribed procedure in electing Philippine citizenship is

certainly not a tedious and painstaking process. All that is required of the elector is to execute
an affidavit of election of Philippine citizenship and, thereafter, file the same with the nearest
civil registry. Ching's unreasonable and unexplained delay in making his election cannot be
simply glossed over.
Philippine citizenship can never be treated like a commodity that can be claimed when needed
and suppressed when convenient. 20 One who is privileged to elect Philippine citizenship has
only an inchoate right to such citizenship. As such, he should avail of the right with fervor,
enthusiasm and promptitude. Sadly, in this case, Ching slept on his opportunity to elect
Philippine citizenship and, as a result. this golden privilege slipped away from his grasp.
IN VIEW OF THE FOREGOING, the Court Resolves to DENY Vicente D. Ching's application
for admission to the Philippine Bar.1wphi1.nt
SO ORDERED.

46

G.R. No. 153883


January 13, 2004
REPUBLIC OF THE PHILIPPINES, petitioner,
vs.
CHULE Y. LIM, respondent.
DECISION
YNARES-SANTIAGO, J.:
This petition for review on certiorari under Rule 45 of the Rules of Court stemmed from a
petition for correction of entries under Rule 108 of the Rules of Court filed by respondent
Chule Y. Lim with the Regional Trial Court of Lanao del Norte, Branch 4, docketed as Sp.
Proc. No. 4933.
In her petition, respondent claimed that she was born on October 29, 1954 in Buru-an, Iligan
City. Her birth was registered in Kauswagan, Lanao del Norte but the Municipal Civil Registrar
of Kauswagan transferred her record of birth to Iligan City. She alleged that both her
Kauswagan and Iligan City records of birth have four erroneous entries, and prays that they
be corrected.
The trial court then issued an Order,1 which reads:
WHEREFORE, finding the petition to be sufficient in form and substance, let the
hearing of this case be set on December 27, 1999 before this Court, Hall of Justice,
Rosario Heights, Tubod, Iligan City at 8:30 oclock in the afternoon at which date,
place and time any interested person may appear and show cause why the petition
should not be granted.
Let this order be published in a newspaper of general circulation in the City of Iligan
and the Province of Lanao del Norte once a week for three (3) consecutive weeks at
the expense of the petitioner.
Furnish copies of this order the Office of the Solicitor General at 134 Amorsolo St.,
Legaspi Vill., Makati City and the Office of the Local Civil Registrar of Iligan City at
Quezon Ave., Pala-o, Iligan City.
SO ORDERED.
During the hearing, respondent testified thus:
First, she claims that her surname "Yu" was misspelled as "Yo". She has been using
"Yu" in all her school records and in her marriage certificate.2 She presented a
clearance from the National Bureau of Investigation (NBI)3 to further show the
consistency in her use of the surname "Yu".
Second, she claims that her fathers name in her birth record was written as "Yo Diu
To (Co Tian)" when it should have been "Yu Dio To (Co Tian)."
Third, her nationality was entered as Chinese when it should have been Filipino considering
that her father and mother never got married. Only her deceased father was Chinese, while
her mother is Filipina. She claims that her being a registered voter attests to the fact that she
is a Filipino citizen.
Finally, it was erroneously indicated in her birth certificate that she was a legitimate child when
she should have been described as illegitimate considering that her parents were never
married.

Placida Anto, respondents mother, testified that she is a Filipino citizen as her parents were
both Filipinos from Camiguin. She added that she and her daughters father were never
married because the latter had a prior subsisting marriage contracted in China.
In this connection, respondent presented a certification attested by officials of the local civil
registries of Iligan City and Kauswagan, Lanao del Norte that there is no record of marriage
between Placida Anto and Yu Dio To from 1948 to the present.
The Republic, through the City Prosecutor of Iligan City, did not present any evidence
although it actively participated in the proceedings by attending hearings and cross-examining
respondent and her witnesses.
On February 22, 2000, the trial court granted respondents petition and rendered judgment as
follows:
WHEREFORE, the foregoing premises considered, to set the records of the
petitioner straight and in their proper perspective, the petition is granted and the
Civil Registrar of Iligan City is directed to make the following corrections in the birth
records of the petitioner, to wit:
1. Her family name from "YO" to "YU";
2. Her fathers name from "YO DIU TO (CO TIAN)" to "YU DIOTO (CO
TIAN)";
3. Her status from "legitimate" to "illegitimate" by changing "YES" to "NO"
in answer to the question "LEGITIMATE?"; and,
4. Her citizenship from "Chinese" to "Filipino".
SO ORDERED.4
The Republic of the Philippines appealed the decision to the Court of Appeals which affirmed
the trial courts decision.5
Hence, this petition on the following assigned errors:
I
THE COURT OF APPEALS ERRED IN ORDERING THE CORRECTION OF THE
CITIZENSHIP OF RESPONDENT CHULE Y. LIM FROM "CHINESE" TO
"FILIPINO" DESPITE THE FACT THAT RESPONDENT NEVER DEMONSTRATED
ANY COMPLIANCE WITH THE LEGAL REQUIREMENTS FOR ELECTION OF
CITIZENSHIP.
II
THE COURT OF APPEALS ERRED IN ALLOWING RESPONDENT TO
CONTINUE USING HER FATHERS SURNAME DESPITE ITS FINDING THAT
RESPONDENT IS AN ILLEGITIMATE CHILD.6
To digress, it is just as well that the Republic did not cite as error respondents recourse to
Rule 108 of the Rules of Court to effect what indisputably are substantial corrections and
changes in entries in the civil register. To clarify, Rule 108 of the Revised Rules of Court
provides the procedure for cancellation or correction of entries in the civil registry. The
proceedings under said rule may either be summary or adversary in nature. If the correction
sought to be made in the civil register is clerical, then the procedure to be adopted is
summary. If the rectification affects the civil status, citizenship or nationality of a party, it is
47

deemed substantial, and the procedure to be adopted is adversary. This is our ruling in
Republic v. Valencia7 where we held that even substantial errors in a civil registry may be
corrected and the true facts established under Rule 108 provided the parties aggrieved by the
error avail themselves of the appropriate adversary proceeding. An appropriate adversary suit
or proceeding is one where the trial court has conducted proceedings where all relevant facts
have been fully and properly developed, where opposing counsel have been given opportunity
to demolish the opposite partys case, and where the evidence has been thoroughly weighed
and considered.8
As likewise observed by the Court of Appeals, we take it that the Republics failure to cite this
error amounts to a recognition that this case properly falls under Rule 108 of the Revised
Rules of Court considering that the proceeding can be appropriately classified as adversarial.
Instead, in its first assignment of error, the Republic avers that respondent did not comply with
the constitutional requirement of electing Filipino citizenship when she reached the age of
majority. It cites Article IV, Section 1(3) of the 1935 Constitution, which provides that the
citizenship of a legitimate child born of a Filipino mother and an alien father followed the
citizenship of the father, unless, upon reaching the age of majority, the child elected Philippine
citizenship.9 Likewise, the Republic invokes the provision in Section 1 of Commonwealth Act
No. 625, that legitimate children born of Filipino mothers may elect Philippine citizenship by
expressing such intention "in a statement to be signed and sworn to by the party concerned
before any officer authorized to administer oaths, and shall be filed with the nearest civil
registry. The said party shall accompany the aforesaid statement with the oath of allegiance to
the Constitution and the Government of the Philippines."10
Plainly, the above constitutional and statutory requirements of electing Filipino citizenship
apply only to legitimate children. These do not apply in the case of respondent who was
concededly an illegitimate child, considering that her Chinese father and Filipino mother were
never married. As such, she was not required to comply with said constitutional and statutory
requirements to become a Filipino citizen. By being an illegitimate child of a Filipino mother,
respondent automatically became a Filipino upon birth. Stated differently, she is a Filipino
since birth without having to elect Filipino citizenship when she reached the age of majority.
In Ching, Re: Application for Admission to the Bar,11 citing In re Florencio Mallare,12 we held:
Esteban Mallare, natural child of Ana Mallare, a Filipina, is therefore himself a
Filipino, and no other act would be necessary to confer on him all the rights and
privileges attached to Philippine citizenship (U.S. vs. Ong Tianse, 29 Phil. 332;
Santos Co vs. Government of the Philippine Islands, 42 Phil. 543; Serra vs.
Republic, L-4223, May 12, 1952; Sy Quimsuan vs. Republic, L-4693, Feb. 16, 1953;
Pitallano vs. Republic, L-5111, June 28, 1954). Neither could any act be taken on
the erroneous belief that he is a non-Filipino divest him of the citizenship privileges
to which he is rightfully entitled.13
This notwithstanding, the records show that respondent elected Filipino citizenship when she
reached the age of majority. She registered as a voter in Misamis Oriental when she was 18
years old.14 The exercise of the right of suffrage and the participation in election exercises
constitute a positive act of election of Philippine citizenship.15
In its second assignment of error, the Republic assails the Court of Appeals decision in
allowing respondent to use her fathers surname despite its finding that she is illegitimate.

The Republics submission is misleading. The Court of Appeals did not allow respondent to
use her fathers surname. What it did allow was the correction of her fathers misspelled
surname which she has been using ever since she can remember. In this regard, respondent
does not need a court pronouncement for her to use her fathers surname.
We agree with the Court of Appeals when it held:
Firstly, Petitioner-appellee is now 47 years old. To bar her at this time from using
her fathers surname which she has used for four decades without any known
objection from anybody, would only sow confusion. Concededly, one of the reasons
allowed for changing ones name or surname is to avoid confusion.
Secondly, under Sec. 1 of Commonwealth Act No. 142, the law regulating the use of
aliases, a person is allowed to use a name "by which he has been known since
childhood."
Thirdly, the Supreme Court has already addressed the same issue. In Pabellar v.
Rep. of the Phils.,16 we held:
Section 1 of Commonwealth Act No. 142, which regulates the use of aliases, allows
a person to use a name "by which he has been known since childhood" (Lim Hok
Albano v. Republic, 104 Phil. 795; People v. Uy Jui Pio, 102 Phil. 679; Republic v.
Taada, infra). Even legitimate children cannot enjoin the illegitimate children of
their father from using his surname (De Valencia v. Rodriguez, 84 Phil. 222).17
While judicial authority is required for a change of name or surname,18 there is no such
requirement for the continued use of a surname which a person has already been using since
childhood.19
The doctrine that disallows such change of name as would give the false impression of family
relationship remains valid but only to the extent that the proposed change of name would in
great probability cause prejudice or future mischief to the family whose surname it is that is
involved or to the community in general.20 In this case, the Republic has not shown that the Yu
family in China would probably be prejudiced or be the object of future mischief. In
respondents case, the change in the surname that she has been using for 40 years would
even avoid confusion to her community in general.
WHEREFORE, in view of the foregoing, the instant petition for review is DENIED. The
decision of the Court of Appeals in CA-G.R. CV No. 68893 dated May 29, 2002, is
AFFIRMED. Accordingly, the Civil Registrar of Iligan City is DIRECTED to make the following
corrections in the birth record of respondent Chule Y. Lim, to wit:
1. Her family name from "YO" to "YU";
2. Her fathers name from "YO DIU TO (CO TIAN)" to "YU DIOTO (CO TIAN)";
3. Her status from "legitimate" to "illegitimate" by changing "YES" to "NO" in answer
to the question "LEGITIMATE?"; and,
4. Her citizenship from "Chinese" to "Filipino".
SO ORDERED.

48

G.R. No. 183133


July 26, 2010
BALGAMELO CABILING MA, FELIX CABILING MA, JR., AND VALERIANO CABILING
MA, Petitioners,
vs.
COMMISSIONER ALIPIO F. FERNANDEZ, JR., ASSOCIATE COMMISSIONER ARTHEL B.
CARONOGAN, ASSOCIATE COMMISSIONER JOSE DL. CABOCHAN, ASSOCIATE
COMMISSIONER TEODORO B. DELARMENTE AND ASSOCIATE COMMISSIONER
FRANKLIN Z. LITTAUA, in their capacities as Chairman and Members of the Board of
Commissioners (Bureau of Immigration), and MAT G. CATRAL, Respondents.
DECISION
PEREZ, J.:
Should children born under the 1935 Constitution of a Filipino mother and an alien father, who
executed an affidavit of election of Philippine citizenship and took their oath of allegiance to
the government upon reaching the age of majority, but who failed to immediately file the
documents of election with the nearest civil registry, be considered foreign nationals subject to
deportation as undocumented aliens for failure to obtain alien certificates of registration?
Positioned upon the facts of this case, the question is translated into the inquiry whether or not
the omission negates their rights to Filipino citizenship as children of a Filipino mother, and
erase the years lived and spent as Filipinos.
The resolution of these questions would significantly mark a difference in the lives of herein
petitioners.
The Facts
Balgamelo Cabiling Ma (Balgamelo), Felix Cabiling Ma, Jr. (Felix, Jr.), Valeriano Cabiling Ma
(Valeriano), Lechi Ann Ma (Lechi Ann), Arceli Ma (Arceli), Nicolas Ma (Nicolas), and Isidro Ma
(Isidro) are the children of Felix (Yao Kong) Ma,1 a Taiwanese, and Dolores Sillona Cabiling, a
Filipina.2
Records reveal that petitioners Felix, Jr., Balgamelo and Valeriano were all born under aegis
of the 1935 Philippine Constitution in the years 1948, 1951, and 1957, respectively.3
They were all raised in the Philippines and have resided in this country for almost sixty (60)
years; they spent their whole lives, studied and received their primary and secondary
education in the country; they do not speak nor understand the Chinese language, have not
set foot in Taiwan, and do not know any relative of their father; they have not even traveled
abroad; and they have already raised their respective families in the Philippines.4
During their age of minority, they secured from the Bureau of Immigration their Alien
Certificates of Registration (ACRs). 5
Immediately upon reaching the age of twenty-one, they claimed Philippine citizenship in
accordance with Section 1(4), Article IV, of the 1935 Constitution, which provides that "(t)hose
whose mothers are citizens of the Philippines and, upon reaching the age of majority, elect
Philippine citizenship" are citizens of the Philippines. Thus, on 15 August 1969, Felix, Jr.
executed his affidavit of election of Philippine citizenship and took his oath of allegiance
before then Judge Jose L. Gonzalez, Municipal Judge, Surigao, Surigao del Norte.6 On 14
January 1972, Balgamelo did the same before Atty. Patrocinio C. Filoteo, Notary Public,
Surigao City, Surigao del Norte.7 In 1978, Valeriano took his oath of allegiance before then

Judge Salvador C. Sering, City Court of Surigao City, the fact of which the latter attested to in
his Affidavit of 7 March 2005.8
Having taken their oath of allegiance as Philippine citizens, petitioners, however, failed to
have the necessary documents registered in the civil registry as required under Section 1 of
Commonwealth Act No. 625 (An Act Providing the Manner in which the Option to Elect
Philippine Citizenship shall be Declared by a Person whose Mother is a Filipino Citizen). It
was only on 27 July 2005 or more than thirty (30) years after they elected Philippine
citizenship that Balgamelo and Felix, Jr. did so.9 On the other hand, there is no showing that
Valeriano complied with the registration requirement.
Individual certifications10 all dated 3 January 2005 issued by the Office of the City Election
Officer, Commission on Elections, Surigao City, show that all of them are registered voters of
Barangay Washington, Precinct No. 0015A since June 1997, and that records on previous
registrations are no longer available because of the mandatory general registration every ten
(10) years. Moreover, aside from exercising their right of suffrage, Balgamelo is one of the
incumbent Barangay Kagawads in Barangay Washington, Surigao City.11
Records further reveal that Lechi Ann and Arceli were born also in Surigao City in 195312 and
1959,13 respectively. The Office of the City Civil Registrar issued a Certification to the effect
that the documents showing that Arceli elected Philippine citizenship on 27 January 1986
were registered in its Office on 4 February 1986. However, no other supporting documents
appear to show that Lechi Ann initially obtained an ACR nor that she subsequently elected
Philippine citizenship upon reaching the age of majority. Likewise, no document exists that will
provide information on the citizenship of Nicolas and Isidro.
The Complaint
On 16 February 2004, the Bureau of Immigration received the Complaint-Affidavit14 of a
certain Mat G. Catral (Mr. Catral), alleging that Felix (Yao Kong) Ma and his seven (7) children
are undesirable and overstaying aliens. Mr. Catral, however, did not participate in the
proceedings, and the Ma family could not but believe that the complaint against them was
politically motivated because they strongly supported a candidate in Surigao City in the 2004
National and Local Elections.15
On 9 November 2004, the Legal Department of the Bureau of Immigration charged them for
violation of Sections 37(a)(7)16 and 45(e)17 of Commonwealth Act No. 613, otherwise known
as the Philippine Immigration Act of 1940, as amended. The Charge Sheet18 docketed as BSID.C. No. AFF-04-574 (OC-STF-04-09/23-1416) reads, in part:
That Respondents x x x, all Chinese nationals, failed and continuously failed to present any
valid document to show their respective status in the Philippines. They likewise failed to
produce documents to show their election of Philippines (sic) citizenship, hence,
undocumented and overstaying foreign nationals in the country.
That respondents, being aliens, misrepresent themselves as Philippine citizens in order to
evade the requirements of the immigration laws.
Ruling of the Board of Commissioners, Bureau of Immigration
After Felix Ma and his seven (7) children were afforded the opportunity to refute the
allegations, the Board of Commissioners (Board) of the Bureau of Immigration (BI), composed
of the public respondents, rendered a Judgment dated 2 February 2005 finding that Felix Ma
and his children violated Commonwealth Act No. 613, Sections 37(a)(7) and 45(e) in relation
49

to BI Memorandum Order Nos. ADD-01-031 and ADD-01-035 dated 6 and 22 August 2001,
respectively.19
The Board ruled that since they elected Philippine citizenship after the enactment of
Commonwealth Act No. 625, which was approved on 7 June 1941, they were governed by the
following rules and regulations:
1. Section 1 of Commonwealth Act No. 625, providing that the election of Philippine
citizenship embodied in a statement sworn before any officer authorized to
administer oaths and the oath of allegiance shall be filed with the nearest civil
registry;20 and Commission of Immigration and Deportation (CID, now Bureau of
Immigration [BI]) Circular dated 12 April 1954,21 detailing the procedural
requirements in the registration of the election of Philippine citizenship.
2. Memorandum Order dated 18 August 195622 of the CID, requiring the filing of a
petition for the cancellation of their alien certificate of registration with the CID, in
view of their election of Philippine citizenship;
3. Department of Justice (DOJ) Opinion No. 182, 19 August 1982; and DOJ
Guidelines, 27 March 1985, requiring that the records of the proceedings be
forwarded to the Ministry (now the Department) of Justice for final determination and
review.23
As regards the documentation of aliens in the Philippines, Administrative Order No. 1-93 of
the Bureau of Immigration24 requires that ACR, E-series, be issued to foreign nationals who
apply for initial registration, finger printing and issuance of an ACR in accordance with the
Alien Registration Act of 1950.25 According to public respondents, any foreign national found
in possession of an ACR other than the E-series shall be considered improperly documented
aliens and may be proceeded against in accordance with the Immigration Act of 1940 or the
Alien Registration Act of 1950, as amended.26
Supposedly for failure to comply with the procedure to prove a valid claim to Philippine
citizenship via election proceedings, public respondents concluded that Felix, Jr. Balgamelo,
Arceli, Valeriano and Lechi Ann are undocumented and/or improperly documented aliens. 27
Nicolas and Isidro, on the other hand, did not submit any document to support their claim that
they are Philippine citizens. Neither did they present any evidence to show that they are
properly documented aliens. For these reasons, public respondents likewise deemed them
undocumented and/or improperly documented aliens.28
The dispositive portion29 of the Judgment of 2 February 2005 reads:
1. Subject to the submission of appropriate clearances, summary deportation of
Felix (Yao Kong) Ma, Felix Ma, Jr., Balgamelo Ma, Valeriano Ma, Lechi Ann Ma,
Nicolas Ma, Arceli Ma and Isidro Ma, Taiwanese [Chinese], under C.A. No. 613,
Sections 37(a)(7), 45(e) and 38 in relation to BI M.O. Nos. ADD-01-031 and ADD01-035 dated 6 and 22 August 2001, respectively;
2. Issuance of a warrant of deportation against Felix (Yao Kong) Ma, Felix Ma, Jr.,
Balgamelo Ma, Valeriano Ma, Lechi Ann Ma, Nicolas Ma, Arceli Ma and Isidro Ma
under C.A. No. 613, Section 37(a);
3. Inclusion of the names of Felix (Yao Kong) Ma, Felix Ma, Jr., Balgamelo Ma,
Valeriano Ma, Lechi Ann Ma, Nicolas Ma, Arceli Ma and Isidro Ma in the
Immigration Blacklist; and

4. Exclusion from the Philippines of Felix (Yao Kong) Ma, Felix Ma, Jr., Balgamelo
Ma, Valeriano Ma, Lechi Ann Ma, Nicolas Ma, Arceli Ma and Isidro Ma under C.A.
No. 613, Section 29(a)(15). (Emphasis supplied.)
In its Resolution30 of 8 April 2005, public respondents partially reconsidered their Judgment of
2 February 2005. They were convinced that Arceli is an immigrant under Commonwealth Act
No. 613, Section 13(g).31 However, they denied the Motion for Reconsideration with respect to
Felix Ma and the rest of his children.32
Ruling of the Court of Appeals
On 3 May 2005, only Balgamelo, Felix, Jr., and Valeriano filed the Petition for Certiorari under
Rule 65 of the 1997 Rules of Civil Procedure before the Court of Appeals, which was
docketed as CA-G.R. SP No. 89532. They sought the nullification of the issuances of the
public respondents, to wit: (1) the Judgment dated 2 February 2005, ordering the summary
deportation of the petitioners, issuance of a warrant of deportation against them, inclusion of
their names in the Immigration Blacklist, and exclusion of the petitioners from the Philippines;
and (2) the Resolution dated 8 April 2005, denying the petitioners Motion for Reconsideration.
On 29 August 2007, the Court of Appeals dismissed the petition33 after finding that the
petitioners "failed to comply with the exacting standards of the law providing for the procedure
and conditions for their continued stay in the Philippines either as aliens or as its nationals."34
On 29 May 2008, it issued a Resolution35 denying the petitioners Motion for Reconsideration
dated 20 September 2007.
To reiterate, a persons continued and uninterrupted stay in the Philippines, his being a
registered voter or an elected public official cannot vest in him Philippine citizenship as the law
specifically lays down the requirements for acquisition of Philippine citizenship by election.
The prescribed procedure in electing Philippine citizenship is certainly not a tedious and
painstaking process. All that is required of the elector is to execute an affidavit of election of
Philippine citizenship and, thereafter, file the same with the nearest civil registry. The
constitutional mandate concerning citizenship must be adhered to strictly. Philippine
citizenship can never be treated like a commodity that can be claimed when needed and
suppressed when convenient. One who is privileged to elect Philippine citizenship has only an
inchoate right to such citizenship. As such, he should avail of the right with fervor, enthusiasm
and promptitude.36
Our Ruling
The 1935 Constitution declares as citizens of the Philippines those whose mothers are
citizens of the Philippines and elect Philippine citizenship upon reaching the age of majority.
The mandate states:
Section 1. The following are citizens of the Philippines:
(1) xxx;
xxxx
(4) Those whose mothers are citizens of the Philippines and, upon
reaching the age of majority, elect Philippine citizenship.37
In 1941, Commonwealth Act No. 625 was enacted. It laid down the manner of electing
Philippine citizenship, to wit:

50

Section 1. The option to elect Philippine citizenship in accordance with subsection (4), Section
1, Article IV, of the Constitution shall be expressed in a statement to be signed and sworn to
by the party concerned before any officer authorized to administer oaths, and shall be filed
with the nearest civil registry. The said party shall accompany the aforesaid statement with the
oath of allegiance to the Constitution and the Government of the Philippines.
The statutory formalities of electing Philippine citizenship are: (1) a statement of election
under oath; (2) an oath of allegiance to the Constitution and Government of the Philippines;
and (3) registration of the statement of election and of the oath with the nearest civil registry.
In Re:Application for Admission to the Philippine Bar, Vicente D. Ching,38 we determined the
meaning of the period of election described by phrase "upon reaching the age of majority."
Our references were the Civil Code of the Philippines, the opinions of the Secretary of Justice,
and the case of Cueco v. Secretary of Justice.39 We pronounced:
x x x [T]he 1935 Constitution and C.A. No. 625 did not prescribe a time period within which the
election of Philippine citizenship should be made. The 1935 Charter only provides that the
election should be made "upon reaching the age of majority." The age of majority then
commenced upon reaching twenty-one (21) years.40 In the opinions of the Secretary of Justice
on cases involving the validity of election of Philippine citizenship, this dilemma was resolved
by basing the time period on the decisions of this Court prior to the effectivity of the 1935
Constitution. In these decisions, the proper period for electing Philippine citizenship was, in
turn, based on the pronouncements of the Department of State of the United States
Government to the effect that the election should be made within a reasonable time after
attaining the age of majority.41 The phrase "reasonable time" has been interpreted to mean
that the elections should be made within three (3) years from reaching the age of majority.42
However, we held in Cue[n]co vs. Secretary of Justice,43 that the three (3) year period is not
an inflexible rule. We said:
It is true that this clause has been construed to mean a reasonable time after reaching the age
of majority, and that the Secretary of Justice has ruled that three (3) years is the reasonable
time to elect Philippine citizenship under the constitutional provision adverted to above, which
period may be extended under certain circumstances, as when the person concerned has
always considered himself a Filipino.
However, we cautioned in Cue[n]co that the extension of the option to elect Philippine
citizenship is not indefinite.
Regardless of the foregoing, petitioner was born on February 16, 1923. He became of age on
February 16, 1944. His election of citizenship was made on May 15, 1951, when he was over
twenty-eight (28) years of age, or over seven (7) years after he had reached the age of
majority. It is clear that said election has not been made "upon reaching the age of majority.44
We reiterated the above ruling in Go, Sr. v. Ramos,45 a case in which we adopted the findings
of the appellate court that the father of the petitioner, whose citizenship was in question, failed
to elect Philippine citizenship within the reasonable period of three (3) years upon reaching
the age of majority; and that "the belated submission to the local civil registry of the affidavit of
election and oath of allegiance x x x was defective because the affidavit of election was
executed after the oath of allegiance, and the delay of several years before their filing with the
proper office was not satisfactorily explained."46

In both cases, we ruled against the petitioners because they belatedly complied with all the
requirements. The acts of election and their registration with the nearest civil registry were all
done beyond the reasonable period of three years upon reaching the age of majority.
The instant case presents a different factual setting. Petitioners complied with the first and
second requirements upon reaching the age of majority. It was only the registration of the
documents of election with the civil registry that was belatedly done.
We rule that under the facts peculiar to the petitioners, the right to elect Philippine citizenship
has not been lost and they should be allowed to complete the statutory requirements for such
election.
Such conclusion, contrary to the finding of the Court of Appeals, is in line with our decisions in
In Re:Florencio Mallare,47 Co v. Electoral Tribunal of the House of Representatives,48 and
Re:Application for Admission to the Philippine Bar, Vicente D. Ching.49
In Mallare, Estebans exercise of the right of suffrage when he came of age was deemed to be
a positive act of election of Philippine citizenship.50 The Court of Appeals, however, said that
the case cannot support herein petitioners cause, pointing out that, unlike petitioner, Esteban
is a natural child of a Filipina, hence, no other act would be necessary to confer on him the
rights and privileges of a Filipino citizen,51 and that Esteban was born in 192952 prior to the
adoption of the 1935 Constitution and the enactment of Commonwealth Act No. 625.53
In the Co case, Jose Ong, Jr. did more than exercise his right of suffrage, as he established
his life here in the Philippines.54 Again, such circumstance, while similar to that of herein
petitioners, was not appreciated because it was ruled that any election of Philippine
citizenship on the part of Ong would have resulted in absurdity, because the law itself had
already elected Philippine citizenship for him55 as, apparently, while he was still a minor, a
certificate of naturalization was issued to his father.56
In Ching, it may be recalled that we denied his application for admission to the Philippine Bar
because, in his case, all the requirements, to wit: (1) a statement of election under oath; (2) an
oath of allegiance to the Constitution and Government of the Philippines; and (3) registration
of the statement of election and of the oath with the nearest civil registry were complied with
only fourteen (14) years after he reached the age of majority. Ching offered no reason for the
late election of Philippine citizenship.57
In all, the Court of Appeals found the petitioners argument of good faith and "informal
election" unacceptable and held:
Their reliance in the ruling contained in Re:Application for Admission to the Philippine Bar,
Vicente D. Ching, [which was decided on 1 October 1999], is obviously flawed. It bears
emphasis that the Supreme Court, in said case, did not adopt the doctrine laid down in In Re:
Florencio Mallare. On the contrary, the Supreme Court was emphatic in pronouncing that "the
special circumstances invoked by Ching, i.e., his continuous and uninterrupted stay in the
Philippines and his being a certified public accountant, a registered voter and a former elected
public official, cannot vest in him Philippine citizenship as the law specifically lays down the
requirements for acquisition of Philippine citizenship by election.58
We are not prepared to state that the mere exercise of suffrage, being elected public official,
continuous and uninterrupted stay in the Philippines, and other similar acts showing exercise
of Philippine citizenship can take the place of election of citizenship. What we now say is that
where, as in petitioners case, the election of citizenship has in fact been done and
51

documented within the constitutional and statutory timeframe, the registration of the
documents of election beyond the frame should be allowed if in the meanwhile positive acts of
citizenship have publicly, consistently, and continuously been done. The actual exercise of
Philippine citizenship, for over half a century by the herein petitioners, is actual notice to the
Philippine public which is equivalent to formal registration of the election of Philippine
citizenship.
For what purpose is registration?
In Pascua v. Court of Appeals,59 we elucidated the principles of civil law on registration:
To register is to record or annotate. American and Spanish authorities are unanimous on the
meaning of the term "to register" as "to enter in a register; to record formally and distinctly; to
enroll; to enter in a list."60 In general, registration refers to any entry made in the books of the
registry, including both registration in its ordinary and strict sense, and cancellation,
annotation, and even the marginal notes. In strict acceptation, it pertains to the entry made in
the registry which records solemnly and permanently the right of ownership and other real
rights.61 Simply stated, registration is made for the purpose of notification.62
Actual knowledge may even have the effect of registration as to the person who has
knowledge thereof. Thus, "[i]ts purpose is to give notice thereof to all persons (and it) operates
as a notice of the deed, contract, or instrument to others."63 As pertinent is the holding that
registration "neither adds to its validity nor converts an invalid instrument into a valid one
between the parties."64 It lays emphasis on the validity of an unregistered document.
Comparable jurisprudence may be consulted.
In a contract of partnership, we said that the purpose of registration is to give notice to third
parties; that failure to register the contract does not affect the liability of the partnership and of
the partners to third persons; and that neither does such failure affect the partnerships
juridical personality.65 An unregistered contract of partnership is valid as among the partners,
so long as it has the essential requisites, because the main purpose of registration is to give
notice to third parties, and it can be assumed that the members themselves knew of the
contents of their contract.66 The non-registration of a deed of donation does not also affect its
validity. Registration is not a requirement for the validity of the contract as between the
parties, for the effect of registration serves chiefly to bind third persons.67
Likewise relevant is the pronouncement that registration is not a mode of acquiring a right. In
an analogous case involving an unrecorded deed of sale, we reiterated the settled rule that
registration is not a mode of acquiring ownership.
Registration does not confer ownership. It is not a mode of acquiring dominion, but only a
means of confirming the fact of its existence with notice to the world at large.68
Registration, then, is the confirmation of the existence of a fact. In the instant case,
registration is the confirmation of election as such election. It is not the registration of the act
of election, although a valid requirement under Commonwealth Act No. 625, that will confer
Philippine citizenship on the petitioners. It is only a means of confirming the fact that
citizenship has been claimed.
Indeed, we even allow the late registration of the fact of birth and of marriage.69 Thus, has it
been admitted through existing rules that the late registration of the fact of birth of a child does
not erase the fact of birth. Also, the fact of marriage cannot be declared void solely because of
the failure to have the marriage certificate registered with the designated government agency.

Notably, the petitioners timely took their oath of allegiance to the Philippines. This was a
serious undertaking. It was commitment and fidelity to the state coupled with a pledge "to
renounce absolutely and forever all allegiance" to any other state. This was unqualified
acceptance of their identity as a Filipino and the complete disavowal of any other nationality.
Petitioners have passed decades of their lives in the Philippines as Filipinos. Their present
status having been formed by their past, petitioners can no longer have any national identity
except that which they chose upon reaching the age of reason.
Corollary to this fact, we cannot agree with the view of the Court of Appeals that since the
ACR presented by the petitioners are no longer valid on account of the new requirement to
present an E-series ACR, they are deemed not properly documented.70 On the contrary,
petitioners should not be expected to secure E-series ACR because it would be inconsistent
with the election of citizenship and its constructive registration through their acts made public,
among others, their exercise of suffrage, election as public official, and continued and
uninterrupted stay in the Philippines since birth. The failure to register as aliens is, obviously,
consistent with petitioners election of Philippine citizenship.
The leanings towards recognition of the citizenship of children of Filipino mothers have been
indicated not alone by the jurisprudence that liberalized the requirement on time of election,
and recognized positive acts of Philippine citizenship.
The favor that is given to such children is likewise evident in the evolution of the constitutional
provision on Philippine citizenship.
Thus, while the 1935 Constitution requires that children of Filipino mothers elect Philippine
citizenship upon reaching their age of majority,71 upon the effectivity of the 1973 Constitution,
they automatically become Filipinos72 and need not elect Philippine citizenship upon reaching
the age of majority. The 1973 provision reads:
Section 1. The following are citizens of the Philippines:
(1) xxx.
(2) Those whose fathers and mothers are citizens of the Philippines.73
Better than the relaxation of the requirement, the 1987 Constitution now classifies them as
natural-born citizens upon election of Philippine citizenship. Thus, Sec. 2, Article IV thereof
provides:
Section 2. Natural-born citizens are those who are citizens of the Philippines from birth without
having to perform any act to acquire or perfect their Philippine citizenship. Those who elect
Philippine citizenship in accordance with paragraph (3), Section 1 hereof74 shall be deemed
natural-born citizens. (Emphasis supplied.)
The constitutional bias is reflected in the deliberations of the 1986 Constitutional Commission.
MR. CONCEPCION. x x x.
xxxx
x x x x As regards those born of Filipino mothers, the 1935 Constitution merely gave them the
option to choose Philippine citizenship upon reaching the age of majority, even, apparently, if
the father were an alien or unknown. Upon the other hand, under the 1973 Constitution,
children of mixed marriages involving an alien father and a Filipino mother are Filipino
citizens, thus liberalizing the counterpart provision in the 1935 Constitution by dispensing with
the need to make a declaration of intention upon reaching the age of majority. I understand
52

that the committee would further liberalize this provision of the 1935 Constitution. The
Committee seemingly proposes to further liberalize the policy of the 1935 Constitution by
making those who became citizens of the Philippines through a declaration of intention to
choose their mothers citizenship upon reaching the majority age by declaring that such
children are natural-born citizens of the Philippines.75
xxxx
xxx Why does the draft resolution adopt the provision of the 1973 Constitution and not that of
the 1935? 76
xxxx
FR. BERNAS. x x x Precisely, the reason behind the modification of the 1935 rule on
citizenship was a recognition of the fact that it reflected a certain male chauvinism, and it was
for the purpose of remedying that this proposed provision was put in. The idea was that we
should not penalize the mother of a child simply because she fell in love with a foreigner. Now,
the question on what citizenship the child would prefer arises. We really have no way of
guessing the preference of the infant. But if we recognize the right of the child to choose, then
let him choose when he reaches the age of majority. I think dual citizenship is just a reality
imposed on us because we have no control of the laws on citizenship of other countries. We
recognize a child of a Filipino mother. But whether or not she is considered a citizen of
another country is something completely beyond our control. But certainly it is within the
jurisdiction of the Philippine government to require that [at] a certain point, a child be made to
choose. But I do not think we should penalize the child before he is even able to choose. I
would, therefore, support the retention of the modification made in 1973 of the male
chauvinistic rule of the 1935 Constitution.77
xxxx
MR. REGALADO. With respect to a child who became a Filipino citizen by election, which the
Committee is now planning to consider a natural-born citizen, he will be so the moment he
opts for Philippine citizenship. Did the Committee take into account the fact that at the time of
birth, all he had was just an inchoate right to choose Philippine citizenship, and yet, by
subsequently choosing Philippine citizenship, it would appear that his choice retroacted to the
date of his birth so much so that under the Gentlemans proposed amendment, he would be a
natural-born citizen?78
FR. BERNAS. But the difference between him and the natural-born who lost his status is that
the natural-born who lost his status, lost it voluntarily; whereas, this individual in the situation
contemplated in Section 1, paragraph 3 never had the chance to choose.79
xxxx
[on the period within which to elect Philippine citizenship]
MR. RODRIGO. [T]his provision becomes very, very important because his election of
Philippine citizenship makes him not only a Filipino citizen but a natural-born Filipino citizen,
entitling him to run for Congress, to be a Justice of the Supreme Court x x x.80
We are guided by this evolvement from election of Philippine citizenship upon reaching the
age of majority under the 1935 Philippine Constitution to dispensing with the election
requirement under the 1973 Philippine Constitution to express classification of these children
as natural-born citizens under the 1987 Constitution towards the conclusion that the omission

of the 1941 statutory requirement of registration of the documents of election should not result
in the obliteration of the right to Philippine citizenship.1avvphi1
Having a Filipino mother is permanent. It is the basis of the right of the petitioners to elect
Philippine citizenship. Petitioners elected Philippine citizenship in form and substance. The
failure to register the election in the civil registry should not defeat the election and resultingly
negate the permanent fact that they have a Filipino mother. The lacking requirements may still
be complied with subject to the imposition of appropriate administrative penalties, if any. The
documents they submitted supporting their allegations that they have already registered with
the civil registry, although belatedly, should be examined for validation purposes by the
appropriate agency, in this case, the Bureau of Immigration. Other requirements embodied in
the administrative orders and other issuances of the Bureau of Immigration and the
Department of Justice shall be complied with within a reasonable time.
WHEREFORE, the Decision dated 29 August 2007, and the Resolution dated 29 May 2008 of
the Court of Appeals in CA-G.R. SP No. 89532 affirming the Judgment dated 2 February
2005, and the Resolution dated 8 April 2005 of the Bureau of Immigration in BSI-D.C. No.
AFF-04-574 OC-STF-04-09/23-1416 are hereby SET ASIDE with respect to petitioners
Balgamelo Cabiling Ma, Felix Cabiling Ma, Jr., and Valeriano Cabiling Ma. Petitioners are
given ninety (90) days from notice within which to COMPLY with the requirements of the
Bureau of Immigration embodied in its Judgment of 2 February 2005. The Bureau of
Immigration shall ENSURE that all requirements, including the payment of their financial
obligations to the state, if any, have been complied with subject to the imposition of
appropriate administrative fines; REVIEW the documents submitted by the petitioners; and
ACT thereon in accordance with the decision of this Court.
SO ORDERED.

53

G.R. No. 187567


February 15, 2012
THE REPUBLIC OF THE PHILIPPINES, Petitioner,
vs.
NORA FE SAGUN, Respondent.
DECISION
VILLARAMA, JR., J.:
Before us is a petition for review on certiorari filed by the Solicitor General on behalf of the
Republic of the Philippines, seeking the reversal of the April 3, 2009 Decision1 of the Regional
Trial Court (RTC), Branch 3, of Baguio City in Spcl. Pro. Case No. 17-R. The RTC granted the
petition2 filed by respondent Nora Fe Sagun entitled "In re: Judicial Declaration of Election of
Filipino Citizenship, Nora Fe Sagun v. The Local Civil Registrar of Baguio City."
The facts follow:
Respondent is the legitimate child of Albert S. Chan, a Chinese national, and Marta Borromeo,
a Filipino citizen. She was born on August 8, 1959 in Baguio City3 and did not elect Philippine
citizenship upon reaching the age of majority. In 1992, at the age of 33 and after getting
married to Alex Sagun, she executed an Oath of Allegiance4 to the Republic of the Philippines.
Said document was notarized by Atty. Cristeta Leung on December 17, 1992, but was not
recorded and registered with the Local Civil Registrar of Baguio City.
Sometime in September 2005, respondent applied for a Philippine passport. Her application
was denied due to the citizenship of her father and there being no annotation on her birth
certificate that she has elected Philippine citizenship. Consequently, she sought a judicial
declaration of her election of Philippine citizenship and prayed that the Local Civil Registrar of
Baguio City be ordered to annotate the same on her birth certificate.
In her petition, respondent averred that she was raised as a Filipino, speaks Ilocano and
Tagalog fluently and attended local schools in Baguio City, including Holy Family Academy
and the Saint Louis University. Respondent claimed that despite her part-Chinese ancestry,
she always thought of herself as a Filipino. She is a registered voter of Precinct No. 0419A of
Barangay Manuel A. Roxas in Baguio City and had voted in local and national elections as
shown in the Voter Certification5 issued by Atty. Maribelle Uminga of the Commission on
Elections of Baguio City.
She asserted that by virtue of her positive acts, she has effectively elected Philippine
citizenship and such fact should be annotated on her record of birth so as to entitle her to the
issuance of a Philippine passport.
On August 7, 2007, the Office of the Solicitor General (OSG) entered its appearance as
counsel for the Republic of the Philippines and authorized the City Prosecutor of Baguio City
to appear in the above mentioned case.6 However, no comment was filed by the City
Prosecutor.
After conducting a hearing, the trial court rendered the assailed Decision on April 3, 2009
granting the petition and declaring respondent a Filipino citizen. The fallo of the decision
reads:
WHEREFORE, the instant petition is hereby GRANTED. Petitioner Nora Fe Sagun y Chan is
hereby DECLARED [a] FILIPINO CITIZEN, having chosen or elected Filipino citizenship.

Upon payment of the required fees, the Local Civil Registrar of Baguio City is hereby directed
to annotate [on] her birth certificate, this judicial declaration of Filipino citizenship of said
petitioner.
IT IS SO ORDERED.7
Contending that the lower court erred in so ruling, petitioner, through the OSG, directly filed
the instant recourse via a petition for review on certiorari before us. Petitioner raises the
following issues:
I
Whether or not an action or proceeding for judicial declaration of Philippine
citizenship is procedurally and jurisdictionally permissible; and,
II
Whether or not an election of Philippine citizenship, made twelve (12) years after
reaching the age of majority, is considered to have been made "within a reasonable
time" as interpreted by jurisprudence.8
Petitioner argues that respondents petition before the RTC was improper on two counts: for
one, law and jurisprudence clearly contemplate no judicial action or proceeding for the
declaration of Philippine citizenship; and for another, the pleaded registration of the oath of
allegiance with the local civil registry and its annotation on respondents birth certificate are
the ministerial duties of the registrar; hence, they require no court order. Petitioner asserts that
respondents petition before the trial court seeking a judicial declaration of her election of
Philippine citizenship undeniably entails a determination and consequent declaration of her
status as a Filipino citizen which is not allowed under our legal system. Petitioner also argues
that if respondents intention in filing the petition is ultimately to have her oath of allegiance
registered with the local civil registry and annotated on her birth certificate, then she does not
have to resort to court proceedings.
Petitioner further argues that even assuming that respondents action is sanctioned, the trial
court erred in finding respondent as having duly elected Philippine citizenship since her
purported election was not in accordance with the procedure prescribed by law and was not
made within a "reasonable time." Petitioner points out that while respondent executed an oath
of allegiance before a notary public, there was no affidavit of her election of Philippine
citizenship. Additionally, her oath of allegiance which was not registered with the nearest local
civil registry was executed when she was already 33 years old or 12 years after she reached
the age of majority. Accordingly, it was made beyond the period allowed by law.
In her Comment,9 respondent avers that notwithstanding her failure to formally elect Filipino
citizenship upon reaching the age of majority, she has in fact effectively elected Filipino
citizenship by her performance of positive acts, among which is the exercise of the right of
suffrage. She claims that she had voted and participated in all local and national elections
from the time she was of legal age. She also insists that she is a Filipino citizen despite the
fact that her "election" of Philippine citizenship was delayed and unregistered.
In reply,10 petitioner argues that the special circumstances invoked by respondent, like her
continuous and uninterrupted stay in the Philippines, her having been educated in schools in
the country, her choice of staying here despite the naturalization of her parents as American
citizens, and her being a registered voter, cannot confer on her Philippine citizenship as the
law specifically provides the requirements for acquisition of Philippine citizenship by election.
54

Essentially, the issues for our resolution are: (1) whether respondents petition for declaration
of election of Philippine citizenship is sanctioned by the Rules of Court and jurisprudence; (2)
whether respondent has effectively elected Philippine citizenship in accordance with the
procedure prescribed by law.
The petition is meritorious.
At the outset, it is necessary to stress that a direct recourse to this Court from the decisions,
final resolutions and orders of the RTC may be taken where only questions of law are raised
or involved. There is a question of law when the doubt or difference arises as to what the law
is on a certain state of facts, which does not call for an examination of the probative value of
the evidence presented by the parties-litigants. On the other hand, there is a question of fact
when the doubt or controversy arises as to the truth or falsity of the alleged facts. Simply put,
when there is no dispute as to fact, the question of whether the conclusion drawn therefrom is
correct or not, is a question of law.11
In the present case, petitioner assails the propriety of the decision of the trial court declaring
respondent a Filipino citizen after finding that respondent was able to substantiate her election
of Filipino citizenship. Petitioner contends that respondents petition for judicial declaration of
election of Philippine citizenship is procedurally and jurisdictionally impermissible. Verily,
petitioner has raised questions of law as the resolution of these issues rest solely on what the
law provides given the attendant circumstances.
In granting the petition, the trial court stated:
This Court believes that petitioner was able to fully substantiate her petition regarding her
election of Filipino citizenship, and the Local Civil Registrar of Baguio City should be ordered
to annotate in her birth certificate her election of Filipino citizenship. This Court adds that the
petitioners election of Filipino citizenship should be welcomed by this country and people
because the petitioner has the choice to elect citizenship of powerful countries like the United
States of America and China, however, petitioner has chosen Filipino citizenship because she
grew up in this country, and has learned to love the Philippines. Her choice of electing Filipino
citizenship is, in fact, a testimony that many of our people still wish to live in the Philippines,
and are very proud of our country.
WHEREFORE, the instant petition is hereby GRANTED. Petitioner Nora Fe Sagun y Chan is
hereby DECLARED as FILIPINO CITIZEN, having chosen or elected Filipino citizenship.12
For sure, this Court has consistently ruled that there is no proceeding established by law, or
the Rules for the judicial declaration of the citizenship of an individual.13 There is no specific
legislation authorizing the institution of a judicial proceeding to declare that a given person is
part of our citizenry.14 This was our ruling in Yung Uan Chu v. Republic15 citing the early case
of Tan v. Republic of the Philippines,16 where we clearly stated:
Under our laws, there can be no action or proceeding for the judicial declaration of the
citizenship of an individual. Courts of justice exist for settlement of justiciable controversies,
which imply a given right, legally demandable and enforceable, an act or omission violative of
said right, and a remedy, granted or sanctioned by law, for said breach of right. As an incident
only of the adjudication of the rights of the parties to a controversy, the court may pass upon,
and make a pronouncement relative to their status. Otherwise, such a pronouncement is
beyond judicial power. x x x

Clearly, it was erroneous for the trial court to make a specific declaration of respondents
Filipino citizenship as such pronouncement was not within the courts competence.
As to the propriety of respondents petition seeking a judicial declaration of election of
Philippine citizenship, it is imperative that we determine whether respondent is required under
the law to make an election and if so, whether she has complied with the procedural
requirements in the election of Philippine citizenship.
When respondent was born on August 8, 1959, the governing charter was the 1935
Constitution, which declares as citizens of the Philippines those whose mothers are citizens of
the Philippines and elect Philippine citizenship upon reaching the age of majority. Sec. 1, Art.
IV of the 1935 Constitution reads:
Section 1. The following are citizens of the Philippines:
xxxx
(4) Those whose mothers are citizens of the Philippines and, upon reaching the age of
majority, elect Philippine citizenship.
Under Article IV, Section 1(4) of the 1935 Constitution, the citizenship of a legitimate child
born of a Filipino mother and an alien father followed the citizenship of the father, unless,
upon reaching the age of majority, the child elected Philippine citizenship. The right to elect
Philippine citizenship was recognized in the 1973 Constitution when it provided that "[t]hose
who elect Philippine citizenship pursuant to the provisions of the Constitution of nineteen
hundred and thirty-five" are citizens of the Philippines.17 Likewise, this recognition by the 1973
Constitution was carried over to the 1987 Constitution which states that "[t]hose born before
January 17, 1973 of Filipino mothers, who elect Philippine citizenship upon reaching the age
of majority" are Philippine citizens.18 It should be noted, however, that the 1973 and 1987
Constitutional provisions on the election of Philippine citizenship should not be understood as
having a curative effect on any irregularity in the acquisition of citizenship for those covered by
the 1935 Constitution. If the citizenship of a person was subject to challenge under the old
charter, it remains subject to challenge under the new charter even if the judicial challenge
had not been commenced before the effectivity of the new Constitution.19
Being a legitimate child, respondents citizenship followed that of her father who is Chinese,
unless upon reaching the age of majority, she elects Philippine citizenship. It is a settled rule
that only legitimate children follow the citizenship of the father and that illegitimate children are
under the parental authority of the mother and follow her nationality.20 An illegitimate child of
Filipina need not perform any act to confer upon him all the rights and privileges attached to
citizens of the Philippines; he automatically becomes a citizen himself.21 But in the case of
respondent, for her to be considered a Filipino citizen, she must have validly elected
Philippine citizenship upon reaching the age of majority.
Commonwealth Act (C.A.) No. 625,22 enacted pursuant to Section 1(4), Article IV of the 1935
Constitution, prescribes the procedure that should be followed in order to make a valid
election of Philippine citizenship, to wit:
Section 1. The option to elect Philippine citizenship in accordance with subsection (4),
[S]ection 1, Article IV, of the Constitution shall be expressed in a statement to be signed and
sworn to by the party concerned before any officer authorized to administer oaths, and shall
be filed with the nearest civil registry. The said party shall accompany the aforesaid statement
with the oath of allegiance to the Constitution and the Government of the Philippines.
55

Based on the foregoing, the statutory formalities of electing Philippine citizenship are: (1) a
statement of election under oath; (2) an oath of allegiance to the Constitution and Government
of the Philippines; and (3) registration of the statement of election and of the oath with the
nearest civil registry.23
Furthermore, no election of Philippine citizenship shall be accepted for registration under C.A.
No. 625 unless the party exercising the right of election has complied with the requirements of
the Alien Registration Act of 1950. In other words, he should first be required to register as an
alien.24 Pertinently, the person electing Philippine citizenship is required to file a petition with
the Commission of Immigration and Deportation (now Bureau of Immigration) for the
cancellation of his alien certificate of registration based on his aforesaid election of Philippine
citizenship and said Office will initially decide, based on the evidence presented the validity or
invalidity of said election.25 Afterwards, the same is elevated to the Ministry (now Department)
of Justice for final determination and review.261wphi1
It should be stressed that there is no specific statutory or procedural rule which authorizes the
direct filing of a petition for declaration of election of Philippine citizenship before the courts.
The special proceeding provided under Section 2, Rule 108 of the Rules of Court on
Cancellation or Correction of Entries in the Civil Registry, merely allows any interested party to
file an action for cancellation or correction of entry in the civil registry, i.e., election, loss and
recovery of citizenship, which is not the relief prayed for by the respondent.
Be that as it may, even if we set aside this procedural infirmity, still the trial courts conclusion
that respondent duly elected Philippine citizenship is erroneous since the records undisputably
show that respondent failed to comply with the legal requirements for a valid election.
Specifically, respondent had not executed a sworn statement of her election of Philippine
citizenship. The only documentary evidence submitted by respondent in support of her claim
of alleged election was her oath of allegiance, executed 12 years after she reached the age of
majority, which was unregistered. As aptly pointed out by the petitioner, even assuming
arguendo that respondents oath of allegiance suffices, its execution was not within a
reasonable time after respondent attained the age of majority and was not registered with the
nearest civil registry as required under Section 1 of C.A. No. 625. The phrase "reasonable
time" has been interpreted to mean that the election should be made generally within three (3)
years from reaching the age of majority.27 Moreover, there was no satisfactory explanation
proffered by respondent for the delay and the failure to register with the nearest local civil
registry.
Based on the foregoing circumstances, respondent clearly failed to comply with the procedural
requirements for a valid and effective election of Philippine citizenship. Respondent cannot
assert that the exercise of suffrage and the participation in election exercises constitutes a
positive act of election of Philippine citizenship since the law specifically lays down the
requirements for acquisition of citizenship by election. The mere exercise of suffrage,
continuous and uninterrupted stay in the Philippines, and other similar acts showing exercise
of Philippine citizenship cannot take the place of election of Philippine citizenship. Hence,
respondent cannot now be allowed to seek the intervention of the court to confer upon her
Philippine citizenship when clearly she has failed to validly elect Philippine citizenship. As we
held in Ching,28 the prescribed procedure in electing Philippine citizenship is certainly not a
tedious and painstaking process. All that is required of the elector is to execute an affidavit of
election of Philippine citizenship and, thereafter, file the same with the nearest civil registry.

Having failed to comply with the foregoing requirements, respondents petition before the trial
court must be denied.
WHEREFORE, the petition is GRANTED. The Decision dated April 3, 2009 of the Regional
Trial Court, Branch 3 of Baguio City in Spcl. Pro. Case No. 17-R is REVERSED and SET
ASIDE. The petition for judicial declaration of election of Philippine citizenship filed by
respondent Nora Fe Sagun is hereby DISMISSED for lack of merit.
No costs.
SO ORDERED.

56

G.R. No. 177721


July 3, 2007
KILOSBAYAN FOUNDATION AND BANTAY KATARUNGAN FOUNDATION, petitioners,
vs.
EXECUTIVE SECRETARY EDUARDO R. ERMITA; SANDIGANBAYAN JUSTICE
GREGORY S. ONG, respondents.
DECISION
AZCUNA, J.:
Filed on May 23, 2007 was this petition for certiorari under Rule 65 of the Rules of Court.
Petitioners are peoples and/or non-governmental organizations engaged in public and civic
causes aimed at protecting the peoples rights to self-governance and justice.
Respondent Executive Secretary is the head of the Office of the President and is in charge of
releasing presidential appointments including those of Supreme Court Justices.
Respondent Gregory S. Ong is allegedly the party whose appointment would fill up the
vacancy in this Court.
Petitioners allege that:
On May 16, 2007, respondent Executive Secretary, in representation of the Office of the
President, announced an appointment in favor of respondent Gregory S. Ong as Associate
Justice of the Supreme Court to fill up the vacancy created by the retirement on April 28, 2007
of Associate Justice Romeo J. Callejo, Sr. The appointment was reported the following day,
May 17, 2007, by the major daily publications.
On May 18, 2007, the major daily publications reported that the appointment was "recalled" or
"held in abeyance" by Malacaang in view of the question relating to the citizenship of
respondent Gregory S. Ong. There is no indication whatever that the appointment has been
cancelled by the Office of the President.
On May 19, 2007, the major daily publications reported that respondent Executive Secretary
stated that the appointment is "still there except that the validation of the issue is being done
by the Judicial and Bar Council (JBC)."
Petitioners contend that the appointment extended to respondent Ong through respondent
Executive Secretary is patently unconstitutional, arbitrary, whimsical and issued with grave
abuse of discretion amounting to lack of jurisdiction.
Petitioners claim that respondent Ong is a Chinese citizen, that this fact is plain and
incontestable, and that his own birth certificate indicates his Chinese citizenship. Petitioners
attached a copy of said birth certificate as Annex "H" to the petition. The birth certificate,
petitioners add, reveals that at the time of respondent Ongs birth on May 25, 1953, his father
was Chinese and his mother was also Chinese.
Petitioners invoke the Constitution:
Section 7 (1) of Article VIII of the 1987 Constitution provides that "No person shall
be appointed Member of the Supreme Court or any lower collegiate court unless he
is a natural-born citizen of the Philippines." Sec. 2 of Art. IV defines "natural-born
citizens as those who are citizens of the Philippines from birth without having to
perform any act to acquire or perfect their Philippine Citizenship."1

Petitioners maintain that even if it were granted that eleven years after respondent Ongs birth
his father was finally granted Filipino citizenship by naturalization, that, by itself, would not
make respondent Ong a natural-born Filipino citizen.
Petitioners further argue that respondent Ongs birth certificate speaks for itself and it states
his nationality as "Chinese" at birth. They invoke the Civil Code:
Article 410 of the Civil Code provides that "[t]he books making up the civil register and all
documents relating thereto x x x shall be prima facie evidence of the facts therein contained."
Therefore, the entry in Ongs birth certificate indicating his nationality as Chinese is prima
facie evidence of the fact that Ongs citizenship at birth is Chinese.
Article 412 of the Civil Code also provides that "[N]o entry in a civil register shall be
changed or corrected without a judicial order." Thus, as long as Ongs birth
certificate is not changed by a judicial order, the Judicial & Bar Council, as well as
the whole world, is bound by what is stated in his birth certificate.2
This birth certificate, petitioners assert, prevails over respondent Ongs new
Identification Certificate issued by the Bureau of Immigration dated October 16,
1996, stating that he is a natural-born Filipino and over the opinion of then Secretary
of Justice Teofisto Guingona that he is a natural-born Filipino. They maintain that
the Department of Justice (DOJ) does not have the power or authority to alter
entries in a birth certificate; that respondent Ongs old Identification Certificate did
not declare that he is a natural-born Filipino; and that respondent Ongs remedy is
an action to correct his citizenship as it appears in his birth certificate.
Petitioners thereupon pray that a writ of certiorari be issued annulling the appointment issued
to respondent Ong as Associate Justice of this Court.
Subsequently, on May 24, 2007, petitioners filed an Urgent Motion for the Issuance of a
Temporary Restraining Order (TRO), praying that a TRO be issued, in accordance with the
Rules of Court, to prevent and restrain respondent Executive Secretary from releasing the
appointment of respondent Ong, and to prevent and restrain respondent Ong from assuming
the office and discharging the functions of Associate Justice of this Court.
The Court required respondents to Comment on the petition.
Respondent Executive Secretary accordingly filed his Comment, essentially stating that the
appointment of respondent Ong as Associate Justice of this Court on May 16, 2007 was made
by the President pursuant to the powers vested in her by Article VIII, Section 9 of the
Constitution, thus:
SEC. 9. The Members of the Supreme Court and Judges of lower courts shall be
appointed by the President from a list of at least three nominees prepared by the
Judicial and Bar Council for every vacancy. Such appointments need no
confirmation.
Respondent Executive Secretary added that the President appointed respondent Ong from
among the list of nominees who were duly screened by and bore the imprimatur of the JBC
created under Article VIII, Section 8 of the Constitution. Said respondent further stated: "The
appointment, however, was not released, but instead, referred to the JBC for validation of
respondent Ongs citizenship."3 To date, however, the JBC has not received the referral.
Supporting the Presidents action and respondent Ongs qualifications, respondent Executive
Secretary submits that:
57

1. The President did not gravely abuse her discretion as she appointed a person,
duly nominated by the JBC, which passed upon the appointees qualifications.
2. Justice Gregory S. Ong is a natural-born citizen as determined by the Bureau of
Immigration and affirmed by the Department of Justice, which have the authority
and jurisdiction to make determination on matters of citizenship.
3. Undisputed evidence disclosed that respondent Ong is a natural-born citizen.
4. Petitioners are not entitled to a temporary restraining order.4
Respondent Ong submitted his Comment with Opposition, maintaining that he is a naturalborn Filipino citizen; that petitioners have no standing to file the present suit; and that the
issue raised ought to be addressed to the JBC as the Constitutional body mandated to review
the qualifications of those it recommends to judicial posts. Furthermore, the petitioners in his
view failed to include the President who is an indispensable party as the one who extended
the appointment.
As to his citizenship, respondent Ong traces his ancestral lines to one Maria Santos of
Malolos, Bulacan, born on November 25, 1881, who was allegedly a Filipino citizen5 who
married Chan Kin, a Chinese citizen; that these two had a son, Juan Santos; that in 1906
Chan Kin died in China, as a result of which Maria Santos reverted to her Filipino citizenship;
that at that time Juan Santos was a minor; that Juan Santos thereby also became a Filipino
citizen;6 that respondent Ongs mother, Dy Guiok Santos, is the daughter of the spouses Juan
Santos and Sy Siok Hian, a Chinese citizen, who were married in 1927; that, therefore,
respondents mother was a Filipino citizen at birth; that Dy Guiok Santos later married a
Chinese citizen, Eugenio Ong Han Seng, thereby becoming a Chinese citizen; that when
respondent Ong was eleven years old his father, Eugenio Ong Han Seng, was naturalized,
and as a result he, his brothers and sisters, and his mother were included in the naturalization.
Respondent Ong subsequently obtained from the Bureau of Immigration and the DOJ a
certification and an identification that he is a natural-born Filipino citizen under Article IV,
Sections 1 and 2 of the Constitution, since his mother was a Filipino citizen when he was
born.
Summarizing, his arguments are as follows:
I. PETITIONERS LACK OF STANDING AND INABILITY TO IMPLEAD AN
INDISPENSABLE PARTY WHOSE OFFICIAL ACTION IS THE VERY ACT
SOUGHT TO BE ANNULLED CONSTITUTE INSUPERABLE LEGAL OBSTACLES
TO THE EXERCISE OF JUDICIAL POWER AND SHOULD PREVENT THIS CASE
FROM PROCEEDING FURTHER FOR DETERMINATION ON THE MERITS BY
THIS HONORABLE COURT.
II. RESPONDENT ONG IS, IN TRUTH AND IN FACT, A NATURAL-BORN CITIZEN
OF THE PHILIPPINES, CONSIDERING THAT:
A. DY GUIOK SANTOS WAS A FILIPINO CITIZEN AT THE TIME OF
HER MARRIAGE TO EUGENIO; and
B. HAVING BEEN BORN BEFORE JANUARY 17, 1973 OF A FILIPINO
MOTHER AND WHO ELECTED FILIPINO CITIZENSHIP UPON
REACHING THE AGE OF MAJORITY, RESPONDENT ONG MEETS
THE REQUIREMENTS UNDER ARTICLE IV, SECTIONS 1 AND 2 OF
THE 1987 CONSTITUTION.

III. THE BIRTH CERTIFICATE OF RESPONDENT ONG AS PRESENTED BY


PETITIONERS CAN, IN NO WAY, WITHOUT MORE, ESTABLISH WITH FINALITY
THAT HE IS A CHINESE NATIONAL, OR DISPROVE CONCLUSIVELY THAT HE
IS, IN FACT, A NATURAL-BORN FILIPINO, DESCENDED FROM "INDIOS."
IV. IT IS NOT NECESSARY FOR RESPONDENT ONG TO RESORT TO JUDICIAL
ACTION UNDER RULE 108 OF THE RULES OF COURT FOR HIM TO BE ABLE
TO CLAIM AND ENJOY HIS RIGHTFUL STATUS AS A NATURAL-BORN
FILIPINO.
V. THE BUREAU OF IMMIGRATION HAS PREEMPTIVE LEGAL AUTHORITY OR
PRIMARY ADMINISTRATIVE JURIDICTION TO MAKE A DETERMINATION AS
REGARDS THE CITIZENSHIP OF RESPONDENT ONG, AND UPON
SUBSEQUENT CONFIRMATION BY THE SECRETARY OF JUSTICE AS
REQUIRED BY THE RULES, ISSUE A DECLARATION (I.E., IDENTIFICATION
CERTIFICATE NO. 113878) RECOGNIZING THAT RESPONDENT ONG IS A
NATURAL-BORN FILIPINO, THEREBY RENDERING NONEXISTENT ANY
CONTITUTIONAL IMPEDIMENT FOR HIM TO ASSUME THE POSITION OF
ASSOCIATE JUSTICE OF THE SUPREME COURT.7
Petitioners, in turn, filed a Consolidated Reply, in which they asserted their standing to file this
suit on the strength of previous decisions of this Court, e.g., Kilosbayan, Incorporated v.
Guingona8 and Kilosbayan, Incorporated v. Morato,9 on the ground that the case is one of
transcendental importance. They claim that the Presidents appointment of respondent Ong as
Supreme Court Justice violates the Constitution and is, therefore, attended with grave abuse
of discretion amounting to lack or excess of jurisdiction. Finally, they reiterate that respondent
Ongs birth certificate, unless corrected by judicial order in non-summary proceedings for the
purpose, is binding on all and is prima facie evidence of what it states, namely, that
respondent Ong is a Chinese citizen. The alleged naturalization of his father when he was a
minor would not make him a natural-born Filipino citizen.
The petition has merit.
First, as to standing. Petitioners have standing to file the suit simply as peoples organizations
and taxpayers since the matter involves an issue of utmost and far-reaching Constitutional
importance, namely, the qualification nay, the citizenship of a person to be appointed a
member of this Court. Standing has been accorded and recognized in similar instances.10
Second, as to having to implead the President as an alleged necessary party. This is not
necessary since the suit impleads the Executive Secretary who is the alter ego of the
President and he has in fact spoken for her in his Comment. Furthermore, the suit does not
seek to stop the President from extending the appointment but only the Executive Secretary
from releasing it and respondent Ong from accepting the same.
Third, as to the proper forum for litigating the issue of respondent Ongs qualification for
memberhip of this Court. This case is a matter of primordial importance involving compliance
with a Constitutional mandate. As the body tasked with the determination of the merits of
conflicting claims under the Constitution,11 the Court is the proper forum for resolving the
issue, even as the JBC has the initial competence to do so.
Fourth, as to the principal issue of the case is respondent Ong a natural-born Filipino
citizen?
58

On this point, the Court takes judicial notice of the records of respondent Ongs petition to be
admitted to the Philippine bar.
In his petition to be admitted to the Philippine bar, docketed as B.E. No. 1398-N filed on
September 14, 1979, under O.R. No. 8131205 of that date, respondent Ong alleged that he is
qualified to be admitted to the Philippine bar because, among others, he is a Filipino citizen;
and that he is a Filipino citizen because his father, Eugenio Ong Han Seng, a Chinese citizen,
was naturalized in 1964 when he, respondent Ong, was a minor of eleven years and thus he,
too, thereby became a Filipino citizen. As part of his evidence, in support of his petition, be
submitted his birth certificate and the naturalization papers of his father. His birth certificate12
states that he was a Chinese citizen at birth and that his mother, Dy Guiok Santos, was a
Chinese citizen and his father, Eugenio Ong Han Seng, was also a Chinese citizen.
Specifically, the following appears in the records:
PETITION
COMES now the undersigned petitioner and to this Honorable Court respectfully
states:
1. That he is single/married/widower/widow, Filipino citizen and 26 years of age,
having been born on May 25, 1953, at SAN JUAN RIZAL, to spouses Eugenio Ong
Han Seng and Dy Guiok Santos who are citizens of the Philippines, as evidenced by
the attached copy of his birth certificate marked as Annex A (if born outside of
wedlock, state so; or if Filipino citizen other than natural born, state how and when
citizenship was acquired and attach the necessary proofs: By Nat. Case #584 of
Eugenio Ong Han Seng (Father) See Attached documents Annex B, B-1, B-2, B-3,
B-4.
xxx
VERIFICATION
Republic of the Philippines )
City of Manila ) S.S.
I, GREGORY SANTOS ONG, after being sworn, depose and state: that I am the
petitioner in the foregoing petition; that the same was prepared by me and/or at my
instance and that the allegations contained therein are true to my knowledge.
(Sgd.) GREGORY SANTOS ONG
Affiant
SUBSCRIBED AND SWORN to before me this 28th day of August, 1979, City of
Manila, Philippines, affiant exhibiting his/her Residence Certificate No. A___________, issued at ________________, on __________________, 19__.

Doc. No. 98;


Page No. 10;
Book No. VIII;
Series of 1979.13
In fact, Emilio R. Rebueno, Deputy Clerk of Court and Bar Confidant, wrote respondent Ong a
letter dated October 3, 1979 stating that in connection with his Petition for Admission to the
1979 Bar Examinations, he has to submit:
1) A certified clear copy of his Birth Certificate; and
2) A certification of non-appeal re his citizenship from the Office of the Solicitor
General.
Respondent Ong complied with these requirements.
It was on the basis of these allegations under oath and the submitted evidence of
naturalization that this Court allowed respondent Ong to take the oath as a lawyer.
It is clear, therefore, that from the records of this Court, respondent Ong is a naturalized
Filipino citizen. The alleged subsequent recognition of his natural-born status by the Bureau of
Immigration and the DOJ cannot amend the final decision of the trial court stating that
respondent Ong and his mother were naturalized along with his father.
Furthermore, as petitioners correctly submit, no substantial change or correction in an entry in
a civil register can be made without a judicial order, and, under the law, a change in
citizenship status is a substantial change. In Labayo-Rowe v. Republic,14 this Court held that:
Changes which affect the civil status or citizenship of a party are substantial in
character and should be threshed out in a proper action depending upon the nature
of the issues in controversy, and wherein all the parties who may be affected by the
entries are notified or represented and evidence is submitted to prove the
allegations of the complaint, and proof to the contrary admitted.15
Republic Act No. 9048 provides in Section 2 (3) that a summary administrative proceeding to
correct clerical or typographical errors in a birth certificate cannot apply to a change in
nationality. Substantial corrections to the nationality or citizenship of persons recorded in the
civil registry should, therefore, be effected through a petition filed in court under Rule 108 of
the Rules of Court.16
The series of events and long string of alleged changes in the nationalities of respondent
Ongs ancestors, by various births, marriages and deaths, all entail factual assertions that
need to be threshed out in proper judicial proceedings so as to correct the existing records on
his birth and citizenship. The chain of evidence would have to show that Dy Guiok Santos,
respondent Ongs mother, was a Filipino citizen, contrary to what still appears in the records
of this Court. Respondent Ong has the burden of proving in court his alleged ancestral tree as
well as his citizenship under the time-line of three Constitutions.17 Until this is done,
respondent Ong cannot accept an appointment to this Court as that would be a violation of the
Constitution. For this reason, he can be prevented by injunction from doing so.
WHEREFORE, the petition is GRANTED as one of injunction directed against respondent
Gregory S. Ong, who is hereby ENJOINED from accepting an appointment to the position of
Associate Justice of the Supreme Court or assuming the position and discharging the
functions of that office, until he shall have successfully completed all necessary steps, through

(Sgd.)
Notary Public
Until December 31, 1979
PTR No. 3114917
January 19, 1979, Pasig, MM

59

the appropriate adversarial proceedings in court, to show that he is a natural-born Filipino


citizen and correct the records of his birth and citizenship.
This Decision is FINAL and IMMEDIATELY EXECUTORY.
No costs.
SO ORDERED.

60

G.R. No. 138496


February 23, 2004
HUBERT TAN CO and ARLENE TAN CO, petitioners,
vs.
THE CIVIL REGISTER OF MANILA and any person having or claiming an interest under
the entry whose cancellation or correction is sought, respondent.
DECISION
CALLEJO, SR., J.:
Before the Court is the petition for review on certiorari filed by Hubert Tan Co and Arlene Tan
Co seeking to reverse and set aside the Order1 dated September 23, 1998 of the Regional
Trial Court of Manila, Branch 26, dismissing their petition for correction of entries in the Civil
Register. Likewise sought to be reversed and set aside is the Order dated April 27, 1999 of
the court a quo denying the petitioners motion for reconsideration of the said order.
The factual antecedents are as follows:
Hubert Tan Co was born on March 23, 1974. His sister, Arlene Tan Co, was born on May 19,
1975. In their respective certificates of birth, it is stated that their parents Co Boon Peng and
Lourdes Vihong K. Tan are Chinese citizens.
Thereafter, Co Boon Peng filed an application for his naturalization as a citizen of the
Philippines with the Special Committee on Naturalization under Letter of Instruction (LOI) No.
270. His application was granted and he was conferred Philippine citizenship under
Presidential Decree (P.D.) No. 1055. The Chairman of the Committee issued on February 15,
1977 Certificate of Naturalization No. 020778 in his favor. Thus, on February 15, 1977, Co
Boon Peng took his oath as a Philippine citizen. In the meantime, Hubert and Arlene Co
finished college and earned their respective degrees in architecture and accountancy in
Philippine schools.
On August 27, 1998, they filed with the Regional Trial Court of Manila a petition under Rule
108 of the Rules of Court for correction of entries in their certificates of birth. The case was
docketed as Sp. Proc. Case No. 98-90470. They alleged, inter alia, in their petition that:
(3) They were born in the Philippines and the legitimate children of CO BOON
PENG;
(4) Co Boon Peng, who is formerly a citizen of China, was conferred Philippine
citizenship by naturalization under Presidential Decree No. 1055 and had taken his
oath of allegiance to the Republic of the Philippines on 15th February, 1977 in the
City of Manila;
(5) At the time of birth of [the] petitioners, their father CO BOON PENG was still a
Chinese citizen that is why entry in their respective birth certificates as to their
fathers citizenship was Chinese;
(6) Upon granting of Philippine citizenship by naturalization to Co Boon Peng in
1977, [the] petitioners who were born in the Philippines and still minors at that time
became Filipino citizens through the derivative mode of naturalization. Our
Naturalization Law, specifically Section 15 of Commonwealth Act No. 473, as
amended by Commonwealth Act No. 535 which provides:
"Minor children of persons naturalized under this law who have been born in the
Philippines shall be considered citizens thereof;"

(7) The naturalization of petitioners father in 1977 was an act or event affecting and
concerning their civil status that must be recorded in the Civil Register, Article 407 of
the New Civil Code of the Philippines which provides:
"Acts, events and judicial decrees concerning the civil status of persons shall be recorded in
the Civil Register."2
The petitioners prayed that, after due proceedings, the trial court render judgment correcting
and changing the entries in their respective birth certificates as to the citizenship of their father
Co Boon Peng, from "Chinese" to "Filipino."3
On September 23, 1998, the court a quo issued an order dismissing the petition outright on
the ground that the petition was insufficient, solely because the petitioners father Co Boon
Peng applied for naturalization under LOI No. 270 and was conferred Philippine citizenship by
naturalization under PD No. 1055 and not under Commonwealth Act (CA) No. 473. 4
The petitioners sought the reconsideration of the assailed order arguing that LOI No. 270 and
CA No. 473 were designed to grant citizenship to deserving aliens; hence, should be
construed together. They averred that the benefit of Section 15 of CA No. 473 should also be
granted to the petitioners whose father was granted naturalization under LOI No. 270.
However, the RTC issued an Order on April 27, 1999, denying their motion for reconsideration
for the following reasons: (a) although Commonwealth Act No. 473 and Letter of Instructions
No. 270 are statutes relating to the same subject matter, they do not provide the same
beneficial effects with respect to the minor children of the applicant. Section 15 of CA No. 473
expressly provides for the effect of the naturalization on the wife and children of the applicant
while LOI No. 270 does not have any proviso to that effect; (b) LOI No. 270 clearly refers to
qualified individuals only. The rules and regulations promulgated by the Committee
established pursuant to LOI No. 270 and the amendments issued by then President Ferdinand
E. Marcos (LOI Nos. 292 and 491) clearly speak of qualified individuals only; no proviso
therein referred to its effect on the wife and children of the individual; (c) Section 15 of CA No.
473 should not be deemed and incorporated in and applied to LOI No. 270; and, (d) the
application of the so-called "pari materia" rule of construction made by the petitioners is
misplaced, as what should be applied in the instant case is the rule on strict construction of
legislative grants or franchise. The court a quo stressed that legislative grants, whether they
be of property, rights or privileges, whether granted to corporations or individuals, must be
strictly construed against the grantee and in favor of the grantor.
Aggrieved, the petitioners now come to this Court assailing the court a quos Order dismissing
their petition outright and its Order denying their motion for the reconsideration of the same.
The petitioners contend that the trial court erred in holding that their petition was insufficient.
They assert that contrary to the ruling of the trial court, they are qualified to claim the benefit of
Section 15 of CA No. 473, which provides that minor children of persons naturalized
thereunder who were born in the Philippines shall likewise be considered citizens thereof.
They contend that although LOI No. 270, under which the petitioners father was naturalized
does not contain a provision similar to Section 15 of CA No. 473, the latter provision should be
deemed incorporated therein. They point out that both laws have the same purpose and
objective, i.e., to grant Philippine citizenship to qualified aliens permanently residing in the
Philippines. The petitioners invoke the rule that statutes in pari materia are to be read
together.5 They posit that CA No. 473 and LOI No. 270 should be harmonized and reconciled
61

since "all statutes relating to the same subject, or having the same general purpose, should
be read in connection with it, and should be construed together as they constitute one law."6
The petitioners maintain that the letter and spirit of LOI No. 270 was to grant the privilege of
Philippine citizenship not only to qualified aliens but also to their minor children who were born
in the country. They assert that this is apparent from paragraph 4-A thereof, which extends the
option to adopt Filipino names not only to qualified applicants for naturalization but also to
their wives and minor children. They submit that when then President Ferdinand E. Marcos
enacted LOI No. 270, he must be presumed to have been acquainted with the provisions of
CA No. 473 and did not intend to abrogate and discontinue the beneficial effects of Section 15
thereof; otherwise, Pres. Marcos would have expressly repealed Section 15 of CA No. 473 in
relation to LOI No. 270. Thus, according to the petitioners, the naturalization of their father
during their minority is an act or event affecting their civil status that must be recorded in the
Civil Register pursuant to Article 407 of the Civil Code.
In his Comment, the Solicitor General contends that the court a quo did not err in issuing the
assailed orders. Contrary to the petitioners theory, LOI No. 270 and CA No. 473 are separate
and distinct laws; therefore, are not in pari materia. He points out that although LOI No. 270
and CA No. 473 both govern the naturalization of aliens, CA No. 473 deals with the
requirements and procedure for naturalization by judicial decree; LOI No. 270, on the other
hand, deals with the requirements and procedure for naturalization by presidential decree.
The Solicitor General further asserts that the petitioners contention that the naturalization of
their father is an event affecting and concerning their civil status envisaged in Article 407 of
the Civil Code has no legal basis. The correction sought and allowed under Rule 108 of the
Rules of Court must be one that reflects a fact existing before or at the time of birth. In the
petitioners case, the naturalization of their father in 1977 took place long after they were born.
Moreover, according to the Solicitor General, under LOI No. 270 and its amendatory laws, the
naturalization of a father did not ipso facto render his children also naturalized. The petitioners
thus cannot invoke Article 407 of the Civil Code and Rule 108 of the Rules of Court to avoid
strict compliance with the naturalization laws.
The petition is meritorious.
The rule on statutory construction provides that:
Statutes in pari materia should be read and construed together because enactments of the
same legislature on the same subject are supposed to form part of one uniform system; later
statutes are supplementary or complimentary (sic) to the earlier enactments and in the
passage of its acts the legislature is supposed to have in mind the existing legislations on the
subject and to have enacted its new act with reference thereto.7
Statutes in pari materia should be construed together to attain the purpose of an expressed
national policy, thus:
On the presumption that whenever the legislature enacts a provision it has in mind the
previous statutes relating to the same subject matter, it is held that in the absence of any
express repeal or amendment therein, the new provision was enacted in accord with the
legislative policy embodied in those prior statutes, and they all should be construed together.
Provisions in an act which are omitted in another act relating to the same subject matter will
be applied in a proceeding under the other act, when not inconsistent with its purpose. Prior
statutes relating to the same subject matter are to be compared with the new provisions; and if

possible by reasonable construction, both are to be construed that effect is given to every
provision of each. Statutes in pari materia, although in apparent conflict, are so far as
reasonably possible construed to be in harmony with each other.8
LOI No. 270 and CA No. 473 are laws governing the naturalization of qualified aliens residing
in the Philippines. While they provide for different procedures, CA No. 473 governs
naturalization by judicial decree while LOI No. 270 governs naturalization by presidential
decree; both statutes have the same purpose and objective: to enable aliens permanently
residing in the Philippines, who, having demonstrated and developed love for and loyalty to
the Philippines, as well as affinity to the culture, tradition and ideals of the Filipino people, and
contributed to the economic, social and cultural development of our country, to be integrated
into the national fabric by being granted Filipino citizenship. Under the LOI, the procedure for
the acquisition of citizenship by naturalization is more expeditious, less cumbersome and less
expensive. The sooner qualified aliens are naturalized, the faster they are able to integrate
themselves into the national fabric, and are thus able to contribute to the cultural, social and
political well- being of the country and its people.
Clearly, LOI No. 270 and CA No. 473 are, as the petitioners correctly posit, statutes in pari
materia. Absent any express repeal of Section 15 of CA No. 473 in LOI No. 270, the said
provision should be read into the latter law as an integral part thereof, not being inconsistent
with its purpose. Thus, Section 15 of CA No. 473,9 which extends the grant of Philippine
citizenship to the minor children of those naturalized thereunder, should be similarly applied to
the minor children of those naturalized under LOI No. 270, like the petitioners in this case.
It is not enough that the petitioners adduce in evidence the certificate of naturalization of their
father, Co Boon Peng, and of his oath of allegiance to the Republic of the Philippines, to
entitle them to Philippine citizenship. They are likewise mandated to prove the following
material allegations in their petition: (a) that they are the legitimate children of Co Boon Peng;
(b) that they were born in the Philippines; and, (c) that they were still minors when Co Boon
Peng was naturalized as a Filipino citizen;
The petitioners recourse to Rule 108 of the Rules of Court, as amended, is appropriate.
Under Article 412 of the New Civil Code, no entry in a civil register shall be changed or
corrected without a judicial order. The law does not provide for a specific procedure of law to
be followed. But the Court approved Rule 108 of the Rules of Court to provide for a procedure
to implement the law.10 The entries envisaged in Article 412 of the New Civil Code are those
provided in Articles 407 and 408 of the New Civil Code which reads:
Art. 407. Acts, events and judicial decrees concerning the civil status of persons shall be
recorded in the civil register.
Art. 408. The following shall be entered in the civil register:
(1) Births; (2) Marriages; (3) deaths; (4) legal separations; (5) annulments of marriage; (6)
judgments declaring marriages void from the beginning; (7) legitimations; (8) adoptions; (9)
acknowledgments of natural children; (10) naturalization; (11) loss, or (12) recovery of
citizenship; (13) civil interdiction; (14) judicial determination of filiation; (15) voluntary
emancipation of a minor; and (16) changes of name.
Specific matters covered by the said provision include not only status but also nationality.11
The acts, events or factual errors envisaged in Article 407 of the New Civil Code include even
those that occur after the birth of the petitioner. However, in such cases, the entries in the
62

certificates of birth will not be corrected or changed. The decision of the court granting the
petition shall be annotated in the certificates of birth and shall form part of the civil register in
the Office of the Local Civil Registrar.12
To correct simply means "to make or set aright; to remove the faults or error from." To change
means "to replace something with something else of the same kind or with something that
serves as a substitute. Article 412 of the New Civil Code does not qualify as to the kind of
entry to be changed or corrected or distinguished on the basis of the effect that the correction
or change may be.13 Such entries include not only those clerical in nature but also substantial
errors. After all, the role of the Court under Rule 108 of the Rules of Court is to ascertain the
truths about the facts recorded therein.14
The proceedings in Rule 108 of the Rules of Court are summary if the entries in the civil
register sought to be corrected are clerical or innocuous in nature. However, where such
entries sought to be corrected or changed are substantial: i.e., the status and nationality of the
petitioners or the citizenship of their parents,15 the proceedings are adversarial in nature as
defined by this Court in Republic v. Valencia, thus:
One having opposing parties; contested, as distinguished from an ex parte application, one of
which the party seeking relief has given legal warning to the other party, and afforded the
latter an opportunity to contest it. Excludes an adoption proceeding.16
In such a proceeding, the parties to be impleaded as respective defendants are (a) the local
civil registrar; and, (b) all persons who have claims any interest which would be affected
thereby.17
In this case, the petitioners alleged in their petition that they are the legitimate children of Co
Boon Peng, who was naturalized as a Filipino citizen, but that their certificates of birth still
indicate that he is a Chinese national. In view of their fathers naturalization, they pray that the
entries in their certificates of birth relating to the citizenship of their father be changed from
"Chinese" to "Filipino."
The petitioners recourse to the procedure in Rule 108 of the Rules of Court, as amended,
being appropriate, it behooved the trial court to do its duty under Section 4, Rule 108 of the
Rules of Court, namely:
Sec. 4. Notice and Publication. Upon the filing of the petition, the court shall, by an order, fix
the time and place for the hearing of the same, and cause reasonable notice thereof to be
given to the person named in the petition. The court shall also cause the order to be published
once a week for three (3) consecutive weeks in a newspaper of general circulation in the
province.
After hearing, the court shall issue an order either dismissing the petition or issue an order
granting the same. In either case, a certified copy of the judgment shall be served upon the
civil registrar concerned who shall annotate the same in the certificates of birth of the
petitioners. The judgment of the court shall form part of the records of the local civil register. 18
In this case, the trial court dismissed the petition outright in violation of Rule 108 of the Rules
of Court. Patently, then, the trial court erred in so doing.
IN THE LIGHT OF THE FOREGOING, the petition is GRANTED. The assailed Orders of the
Regional Trial Court of Manila, Branch 26, are SET ASIDE and REVERSED. The trial court is
DIRECTED to reinstate the petition in Special Proceedings NO. 98-90470 in the court docket,

and ORDERED to continue with the proceedings in the said case under Rule 108 of the Rules
of Court, as amended.
SO ORDERED.

63

G.R. No. 160869


May 11, 2007
AASJS (ADVOCATES AND ADHERENTS OF SOCIAL JUSTICE FOR SCHOOL
TEACHERS AND ALLIED WORKERS) MEMBER - HECTOR GUMANGAN CALILUNG,
Petitioner,
vs.
THE HONORABLE SIMEON DATUMANONG, in his official capacity as the Secretary of
Justice, Respondent.
DECISION
QUISUMBING, J.:
This is an original action for prohibition under Rule 65 of the 1997 Revised Rules of Civil
Procedure.
Petitioner filed the instant petition against respondent, then Secretary of Justice Simeon
Datumanong, the official tasked to implement laws governing citizenship.1 Petitioner prays that
a writ of prohibition be issued to stop respondent from implementing Republic Act No. 9225,
entitled "An Act Making the Citizenship of Philippine Citizens Who Acquire Foreign Citizenship
Permanent, Amending for the Purpose Commonwealth Act No. 63, As Amended, and for
Other Purposes." Petitioner avers that Rep. Act No. 9225 is unconstitutional as it violates
Section 5, Article IV of the 1987 Constitution that states, "Dual allegiance of citizens is inimical
to the national interest and shall be dealt with by law."
Rep. Act No. 9225, signed into law by President Gloria M. Arroyo on August 29, 2003, reads:
SECTION 1. Short Title.-This Act shall be known as the "Citizenship Retention and
Reacquisition Act of 2003."
SEC. 2. Declaration of Policy.-It is hereby declared the policy of the State that all Philippine
citizens who become citizens of another country shall be deemed not to have lost their
Philippine citizenship under the conditions of this Act.
SEC. 3. Retention of Philippine Citizenship.-Any provision of law to the contrary
notwithstanding, natural-born citizens of the Philippines who have lost their Philippine
citizenship by reason of their naturalization as citizens of a foreign country are hereby deemed
to have reacquired Philippine citizenship upon taking the following oath of allegiance to the
Republic:
"I ___________________________, solemnly swear (or affirm) that I will support and defend
the Constitution of the Republic of the Philippines and obey the laws and legal orders
promulgated by the duly constituted authorities of the Philippines; and I hereby declare that I
recognize and accept the supreme authority of the Philippines and will maintain true faith and
allegiance thereto; and that I impose this obligation upon myself voluntarily without mental
reservation or purpose of evasion."
Natural-born citizens of the Philippines who, after the effectivity of this Act, become citizens of
a foreign country shall retain their Philippine citizenship upon taking the aforesaid oath.
SEC. 4. Derivative Citizenship. - The unmarried child, whether legitimate, illegitimate or
adopted, below eighteen (18) years of age, of those who reacquire Philippine citizenship upon
effectivity of this Act shall be deemed citizens of the Philippines.
SEC. 5. Civil and Political Rights and Liabilities. - Those who retain or reacquire Philippine
citizenship under this Act shall enjoy full civil and political rights and be subject to all attendant

liabilities and responsibilities under existing laws of the Philippines and the following
conditions:
(1) Those intending to exercise their right of suffrage must meet the requirements
under Section 1, Article V of the Constitution, Republic Act No. 9189, otherwise
known as "The Overseas Absentee Voting Act of 2003" and other existing laws;
(2) Those seeking elective public office in the Philippines shall meet the
qualifications for holding such public office as required by the Constitution and
existing laws and, at the time of the filing of the certificate of candidacy, make a
personal and sworn renunciation of any and all foreign citizenship before any public
officer authorized to administer an oath;
(3) Those appointed to any public office shall subscribe and swear to an oath of
allegiance to the Republic of the Philippines and its duly constituted authorities prior
to their assumption of office: Provided, That they renounce their oath of allegiance
to the country where they took that oath;
(4) Those intending to practice their profession in the Philippines shall apply with the
proper authority for a license or permit to engage in such practice; and
(5) That right to vote or be elected or appointed to any public office in the Philippines
cannot be exercised by, or extended to, those who:
(a) are candidates for or are occupying any public office in the country of which they
are naturalized citizens; and/or
(b) are in the active service as commissioned or noncommissioned officers in the
armed forces of the country which they are naturalized citizens.
SEC. 6. Separability Clause. - If any section or provision of this Act is held unconstitutional or
invalid, any other section or provision not affected thereby shall remain valid and effective.
SEC. 7. Repealing Clause. - All laws, decrees, orders, rules and regulations inconsistent with
the provisions of this Act are hereby repealed or modified accordingly.
SEC. 8. Effectivity Clause. - This Act shall take effect after fifteen (15) days following its
publication in the Official Gazette or two (2) newspapers of general circulation.
In this petition for prohibition, the following issues have been raised: (1) Is Rep. Act No. 9225
unconstitutional? (2) Does this Court have jurisdiction to pass upon the issue of dual
allegiance?
We shall discuss these issues jointly.
Petitioner contends that Rep. Act No. 9225 cheapens Philippine citizenship. He avers that
Sections 2 and 3 of Rep. Act No. 9225, together, allow dual allegiance and not dual
citizenship. Petitioner maintains that Section 2 allows all Filipinos, either natural-born or
naturalized, who become foreign citizens, to retain their Philippine citizenship without losing
their foreign citizenship. Section 3 permits dual allegiance because said law allows naturalborn citizens of the Philippines to regain their Philippine citizenship by simply taking an oath of
allegiance without forfeiting their foreign allegiance.2 The Constitution, however, is categorical
that dual allegiance is inimical to the national interest.
The Office of the Solicitor General (OSG) claims that Section 2 merely declares as a state
policy that "Philippine citizens who become citizens of another country shall be deemed not to
have lost their Philippine citizenship." The OSG further claims that the oath in Section 3 does
64

not allow dual allegiance since the oath taken by the former Filipino citizen is an effective
renunciation and repudiation of his foreign citizenship. The fact that the applicant taking the
oath recognizes and accepts the supreme authority of the Philippines is an unmistakable and
categorical affirmation of his undivided loyalty to the Republic.3
In resolving the aforecited issues in this case, resort to the deliberations of Congress is
necessary to determine the intent of the legislative branch in drafting the assailed law. During
the deliberations, the issue of whether Rep. Act No. 9225 would allow dual allegiance had in
fact been the subject of debate. The record of the legislative deliberations reveals the
following:
xxxx
Pursuing his point, Rep. Dilangalen noted that under the measure, two situations exist - - the
retention of foreign citizenship, and the reacquisition of Philippine citizenship. In this case, he
observed that there are two citizenships and therefore, two allegiances. He pointed out that
under the Constitution, dual allegiance is inimical to public interest. He thereafter asked
whether with the creation of dual allegiance by reason of retention of foreign citizenship and
the reacquisition of Philippine citizenship, there will now be a violation of the Constitution
Rep. Locsin underscored that the measure does not seek to address the constitutional
injunction on dual allegiance as inimical to public interest. He said that the proposed law aims
to facilitate the reacquisition of Philippine citizenship by speedy means. However, he said that
in one sense, it addresses the problem of dual citizenship by requiring the taking of an oath.
He explained that the problem of dual citizenship is transferred from the Philippines to the
foreign country because the latest oath that will be taken by the former Filipino is one of
allegiance to the Philippines and not to the United States, as the case may be. He added that
this is a matter which the Philippine government will have no concern and competence over.
Rep. Dilangalen asked why this will no longer be the country's concern, when dual allegiance
is involved.
Rep. Locsin clarified that this was precisely his objection to the original version of the bill,
which did not require an oath of allegiance. Since the measure now requires this oath, the
problem of dual allegiance is transferred from the Philippines to the foreign country
concerned, he explained.
xxxx
Rep. Dilangalen asked whether in the particular case, the person did not denounce his foreign
citizenship and therefore still owes allegiance to the foreign government, and at the same
time, owes his allegiance to the Philippine government, such that there is now a case of dual
citizenship and dual allegiance.
Rep. Locsin clarified that by swearing to the supreme authority of the Republic, the person
implicitly renounces his foreign citizenship. However, he said that this is not a matter that he
wishes to address in Congress because he is not a member of a foreign parliament but a
Member of the House.
xxxx
Rep. Locsin replied that it is imperative that those who have dual allegiance contrary to
national interest should be dealt with by law. However, he said that the dual allegiance
problem is not addressed in the bill. He then cited the Declaration of Policy in the bill which
states that "It is hereby declared the policy of the State that all citizens who become citizens of

another country shall be deemed not to have lost their Philippine citizenship under the
conditions of this Act." He stressed that what the bill does is recognize Philippine citizenship
but says nothing about the other citizenship.
Rep. Locsin further pointed out that the problem of dual allegiance is created wherein a
natural-born citizen of the Philippines takes an oath of allegiance to another country and in
that oath says that he abjures and absolutely renounces all allegiance to his country of origin
and swears allegiance to that foreign country. The original Bill had left it at this stage, he
explained. In the present measure, he clarified, a person is required to take an oath and the
last he utters is one of allegiance to the country. He then said that the problem of dual
allegiance is no longer the problem of the Philippines but of the other foreign country.4
(Emphasis supplied.)
From the above excerpts of the legislative record, it is clear that the intent of the legislature in
drafting Rep. Act No. 9225 is to do away with the provision in Commonwealth Act No. 635
which takes away Philippine citizenship from natural-born Filipinos who become naturalized
citizens of other countries. What Rep. Act No. 9225 does is allow dual citizenship to naturalborn Filipino citizens who have lost Philippine citizenship by reason of their naturalization as
citizens of a foreign country. On its face, it does not recognize dual allegiance. By swearing to
the supreme authority of the Republic, the person implicitly renounces his foreign citizenship.
Plainly, from Section 3, Rep. Act No. 9225 stayed clear out of the problem of dual allegiance
and shifted the burden of confronting the issue of whether or not there is dual allegiance to the
concerned foreign country. What happens to the other citizenship was not made a concern of
Rep. Act No. 9225.
Petitioner likewise advances the proposition that although Congress has not yet passed any
law on the matter of dual allegiance, such absence of a law should not be justification why this
Court could not rule on the issue. He further contends that while it is true that there is no
enabling law yet on dual allegiance, the Supreme Court, through Mercado v. Manzano,6
already had drawn up the guidelines on how to distinguish dual allegiance from dual
citizenship.7
For its part, the OSG counters that pursuant to Section 5, Article IV of the 1987 Constitution,
dual allegiance shall be dealt with by law. Thus, until a law on dual allegiance is enacted by
Congress, the Supreme Court is without any jurisdiction to entertain issues regarding dual
allegiance.8
To begin with, Section 5, Article IV of the Constitution is a declaration of a policy and it is not a
self-executing provision. The legislature still has to enact the law on dual allegiance. In
Sections 2 and 3 of Rep. Act No. 9225, the framers were not concerned with dual citizenship
per se, but with the status of naturalized citizens who maintain their allegiance to their
countries of origin even after their naturalization.9 Congress was given a mandate to draft a
law that would set specific parameters of what really constitutes dual allegiance.10 Until this is
done, it would be premature for the judicial department, including this Court, to rule on issues
pertaining to dual allegiance.
Neither can we subscribe to the proposition of petitioner that a law is not needed since the
case of Mercado had already set the guidelines for determining dual allegiance. Petitioner
misreads Mercado. That case did not set the parameters of what constitutes dual allegiance
but merely made a distinction between dual allegiance and dual citizenship.
65

Moreover, in Estrada v. Sandiganbayan,11 we said that the courts must assume that the
legislature is ever conscious of the borders and edges of its plenary powers, and passed laws
with full knowledge of the facts and for the purpose of promoting what is right and advancing
the welfare of the majority. Hence, in determining whether the acts of the legislature are in
tune with the fundamental law, we must proceed with judicial restraint and act with caution
and forbearance.12 The doctrine of separation of powers demands no less. We cannot
arrogate the duty of setting the parameters of what constitutes dual allegiance when the
Constitution itself has clearly delegated the duty of determining what acts constitute dual
allegiance for study and legislation by Congress.
WHEREFORE, the petition is hereby DISMISSED for lack of merit.
SO ORDERED.

66

G.R. No. 182701


July 23, 2008
EUSEBIO EUGENIO K. LOPEZ, Petitioner,
vs.
COMMISSION ON ELECTIONS and TESSIE P. VILLANUEVA, Respondents.
RESOLUTION
REYES, R.T., J.:
A Filipino-American or any dual citizen cannot run for any elective public position in the
Philippines unless he or she personally swears to a renunciation of all foreign citizenship at
the time of filing the certificate of candidacy.
This is a petition for certiorari under Rule 65, in relation to Rule 64 of the Rules on Civil
Procedure assailing the (1) Resolution1 and (2) Omnibus Order2 of the Commission on
Elections (COMELEC), Second Division, disqualifying petitioner from running as Barangay
Chairman.
Petitioner Eusebio Eugenio K. Lopez was a candidate for the position of Chairman of
Barangay Bagacay, San Dionisio, Iloilo City in the synchronized Barangay and Sangguniang
Kabataan Elections held on October 29, 2007.
On October 25, 2007, respondent Tessie P. Villanueva filed a petition3 before the Provincial
Election Supervisor of the Province of Iloilo, praying for the disqualification of petitioner on the
ground that he is an American citizen, hence, ineligible from running for any public office. In
his Answer,4 petitioner argued that he is a dual citizen, a Filipino and at the same time an
American, by virtue of Republic Act (R.A.) No. 9225, otherwise known as the Citizenship
Retention and Re-acquisition Act of 2003.5 He returned to the Philippines and resided in
Barangay Bagacay. Thus, he said, he possessed all the qualifications to run for Barangay
Chairman.
After the votes for Barangay Chairman were canvassed, petitioner emerged as the winner.6
On February 6, 2008, COMELEC issued the assailed Resolution granting the petition for
disqualification, disposing as follows:
WHEREFORE, premises considered, the instant Petition for Disqualification is GRANTED and
respondent Eusebio Eugenio K. Lopez is DISQUALIFIED from running as Barangay Chairman
of Barangay Bagacay, San Dionisio, Iloilo.
SO ORDERED.7
In ruling against petitioner, the COMELEC found that he was not able to regain his Filipino
citizenship in the manner provided by law. According to the poll body, to be able to qualify as
a candidate in the elections, petitioner should have made a personal and sworn renunciation
of any and all foreign citizenship. This, petitioner failed to do.
His motion for reconsideration having been denied, petitioner resorted to the present petition,
imputing grave abuse of discretion on the part of the COMELEC for disqualifying him from
running and assuming the office of Barangay Chairman.
We dismiss the petition.
Relying on Valles v. Commission on Elections,8 petitioner argues that his filing of a certificate
of candidacy operated as an effective renunciation of foreign citizenship.

We note, however, that the operative facts that led to this Courts ruling in Valles are
substantially different from the present case. In Valles, the candidate, Rosalind Ybasco Lopez,
was a dual citizen by accident of birth on foreign soil.9 Lopez was born of Filipino parents in
Australia, a country which follows the principle of jus soli.lauuphi1 As a result, she acquired
Australian citizenship by operation of Australian law, but she was also considered a Filipino
citizen under Philippine law. She did not perform any act to swear allegiance to a country
other than the Philippines.
In contrast, petitioner was born a Filipino but he deliberately sought American citizenship and
renounced his Filipino citizenship. He later on became a dual citizen by re-acquiring Filipino
citizenship.1awphi1
More importantly, the Courts 2000 ruling in Valles has been superseded by the enactment of
R.A. No. 922510 in 2003. R.A. No. 9225 expressly provides for the conditions before those
who re-acquired Filipino citizenship may run for a public office in the Philippines. Section 5 of
the said law states:
Section 5. Civil and Political Rights and Liabilities. Those who retain or re-acquire
Philippine citizenship under this Act shall enjoy full civil and political rights and be subject to all
attendant liabilities and responsibilities under existing laws of the Philippines and the following
conditions:
xxxx
(2) Those seeking elective public office in the Philippines shall meet the qualification for
holding such public office as required by the Constitution and existing laws and, at the time of
the filing of the certificate of candidacy, make a personal and sworn renunciation of any and
all foreign citizenship before any public officer authorized to administer an oath. (Emphasis
added)
Petitioner re-acquired his Filipino citizenship under the cited law. This new law explicitly
provides that should one seek elective public office, he should first "make a personal and
sworn renunciation of any and all foreign citizenship before any public officer authorized to
administer an oath."
Petitioner failed to comply with this requirement. We quote with approval the COMELEC
observation on this point:
While respondent was able to regain his Filipino Citizenship by virtue of the Dual Citizenship
Law when he took his oath of allegiance before the Vice Consul of the Philippine Consulate
Generals Office in Los Angeles, California, the same is not enough to allow him to run for a
public office. The above-quoted provision of law mandates that a candidate with dual
citizenship must make a personal and sworn renunciation of any and all foreign citizenship
before any public officer authorized to administer an oath. There is no evidence presented that
will show that respondent complied with the provision of R.A. No. 9225. Absent such proof we
cannot allow respondent to run for Barangay Chairman of Barangay Bagacay.
For the renunciation to be valid, it must be contained in an affidavit duly executed before an
officer of law who is authorized to administer an oath. The affiant must state in clear and
unequivocal terms that he is renouncing all foreign citizenship for it to be effective. In the
instant case, respondent Lopezs failure to renounce his American citizenship as proven by
the absence of an affidavit that will prove the contrary leads this Commission to believe that
he failed to comply with the positive mandate of law. For failure of respondent to prove that he
67

abandoned his allegiance to the United States, this Commission holds him disqualified from
running for an elective position in the Philippines.11 (Emphasis added)
While it is true that petitioner won the elections, took his oath and began to discharge the
functions of Barangay Chairman, his victory can not cure the defect of his candidacy.
Garnering the most number of votes does not validate the election of a disqualified candidate
because the application of the constitutional and statutory provisions on disqualification is not
a matter of popularity.12
In sum, the COMELEC committed no grave abuse of discretion in disqualifying petitioner as
candidate for Chairman in the Barangay elections of 2007.
WHEREFORE, the petition is DISMISSED.
SO ORDERED.

68

G.R. No. 179848


November 27, 2008
NESTOR A. JACOT, petitioner,
vs.
ROGEN T. DAL and COMMISSION ON ELECTIONS, respondents.
DECISION
CHICO-NAZARIO, J.:
Petitioner Nestor A. Jacot assails the Resolution1 dated 28 September 2007 of the
Commission on Elections (COMELEC) En Banc in SPA No. 07-361, affirming the Resolution
dated 12 June 2007 of the COMELEC Second Division2 disqualifying him from running for the
position of Vice-Mayor of Catarman, Camiguin, in the 14 May 2007 National and Local
Elections, on the ground that he failed to make a personal renouncement of his United States
(US) citizenship.
Petitioner was a natural born citizen of the Philippines, who became a naturalized citizen of
the US on 13 December 1989. 3
Petitioner sought to reacquire his Philippine citizenship under Republic Act No. 9225,
otherwise known as the Citizenship Retention and Re-Acquisition Act. He filed a request for
the administration of his Oath of Allegiance to the Republic of the Philippines with the
Philippine Consulate General (PCG) of Los Angeles, California. The Los Angeles PCG issued
on 19 June 2006 an Order of Approval4 of petitioners request, and on the same day,
petitioner took his Oath of Allegiance to the Republic of the Philippines before Vice Consul
Edward C. Yulo. 5 On 27 September 2006, the Bureau of Immigration issued Identification
Certificate No. 06-12019 recognizing petitioner as a citizen of the Philippines.6
Six months after, on 26 March 2007, petitioner filed his Certificate of Candidacy for the
Position of Vice-Mayor of the Municipality of Catarman, Camiguin. 7
On 2 May 2007, respondent Rogen T. Dal filed a Petition for Disqualification8 before the
COMELEC Provincial Office in Camiguin against petitioner, arguing that the latter failed to
renounce his US citizenship, as required under Section 5(2) of Republic Act No. 9225, which
reads as follows:
Section 5. Civil and Political Rights and Liabilities.Those who retain or reacquire
Philippine citizenship under this Act shall enjoy full civil and political rights and be
subject to all attendant liabilities and responsibilities under existing laws of the
Philippines and the following conditions:
xxxx
(2) Those seeking elective public office in the Philippines shall meet the
qualifications for holding such public office as required by the Constitution and
existing laws and, at the time of the filing of the certificate of candidacy, make a
personal and sworn renunciation of any and all foreign citizenship before any public
officer authorized to administer an oath.
In his Answer9 dated 6 May 2007 and Position Paper10 dated 8 May 2007, petitioner countered
that his Oath of Allegiance to the Republic of the Philippines made before the Los Angeles
PCG and the oath contained in his Certificate of Candidacy operated as an effective
renunciation of his foreign citizenship.

In the meantime, the 14 May 2007 National and Local Elections were held. Petitioner
garnered the highest number of votes for the position of Vice Mayor.
On 12 June 2007, the COMELEC Second Division finally issued its Resolution11 disqualifying
the petitioner from running for the position of Vice-Mayor of Catarman, Camiguin, for failure to
make the requisite renunciation of his US citizenship. The COMELEC Second Division
explained that the reacquisition of Philippine citizenship under Republic Act No. 9225 does not
automatically bestow upon any person the privilege to run for any elective public office. It
additionally ruled that the filing of a Certificate of Candidacy cannot be considered as a
renunciation of foreign citizenship. The COMELEC Second Division did not consider Valles v.
COMELEC12 and Mercado v. Manzano13 applicable to the instant case, since Valles and
Mercado were dual citizens since birth, unlike the petitioner who lost his Filipino citizenship by
means of naturalization. The COMELEC, thus, decreed in the aforementioned Resolution that:
ACCORDINGLY, NESTOR ARES JACOT is DISQUALIFIED to run for the position
of Vice-Mayor of Catarman, Camiguin for the May 14, 2007 National and Local
Elections. If proclaimed, respondent cannot thus assume the Office of Vice-Mayor of
said municipality by virtue of such disqualification.14
Petitioner filed a Motion for Reconsideration on 29 June 2007 reiterating his position that his
Oath of Allegiance to the Republic of the Philippines before the Los Angeles PCG and his
oath in his Certificate of Candidacy sufficed as an effective renunciation of his US citizenship.
Attached to the said Motion was an "Oath of Renunciation of Allegiance to the United States
and Renunciation of Any and All Foreign Citizenship" dated 27 June 2007, wherein petitioner
explicitly renounced his US citizenship.15 The COMELEC en banc dismissed petitioners
Motion in a Resolution16 dated 28 September 2007 for lack of merit.
Petitioner sought remedy from this Court via the present Special Civil Action for Certiorari
under Rule 65 of the Revised Rules of Court, where he presented for the first time an
"Affidavit of Renunciation of Allegiance to the United States and Any and All Foreign
Citizenship"17 dated 7 February 2007. He avers that he executed an act of renunciation of his
US citizenship, separate from the Oath of Allegiance to the Republic of the Philippines he took
before the Los Angeles PCG and his filing of his Certificate of Candidacy, thereby changing
his theory of the case during the appeal. He attributes the delay in the presentation of the
affidavit to his former counsel, Atty. Marciano Aparte, who allegedly advised him that said
piece of evidence was unnecessary but who, nevertheless, made him execute an identical
document entitled "Oath of Renunciation of Allegiance to the United States and Renunciation
of Any and All Foreign Citizenship" on 27 June 2007 after he had already filed his Certificate
of Candidacy.18
Petitioner raises the following issues for resolution of this Court:
I
WHETHER OR NOT PUBLIC RESPONDENT EXERCISED GRAVE ABUSE OF
DISCRETION WHEN IT HELD THAT PETITIONER FAILED TO COMPLY WITH
THE PROVISIONS OF R.A. 9225, OTHERWISE KNOWN AS THE "CITIZENSHIP
RETENTION AND RE-ACQUISITION ACT OF 2003," SPECIFICALLY SECTION
5(2) AS TO THE REQUIREMENTS FOR THOSE SEEKING ELECTIVE PUBLIC
OFFICE;
II
69

WHETHER OR NOT PUBLIC RESPONDENT EXERCISED GRAVE ABUSE OF


DISCRETION WHEN IT HELD THAT PETITIONER FAILED TO COMPLY WITH
THE PROVISIONS OF THE COMELEC RULES OF PROCEDURE AS REGARDS
THE PAYMENT OF THE NECESSARY MOTION FEES; AND
III
WHETHER OR NOT UPHOLDING THE DECISION OF PUBLIC RESPONDENT
WOULD RESULT IN THE FRUSTRATION OF THE WILL OF THE PEOPLE OF
CATARMAN, CAMIGUIN.19
The Court determines that the only fundamental issue in this case is whether petitioner is
disqualified from running as a candidate in the 14 May 2007 local elections for his failure to
make a personal and sworn renunciation of his US citizenship.
This Court finds that petitioner should indeed be disqualified.
Contrary to the assertions made by petitioner, his oath of allegiance to the Republic of the
Philippines made before the Los Angeles PCG and his Certificate of Candidacy do not
substantially comply with the requirement of a personal and sworn renunciation of foreign
citizenship because these are distinct requirements to be complied with for different purposes.
Section 3 of Republic Act No. 9225 requires that natural-born citizens of the Philippines,
who are already naturalized citizens of a foreign country, must take the following oath of
allegiance to the Republic of the Philippines to reacquire or retain their Philippine
citizenship:
SEC. 3. Retention of Philippine Citizenship.Any provision of law to the contrary
notwithstanding, natural-born citizens of the Philippines who have lost their
Philippine citizenship by reason of their naturalization as citizens of a foreign country
are hereby deemed to have reacquired Philippine citizenship upon taking the
following oath of allegiance to the Republic:
"I __________ solemnly swear (or affirm) that I will support and defend the
Constitution of the Republic of the Philippines and obey the laws and legal orders
promulgated by the duly constituted authorities of the Philippines; and I hereby
declare that I recognize and accept the supreme authority of the Philippines and will
maintain true faith and allegiance thereto; and that I impose this obligation upon
myself voluntarily, without mental reservation or purpose of evasion."
Natural-born citizens of the Philippines who, after the effectivity of this Act, become
citizens of a foreign country shall retain their Philippine citizenship upon taking the
aforesaid oath.
By the oath dictated in the afore-quoted provision, the Filipino swears allegiance to the
Philippines, but there is nothing therein on his renunciation of foreign citizenship. Precisely, a
situation might arise under Republic Act No. 9225 wherein said Filipino has dual citizenship by
also reacquiring or retaining his Philippine citizenship, despite his foreign citizenship.
The afore-quoted oath of allegiance is substantially similar to the one contained in the
Certificate of Candidacy which must be executed by any person who wishes to run for
public office in Philippine elections. Such an oath reads:
I am eligible for the office I seek to be elected. I will support and defend the
Constitution of the Philippines and will maintain true faith and allegiance thereto;

that I will obey the laws, legal orders and decrees promulgated by the duly
constituted authorities of the Republic of the Philippines; and that I impose this
obligation upon myself voluntarily, without mental reservation or purpose of evasion.
I hereby certify that the facts stated herein are true and correct of my own personal
knowledge.
Now, Section 5(2) of Republic Act No. 9225 specifically provides that:
Section 5. Civil and Political Rights and Liabilities.Those who retain or reacquire
Philippine citizenship under this Act shall enjoy full civil and political rights and be
subject to all attendant liabilities and responsibilities under existing laws of the
Philippines and the following conditions:
xxxx
(2) Those seeking elective public office in the Philippines shall meet the
qualifications for holding such public office as required by the Constitution and
existing laws and, at the time of the filing of the certificate of candidacy, make a
personal and sworn renunciation of any and all foreign citizenship before any public
officer authorized to administer an oath.
The law categorically requires persons seeking elective public office, who either retained their
Philippine citizenship or those who reacquired it, to make a personal and sworn renunciation
of any and all foreign citizenship before a public officer authorized to administer an oath
simultaneous with or before the filing of the certificate of candidacy.20
Hence, Section 5(2) of Republic Act No. 9225 compels natural-born Filipinos, who have
been naturalized as citizens of a foreign country, but who reacquired or retained their
Philippine citizenship (1) to take the oath of allegiance under Section 3 of Republic Act
No. 9225, and (2) for those seeking elective public offices in the Philippines, to
additionally execute a personal and sworn renunciation of any and all foreign citizenship
before an authorized public officer prior or simultaneous to the filing of their certificates of
candidacy, to qualify as candidates in Philippine elections.
Clearly Section 5(2) of Republic Act No. 9225 (on the making of a personal and sworn
renunciation of any and all foreign citizenship) requires of the Filipinos availing themselves of
the benefits under the said Act to accomplish an undertaking other than that which they have
presumably complied with under Section 3 thereof (oath of allegiance to the Republic of the
Philippines). This is made clear in the discussion of the Bicameral Conference Committee on
Disagreeing Provisions of House Bill No. 4720 and Senate Bill No. 2130 held on 18 August
2003 (precursors of Republic Act No. 9225), where the Hon. Chairman Franklin Drilon and
Hon. Representative Arthur Defensor explained to Hon. Representative Exequiel Javier that
the oath of allegiance is different from the renunciation of foreign citizenship:
CHAIRMAN DRILON. Okay. So, No. 2. "Those seeking elective public office in the
Philippines shall meet the qualifications for holding such public office as required by
the Constitution and existing laws and, at the time of the filing of the certificate of
candidacy, make a personal and sworn renunciation of any and all foreign
citizenship before any public officer authorized to administer an oath." I think its
very good, ha? No problem?
REP. JAVIER. I think its already covered by the oath.
CHAIRMAN DRILON. Renouncing foreign citizenship.
70

Lopez v. Commission on Elections24 is the more fitting precedent for this case since they both
share the same factual milieu. In Lopez, therein petitioner Lopez was a natural-born Filipino
who lost his Philippine citizenship after he became a naturalized US citizen. He later
reacquired his Philippine citizenship by virtue of Republic Act No. 9225. Thereafter, Lopez
filed his candidacy for a local elective position, but failed to make a personal and sworn
renunciation of his foreign citizenship. This Court unequivocally declared that despite having
garnered the highest number of votes in the election, Lopez is nonetheless disqualified as a
candidate for a local elective position due to his failure to comply with the requirements of
Section 5(2) of Republic Act No. 9225.
Petitioner presents before this Court for the first time, in the instant Petition for Certiorari, an
"Affidavit of Renunciation of Allegiance to the United States and Any and All Foreign
Citizenship,"25 which he supposedly executed on 7 February 2007, even before he filed his
Certificate of Candidacy on 26 March 2007. With the said Affidavit, petitioner puts forward in
the Petition at bar a new theory of his casethat he complied with the requirement of making a
personal and sworn renunciation of his foreign citizenship before filing his Certificate of
Candidacy. This new theory constitutes a radical change from the earlier position he took
before the COMELECthat he complied with the requirement of renunciation by his oaths of
allegiance to the Republic of the Philippines made before the Los Angeles PCG and in his
Certificate of Candidacy, and that there was no more need for a separate act of renunciation.
As a rule, no question will be entertained on appeal unless it has been raised in the
proceedings below. Points of law, theories, issues and arguments not brought to the attention
of the lower court, administrative agency or quasi-judicial body need not be considered by a
reviewing court, as they cannot be raised for the first time at that late stage. Basic
considerations of fairness and due process impel this rule.26 Courts have neither the time nor
the resources to accommodate parties who chose to go to trial haphazardly.27
Likewise, this Court does not countenance the late submission of evidence.28 Petitioner should
have offered the Affidavit dated 7 February 2007 during the proceedings before the
COMELEC.
Section 1 of Rule 43 of the COMELEC Rules of Procedure provides that "In the absence of
any applicable provisions of these Rules, the pertinent provisions of the Rules of Court in the
Philippines shall be applicable by analogy or in suppletory character and effect." Section 34 of
Rule 132 of the Revised Rules of Court categorically enjoins the admission of evidence not
formally presented:
SEC. 34. Offer of evidence. - The court shall consider no evidence which has not
been formally offered. The purpose for which the evidence is offered must be
specified.
Since the said Affidavit was not formally offered before the COMELEC, respondent had no
opportunity to examine and controvert it. To admit this document would be contrary to due
process. 29 Additionally, the piecemeal presentation of evidence is not in accord with orderly
justice.30
The Court further notes that petitioner had already presented before the COMELEC an
identical document, "Oath of Renunciation of Allegiance to the United States and
Renunciation of Any and All Foreign Citizenship" executed on 27 June 2007, subsequent to
his filing of his Certificate of Candidacy on 26 March 2007. Petitioner attached the said Oath
of 27 June 2007 to his Motion for Reconsideration with the COMELEC en banc. The

REP. JAVIER. Ah but he has taken his oath already.


CHAIRMAN DRILON. Nono, renouncing foreign citizenship.
xxxx
CHAIRMAN DRILON. Can I go back to No. 2. Whats your problem, Boy? Those
seeking elective office in the Philippines.
REP. JAVIER. They are trying to make him renounce his citizenship thinking that
ano
CHAIRMAN DRILON. His American citizenship.
REP. JAVIER. To discourage him from running?
CHAIRMAN DRILON. No.
REP. A.D. DEFENSOR. No. When he runs he will only have one citizenship.
When he runs for office, he will have only one. (Emphasis ours.)
There is little doubt, therefore, that the intent of the legislators was not only for Filipinos
reacquiring or retaining their Philippine citizenship under Republic Act No. 9225 to take their
oath of allegiance to the Republic of the Philippines, but also to explicitly renounce their
foreign citizenship if they wish to run for elective posts in the Philippines. To qualify as a
candidate in Philippine elections, Filipinos must only have one citizenship, namely, Philippine
citizenship.
By the same token, the oath of allegiance contained in the Certificate of Candidacy, which is
substantially similar to the one contained in Section 3 of Republic Act No. 9225, does not
constitute the personal and sworn renunciation sought under Section 5(2) of Republic Act No.
9225. It bears to emphasize that the said oath of allegiance is a general requirement for all
those who wish to run as candidates in Philippine elections; while the renunciation of foreign
citizenship is an additional requisite only for those who have retained or reacquired Philippine
citizenship under Republic Act No. 9225 and who seek elective public posts, considering their
special circumstance of having more than one citizenship.
Petitioner erroneously invokes the doctrine in Valles21 and Mercado,22 wherein the filing by a
person with dual citizenship of a certificate of candidacy, containing an oath of allegiance, was
already considered a renunciation of foreign citizenship. The ruling of this Court in Valles and
Mercado is not applicable to the present case, which is now specially governed by Republic
Act No. 9225, promulgated on 29 August 2003.
In Mercado, which was cited in Valles, the disqualification of therein private respondent
Manzano was sought under another law, Section 40(d) of the Local Government Code, which
reads:
SECTION 40. Disqualifications. The following persons are disqualified from running
for any elective local position:
xxxx
(d) Those with dual citizenship.
The Court in the aforesaid cases sought to define the term "dual citizenship" vis--vis the
concept of "dual allegiance." At the time this Court decided the cases of Valles and Mercado
on 26 May 1999 and 9 August 2000, respectively, the more explicitly worded requirements of
Section 5(2) of Republic Act No. 9225 were not yet enacted by our legislature.23
71

COMELEC en banc eventually refused to reconsider said document for being belatedly
executed. What was extremely perplexing, not to mention suspect, was that petitioner did not
submit the Affidavit of 7 February 2007 or mention it at all in the proceedings before the
COMELEC, considering that it could have easily won his case if it was actually executed on
and in existence before the filing of his Certificate of Candidacy, in compliance with law.
The justification offered by petitioner, that his counsel had advised him against presenting this
crucial piece of evidence, is lame and unconvincing. If the Affidavit of 7 February 2007 was in
existence all along, petitioners counsel, and even petitioner himself, could have easily
adduced it to be a crucial piece of evidence to prove compliance with the requirements of
Section 5(2) of Republic Act No. 9225. There was no apparent danger for petitioner to submit
as much evidence as possible in support of his case, than the risk of presenting too little for
which he could lose.
And even if it were true, petitioners excuse for the late presentation of the Affidavit of 7
February 2007 will not change the outcome of petitioners case.
It is a well-settled rule that a client is bound by his counsels conduct, negligence, and
mistakes in handling the case, and the client cannot be heard to complain that the result might
have been different had his lawyer proceeded differently.31 The only exceptions to the general
rule -- that a client is bound by the mistakes of his counsel -- which this Court finds acceptable
are when the reckless or gross negligence of counsel deprives the client of due process of
law, or when the application of the rule results in the outright deprivation of ones property
through a technicality.32 These exceptions are not attendant in this case.
The Court cannot sustain petitioners averment that his counsel was grossly negligent in
deciding against the presentation of the Affidavit of 7 February 2007 during the proceedings
before the COMELEC. Mistakes of attorneys as to the competency of a witness; the
sufficiency, relevancy or irrelevancy of certain evidence; the proper defense or the burden of
proof, failure to introduce evidence, to summon witnesses and to argue the case -- unless
they prejudice the client and prevent him from properly presenting his case -- do not constitute
gross incompetence or negligence, such that clients may no longer be bound by the acts of
their counsel.33
Also belying petitioners claim that his former counsel was grossly negligent was the fact that
petitioner continuously used his former counsels theory of the case. Even when the
COMELEC already rendered an adverse decision, he persistently argues even to this Court
that his oaths of allegiance to the Republic of the Philippines before the Los Angeles PCG and
in his Certificate of Candidacy amount to the renunciation of foreign citizenship which the law
requires. Having asserted the same defense in the instant Petition, petitioner only
demonstrates his continued reliance on and complete belief in the position taken by his former
counsel, despite the formers incongruous allegations that the latter has been grossly
negligent.
Petitioner himself is also guilty of negligence. If indeed he believed that his counsel was inept,
petitioner should have promptly taken action, such as discharging his counsel earlier and/or
insisting on the submission of his Affidavit of 7 February 2007 to the COMELEC, instead of
waiting until a decision was rendered disqualifying him and a resolution issued dismissing his
motion for reconsideration; and, thereupon, he could have heaped the blame on his former
counsel. Petitioner could not be so easily allowed to escape the consequences of his former

counsels acts, because, otherwise, it would render court proceedings indefinite, tentative, and
subject to reopening at any time by the mere subterfuge of replacing counsel. 34
Petitioner cites De Guzman v. Sandiganbayan,35 where therein petitioner De Guzman was
unable to present a piece of evidence because his lawyer proceeded to file a demurrer to
evidence, despite the Sandiganbayans denial of his prior leave to do so. The wrongful
insistence of the lawyer in filing a demurrer to evidence had totally deprived De Guzman of
any chance to present documentary evidence in his defense. This was certainly not the case
in the Petition at bar.
Herein, petitioner was in no way deprived of due process. His counsel actively defended his
suit by attending the hearings, filing the pleadings, and presenting evidence on petitioners
behalf. Moreover, petitioners cause was not defeated by a mere technicality, but because of a
mistaken reliance on a doctrine which is not applicable to his case. A case lost due to an
untenable legal position does not justify a deviation from the rule that clients are bound by the
acts and mistakes of their counsel.36
Petitioner also makes much of the fact that he received the highest number of votes for the
position of Vice-Mayor of Catarman during the 2007 local elections. The fact that a candidate,
who must comply with the election requirements applicable to dual citizens and failed to do so,
received the highest number of votes for an elective position does not dispense with, or
amount to a waiver of, such requirement.37 The will of the people as expressed through the
ballot cannot cure the vice of ineligibility, especially if they mistakenly believed that the
candidate was qualified. The rules on citizenship qualifications of a candidate must be strictly
applied. If a person seeks to serve the Republic of the Philippines, he must owe his loyalty to
this country only, abjuring and renouncing all fealty and fidelity to any other state.38 The
application of the constitutional and statutory provisions on disqualification is not a matter of
popularity.39
WHEREFORE, the instant appeal is DISMISSED. The Resolution dated 28 September 2007
of the COMELEC en banc in SPA No. 07-361, affirming the Resolution dated 12 June 2007 of
the COMELEC Second Division, is AFFIRMED. Petitioner is DISQUALIFIED to run for the
position of Vice-Mayor of Catarman, Camiguin in the 14 May 2007 National and Local
Elections, and if proclaimed, cannot assume the Office of Vice-Mayor of said municipality by
virtue of such disqualification. Costs against petitioner.
SO ORDERED.

72

G.R. No. 180048


June 19, 2009
ROSELLER DE GUZMAN, Petitioner,
vs.
COMMISSION ON ELECTIONS and ANGELINA DG. DELA CRUZ, Respondents.
DECISION
YNARES-SANTIAGO, J.:
This petition1 for certiorari with prayer for preliminary injunction and temporary restraining
order assails the June 15, 2007 Resolution2 of the First Division of the Commission on
Elections (COMELEC) in SPA No. 07-211, disqualifying petitioner Roseller De Guzman from
running as vice-mayor in the May 14, 2007 Synchronized National and Local Elections. Also
assailed is the October 9, 2007 Resolution3 of the COMELEC En Banc denying petitioners
motion for reconsideration.
Petitioner De Guzman and private respondent Angelina DG. Dela Cruz were candidates for
vice-mayor of Guimba, Nueva Ecija in the May 14, 2007 elections. On April 3, 2007, private
respondent filed against petitioner a petition4 for disqualification docketed as SPA No. 07-211,
alleging that petitioner is not a citizen of the Philippines, but an immigrant and resident of the
United States of America.
In his answer, petitioner admitted that he was a naturalized American. However, on January
25, 2006, he applied for dual citizenship under Republic Act No. 9225 (R.A. No. 9225),
otherwise known as the Citizenship Retention and Re-Acquisition Act of 2003.5 Upon approval
of his application, he took his oath of allegiance to the Republic of the Philippines on
September 6, 2006. He argued that, having re-acquired Philippine citizenship, he is entitled to
exercise full civil and political rights. As such, he is qualified to run as vice-mayor of Guimba,
Nueva Ecija.
During the May 14, 2007 elections, private respondent won as vice-mayor. Petitioner filed an
election protest on grounds of irregularities and massive cheating. The case was filed before
Branch 31 of the Regional Trial Court of Guimba, Nueva Ecija and was docketed as Election
Protest No. 07-01.
Meanwhile, in SPA No. 07-211, the COMELEC First Division rendered its June 15, 2007
Resolution disqualifying petitioner, which reads as follows:
Section 3 of R.A. No. 9225 states:
"Retention of Philippine Citizenship. Natural-born citizens of the Philippines who have lost
their Philippine citizenship by reason of their naturalization as citizens of a foreign country are
hereby deemed to have reacquired Philippine citizenship upon taking the following oath of
allegiance to the Republic: x x x"
Hence, under the provisions of the aforementioned law, respondent has validly reacquired
Filipino citizenship. By taking this Oath of Allegiance to the Republic of the Philippines on
September 6, 2006 before Mary Jo Bernardo Aragon, Deputy Consul General at the Philippine
Consulate General, Los Angeles, California respondent was deemed a dual citizen,
possessing both Filipino and American citizenship.
However, subparagraph (2), Section 5 of the aforementioned Act also provides:
Section 5. Civil and Political Rights and Liabilities -- Those who retain or re-acquire Philippine
Citizenship under this Act shall enjoy full civil and political rights and be subject to all attendant

liabilities and responsibilities under existing laws of the Philippines and the following
conditions:
xxxx
(2) Those seeking elective public office in the Philippines shall meet the qualifications for
holding such public office as required by the Constitution and existing laws and, at the time of
the filing of the certificate of candidacy, make a personal and sworn renunciation of any and
all foreign citizenship before any public officer authorized to administer an oath.
As can be gleaned from the above cited provision, respondent [herein petitioner] should have
renounced his American citizenship before he can run for any public elective position. This
respondent did not do. The Oath of Allegiance taken by respondent was for the purpose of reacquiring Philippine citizenship. It did not, at the same time, mean that respondent has
renounced his American citizenship. Thus, at the time respondent filed his certificate of
candidacy for the position of Vice-Mayor of Guimba, Nueva Ecija he was, and still is, a dual
citizen, possessing both Philippine and American citizenship. For this reason alone,
respondent is disqualified to run for the abovementioned elective position.
WHEREFORE, premises considered, the Commission (First Division) RESOLVED, as it
hereby RESOLVES, to GRANT the instant petition finding it IMBUED WITH MERIT. Hence,
respondent (petitioner herein) Roseller T. De Guzman is disqualified to run as Vice-Mayor of
Guimba, Nueva Ecija in the May 14, 2007 Synchronized National and Local Elections.6
Petitioner filed a motion for reconsideration but it was dismissed on October 9, 2007 by the
COMELEC En Banc for having been rendered moot in view of private respondents victory.
Thereafter, the trial court in Election Protest No. 07-01 rendered a Decision,7 dated November
26, 2007, declaring petitioner as the winner for the Vice-Mayoralty position. It held:
WHEREFORE, judgment is hereby rendered declaring protestant ROSELLER T. DE
GUZMAN, as the winner for the Vice-Mayoralty position with a plurality of 776 votes over the
protestee, ANGELINA D.G. DELA CRUZ, in the May 14, 2007 Local Elections in Guimba,
Nueva Ecija. With costs against the protestee.
There being no evidence presented as to the damages by both parties, the same are hereby
denied.
SO ORDERED.8
Petitioner filed the instant petition for certiorari, alleging that the COMELEC acted with grave
abuse of discretion in disqualifying him from running as Vice-Mayor because of his failure to
renounce his American citizenship, and in dismissing the motion for reconsideration for being
moot.
Petitioner invokes the rulings in Frivaldo v. Commission on Elections9 and Mercado v.
Manzano,10 that the filing by a person with dual citizenship of a certificate of candidacy,
containing an oath of allegiance, constituted as a renunciation of his foreign citizenship.
Moreover, he claims that the COMELEC En Banc prematurely dismissed the motion for
reconsideration because at that time, there was a pending election protest which was later
decided in his favor.
Meanwhile, private respondent claims that the passage of R.A. No. 9225 effectively
abandoned the Courts rulings in Frivaldo and Mercado; that the current law requires a
personal and sworn renunciation of any and all foreign citizenship; and that petitioner, having
73

failed to renounce his American citizenship, remains a dual citizen and is therefore disqualified
from running for an elective public position under Section 4011 of Republic Act No. 7160,
otherwise known as the Local Government Code of 1991 (LGC).
The issues for resolution are: 1) whether the COMELEC gravely abused its discretion in
dismissing petitioners motion for reconsideration for being moot; and 2) whether petitioner is
disqualified from running for vice-mayor of Guimba, Nueva Ecija in the May 14, 2007 elections
for having failed to renounce his American citizenship in accordance with R.A. No. 9225.
An issue becomes moot when it ceases to present a justifiable controversy so that a
determination thereof would be without practical use and value.12 In this case, the pendency of
petitioners election protest assailing the results of the election did not render moot the motion
for reconsideration which he filed assailing his disqualification. Stated otherwise, the issue of
petitioners citizenship did not become moot; the resolution of the issue remained relevant
because it could significantly affect the outcome of the election protest. Philippine citizenship
is an indispensable requirement for holding an elective office. As mandated by law: "An
elective local official must be a citizen of the Philippines."13 It bears stressing that the Regional
Trial Court later ruled in favor of petitioner in the election protest and declared him the winner.
In view thereof, a definitive ruling on the issue of petitioners citizenship was clearly
necessary. Hence, the COMELEC committed grave abuse of discretion in dismissing
petitioners motion for reconsideration solely on the ground that the same was rendered moot
because he lost to private respondent.
Anent the second issue, we find that petitioner is disqualified from running for public office in
view of his failure to renounce his American citizenship.
R.A. No. 9225 was enacted to allow re-acquisition and retention of Philippine citizenship for:
1) natural-born citizens who have lost their Philippine citizenship by reason of their
naturalization as citizens of a foreign country; and 2) natural-born citizens of the Philippines
who, after the effectivity of the law, become citizens of a foreign country. The law provides that
they are deemed to have re-acquired or retained their Philippine citizenship upon taking the
oath of allegiance.14
Petitioner falls under the first category, being a natural-born citizen who lost his Philippine
citizenship upon his naturalization as an American citizen. In the instant case, there is no
question that petitioner re-acquired his Philippine citizenship after taking the oath of allegiance
on September 6, 2006. However, it must be emphasized that R.A. No. 9225 imposes an
additional requirement on those who wish to seek elective public office, as follows:
Section 5. Civil and Political Rights and Liabilities. Those who retain or re-acquire Philippine
Citizenship under this Act shall enjoy full civil and political rights and be subject to all attendant
liabilities and responsibilities under existing laws of the Philippines and the following
conditions:
xxxx
(2) Those seeking elective public office in the Philippines shall meet the qualifications for
holding such public office as required by the Constitution and existing laws and, at the time of
the filing of the certificate of candidacy, make a personal and sworn renunciation of any and
all foreign citizenship before any public officer authorized to administer an oath.
Contrary to petitioners claims, the filing of a certificate of candidacy does not ipso facto
amount to a renunciation of his foreign citizenship under R.A. No. 9225. Our rulings in the

cases of Frivaldo and Mercado are not applicable to the instant case because R.A. No. 9225
provides for more requirements.
Thus, in Japzon v. COMELEC,15 the Court held that Section 5(2) of R.A. No. 9225 requires the
twin requirements of swearing to an Oath of Allegiance and executing a Renunciation of
Foreign Citizenship, viz:
Breaking down the afore-quoted provision, for a natural born Filipino, who reacquired or
retained his Philippine citizenship under Republic Act No. 9225, to run for public office, he
must: (1) meet the qualifications for holding such public office as required by the Constitution
and existing laws; and (2) make a personal and sworn renunciation of any and all foreign
citizenships before any public officer authorized to administer an oath.1awphi1
Further, in Jacot v. Dal and COMELEC,16 the Court ruled that a candidates oath of allegiance
to the Republic of the Philippines and his Certificate of Candidacy do not substantially comply
with the requirement of a personal and sworn renunciation of foreign citizenship. Thus:
The law categorically requires persons seeking elective public office, who either retained their
Philippine citizenship or those who reacquired it, to make a personal and sworn renunciation
of any and all foreign citizenship before a public officer authorized to administer an oath
simultaneous with or before the filing of the certificate of candidacy.
Hence, Section 5(2) of Republic Act No. 9225 compels natural-born Filipinos, who have been
naturalized as citizens of a foreign country, but who reacquired or retained their Philippine
citizenship (1) to take the oath of allegiance under Section 3 of Republic Act No. 9225, and (2)
for those seeking elective public offices in the Philippines, to additionally execute a personal
and sworn renunciation of any and all foreign citizenship before an authorized public officer
prior or simultaneous to the filing of their certificates of candidacy, to qualify as candidates in
Philippine elections.
Clearly Section 5(2) of Republic Act No. 9225 (on the making of a personal and sworn
renunciation of any and all foreign citizenship) requires of the Filipinos availing themselves of
the benefits under the said Act to accomplish an undertaking other than that which they have
presumably complied with under Section 3 thereof (oath of allegiance to the Republic of the
Philippines). This is made clear in the discussion of the Bicameral Conference Committee on
Disagreeing Provisions of House Bill No. 4720 and Senate Bill No. 2130 held on 18 August
2003 (precursors of Republic Act No. 9225), where the Hon. Chairman Franklin Drilon and
Hon. Representative Arthur Defensor explained to Hon. Representative Exequiel Javier that
the oath of allegiance is different from the renunciation of foreign citizenship:
CHAIRMAN DRILON. Okay. So, No. 2. "Those seeking elective public office in the
Philippines shall meet the qualifications for holding such public office as required by
the Constitution and existing laws and, at the time of the filing of the certificate of
candidacy, make a personal and sworn renunciation of any and all foreign
citizenship before any public officer authorized to administer an oath." I think its
very good, ha? No problem?
REP. JAVIER. I think its already covered by the oath.
CHAIRMAN DRILON. Renouncing foreign citizenship.
REP. JAVIER. Ah but he has taken his oath already.
CHAIRMAN DRILON. Nono, renouncing foreign citizenship.
74

xxxx
CHAIRMAN DRILON. Can I go back to No. 2. Whats your problem, Boy? Those
seeking elective office in the Philippines.
REP. JAVIER. They are trying to make him renounce his citizenship thinking that
ano
CHAIRMAN DRILON. His American citizenship.
REP. JAVIER. To discourage him from running?
CHAIRMAN DRILON. No.
REP. A.D. DEFENSOR. No. When he runs he will only have one citizenship. When
he runs for office, he will have only one. (Emphasis ours.)
There is little doubt, therefore, that the intent of the legislators was not only for Filipinos
reacquiring or retaining their Philippine citizenship under Republic Act No. 9225 to take their
oath of allegiance to the Republic of the Philippines, but also to explicitly renounce their
foreign citizenship if they wish to run for elective posts in the Philippines. To qualify as a
candidate in Philippine elections, Filipinos must only have one citizenship, namely, Philippine
citizenship.
By the same token, the oath of allegiance contained in the Certificate of Candidacy, which is
substantially similar to the one contained in Section 3 of Republic Act No. 9225, does not
constitute the personal and sworn renunciation sought under Section 5(2) of Republic Act No.
9225. It bears to emphasize that the said oath of allegiance is a general requirement for all
those who wish to run as candidates in Philippine elections; while the renunciation of foreign
citizenship is an additional requisite only for those who have retained or reacquired Philippine
citizenship under Republic Act No. 9225 and who seek elective public posts, considering their
special circumstance of having more than one citizenship.
In the instant case, petitioners Oath of Allegiance and Certificate of Candidacy did not comply
with Section 5(2) of R.A. No. 9225 which further requires those seeking elective public office
in the Philippines to make a personal and sworn renunciation of foreign citizenship. Petitioner
failed to renounce his American citizenship; as such, he is disqualified from running for vicemayor of Guimba, Nueva Ecija in the May 14, 2007 elections.
WHEREFORE, the petition is DISMISSED. Petitioner is declared DISQUALIFIED from
running for Vice-Mayor of Guimba, Nueva Ecija in the May 14, 2007 elections because of his
failure to renounce his foreign citizenship pursuant to Section 5(2) of R.A. No. 9225.
SO ORDERED.

75

G.R. No. 198742


August 10, 2012
TEODORA SOBEJANA-CONDON, Petitioner,
vs.
COMMISSION ON ELECTIONS, LUIS M. BAUTISTA, ROBELITO V. PICAR and WILMA P.
PAGADUAN, Respondents.
DECISION
REYES, J.:
Failure to renounce foreign citizenship in accordance with the exact tenor of Section 5(2) of
Republic Act (R.A.) No. 9225 renders a dual citizen ineligible to run for and thus hold any
elective public office.
The Case
1
At bar is a special civil action for certiorari under Rule 64 of the Rules of Court seeking to
nullify Resolution2 dated September 6, 2011 of the Commission on Elections (COMELEC) en
banc in EAC (AE) No. A-44-2010. The assailed resolution (a) reversed the Order3 dated
November 30, 2010 of COMELEC Second Division dismissing petitioners appeal; and (b)
affirmed the consolidated Decision4 dated October 22, 2010 of the Regional Trial Court (RTC),
Bauang, La Union, Branch 33, declaring petitioner Teodora Sobejana-Condon (petitioner)
disqualified and ineligible to her position as Vice-Mayor of Caba, La Union.
The Undisputed Facts
The petitioner is a natural-born Filipino citizen having been born of Filipino parents on August
8, 1944. On December 13, 1984, she became a naturalized Australian citizen owing to her
marriage to a certain Kevin Thomas Condon.
On December 2, 2005, she filed an application to re-acquire Philippine citizenship before the
Philippine Embassy in Canberra, Australia pursuant to Section 3 of R.A. No. 9225 otherwise
known as the "Citizenship Retention and Re-Acquisition Act of 2003."5 The application was
approved and the petitioner took her oath of allegiance to the Republic of the Philippines on
December 5, 2005.
On September 18, 2006, the petitioner filed an unsworn Declaration of Renunciation of
Australian Citizenship before the Department of Immigration and Indigenous Affairs,
Canberra, Australia, which in turn issued the Order dated September 27, 2006 certifying that
she has ceased to be an Australian citizen.6
The petitioner ran for Mayor in her hometown of Caba, La Union in the 2007 elections. She
lost in her bid. She again sought elective office during the May 10, 2010 elections this time for
the position of Vice-Mayor. She obtained the highest numbers of votes and was proclaimed as
the winning candidate. She took her oath of office on May 13, 2010.
Soon thereafter, private respondents Robelito V. Picar, Wilma P. Pagaduan7 and Luis M.
Bautista,8 (private respondents) all registered voters of Caba, La Union, filed separate
petitions for quo warranto questioning the petitioners eligibility before the RTC. The petitions
similarly sought the petitioners disqualification from holding her elective post on the ground
that she is a dual citizen and that she failed to execute a "personal and sworn renunciation of
any and all foreign citizenship before any public officer authorized to administer an oath" as
imposed by Section 5(2) of R.A. No. 9225.

The petitioner denied being a dual citizen and averred that since September 27, 2006, she
ceased to be an Australian citizen. She claimed that the Declaration of Renunciation of
Australian Citizenship she executed in Australia sufficiently complied with Section 5(2), R.A.
No. 9225 and that her act of running for public office is a clear abandonment of her Australian
citizenship.
Ruling of the RTC
In its consolidated Decision dated October 22, 2010, the trial court held that the petitioners
failure to comply with Section 5(2) of R.A. No. 9225 rendered her ineligible to run and hold
public office. As admitted by the petitioner herself during trial, the personal declaration of
renunciation she filed in Australia was not under oath. The law clearly mandates that the
document containing the renunciation of foreign citizenship must be sworn before any public
officer authorized to administer oath. Consequently, the RTCs decision disposed as follows:
WHEREFORE, premises considered, the Court renders judgment in FAVOR of [private
respondents] and AGAINST (petitioner):
1) DECLARING [petitioner] TEODORA SOBEJANA-CONDON, disqualified and ineligible to
hold the office of Vice-Mayor of Caba, La Union;
2) NULLIFYING her proclamation as the winning candidate for Vice-Mayor of said
municipality; and
3) DECLARING the position of Vice-Mayor in said municipality vacant.
SO ORDERED.9
Ruling of the COMELEC
The petitioner appealed to the COMELEC but the appeal was dismissed by the Second
Division in its Order10 dated November 30, 2010 for failure to pay the docket fees within the
prescribed period. On motion for reconsideration, the appeal was reinstated by the COMELEC
en banc in its Resolution11 dated September 6, 2011. In the same issuance, the substantive
merits of the appeal were given due course. The COMELEC en banc concurred with the
findings and conclusions of the RTC; it also granted the Motion for Execution Pending Appeal
filed by the private respondents.
The decretal portion of the resolution reads:
WHEREFORE, premises considered the Commission RESOLVED as it hereby RESOLVES
as follows:
1. To DISMISS the instant appeal for lack of merit;
2. To AFFIRM the DECISION dated 22 October 2010 of the court a quo; and
3. To GRANT the Motion for Execution filed on November 12, 2010.
SO ORDERED.12 (Emphasis supplied)
Hence, the present petition ascribing grave abuse of discretion to the COMELEC en banc.
The Petitioners Arguments
The petitioner contends that since she ceased to be an Australian citizen on September 27,
2006, she no longer held dual citizenship and was only a Filipino citizen when she filed her
certificate of candidacy as early as the 2007 elections. Hence, the "personal and sworn
renunciation of foreign citizenship" imposed by Section 5(2) of R.A. No. 9225 to dual citizens
seeking elective office does not apply to her.
76

She further argues that a sworn renunciation is a mere formal and not a mandatory
requirement. In support thereof, she cites portions of the Journal of the House of
Representatives dated June 2 to 5, 2003 containing the sponsorship speech for House Bill
(H.B.) No. 4720, the precursor of R.A. No. 9225.
She claims that the private respondents are estopped from questioning her eligibility since
they failed to do so when she filed certificates of candidacy for the 2007 and 2010 elections.
Lastly, she disputes the power of the COMELEC en banc to: (a) take cognizance of the
substantive merits of her appeal instead of remanding the same to the COMELEC Second
Division for the continuation of the appeal proceedings; and (b) allow the execution pending
appeal of the RTCs judgment.
The Issues
Posed for resolution are the following issues: I) Whether the COMELEC en banc may resolve
the merits of an appeal after ruling on its reinstatement; II) Whether the COMELEC en banc
may order the execution of a judgment rendered by a trial court in an election case; III)
Whether the private respondents are barred from questioning the qualifications of the
petitioner; and IV) For purposes of determining the petitioners eligibility to run for public office,
whether the "sworn renunciation of foreign citizenship" in Section 5(2) of R.A. No. 9225 is a
mere pro-forma requirement.
The Courts Ruling
I. An appeal may be simultaneously
reinstated and definitively resolved
by the COMELEC en banc in a
resolution disposing of a motion for
reconsideration.
The power to decide motions for reconsideration in election cases is arrogated unto the
COMELEC en banc by Section 3, Article IX-C of the Constitution, viz:
Sec. 3. The Commission on Elections may sit en banc or in two divisions, and shall
promulgate its rules of procedure in order to expedite disposition of election cases, including
pre-proclamation controversies. All such election cases shall be heard and decided in division,
provided that motions for reconsideration of decisions shall be decided by the Commission en
banc.
A complementary provision is present in Section 5(c), Rule 3 of the COMELEC Rules of
Procedure, to wit:
Any motion to reconsider a decision, resolution, order or ruling of a Division shall be resolved
by the Commission en banc except motions on interlocutory orders of the division which shall
be resolved by the division which issued the order.
Considering that the above cited provisos do not set any limits to the COMELEC en bancs
prerogative in resolving a motion for reconsideration, there is nothing to prevent the body from
directly adjudicating the substantive merits of an appeal after ruling for its reinstatement
instead of remanding the same to the division that initially dismissed it.
We thus see no impropriety much more grave abuse of discretion on the part of the
COMELEC en banc when it proceeded to decide the substantive merits of the petitioners
appeal after ruling for its reinstatement.

Further, records show that, in her motion for reconsideration before the COMELEC en banc,
the petitioner not only proffered arguments on the issue on docket fees but also on the issue
of her eligibility. She even filed a supplemental motion for reconsideration attaching therewith
supporting documents13 to her contention that she is no longer an Australian citizen. The
petitioner, after obtaining an unfavorable decision, cannot be permitted to disavow the en
bancs exercise of discretion on the substantial merits of her appeal when she herself invoked
the same in the first place.
The fact that the COMELEC en banc had remanded similar appeals to the Division that
initially dismissed them cannot serve as a precedent to the disposition of the petitioners
appeal. A decision or resolution of any adjudicating body can be disposed in several ways. To
sustain petitioners argument would be virtually putting a straightjacket on the COMELEC en
bancs adjudicatory powers.
More significantly, the remand of the appeal to the COMELEC Second Division would be
unnecessarily circuitous and repugnant to the rule on preferential disposition of quo warranto
cases espoused in Rule 36, Section 15 of the COMELEC Rules of Procedure.14
II. The COMELEC en banc has the
power to order discretionary
execution of judgment.
We cannot subscribe to petitioners submission that the COMELEC en banc has no power to
order the issuance of a writ of execution and that such function belongs only to the court of
origin.
There is no reason to dispute the COMELECs authority to order discretionary execution of
judgment in view of the fact that the suppletory application of the Rules of Court is expressly
sanctioned by Section 1, Rule 41 of the COMELEC Rules of Procedure.15
Under Section 2, Rule 39 of the Rules of Court, execution pending appeal may be issued by
an appellate court after the trial court has lost jurisdiction. In Batul v. Bayron,16 we stressed the
import of the provision vis--vis election cases when we held that judgments in election cases
which may be executed pending appeal includes those decided by trial courts and those
rendered by the COMELEC whether in the exercise of its original or appellate jurisdiction.
III. Private respondents are not
estopped from questioning
petitioners eligibility to hold public
office.
The fact that the petitioners qualifications were not questioned when she filed certificates of
candidacy for 2007 and 2010 elections cannot operate as an estoppel to the petition for quo
warranto before the RTC.
Under the Batas Pambansa Bilang 881 (Omnibus Election Code), there are two instances
where a petition questioning the qualifications of a registered candidate to run for the office for
which his certificate of candidacy was filed can be raised, to wit:
(1) Before election, pursuant to Section 78 thereof which provides that:
Sec. 78. Petition to deny due course or to cancel a certificate of candidacy. A
verified petition seeking to deny due course or to cancel a certificate of candidacy
may be filed by any person exclusively on the ground that any material
representation contained therein as required under Section 74 hereof is false. The
77

petition may be filed at any time not later than twenty-five days from the time of the
filing of the certificate of candidacy and shall be decided, after due notice and
hearing, not later than fifteen days before the election; and
(2) After election, pursuant to Section 253 thereof, viz:
Sec. 253. Petition for quo warranto. Any voter contesting the election of any
Member of the Batasang Pambansa, regional, provincial, or city officer on the
ground of ineligibility or of disloyalty to the Republic of the Philippines shall file a
sworn petition for quo warranto with the Commission within ten days after the
proclamation of the results of the election. (Emphasis ours)
Hence, if a person qualified to file a petition to disqualify a certain candidate fails to file the
petition within the twenty-five (25)-day period prescribed by Section 78 of the Omnibus
Election Code for whatever reasons, the elections laws do not leave him completely helpless
as he has another chance to raise the disqualification of the candidate by filing a petition for
quo warranto within ten (10) days from the proclamation of the results of the election, as
provided under Section 253 of the Omnibus Election Code.17
The above remedies were both available to the private respondents and their failure to utilize
Section 78 of the Omnibus Election Code cannot serve to bar them should they opt to file, as
they did so file, a quo warranto petition under Section 253.
IV. Petitioner is disqualified from
running for elective office for
failure to renounce her Australian
citizenship in accordance with
Section 5(2) of R.A. No. 9225.
R.A. No. 9225 allows the retention and re-acquisition of Filipino citizenship for natural-born
citizens who have lost their Philippine citizenship18 by taking an oath of allegiance to the
Republic, thus:
Section 3. Retention of Philippine Citizenship. Any provision of law to the contrary
notwithstanding, natural-born citizens of the Philippines who have lost their Philippine
citizenship by reason of their naturalization as citizens of a foreign country are hereby deemed
to have re-acquired Philippine citizenship upon taking the following oath of allegiance to the
Republic:
"I, _____________________, solemnly swear (or affirm) that I will support and defend the
Constitution of the Republic of the Philippines and obey the laws and legal orders
promulgated by the duly constituted authorities of the Philippines; and I hereby declare that I
recognize and accept the supreme authority of the Philippines and will maintain true faith and
allegiance thereto; and that I imposed this obligation upon myself voluntarily without mental
reservation or purpose of evasion."
Natural-born citizens of the Philippines who, after the effectivity of this Act, become citizens of
a foreign country shall retain their Philippine citizenship upon taking the aforesaid oath.
The oath is an abbreviated repatriation process that restores ones Filipino citizenship and all
civil and political rights and obligations concomitant therewith, subject to certain conditions
imposed in Section 5, viz:
Sec. 5. Civil and Political Rights and Liabilities. Those who retain or re-acquire Philippine
citizenship under this Act shall enjoy full civil and political rights and be subject to all attendant

liabilities and responsibilities under existing laws of the Philippines and the following
conditions:
(1) Those intending to exercise their right of suffrage must meet the requirements under
Section 1, Article V of the Constitution, Republic Act No. 9189, otherwise known as "The
Overseas Absentee Voting Act of 2003" and other existing laws;
(2) Those seeking elective public office in the Philippines shall meet the qualification for
holding such public office as required by the Constitution and existing laws and, at the time of
the filing of the certificate of candidacy, make a personal and sworn renunciation of any and
all foreign citizenship before any public officer authorized to administer an oath;
(3) Those appointed to any public office shall subscribe and swear to an oath of allegiance to
the Republic of the Philippines and its duly constituted authorities prior to their assumption of
office: Provided, That they renounce their oath of allegiance to the country where they took
that oath;
(4) Those intending to practice their profession in the Philippines shall apply with the proper
authority for a license or permit to engage in such practice; and
(5) That right to vote or be elected or appointed to any public office in the Philippines cannot
be exercised by, or extended to, those who:
(a) are candidates for or are occupying any public office in the country of which they
are naturalized citizens; and/or
(b) are in active service as commissioned or non-commissioned officers in the
armed forces of the country which they are naturalized citizens. (Emphasis ours)
Under the provisions of the aforementioned law, the petitioner has validly re-acquired her
Filipino citizenship when she took an Oath of Allegiance to the Republic of the Philippines on
December 5, 2005. At that point, she held dual citizenship, i.e., Australian and Philippine.
On September 18, 2006, or a year before she initially sought elective public office, she filed a
renunciation of Australian citizenship in Canberra, Australia. Admittedly, however, the same
was not under oath contrary to the exact mandate of Section 5(2) that the renunciation of
foreign citizenship must be sworn before an officer authorized to administer oath.
To obviate the fatal consequence of her inutile renunciation, the petitioner pleads the Court to
interpret the "sworn renunciation of any and all foreign citizenship" in Section 5(2) to be a
mere pro forma requirement in conformity with the intent of the Legislature. She anchors her
submission on the statement made by Representative Javier during the floor deliberations on
H.B. No. 4720, the precursor of R.A. No. 9225.
At the outset, it bears stressing that the Courts duty to interpret the law according to its true
intent is exercised only when the law is ambiguous or of doubtful meaning. The first and
fundamental duty of the Court is to apply the law. As such, when the law is clear and free from
any doubt, there is no occasion for construction or interpretation; there is only room for
application.19 Section 5(2) of R.A. No. 9225 is one such instance.
Ambiguity is a condition of admitting two or more meanings, of being understood in more than
one way, or of referring to two or more things at the same time. For a statute to be considered
ambiguous, it must admit of two or more possible meanings.20
The language of Section 5(2) is free from any ambiguity. In Lopez v. COMELEC,21 we
declared its categorical and single meaning: a Filipino American or any dual citizen cannot run
78

It is conclusively presumed to be the meaning that the Legislature has intended to convey.26
Even a resort to the Journal of the House of Representatives invoked by the petitioner leads
to the same inference, viz:
INTERPELLATION OF REP. JAVIER
Rep. Javier initially inquired whether under the Bill, dual citizenship is only limited to naturalborn Filipinos and not to naturalized Filipinos.
Rep. Libanan replied in the affirmative.
Rep. Javier subsequently adverted to Section 5 of the Bill which provides that natural-born
Filipinos who have dual citizenship shall continue to enjoy full civil and political rights. This
being the case, he sought clarification as to whether they can indeed run for public office
provided that they renounce their foreign citizenship.
Rep. Libanan replied in the affirmative, citing that these citizens will only have to make a
personal and sworn renunciation of foreign citizenship before any authorized public officer.
Rep. Javier sought further clarification on this matter, citing that while the Bill provides them
with full civil and political rights as Filipino citizens, the measure also discriminates against
them since they are required to make a sworn renunciation of their other foreign citizenship if
and when they run for public office. He thereafter proposed to delete this particular provision.
In his rejoinder, Rep. Libanan explained that this serves to erase all doubts regarding any
issues that might be raised pertaining to the citizenship of any candidate. He subsequently
cited the case of Afroyim vs. Rusk, wherein the United States considered a naturalized
American still as an American citizen even when he cast his vote in Israel during one of its
elections.
Rep. Javier however pointed out that the matter of voting is different because in voting, one is
not required to renounce his foreign citizenship. He pointed out that under the Bill, Filipinos
who run for public office must renounce their foreign citizenship. He pointed out further that
this is a contradiction in the Bill.
Thereafter, Rep. Javier inquired whether Filipino citizens who had acquired foreign citizenship
and are now entitled to reacquire their Filipino citizenship will be considered as natural-born
citizens. As such, he likewise inquired whether they will also be considered qualified to run for
the highest elective positions in the country.
Rep. Libanan replied in the affirmative, citing that the only requirement is that they make a
sworn renunciation of their foreign citizenship and that they comply with the residency and
registration requirements as provided for in the Constitution.
Whereupon, Rep. Javier noted that under the Constitution, natural-born citizens are those who
are citizens at the time of birth without having to perform an act to complete or perfect his/her
citizenship.
Rep. Libanan agreed therewith, citing that this is the reason why the Bill seeks the repeal of
CA No. 63. The repeal, he said, would help
Filipino citizens who acquired foreign citizenship to retain their citizenship. With regard then to
Section 5 of the Bill, he explained that the Committee had decided to include this provision
because Section 18, Article XI of the Constitution provides for the accountability of public
officers.

for any elective public position in the Philippines unless he or she personally swears to a
renunciation of all foreign citizenship at the time of filing the certificate of candidacy. We also
expounded on the form of the renunciation and held that to be valid, the renunciation must be
contained in an affidavit duly executed before an officer of the law who is authorized to
administer an oath stating in clear and unequivocal terms that affiant is renouncing all foreign
citizenship.
The same meaning was emphasized in Jacot v. Dal,22 when we held that Filipinos re-acquiring
or retaining their Philippine citizenship under R.A. No. 9225 must explicitly renounce their
foreign citizenship if they wish to run for elective posts in the Philippines, thus:
The law categorically requires persons seeking elective public office, who either retained their
Philippine citizenship or those who reacquired it, to make a personal and sworn renunciation
of any and all foreign citizenship before a public officer authorized to administer an oath
simultaneous with or before the filing of the certificate of candidacy.
Hence, Section 5(2) of Republic Act No. 9225 compels natural-born Filipinos, who have been
naturalized as citizens of a foreign country, but who reacquired or retained their Philippine
citizenship (1) to take the oath of allegiance under Section 3 of Republic Act No. 9225, and (2)
for those seeking elective public offices in the Philippines, to additionally execute a personal
and sworn renunciation of any and all foreign citizenship before an authorized public officer
prior or simultaneous to the filing of their certificates of candidacy, to qualify as candidates in
Philippine elections.
Clearly Section 5(2) of Republic Act No. 9225 (on the making of a personal and sworn
renunciation of any and all foreign citizenship) requires of the Filipinos availing themselves of
the benefits under the said Act to accomplish an undertaking other than that which they have
presumably complied with under Section 3 thereof (oath of allegiance to the Republic of the
Philippines). This is made clear in the discussion of the Bicameral Conference Committee on
Disagreeing Provisions of House Bill No. 4720 and Senate Bill No. 2130 held on 18 August
2003 (precursors of Republic Act No. 9225), where the Hon. Chairman Franklin Drilon and
Hon. Representative Arthur Defensor explained to Hon. Representative Exequiel Javier that
the oath of allegiance is different from the renunciation of foreign citizenship;
xxxx
The intent of the legislators was not only for Filipinos reacquiring or retaining their Philippine
citizenship under Republic Act No. 9225 to take their oath of allegiance to the Republic of the
Philippines, but also to explicitly renounce their foreign citizenship if they wish to run for
elective posts in the Philippines. To qualify as a candidate in Philippine elections, Filipinos
must only have one citizenship, namely, Philippine citizenship.23 (Citation omitted and italics
and underlining ours)
Hence, in De Guzman v. COMELEC,24 we declared petitioner therein to be disqualified from
running for the position of vice-mayor for his failure to make a personal and sworn
renunciation of his American citizenship.
We find no reason to depart from the mandatory nature infused by the above rulings to the
phrase "sworn renunciation". The language of the provision is plain and unambiguous. It
expresses a single, definite, and sensible meaning and must thus be read literally.25 The
foreign citizenship must be formally rejected through an affidavit duly sworn before an officer
authorized to administer oath.
79

In his rejoinder, Rep. Javier maintained that in this case, the sworn renunciation of a foreign
citizenship will only become a pro forma requirement.
On further queries of Rep. Javier, Rep. Libanan affirmed that natural-born Filipino citizens who
became foreign citizens and who have reacquired their Filipino citizenship under the Bill will
be considered as natural-born citizens, and therefore qualified to run for the presidency, the
vice-presidency or for a seat in Congress. He also agreed with the observation of Rep. Javier
that a natural-born citizen is one who is a citizen of the country at the time of birth. He also
explained that the Bill will, in effect, return to a Filipino citizen who has acquired foreign
citizenship, the status of being a natural-born citizen effective at the time he lost his Filipino
citizenship.
As a rejoinder, Rep. Javier opined that doing so would be discriminating against naturalized
Filipino citizens and Filipino citizens by election who are all disqualified to run for certain public
offices. He then suggested that the Bill be amended by not considering as natural-born
citizens those Filipinos who had renounced their Filipino citizenship and acquired foreign
citizenship. He said that they should be considered as repatriated citizens.
In reply, Rep. Libanan assured Rep. Javier that the Committee will take note of the latters
comments on the matter. He however stressed that after a lengthy deliberation on the subject,
the Committees on Justice, and Foreign Affairs had decided to revert back to the status of
being natural-born citizens those natural-born Filipino citizens who had acquired foreign
citizenship but now wished to reacquire their Filipino citizenship.
Rep. Javier then explained that a Filipina who loses her Filipino citizenship by virtue of her
marriage to a foreigner can regain her repatriated Filipino citizenship, upon the death of her
husband, by simply taking her oath before the Department of Justice (DOJ).
Rep. Javier said that he does not oppose the Bill but only wants to be fair to other Filipino
citizens who are not considered natural-born. He reiterated that natural-born Filipino citizens
who had renounced their citizenship by pledging allegiance to another sovereignty should not
be allowed to revert back to their status of being natural-born citizens once they decide to
regain their Filipino citizenship. He underscored that this will in a way allow such Filipinos to
enjoy dual citizenship.
On whether the Sponsors will agree to an amendment incorporating the position of Rep.
Javier, Rep. Libanan stated that this will defeat the purpose of the Bill.
Rep. Javier disagreed therewith, adding that natural-born Filipino citizens who acquired
foreign citizenships and later decided to regain their Filipino citizenship, will be considered as
repatriated citizens.
Rep. Libanan cited the case of Bengzon vs. HRET wherein the Supreme Court had ruled that
only naturalized Filipino citizens are not considered as natural-born citizens.
In reaction, Rep. Javier clarified that only citizens by election or those whose mothers are
Filipino citizens under the 1935 Constitution and who elected Filipino citizenship upon
reaching the age of maturity, are not deemed as natural-born citizens.
In response, Rep. Libanan maintained that in the Bengzon case, repatriation results in the
recovery of ones original nationality and only naturalized citizens are not considered as
natural-born citizens.

On whether the Sponsors would agree to not giving back the status of being natural-born
citizens to natural-born Filipino citizens who acquired foreign citizenship, Rep. Libanan
remarked that the Body in plenary session will decide on the matter.27
The petitioner obviously espouses an isolated reading of Representative Javiers statement;
she conveniently disregards the preceding and succeeding discussions in the records.
The above-quoted excerpts of the legislative record show that Representative Javiers
statement ought to be understood within the context of the issue then being discussed, that is
whether former natural-born citizens who re-acquire their Filipino citizenship under the
proposed law will revert to their original status as natural-born citizens and thus be qualified to
run for government positions reserved only to natural-born Filipinos, i.e. President, VicePresident and Members of the Congress.
It was Representative Javiers position that they should be considered as repatriated Filipinos
and not as natural-born citizens since they will have to execute a personal and sworn
renunciation of foreign citizenship. Natural-born citizens are those who need not perform an
act to perfect their citizenship. Representative Libanan, however, maintained that they will
revert to their original status as natural-born citizens. To reconcile the renunciation imposed
by Section 5(2) with the principle that natural-born citizens are those who need not perform
any act to perfect their citizenship, Representative Javier suggested that the sworn
renunciation of foreign citizenship be considered as a mere pro forma requirement.
Petitioners argument, therefore, loses its point. The "sworn renunciation of foreign citizenship"
must be deemed a formal requirement only with respect to the re-acquisition of ones status
as a natural-born Filipino so as to override the effect of the principle that natural-born citizens
need not perform any act to perfect their citizenship. Never was it mentioned or even alluded
to that, as the petitioner wants this Court to believe, those who re-acquire their Filipino
citizenship and thereafter run for public office has the option of executing an unsworn affidavit
of renunciation.
It is also palpable in the above records that Section 5 was intended to complement Section
18, Article XI of the Constitution on public officers primary accountability of allegiance and
loyalty, which provides:
Sec. 18. Public officers and employees owe the State and this Constitution allegiance at all
times and any public officer or employee who seeks to change his citizenship or acquire the
status of an immigrant of another country during his tenure shall be dealt with by law.
An oath is a solemn declaration, accompanied by a swearing to God or a revered person or
thing, that ones statement is true or that one will be bound to a promise. The person making
the oath implicitly invites punishment if the statement is untrue or the promise is broken. The
legal effect of an oath is to subject the person to penalties for perjury if the testimony is false. 28
Indeed, the solemn promise, and the risk of punishment attached to an oath ensures
truthfulness to the prospective public officers abandonment of his adopted state and promise
of absolute allegiance and loyalty to the Republic of the Philippines.
To hold the oath to be a mere pro forma requirement is to say that it is only for ceremonial
purposes; it would also accommodate a mere qualified or temporary allegiance from
government officers when the Constitution and the legislature clearly demand otherwise.
Petitioner contends that the Australian Citizenship Act of 1948, under which she is already
deemed to have lost her citizenship, is entitled to judicial notice. We disagree.
80

Foreign laws are not a matter of judicial notice. Like any other fact, they must be alleged and
proven.29 To prove a foreign law, the party invoking it must present a copy thereof and comply
with Sections 24 and 25 of Rule 132 of the Revised Rules of Court which reads:
Sec. 24. Proof of official record. The record of public documents referred to in paragraph (a)
of Section 19, when admissible for any purpose, may be evidenced by an official publication
thereof or by a copy attested by the officer having the legal custody of the record, or by his
deputy, and accompanied, if the record is not kept in the Philippines, with a certificate that
such officer has the custody. If the office in which the record is kept is in a foreign country, the
certificate may be made by a secretary of the embassy or legation, consul general, consul,
vice- consul, or consular agent or by any officer in the foreign service of the Philippines
stationed in the foreign country in which the record is kept, and authenticated by the seal of
his office. (Emphasis ours)
Sec. 25. What attestation of copy must state. Whenever a copy of a document or record is
attested for the purpose of the evidence, the attestation must state, in substance, that the
copy is a correct copy of the original, or a specific part thereof, as the case may be. The
attestation must be under the official seal of the attesting officer, if there be any, or if he be the
clerk of a court having a seal, under the seal of such court.
The Court has admitted certain exceptions to the above rules and held that the existence of a
foreign law may also be established through: (1) a testimony under oath of an expert witness
such as an attorney-at-law in the country where the foreign law operates wherein he quotes
verbatim a section of the law and states that the same was in force at the time material to the
facts at hand; and (2) likewise, in several naturalization cases, it was held by the Court that
evidence of the law of a foreign country on reciprocity regarding the acquisition of citizenship,
although not meeting the prescribed rule of practice, may be allowed and used as basis for
favorable action, if, in the light of all the circumstances, the Court is "satisfied of the
authenticity of the written proof offered." Thus, in a number of decisions, mere authentication
of the Chinese Naturalization Law by the Chinese Consulate General of Manila was held to be
a competent proof of that law.30
The petitioner failed to prove the Australian Citizenship Act of 1948 through any of the above
methods.1wphi1 As uniformly observed by the RTC and COMELEC, the petitioner failed to
show proof of the existence of the law during trial. Also, the letter issued by the Australian
government showing that petitioner already renounced her Australian citizenship was
unauthenticated hence, the courts a quo acted judiciously in disregarding the same.
We are bound to arrive at a similar conclusion even if we were to admit as competent
evidence the said letter in view of the photocopy of a Certificate of Authentication issued by
Consular Section of the Philippine Embassy in Canberra, Australia attached to the petitioners
motion for reconsideration.
We have stressed in Advocates and Adherents of Social Justice for School Teachers and
Allied Workers (AASJS) Member v. Datumanong31 that the framers of R.A. No. 9225 did not
intend the law to concern itself with the actual status of the other citizenship.
This Court as the government branch tasked to apply the enactments of the legislature must
do so conformably with the wisdom of the latter sans the interference of any foreign law. If we
were to read the Australian Citizen Act of 1948 into the application and operation of R.A. No.
9225, we would be applying not what our legislative department has deemed wise to require.

To do so would be a brazen encroachment upon the sovereign will and power of the people of
this Republic.32
The petitioners act of running for public office does not suffice to serve as an effective
renunciation of her Australian citizenship. While this Court has previously declared that the
filing by a person with dual citizenship of a certificate of candidacy is already considered a
renunciation of foreign citizenship,33 such ruling was already adjudged superseded by the
enactment of R.A. No. 9225 on August 29, 2003 which provides for the additional condition of
a personal and sworn renunciation of foreign citizenship.34
The fact that petitioner won the elections can not cure the defect of her candidacy. Garnering
the most number of votes does not validate the election of a disqualified candidate because
the application of the constitutional and statutory provisions on disqualification is not a matter
of popularity.35
In fine, R.A. No. 9225 categorically demands natural-born Filipinos who re-acquire their
citizenship and seek elective office, to execute a personal and sworn renunciation of any and
all foreign citizenships before an authorized public officer prior to or simultaneous to the filing
of their certificates of candidacy, to qualify as candidates in Philippine elections.36 The rule
applies to all those who have re-acquired their Filipino citizenship, like petitioner, without
regard as to whether they are still dual citizens or not. It is a pre-requisite imposed for the
exercise of the right to run for public office.
Stated differently, it is an additional qualification for elective office specific only to Filipino
citizens who re-acquire their citizenship under Section 3 of R.A. No. 9225. It is the operative
act that restores their right to run for public office. The petitioner's failure to comply therewith
in accordance with the exact tenor of the law, rendered ineffectual the Declaration of
Renunciation of Australian Citizenship she executed on September 18, 2006. As such, she is
yet to regain her political right to seek elective office. Unless she executes a sworn
renunciation of her Australian citizenship, she is ineligible to run for and hold any elective
office in the Philippines.
WHEREFORE, in view of all the foregoing, the petition is hereby DISMISSED. The Resolution
dated September 6, 2011 of the Commission on Elections en bane in EAC (AE) No. A-442010 is AFFIRMED in toto.
SO ORDERED.

81

G.R. No. 104654 June 6, 1994


REPUBLIC OF THE PHILIPPINES, petitioner,
vs.
HON. ROSALIO G. DE LA ROSA, PRESIDING JUDGE OF THE REGIONAL TRIAL
COURT, BRANCH 28, MANILA and JUAN G. FRIVALDO, respondents.
G.R. No. 105715 June 6, 1994
RAUL R. LEE, petitioner,
vs.
COMMISSION ON ELECTIONS and JUAN G. FRIVALDO, respondents.
G.R. No. 105735 June 6, 1994
RAUL R. LEE, petitioner,
vs.
COMMISSION ON ELECTIONS and JUAN G. FRIVALDO, respondents.
The Solicitor General for petitioner in G.R. No. 104654.
Yolando F. Lim counsel for private respondent.

that the deadline for filing the certificate of candidacy was March 15, one day before the
scheduled hearing. He asked that the hearing set on March 16 be cancelled and be moved to
January 24 (Rollo, pp. 27-28).
The motion was granted in an Order dated January 24, 1992, wherein the hearing of the
petition was moved to February 21, 1992. The said order was not published nor a copy
thereof posted.
On February 21, the hearing proceeded with private respondent as the sole witness. He
submitted the following documentary evidence: (1) Affidavit of Publication of the Order dated
October 7, 1991 issued by the publisher of The Philippine Star (Exh. "A"); (2) Certificate of
Publication of the order issued
by the National Printing Office (Exh. "B"); (3) Notice of Hearing of Petition (Exh. "B-1"); (4)
Photocopy of a Citation issued by the National Press Club with private respondents picture
(Exhs. "C" and "C-2"); (5) Certificate of Appreciation issued by the Rotary Club of Davao (Exh.
"D"); (6) Photocopy
of a Plaque of Appreciation issued by the Republican College, Quezon City (Exh. "E"); (7)
Photocopy of a Plaque of Appreciation issued by the Davao-Bicol Association (Exh. "F"); (8)
Certification issued by the Records Management and Archives Office that the record of birth of
private respondent was not on file (Exh. "G"); and (8) Certificate of Naturalization issued by
the United States District Court (Exh. "H").
Six days later, on February 27, respondent Judge rendered the assailed Decision, disposing
as follows:
WHEREFORE, the petition is GRANTED. Petitioner JUAN G. FRIVALDO,
is re-admitted as a citizen of the Republic of the Philippines by
naturalization, thereby vesting upon him, all the rights and privileges of a
natural born Filipino citizen (Rollo, p. 33).
On the same day, private respondent was allowed to take his oath of allegiance before
respondent Judge (Rollo, p. 34).
On March 16, a "Motion for Leave of Court to Intervene and to Admit Motion for
Reconsideration" was filed by Quiterio H. Hermo. He alleged that the proceedings were
tainted with jurisdictional defects, and prayed for a new trial to conform with the requirements
of the Naturalization Law.
After receiving a copy of the Decision on March 18, 1992, the Solicitor General interposed a
timely appeal directly with the Supreme Court.
G.R. No. 105715
This is a petition for certiorari, mandamus with injunction under Rule 65 of the Revised Rules
of Court in relation to Section 5(2) of Article VIII of the Constitution with prayer for temporary
restraining order filed by Raul R. Lee against the Commission on Elections (COMELEC) and
private respondent, to annul the en banc Resolution of the COMELEC, which dismissed his
petition docketed as SPC Case No. 92-273. The said petition sought to annul the
proclamation of private respondent as Governor-elect of the Province of Sorsogon.
Petitioner was the official candidate of the Laban ng Demokratikong Pilipino (LDP) for the
position of governor of the Province of Sorsogon in the May 1992 elections. Private
respondent was the official candidate of the Lakas-National Union of Christian Democrats
(Lakas-NUCD) for the same position.

QUIASON, J.:
In Frivaldo v. Commission on Elections, 174 SCRA 245 (1989), this Court declared private
respondent, Juan G. Frivaldo, an alien and therefore disqualified from serving as Governor of
the Province of Sorsogon.
Once more, the citizenship of private respondent is put in issue in
these petitions docketed as G.R. No.104654 and G.R. No. 105715 and G.R. No. 105735. The
petitions were consolidated since they principally involve the same issues and parties.
I
G.R. No. 104654
This is a petition for certiorari under Rule 45 of the Revised Rules of Court in relation to R.A.
No. 5440 and Section 25 of the Interim Rules, filed by the Republic of the Philippines: (1) to
annul the Decision dated February 27, 1992 of the Regional Trial Court, Branch 28, Manila, in
SP Proc. No. 91-58645, which re-admitted private respondent as a Filipino citizen under the
Revised Naturalization Law (C.A. No. 63 as amended by C.A. No. 473); and (2) to nullify the
oath of allegiance taken by private respondent on February 27, 1992.
On September 20, 1991, petitioner filed a petition for naturalization captioned: "In the Matter
of Petition of Juan G. Frivaldo to be Re-admitted as a Citizen of the Philippines under
Commonwealth Act No. 63" (Rollo, pp. 17-23).
In an Order dated October 7, 1991 respondent Judge set the petition for hearing on March 16,
1992, and directed the publication of the said order and petition in the Official Gazette and a
newspaper of general circulation, for three consecutive weeks, the last publication of which
should be at least six months before the said date of hearing. The order further required the
posting of a copy thereof and the petition in a conspicuous place in the Office of the Clerk of
Court of the Regional Trial Court, Manila (Rollo, pp. 24-26).
On January 14, 1992, private respondent filed a "Motion to Set Hearing Ahead of Schedule,"
where he manifested his intention to run for public office in the May 1992 elections. He alleged
82

Private respondent was proclaimed winner on May 22, 1992.


On June 1, petitioner filed a petition with the COMELEC to annul the proclamation of private
respondent as Governor-elect of the Province of Sorsogon on the grounds: (1) that the
proceedings and composition of the Provincial Board of Canvassers were not in accordance
with law; (2) that private respondent is an alien, whose grant of Philippine citizenship is being
questioned by the State in G.R. No. 104654; and (3) that private respondent is not a duly
registered voter. Petitioner further prayed that the votes case in favor of private respondent be
considered as stray votes, and that he, on the basis of the remaining valid votes cast, be
proclaimed winner.
On June 10, the COMELEC issued the questioned en banc resolution which dismissed the
petition for having been filed out of time, citing Section 19 of R.A. No. 7166. Said section
provides that the period to appeal a ruling of the board of canvassers on questions affecting
its composition or proceedings was three days.
In this petition, petitioner argues that the COMELEC acted with grave abuse of discretion
when it ignored the fundamental issue of private respondents disqualification in the guise of
technicality.
Petitioner claims that the inclusion of private respondents name in the list of registered voters
in Sta. Magdalena, Sorsogon was invalid because at the time he registered as a voter in 1987,
he was as American citizen.
Petitioner further claims that the grant of Filipino citizenship to private respondent is not yet
conclusive because the case is still on appeal before us.
Petitioner prays for: (1) the annulment of private respondents proclamation as Governor of
the Province of Sorsogon; (2) the deletion of private respondents name from the list of
candidates for the position of governor; (3) the proclamation of the governor-elect based on
the remaining votes, after the exclusion of the votes for private respondent; (4) the issuance of
a temporary restraining order to enjoin private respondent from taking his oath and assuming
office; and (5) the issuance of a writ of mandamus to compel the COMELEC to resolve the
pending disqualification case docketed as SPA Case No. 92-016, against private respondent.
G.R. No. 105735
This is a petition for mandamus under Rule 65 of the Revised Rules of Court in relation to
Section 5(2) of Article VIII of the Constitution, with prayer for temporary restraining order. The
parties herein are identical with the parties in G.R. No. 105715.
In substance, petitioner prays for the COMELECs immediate resolution of SPA Case No. 92016, which is a petition for the cancellation of private respondents certificate of candidacy
filed on March 23, 1992 by Quiterio H. Hermo, the intervenor in G.R. No. 104654 (Rollo, p.
18).
The petition for cancellation alleged: (1) that private respondent is an American citizen, and
therefore ineligible to run as candidate for the position of governor of the Province of
Sorsogon; (2) that the trial courts decision
re-admitting private respondent as a Filipino citizen was fraught with legal infirmities rendering
it null and void; (3) that assuming the decision to be valid, private respondents oath of
allegiance, which was taken on the same day the questioned decision was promulgated,
violated Republic Act No. 530, which provides for a two-year waiting period before the oath of
allegiance can be taken by the applicant; and (4) that the hearing of the petition on February

27, 1992, was held less than four months from the date of the last publication of the order and
petition. The petition prayed for the cancellation of private respondents certificate of
candidacy and the deletion of his name from the list of registered voters in Sta. Magdalena,
Sorsogon.
In his answer to the petition for cancellation, private respondent denied the allegations therein
and averred: (1) that Quiterio H. Hermo, not being a candidate for the same office for which
private respondent was aspiring, had no standing to file the petition; (2) that the decision readmitting him to Philippine citizenship was presumed to be valid; and (3) that no case had
been filed to exclude his name as a registered voter.
Raul R. Lee intervened in the petition for cancellation of private respondents certificate of
candidacy (Rollo, p. 37.).
On May 13, 1992, said intervenor urged the COMELEC to decide the petition for cancellation,
citing Section 78 of the Omnibus Election Code, which provides that all petitions on matters
involving the cancellation of a certificate of candidacy must be decided "not later than fifteen
days before election," and the case of Alonto v. Commission on Election, 22 SCRA 878
(1968), which ruled that all pre-proclamation controversies should be summarily decided
(Rollo,
p. 50).
The COMELEC concedes that private respondent has not yet reacquired his Filipino
citizenship because the decision granting him the same is not yet final and executory (Rollo,
p. 63). However, it submits that the issue of disqualification of a candidate is not among the
grounds allowed in a
pre-proclamation controversy, like SPC Case No. 92-273. Moreover, the said petition was filed
out of time.
The COMELEC contends that the preparation for the elections occupied much of its time, thus
its failure to immediately resolve SPA Case No. 92-016. It argues that under Section 5 of Rule
25 of the COMELEC Rules of Procedure, it is excused from deciding a disqualification case
within the period provided by law for reasons beyond its control. It also assumed that the
same action was subsequently abandoned by petitioner when he filed before it a petition for
quo warranto docketed as EPC No. 92-35. The quo warranto proceedings sought private
respondents disqualification because of his American citizenship.
II
G.R. No. 104654
We shall first resolve the issue concerning private respondents citizenship.
In his comment to the States appeal of the decision granting him Philippine citizenship in G.R.
No. 104654, private respondent alleges that the precarious political atmosphere in the country
during Martial Law compelled him to seek political asylum in the United States, and eventually
to renounce his Philippine citizenship.
He claims that his petition for naturalization was his only available remedy for his reacquisition
of Philippine citizenship. He tried to reacquire his Philippine citizenship through repatriation
and direct act of Congress. However, he was later informed that repatriation proceedings were
limited to army deserters or Filipino women who had lost their citizenship by reason of their
marriage to foreigners (Rollo, pp. 49-50). His request to Congress for sponsorship of a bill
allowing him to reacquire his Philippine citizenship failed to materialize, notwithstanding the
83

endorsement of several members of the House of Representatives in his favor (Rollo, p. 51).
He attributed this to the maneuvers of his political rivals.
He also claims that the re-scheduling of the hearing of the petition to an earlier date, without
publication, was made without objection from the Office of the Solicitor General. He makes
mention that on the date of the hearing, the court was jam-packed.
It is private respondents posture that there was substantial compliance with the law and that
the public was well-informed of his petition for naturalization due to the publicity given by the
media.
Anent the issue of the mandatory two-year waiting period prior to the taking of the oath of
allegiance, private respondent theorizes that the rationale of the law imposing the waiting
period is to grant the public an opportunity to investigate the background of the applicant and
to oppose the grant of Philippine citizenship if there is basis to do so. In his case, private
respondent alleges that such requirement may be dispensed with, claiming that his life, both
private and public, was well-known. Private respondent cites his achievement as a freedom
fighter and a former Governor of the Province of Sorsogon for six terms.
The appeal of the Solicitor General in behalf of the Republic of the Philippines is meritorious.
The naturalization proceedings in SP Proc. No. 91-58645 was full of procedural flaws,
rendering the decision an anomaly.
Private respondent, having opted to reacquire Philippine citizenship thru naturalization under
the Revised Naturalization Law, is duty bound to follow the procedure prescribed by the said
law. It is not for an applicant to decide for himself and to select the requirements which he
believes, even sincerely, are applicable to his case and discard those which be believes are
inconvenient or merely of nuisance value. The law does not distinguish between an applicant
who was formerly a Filipino citizen and one who was never such a citizen. It does not provide
a special procedure for the reacquisition of Philippine citizenship by former Filipino citizens
akin to the repatriation of a woman who had lost her Philippine citizenship by reason of her
marriage to an alien.
The trial court never acquired jurisdiction to hear the petition for naturalization of private
respondent. The proceedings conducted, the decision rendered and the oath of allegiance
taken therein, are null and void for failure to comply with the publication and posting
requirements under the Revised Naturalization Law.
Under Section 9 of the said law, both the petition for naturalization and the order setting it for
hearing must be published once a week for three consecutive weeks in the Official Gazette
and a newspaper of general circulation respondent cites his achievements as a freedom
fighter and a former Governor of the Province of Sorsogon for six terms.
The appeal of the Solicitor General in behalf of the Republic of
the Philippines is meritorious. The naturalization proceedings in SP Proc.
No. 91-58645 was full of procedural flaws, rendering the decision an anomaly.
Private respondent, having opted to reacquire Philippine citizenship thru naturalization under
the Revised Naturalization Law, is duty bound to follow the procedure prescribed by the said
law. It is not for an applicant to decide for himself and to select the requirements which he
believes, even sincerely, are applicable to his case and discard those which he believes are
inconvenient or merely of nuisance value. The law does not distinguish between an applicant
who was formerly a Filipino citizen and one who was never such a citizen. It does not provide

a special procedure for the reacquisition of Philippine citizenship by former Filipino citizens
akin to the repatriation of a woman who had lost her Philippine citizenship by reason of her
marriage to an alien.
The trial court never acquired jurisdiction to hear the petition for naturalization of private
respondent. The proceedings conducted, the decision rendered and the oath of allegiance
taken therein, are null and void for failure to comply with the publication and posting
requirements under the Revised Naturalization Law.
Under Section 9 of the said law, both the petition for naturalization and the order setting it for
hearing must be published once a week for three consecutive weeks in the Official Gazette
and a newspaper of general circulation. Compliance therewith is jurisdictional (Po Yi Bo v.
Republic, 205 SCRA 400 [1992]). Moreover, the publication and posting of the petition and the
order must be in its full test for the court to acquire jurisdiction (Sy v. Republic, 55 SCRA 724
[1974]).
The petition for naturalization lacks several allegations required by Sections 2 and 6 of the
Revised Naturalization Law, particularly: (1) that the petitioner is of good moral character; (2)
that he resided continuously in the Philippines for at least ten years; (3) that he is able to
speak and write English and any one of the principal dialects; (4) that he will reside
continuously in the Philippines from the date of the filing of the petition until his admission to
Philippine citizenship; and (5) that he has filed a declaration of intention or if he is excused
from said filing, the justification therefor.
The absence of such allegations is fatal to the petition (Po Yi Bi v. Republic, 205 SCRA 400
[1992]).
Likewise, the petition is not supported by the affidavit of at least two credible persons who
vouched for the good moral character of private respondent as required by Section 7 of the
Revised Naturalization Law. Private respondent also failed to attach a copy of his certificate of
arrival to the petition as required by Section 7 of the said law.
The proceedings of the trial court was marred by the following irregularities: (1) the hearing of
the petition was set ahead of the scheduled date of hearing, without a publication of the order
advancing the date of hearing, and the petition itself; (2) the petition was heard within six
months from the last publication of the petition; (3) petitioner was allowed to take his oath of
allegiance before the finality of the judgment; and (4) petitioner took his oath of allegiance
without observing the two-year waiting period.
A decision in a petition for naturalization becomes final only after 30 days from its
promulgation and, insofar as the Solicitor General is concerned, that period is counted from
the date of his receipt of the copy of the decision (Republic v. Court of First Instance of Albay,
60 SCRA 195 [1974]).
Section 1 of R.A. No. 530 provides that no decision granting citizenship in naturalization
proceedings shall be executory until after two years from its promulgation in order to be able
to observe if: (1) the applicant has left the country; (2) the applicant has dedicated himself
continuously to a lawful calling or profession; (3) the applicant has not been convicted of any
offense or violation of government promulgated rules; and (4) the applicant has committed any
act prejudicial to the interest of the country or contrary to government announced policies.
Even discounting the provisions of R.A. No. 530, the courts cannot implement any decision
granting the petition for naturalization before its finality.
84

G.R. No. 105715


In view of the finding in G.R. No. 104654 that private respondent is not yet a Filipino citizen,
we have to grant the petition in G.R. No. 105715 after treating it as a petition for certiorari
instead of a petition for mandamus. Said petition assails the en banc resolution of the
COMELEC, dismissing SPC Case No. 92-273, which in turn is a petition to annul private
respondents proclamation on three grounds: 1) that the proceedings and composition of the
Provincial Board of Canvassers were not in accordance with law; 2) that private respondent is
an alien, whose grant of Filipino citizenship is being questioned by the State in G.R. No.
104654; and 3) that private respondent is not a duly registered voter. The COMELEC
dismissed the petition on the grounds that it was filed outside the three-day period for
questioning the proceedings
and composition of the Provincial Board of Canvassers under Section 19 of R.A. No. 7166.
The COMELEC failed to resolve the more serious issue the disqualification of private
respondent to be proclaimed Governor on grounds of lack of Filipino citizenship. In this
aspect, the petition is one for quo warranto. In Frivaldo v. Commission on Elections, 174
SCRA 245 (1989), we held that a petition for quo warranto, questioning the respondents title
and seeking to prevent him from holding office as Governor for alienage, is not covered by the
ten-day period for appeal prescribed in Section 253 of the Omnibus Election Code.
Furthermore, we explained that "qualifications for public office are continuing requirements
and must be possessed not only at the time of appointment or election or assumption of office
but during the officers entire tenure; once any of the required qualification is lost, his title may
be seasonably challenged."
Petitioners argument, that to unseat him will frustrate the will of the electorate, is untenable.
Both the Local Government Code and the Constitution require that only Filipino citizens can
run and be elected to public office. We can only surmise that the electorate, at the time they
voted for private respondent, was of the mistaken belief that he had legally reacquired Filipino
citizenship.
Petitioner in G.R. No. 105715, prays that the votes cast in favor of private respondent be
considered stray and that he, being the candidate obtaining the second highest number of
votes, be declared winner. In Labo, Jr. v. COMELEC, 176 SCRA 1 (1989), we ruled that
where the candidate who obtained the highest number of votes is later declared to be
disqualified to hold the office to which he was elected, the candidate who garnered the second
highest number of votes is not entitled to be declared winner (See also Geronimo v. Ramos,
136 SCRA 435 [1985]; Topacio v. Paredes, 23 Phil. 238 [1912]).
G.R. No. 105735
In view of the discussions of G.R. No. 104654 and G.R. No. 105715, we find the petition in
G.R. No. 105735 moot and academic.
WHEREFORE, the petitions in G.R. No. 104654 and G.R. No. 105715 are both GRANTED
while the petition in G.R. No. 105735 is DISMISSED. Private respondent is declared NOT a
citizen of the Philippines and therefore DISQUALIFIED from continuing to serve as
GOVERNOR of the Province of Sorsogon. He is ordered to VACATE his office and to
SURRENDER the same to the Vice-Governor of the Province of Sorsogon once this decision
becomes final and executory. No pronouncement as to costs.
SO ORDERED.
85

G.R. No. 167824


July 2, 2010
GERALDINE GAW GUY and GRACE GUY CHEU, Petitioners,
vs.
ALVIN AGUSTIN T. IGNACIO, Respondent.
x - - - - - - - - - - - - - - - - - - - - - - -x
G.R. No. 168622
GERALDINE GAW GUY and GRACE GUY CHEU, Petitioners,
vs.
THE BOARD OF COMMISSIONERS OF THE BUREAU OF IMMIGRATION, HON. MARICEL
U. SALCEDO, MAYNARDO MARINAS, RICARDO CABOCHAN and ELISEO EXCONDE,
Respondents.
DECISION
PERALTA, J.:
This is a petition for review on certiorari1 under Rule 45 of the 1997 Rules of Civil Procedure
seeking, among others, to annul and set aside the Decisions dated January 6, 20052 and April
20, 20053 and Resolutions dated March 10, 20054 and June 29, 20055 rendered by the Court
of Appeals (CA), reversing and setting aside the Writ of Preliminary Injunction issued by the
Regional Trial Court6 (RTC), Branch 37, Manila.
The antecedent facts follow.
The father of petitioners Geraldine Gaw Guy and Grace Guy Cheu became a naturalized7
Filipino citizen sometime in 1959. The said petitioners, being minors at that time, were also
recognized8 as Filipino citizens.
Respondent Atty. Alvin Agustin T. Ignacio, filed a Complaint9 dated March 5, 2004 for
blacklisting and deportation against petitioners Geraldine and Grace before the Bureau of
Immigration (BI) on the basis that the latter two are Canadian citizens who are illegally
working in the Philippines, petitioners having been issued Canadian passports.
Acting upon the Complaint, respondent Maricel U. Salcedo, Special Prosecutor, Special Task
Force of the BI Commissioner, directed the petitioners, through the issuance of a
subpoenae,10 to appear before her and to bring pertinent documents relative to their current
immigration status, to which the petitioners objected by filing with the Special Task Force of
the BI Commissioner a Comment/Opposition with Motion Ad Cautelam to Quash Re:
Subpoena11 dated 30 April 2004 (Duces Tecum/Ad Testificandum), which was eventually
denied by respondent Salcedo in an Order12 dated May 14, 2004.
Respondent Board of Commissioners (BOC) filed a Charge Sheet13 dated June 1, 2004 for
Violation of Sections 37 (a) 7, 45 (e) and 45-A of the Philippine Immigration Act of 1940, as
amended, which reads as follows:
The undersigned Special Prosecutor charges GRACE GUY CHEU and GERALDINE GAW
GUY, both Canadian citizens, for working without permit, for fraudulently representing
themselves as Philippine citizens in order to evade immigration laws and for failure to comply
with the subpoena duces tecum/ad testificandum, in violation of the Philippine Immigration Act
of 1940, as amended, committed as follows:
That respondents GRACE GUY CHEU and GERALDINE GAW GUY, knowingly, willfully and
unlawfully engage in gainful activities in the Philippines without appropriate permit by working

as the Vice-President for Finance & Treasurer and General Manager, respectively, of
Northern Islands Company, Inc., with office address at No. 3 Mercury Avenue, Libis, Quezon
City;
That both respondents, knowingly, willfully and fraudulently misrepresent themselves as
Philippine citizens as reflected in the general Information Sheet of Northern Islands Company,
Inc., for 2004, in order to evade any requirement of the Philippine Immigration Laws;
That both respondents, duly served with subpoenas duces tecum/ad testificandum, dated
April 20, 2004, knowingly, willfully and unlawfully failed to comply with requirements
thereof.1avvphi1
CONTRARY TO LAW.
As a remedy, petitioners filed a Petition for Certiorari with Damages and a Prayer for Issuance
of a Temporary Restraining Order and Preliminary Injunction14 dated May 31, 2004 before the
RTC of Manila, Branch 37.15
The trial court, after hearing petitioner's application for issuance of a temporary restraining
order (TRO) and writ of preliminary injunction, issued an Order16 dated June 28, 2004, the
dispositive portion of which reads:
WHEREFORE, premises considered, the application for temporary restraining order is hereby
GRANTED. The respondents and all persons acting in their behalf and those under their
instructions are directed to cease and desist from continuing with the deportation proceedings
involving the petitioners. In the meantime set the case for hearing on preliminary injunction on
July 5 and 6, 2004, both at 2:00 o'clock in the afternoon and the respondents are directed to
show cause why writ of preliminary injunction should not issue.
SO ORDERED.
On July 5, 2004, public respondents filed their Answer17 and on July 13, 2004, filed a
Supplement (To the Special and Affirmative Defenses/Opposition to the Issuance of a Writ of
Preliminary Injunction).18 The parties were then directed to file their respective memoranda as
to the application for issuance of a writ of preliminary injunction and public respondents'
special and affirmative defenses. On July 16, 2004, public respondents as well as the
petitioners,19 filed their respective Memoranda.20 On the same day, respondent Atty. Ignacio
filed his Answer21 to the petition.
In an Order22 dated July 19, 2004, the trial court granted the application for preliminary
injunction enjoining public respondents from further continuing with the deportation
proceedings. The Order reads, in part:
In view of the foregoing, the Court finds that, indeed, there exists a pressing reason to issue a
writ of preliminary injunction to protect the rights of the petitioners pending hearing of the main
case on the merits and unless this Court issues a writ, grave irreparable injury would be
caused against the petitioners.
WHEREFORE, premises considered, the application for the Writ of Preliminary Injunction is
hereby GRANTED. The respondents and all persons acting on their behalf and those under
their instructions are directed to cease and desist from continuing with the deportation
proceedings involving the petitioners during the pendency of the instant case. The petitioners
are directed to post a bond in the amount of P50,000.00 to answer for whatever damages that
may be sustained by the respondent should the court finally resolve that the petitioners are
not entitled thereto.
86

SO ORDERED.
As a consequence, public respondents, on September 10, 2004, filed a Petition for Certiorari
with Prayer for Issuance of Temporary Restraining Order and Writ of Preliminary Injunction 23
before the CA24 and, on September 17, 2004, respondent Atty. Ignacio filed a Petition for
Certiorari,25 also with the CA.26 Both petitions prayed for the nullification of the Orders dated
June 28, 2004 and July 19, 2004 issued by the RTC in Civil Case No. 04-110179 and for the
dismissal of the petition therein. Later on, petitioner Geraldine filed a Motion to Consolidate
both petitions.
On January 6, 2005, the Ninth Division of the CA granted the petition filed by respondent Atty.
Ignacio and annulled the writ of preliminary injunction issued by the trial court, the dispositive
portion of the Decision27 reads:
WHEREFORE, the instant petition is GRANTED and the Order of the Regional Trial Court,
Branch 37, Manila, dated July 19, 2004, is hereby ANNULLED and SET ASIDE.
SO ORDERED.
On January 21, 2005, petitioners filed a Motion for Reconsideration.28
On March 1, 2005, petitioners reiterated29 their prayer for the consolidation of the petitions in
the Eighth and Ninth Divisions. In its Resolution30 dated March 10, 2005, the CA Ninth Division
denied petitioners' Motion for Reconsideration.
Hence, petitioners filed before this Court a Petition for Review on Certiorari31 dated March 31,
2005 praying for the reversal of the Decision rendered by the CA's Ninth Division, which is
now docketed as G.R. No. 167824.
Thereafter, the CA's Eighth Division rendered its own Decision32 dated April 29, 2005 granting
the petition therein and nullifying the Orders dated June 28 and July 19, 2004 in Civil Case
No. 04-110179, the dispositive portion of which reads as follows:
WHEREFORE, finding the instant petition impressed with merit and in accordance with our
decision in CA-G.R. SP No. 86432, the same is GIVEN DUE COURSE and is GRANTED. The
assailed Orders of the respondent court dated 28 June and 19 July 2004 are hereby
NULLIFIED and SET ASIDE.
SO ORDERED.
Petitioners filed their Motion for Reconsideration33 from the said Decision, which the CA
denied in its Resolution34 dated June 21, 2005.
Thus, petitioners filed before this Court a Petition for Review on Certiorari35 dated July 12,
2005 seeking to reverse and set aside the said Decision and Resolution rendered by the
Eighth Division of the CA and is now docketed as G.R. No. 168622. In its Resolution36 dated
August 10, 2005, the Court dismissed the said petition and said dismissal, despite petitioners'
motion for reconsideration,37 was affirmed in a Resolution38 dated October 17, 2005. This
Court, however, upon another motion for reconsideration39 filed by the petitioners, reinstated
the petition and ordered its consolidation with G.R. No. 167824.40
On September 7, 2007, a Manifestation41 was filed informing this Court that petitioner Grace
Guy Cheu died intestate on August 12, 2007 in the United States of America.
Petitioners raised the following grounds in their Consolidated Memorandum42 dated March 27,
2007:
I.

THE COURT OF APPEALS GRAVELY ABUSED ITS DISCRETION AND ERRED IN


HOLDING THAT THE LOWER COURT HAS NO JURISDICTION OVER CIVIL CASE NO. 04110179 AND ISSUE A WRIT OF PRELIMINARY INJUNCTION THEREIN CONSIDERING
THAT THE INSTANT CASE IS AN EXCEPTION TO THE RULE ON PRIMARY
JURISDICTION DOCTRINE AND WARRANTS PETITIONERS' IMMEDIATE RESORT TO
JUDICIAL INTERVENTION.
A.
CONSIDERING THAT PROOF OF PETITIONERS' PHILIPPINE CITIZENSHIP IS
SUBSTANTIAL, PETITIONERS ARE ALLOWED UNDER THIS HONORABLE
COURT'S RULING IN BID V. DELA ROSA, SUPRA, TO SEEK INJUNCTIVE
RELIEF FROM THE REGIONAL TRIAL COURT TO ENJOIN THE DEPORTATION
PROCEEDINGS CONDUCTED AGAINST THEM.
B.
LIKEWISE, CONSIDERING THAT PETITIONERS STAND TO SUFFER GRAVE
AND IRREPARABLE INJURIES SHOULD THE DEPORTATION PROCEEDINGS
AGAINST THEM BE ALLOWED TO CONTINUE, PETITIONERS ARE ALLOWED
UNDER TE LAW TO IMMEDIATELY SEEK JUDICIAL RELIEF DESPITE THE
PENDENCY OF THE ADMINISTRATIVE PROCEEDINGS.
II.
FURTHER, IT IS RESPECTFULLY SUBMITTED THAT THE RULING OF THIS HONORABLE
COURT IN DWIKARNA V. DOMINGO, 433 SCRA 748 (2004) DID NOT STRIP THE LOWER
COURT OF ITS AUTHORITY TO ENTERTAIN THE PETITION IN CIVIL CASE NO. 04110179 AND TO ISSUE A WRIT OF PRELIMINARY INJUNCTION IN THE AFORESAID
CASE.
III.
EVEN IF THE RULING OF THIS HONORABLE COURT IN DWIKARNA V. DOMINGO,
SUPRA, DID STRIP THE LOWER COURT OF ITS JURISDICTION IN BID V. DELA ROSA,
SUPRA, TO ENJOIN DEPORTATION PROCEEDINGS, THE RULING CAN ONLY HAVE
PROSPECTIVE EFFECT.
Basically, petitioners argue that the doctrine of primary jurisdiction, relied upon by the CA in its
decision, does not apply in the present case because it falls under an exception. Citing Board
of Commissioners (CID) v. Dela Rosa,43 petitioners assert that immediate judicial intervention
in deportation proceedings is allowed where the claim of citizenship is so substantial that there
are reasonable grounds to believe that the claim is correct. In connection therewith, petitioners
assail the applicability of Dwikarna v. Domingo in the present case, which the CA relied upon
in ruling against the same petitioners.
After a careful study of the arguments presented by the parties, this Court finds the petition
meritorious.
Petitioners rely on Board of Commissioners (CID) v. Dela Rosa,44 wherein this Court ruled that
when the claim of citizenship is so substantial as to reasonably believe it to be true, a
respondent in a deportation proceeding can seek judicial relief to enjoin respondent BOC from
proceeding with the deportation case. In particular, petitioners cited the following portions in
this Court's decision:
87

True, it is beyond cavil that the Bureau of Immigration has the exclusive authority and
jurisdiction to try and hear cases against an alleged alien, and in the process, determine also
their citizenship (Lao vs. Court of Appeals, 180 SCRA 756 [1089]. And a mere claim of
citizenship cannot operate to divest the Board of Commissioners of its jurisdiction in
deportation proceedings (Miranda vs. Deportation Board, 94 Phil. 531 [1951]).
However, the rule enunciated in the above-cases admits of an exception, at least insofar
as deportation proceedings are concerned. Thus, what if the claim to citizenship of the
alleged deportee is satisfactory? Should the deportation proceedings be allowed to continue
or should the question of citizenship be ventilated in a judicial proceeding? In Chua Hiong vs.
Deportation Board (96 Phil. 665 [1955]), this Court answered the question in the affirmative,
and We quote:
When the evidence submitted by a respondent is conclusive of his citizenship, the right
to immediate review should also be recognized and the courts should promptly enjoin
the deportation proceedings. A citizen is entitled to live in peace, without molestation
from any official or authority, and if he is disturbed by a deportation proceeding, he has
the unquestionable right to resort to the courts for his protection, either by a writ of
habeas corpus or of prohibition, on the legal ground that the Board lacks jurisdiction. If
he is a citizen and evidence thereof is satisfactory, there is no sense nor justice in allowing the
deportation proceedings to continue, granting him the remedy only after the Board has
finished its investigation of his undesirability.
x x x And if the right (to peace) is precious and valuable at all, it must also be protected
on time, to prevent undue harassment at the hands of ill-meaning or misinformed
administrative officials. Of what use is this much boasted right to peace and liberty if it
can be availed of only after the Deportation Board has unjustly trampled upon it,
besmirching the citizen's name before the bar of public opinion?
The doctrine of primary jurisdiction of petitioners Board of Commissioners over
deportation proceedings is, therefore, not without exception (Calayday vs. Vivo, 33
SCRA 413 [1970]; Vivo vs. Montesa, 24 SCRA 155 [1967]). Judicial intervention, however,
should be granted in cases where the claim of citizenship is so substantial that there are
reasonable grounds to believe that the claim is correct. In other words, the remedy should
be allowed only on sound discretion of a competent court in a proper proceeding (Chua
Hiong v. Deportation Board, supra; Co vs. Deportation Board, 78 SCRA 107 [1977]). It
appearing from the records that respondent's claim of citizenship is substantial, as We
shall show later, judicial intervention should be allowed.45
The present case, as correctly pointed out by petitioners and wrongfully found by the CA, falls
within the above-cited exception considering that proof of their Philippine citizenship had been
adduced, such as, the identification numbers46 issued by the Bureau of Immigration confirming
their Philippine citizenship, they have duly exercised and enjoyed all the rights and privileges
exclusively accorded to Filipino citizens, i.e., their Philippine passports47 issued by the
Department of Foreign Affairs.
In BOC v. Dela Rosa, it is required that before judicial intervention is sought, the claim of
citizenship of a respondent in a deportation proceeding must be so substantial that there are
reasonable grounds to believe that such claim is correct. In the said case, the proof adduced
by the respondent therein was so substantial and conclusive as to his citizenship that it
warranted a judicial intervention. In the present case, there is a substantial or conclusive

evidence that petitioners are Filipino citizens. Without necessarily judging the case on its
merits, as to whether petitioners had lost their Filipino citizenship by having a Canadian
passport, the fact still remains, through the evidence adduced and undisputed by the
respondents, that they are naturalized Filipinos, unless proven otherwise.
However, this Court cannot pass upon the issue of petitioners' citizenship as this was not
raised as an issue. The issue in this petition is on the matter of jurisdiction, and as discussed
above, the trial court has jurisdiction to pass upon the issue whether petitioners have
abandoned their Filipino citizenship or have acquired dual citizenship within the confines of
the law.
In this regard, it must be remembered though that this Court's ruling in Dwikarna v. Domingo
did not abandon the doctrine laid down in BOC v. Dela Rosa. The exception remains.
Dwikarna merely reiterated the doctrine of primary jurisdiction when this Court ruled that if the
petitioner is dissatisfied with the decision of the Board of Commissioners of the
Bureau of Immigration, he can move for its reconsideration and if his motion is denied,
then he can elevate his case by way of a petition for review before the Court of
Appeals, pursuant to Section 1, Rule 43 of the Rules of Civil Procedure. However, utmost
caution must be exercised in availing of the exception laid down in BOC v. Dela Rosa in order
to avoid trampling on the time-honored doctrine of primary jurisdiction. The court cannot or will
not determine a controversy involving a question which is within the jurisdiction of the
administrative tribunal prior to resolving the same, where the question demands the exercise
of sound administrative discretion requiring special knowledge, experience and services in
determining technical and intricate matters of fact.48 In cases where the doctrine of primary
jurisdiction is clearly applicable, the court cannot arrogate unto itself the authority to resolve a
controversy, the jurisdiction over which is initially lodged with an administrative body of special
competence.49
Above all else, this Court still upholds the doctrine of primary jurisdiction. As enunciated in
Republic v. Lacap:50
The general rule is that before a party may seek the intervention of the court, he should first
avail of all the means afforded him by administrative processes.51 The issues which
administrative agencies are authorized to decide should not be summarily taken from them
and submitted to a court without first giving such administrative agency the opportunity to
dispose of the same after due deliberation.52
Corollary to the doctrine of exhaustion of administrative remedies is the doctrine of primary
jurisdiction; that is, courts cannot or will not determine a controversy involving a question
which is within the jurisdiction of the administrative tribunal prior to the resolution of that
question by the administrative tribunal, where the question demands the exercise of sound
administrative discretion requiring the special knowledge, experience and services of the
administrative tribunal to determine technical and intricate matters of fact.53
Nonetheless, the doctrine of exhaustion of administrative remedies and the corollary doctrine
of primary jurisdiction, which are based on sound public policy and practical considerations,
are not inflexible rules. There are many accepted exceptions, such as: (a) where there is
estoppel on the part of the party invoking the doctrine; (b) where the challenged administrative
act is patently illegal, amounting to lack of jurisdiction; (c) where there is unreasonable delay
or official inaction that will irretrievably prejudice the complainant; (d) where the amount
involved is relatively small so as to make the rule impractical and oppressive; (e) where the
88

question involved is purely legal and will ultimately have to be decided by the courts of
justice;54 (f) where judicial intervention is urgent; (g) when its application may cause great and
irreparable damage; (h) where the controverted acts violate due process; (i) when the issue of
non-exhaustion of administrative remedies has been rendered moot;55 (j) when there is no
other plain, speedy and adequate remedy; (k) when strong public interest is involved; and, (l)
in quo warranto proceedings. x x x56
WHEREFORE, the petition is GRANTED. Consequently, the Decisions dated January 6, 2005
and April 20, 2005, and the Resolutions dated March 10, 2005 and June 29, 2005 of the Court
of Appeals, nullifying and setting aside the Writ of Preliminary Injunction issued by the
Regional Trial Court (RTC), Branch 37, Manila, are hereby NULLIFIED and SET ASIDE. The
case is hereby remanded to the trial court for further proceedings, with dispatch.
SO ORDERED.

89

furtherance of said motion for release dated 14 December 1988, 11 motion to set case for oral
argument dated 8 December 1988. 12
Acting on the motion to lift the temporary restraining order (issued on 7 December 1988)
dated 9 December 1988, 13 and the vigorous opposition to lift restraining order dated 15
December 1988, 14 the Court resolved to give petitioner Yu a non-extendible period of three
(3) days from notice within which to explain and prove why he should still be considered a
citizen of the Philippines despite his acquisition and use of a Portuguese passport. 15
Petitioner filed his compliance with the resolution of 15 December 1988 on 20 December 1988
16
followed by an earnest request for temporary release on 22 December 1988. Respondent
filed on 2 January 1989 her comment reiterating her previous motion to lift temporary
restraining order. Petitioner filed a reply thereto on 6 January 1989.
Petitioner's own compliance reveals that he was originally issued a Portuguese passport in
1971, 17 valid for five (5) years and renewed for the same period upon presentment before the
proper Portuguese consular officer. Despite his naturalization as a Philippine citizen on 10
February 1978, on 21 July 1981, petitioner applied for and was issued Portuguese Passport
No. 35/81 serial N. 1517410 by the Consular Section of the Portuguese Embassy in Tokyo.
Said Consular Office certifies that his Portuguese passport expired on 20 July 1986. 18 While
still a citizen of the Philippines who had renounced, upon his naturalization, "absolutely and
forever all allegiance and fidelity to any foreign prince, potentate, state or sovereignty" and
pledged to "maintain true faith and allegiance to the Republic of the Philippines," 19 he
declared his nationality as Portuguese in commercial documents he signed, specifically, the
Companies registry of Tai Shun Estate Ltd. 20 filed in Hongkong sometime in April 1980.
To the mind of the Court, the foregoing acts considered together constitute an express
renunciation of petitioner's Philippine citizenship acquired through naturalization. In Board of
Immigration Commissioners us, Go Gallano, 21 express renunciation was held to mean a
renunciation that is made known distinctly and explicitly and not left to inference or implication.
Petitioner, with full knowledge, and legal capacity, after having renounced Portuguese
citizenship upon naturalization as a Philippine citizen 22 resumed or reacquired his prior status
as a Portuguese citizen, applied for a renewal of his Portuguese passport 23 and represented
himself as such in official documents even after he had become a naturalized Philippine
citizen. Such resumption or reacquisition of Portuguese citizenship is grossly inconsistent with
his maintenance of Philippine citizenship.
This Court issued the aforementioned TRO pending hearings with the Board of Special
Inquiry, CID. However, pleadings submitted before this Court after the issuance of said TRO
have unequivocally shown that petitioner has expressly renounced his Philippine citizenship.
The material facts are not only established by the pleadings they are not disputed by
petitioner. A rehearing on this point with the CID would be unnecessary and superfluous.
Denial, if any, of due process was obviated when petitioner was given by the Court the
opportunity to show proof of continued Philippine citizenship, but he has failed.
While normally the question of whether or not a person has renounced his Philippine
citizenship should be heard before a trial court of law in adversary proceedings, this has
become unnecessary as this Court, no less, upon the insistence of petitioner, had to look into
the facts and satisfy itself on whether or not petitioner's claim to continued Philippine
citizenship is meritorious.

G.R. No. L-83882 January 24, 1989


IN RE PETITION FOR HABEAS CORPUS OF WILLIE YU, petitioner,
vs.
MIRIAM DEFENSOR-SANTIAGO, BIENVENIDO P. ALANO, JR., MAJOR PABALAN,
DELEO HERNANDEZ, BLODDY HERNANDEZ, BENNY REYES and JUN ESPIRITU
SANTO, respondent.
Pelaez, Adriano and Gregorio and Bonifacio A. Alentajan for petitioner.
Chavez, Hechanova & Lim Law Offices collaborating counsel for petitioner.
Augusto Jose y. Arreza for respondents.
PADILLA, J.:
The present controversy originated with a petition for habeas corpus filed with the Court on 4
July 1988 seeking the release from detention of herein petitioner. 1 After manifestation and
motion of the Solicitor General of his decision to refrain from filing a return of the writ on behalf
of the CID, respondent Commissioner thru counsel filed the return. 2 Counsel for the parties
were heard in oral argument on 20 July 1988. The parties were allowed to submit marked
exhibits, and to file memoranda. 3 An internal resolution of 7 November 1988 referred the case
to the Court en banc. In its 10 November 1988 resolution, denying the petition for habeas
corpus, the Court disposed of the pending issues of (1) jurisdiction of the CID over a
naturalized Filipino citizen and (2) validity of warrantless arrest and detention of the same
person.
Petitioner filed a motion for reconsideration with prayer for restraining order dated 24
November 1988. 4 On 29 November 1988, the Court resolved to deny with finality the
aforesaid motion for reconsideration, and further resolved to deny the urgent motion for
issuance of a restraining order dated 28 November 1988. 5
Undaunted, petitioner filed a motion for clarification with prayer for restraining order on 5
December 1988.
Acting on said motion, a temporary restraining order was issued by the Court on 7 December
1988. 6 Respondent Commissioner filed a motion to lift TRO on 13 December 1988, the basis
of which is a summary judgment of deportation against Yu issued by the CID Board of
Commissioners on 2 December 1988. 7 Petitioner also filed a motion to set case for oral
argument on 8 December 1988.
In the meantime, an urgent motion for release from arbitrary detention 8 was filed by petitioner
on 13 December 1988. A memorandum in furtherance of said motion for release dated 14
December 1988 was filed on 15 December 1988 together with a vigorous opposition to the
lifting of the TRO.
The lifting of the Temporary Restraining Order issued by the Court on 7 December 1988 is
urgently sought by respondent Commissioner who was ordered to cease and desist from
immediately deporting petitioner Yu pending the conclusion of hearings before the Board of
Special Inquiry, CID. To finally dispose of the case, the Court will likewise rule on petitioner's
motion for clarification with prayer for restraining order dated 5 December 1988, 9 urgent
motion for release from arbitrary detention dated 13 December 1988, 10 the memorandum in

90

Philippine citizenship, it must be stressed, is not a commodity or were to be displayed when


required and suppressed when convenient. This then resolves adverse to the petitioner his
motion for clarification and other motions mentioned in the second paragraph, page 3 of this
Decision.
WHEREFORE, premises considered, petitioner's motion for release from detention is
DENIED. Respondent's motion to lift the temporary restraining order is GRANTED. This
Decision is immediately executory.
SO ORDERED.

91

G.R. No. 83820 May 25, 1990


JOSE B. AZNAR (as Provincial Chairman of PDP Laban in Cebu), petitioner,
vs.
COMMISSION ON ELECTIONS and EMILIO MARIO RENNER OSMEA, respondents.
Rufino B. Requina for petitioner.
Angara, Abello, Concepcion, Regala & Cruz for private respondent.

birth and has not gone out of the country for more than six months; and that he has been a
registered voter in the Philippines since 1965. (pp. 107-108, Rollo)
On March 3, 1988, COMELEC (First Division) directed the Board of Canvassers to proclaim
the winning candidates. Having obtained the highest number of votes, private respondent was
proclaimed the Provincial Governor of Cebu.
Thereafter, on June 11, 1988, COMELEC (First Division) dismissed the petition for
disqualification for not having been timely filed and for lack of sufficient proof that private
respondent is not a Filipino citizen.
Hence, the present petition.
The petition is not meritorious.
There are two instances where a petition questioning the qualifications of a registered
candidate to run for the office for which his certificate of candidacy was filed can be raised
under the Omnibus Election Code (B.P. Blg. 881), to wit:
(1) Before election, pursuant to Section 78 thereof which provides that:
'Section 78. Petition to deny due course or to cancel a certificate of
candidacy. A verified petition seeking to deny due course or to cancel a
certificate of candidacy may be filed by any person exclusively on the
ground that any material representation contained therein as required
under Section 74 hereof is false. The petition may be filed at any time not
later than twenty-five days from the time of the filing of the certificate of
candidacy and shall be decided, after the notice and hearing, not later
than fifteen days before the election.
and
(2) After election, pursuant to Section 253 thereof, viz:
'Sec. 253. Petition for quo warranto. Any voter contesting the election
of any Member of the Batasang Pambansa, regional, provincial, or city
officer on the ground of ineligibility or of disloyalty to the Republic of the
Philippines shall file a sworn petition for quo warranto with the
Commission within ten days after the proclamation of the results of the
election.
The records show that private respondent filed his certificate of candidacy on November 19,
1987 and that the petitioner filed its petition for disqualification of said private respondent on
January 22, 1988. Since the petition for disqualification was filed beyond the twenty five-day
period required in Section 78 of the Omnibus Election Code, it is clear that said petition was
filed out of time.
The petition for the disqualification of private respondent cannot also be treated as a petition
for quo warranto under Section 253 of the same Code as it is unquestionably premature,
considering that private respondent was proclaimed Provincial Governor of Cebu only on
March 3, 1988.
However, We deem it is a matter of public interest to ascertain the respondent's citizenship
and qualification to hold the public office to which he has been proclaimed elected. There is
enough basis for us to rule directly on the merits of the case, as the COMELEC did below.

PARAS, J.:
Before Us is a petition for certiorari assailing the Resolution of the Commission on Elections
(COMELEC) dated June 11, 1988, which dismissed the petition for the disqualification of
private respondent Emilio "Lito" Osmea as candidate for Provincial Governor of Cebu
Province.
The facts of the case are briefly as follows:
On November 19, 1987, private respondent Emilio "Lito" Osmea filed his certificate of
candidacy with the COMELEC for the position of Provincial Governor of Cebu Province in the
January 18, 1988 local elections.
On January 22, 1988, the Cebu PDP-Laban Provincial Council (Cebu-PDP Laban, for short),
as represented by petitioner Jose B. Aznar in his capacity as its incumbent Provincial
Chairman, filed with the COMELEC a petition for the disqualification of private respondent on
the ground that he is allegedly not a Filipino citizen, being a citizen of the United States of
America.
On January 27, 1988, petitioner filed a Formal Manifestation submitting a Certificate issued by
the then Immigration and Deportation Commissioner Miriam Defensor Santiago certifying that
private respondent is an American and is a holder of Alien Certificate of Registration (ACR)
No. B-21448 and Immigrant Certificate of Residence (ICR) No. 133911, issued at Manila on
March 27 and 28, 1958, respectively. (Annex "B-1").
The petitioner also filed a Supplemental Urgent Ex-Parte Motion for the Issuance of a
Temporary Restraining Order to temporarily enjoin the Cebu Provincial Board of Canvassers
from tabulating/canvassing the votes cast in favor of private respondent and proclaiming him
until the final resolution of the main petition.
Thus, on January 28, 1988, the COMELEC en banc resolved to order the Board to continue
canvassing but to suspend the proclamation.
At the hearing before the COMELEC (First Division), the petitioner presented the following
exhibits tending to show that private respondent is an American citizen: Application for Alien
Registration Form No. 1 of the Bureau of Immigration signed by private respondent dated
November 21, 1979 (Exh. "B"); Alien Certificate of Registration No. 015356 in the name of
private respondent dated November 21, 1979 (Exh. "C"); Permit to Re-enter the Philippines
dated November 21, 1979 (Exh. "D"); Immigration Certificate of Clearance dated January 3,
1980 (Exh. "E"). (pp. 117-118, Rollo)
Private respondent, on the other hand, maintained that he is a Filipino citizen, alleging: that he
is the legitimate child of Dr. Emilio D. Osmea, a Filipino and son of the late President Sergio
Osmea, Sr.; that he is a holder of a valid and subsisting Philippine Passport No. 0855103
issued on March 25, 1987; that he has been continuously residing in the Philippines since
92

Petitioner's contention that private respondent is not a Filipino citizen and, therefore,
disqualified from running for and being elected to the office of Provincial Governor of Cebu, is
not supported by substantial and convincing evidence.
In the proceedings before the COMELEC, the petitioner failed to present direct proof that
private respondent had lost his Filipino citizenship by any of the modes provided for under
C.A. No. 63. Among others, these are: (1) by naturalization in a foreign country; (2) by express
renunciation of citizenship; and (3) by subscribing to an oath of allegiance to support the
Constitution or laws of a foreign country. From the evidence, it is clear that private respondent
Osmea did not lose his Philippine citizenship by any of the three mentioned hereinabove or
by any other mode of losing Philippine citizenship.
In concluding that private respondent had been naturalized as a citizen of the United States of
America, the petitioner merely relied on the fact that private respondent was issued alien
certificate of registration and was given clearance and permit to re-enter the Philippines by the
Commission on Immigration and Deportation. Petitioner assumed that because of the
foregoing, the respondent is an American and "being an American", private respondent "must
have taken and sworn to the Oath of Allegiance required by the U.S. Naturalization Laws." (p.
81, Rollo)
Philippine courts are only allowed to determine who are Filipino citizens and who are not.
Whether or not a person is considered an American under the laws of the United States does
not concern Us here.
By virtue of his being the son of a Filipino father, the presumption that private respondent is a
Filipino remains. It was incumbent upon the petitioner to prove that private respondent had
lost his Philippine citizenship. As earlier stated, however, the petitioner failed to positively
establish this fact.
The cases of Juan Gallanosa Frivaldo v. COMELEC et al, (G.R. No. 87193, June 21, 1989)
and Ramon L. Labo v. COMELEC et al (G.R. No. 86564, August 1, 1989) are not applicable to
the case at bar.
In the Frivaldo case, evidence shows that he was naturalized as a citizen of the United States
in 1983 per certification from the United States District Court, Northern District of California, as
duly authenticated by Vice Consul Amado P. Cortez of the Philippine Consulate General in
San Francisco, California, U.S.A.
Frivaldo expressly admitted in his answer that he was naturalized in the United States but
claimed that he was forced to embrace American citizenship to protect himself from the
persecution of the Marcos government. The Court, however, found this suggestion of
involuntariness unacceptable, pointing out that there were many other Filipinos in the United
States similarly situated as Frivaldo who did not find it necessary to abandon their status as
Filipinos.
Likewise, in the case of Labo, records show that Labo was married to an Australian citizen
and that he was naturalized as an Australian citizen in 1976, per certification from the
Australian Government through its Consul in the Philippines. This was later affirmed by the
Department of Foreign Affairs.
The authenticity of the above evidence was not disputed by Labo. In fact, in a number of
sworn statements, Labo categorically declared that he was a citizen of Australia.

In declaring both Frivaldo and Labo not citizens of the Philippines, therefore, disqualified from
serving as Governor of the Province of Sorsogon and Mayor of Baguio City, respectively, the
Court considered the fact that by their own admissions, they are indubitably aliens, no longer
owing any allegiance to the Republic of the Philippines since they have sworn their total
allegiance to a foreign state.
In the instant case, private respondent vehemently denies having taken the oath of allegiance
of the United States (p. 81, Rollo). He is a holder of a valid and subsisting Philippine passport
and has continuously participated in the electoral process in this country since 1963 up to the
present, both as a voter and as a candidate (pp. 107-108, Rollo). Thus, private respondent
remains a Filipino and the loss of his Philippine citizenship cannot be presumed.
In the learned dissent of Mr. Justice Teodoro Padilla, he stresses the fact that because
Osmea obtained Certificates of Alien Registration as an American citizen, the first in 1958
when he was 24 years old and the second in 1979, he, Osmea should be regarded as having
expressly renounced Philippine citizenship. To Our mind, this is a case of non sequitur (It
does not follow). Considering the fact that admittedly Osmea was both a Filipino and an
American, the mere fact that he has a Certificate stating he is an American does not mean
that he is not still a Filipino. Thus, by way of analogy, if a person who has two brothers named
Jose and Mario states or certifies that he has a brother named Jose, this does not mean that
he does not have a brother named Mario; or if a person is enrolled as student simultaneously
in two universities, namely University X and University Y, presents a Certification that he is a
student of University X, this does not necessarily mean that he is not still a student of
University Y. In the case of Osmea, the Certification that he is an American does not mean
that he is not still a Filipino, possessed as he is, of both nationalities or citizenships. Indeed,
there is no express renunciation here of Philippine citizenship; truth to tell, there is even no
implied renunciation of said citizenship. When We consider that the renunciation needed to
lose Philippine citizenship must be "express", it stands to reason that there can be no such
loss of Philippine 'citizenship when there is no renunciation either "'express" or "implied".
Parenthetically, the statement in the 1987 Constitution that "dual allegiance of citizens is
inimical to the national interest and shall be dealt with by law"(Art. IV, Sec. 5) has no
retroactive effect. And while it is true that even before the 1987 Constitution, Our country had
already frowned upon the concept of dual citizenship or allegiance, the fact is it actually
existed. Be it noted further that under the aforecited proviso, the effect of such dual citizenship
or allegiance shall be dealt with by a future law. Said law has not yet been enacted.
WHEREFORE, the petition for certiorari is hereby DISMISSED and the Resolution of the
COMELEC is hereby AFFIRMED.
SO ORDERED.

93

G.R. No. 137000


August 9, 2000
CIRILO R. VALLES, petitioner,
vs.
COMMISSION ON ELECTIONS and ROSALIND YBASCO LOPEZ, respondents.
DECISION
PURISIMA, J.:
This is a petition for certiorari under Rule 65, pursuant to Section 2, Rule 64 of the 1997 Rules
of Civil Procedure, assailing Resolutions dated July 17, 1998 and January 15, 1999,
respectively, of the Commission on Elections in SPA No. 98-336, dismissing the petition for
disqualification filed by the herein petitioner, Cirilo R. Valles, against private respondent
Rosalind Ybasco Lopez, in the May 1998 elections for governor of Davao Oriental.
Rosalind Ybasco Lopez was born on May 16, 1934 in Napier Terrace, Broome, Western
Australia, to the spouses, Telesforo Ybasco, a Filipino citizen and native of Daet, Camarines
Norte, and Theresa Marquez, an Australian. In 1949, at the age of fifteen, she left Australia
and came to settle in the Philippines.
On June 27, 1952, she was married to Leopoldo Lopez, a Filipino citizen, at the Malate
Catholic Church in Manila. Since then, she has continuously participated in the electoral
process not only as a voter but as a candidate, as well. She served as Provincial Board
Member of the Sangguniang Panlalawigan of Davao Oriental. In 1992, she ran for and was
elected governor of Davao Oriental. Her election was contested by her opponent, Gil Taojo,
Jr., in a petition for quo warranto, docketed as EPC No. 92-54, alleging as ground therefor her
alleged Australian citizenship. However, finding no sufficient proof that respondent had
renounced her Philippine citizenship, the Commission on Elections en banc dismissed the
petition, ratiocinating thus:
"A cursory reading of the records of this case vis-a-vis the impugned resolution shows that
respondent was able to produce documentary proofs of the Filipino citizenship of her late
father... and consequently, prove her own citizenship and filiation by virtue of the Principle of
Jus Sanguinis, the perorations of the petitioner to the contrary notwithstanding.
On the other hand, except for the three (3) alleged important documents . . . no other
evidence substantial in nature surfaced to confirm the allegations of petitioner that respondent
is an Australian citizen and not a Filipino. Express renunciation of citizenship as a mode of
losing citizenship under Commonwealth Act No. 63 is an equivocal and deliberate act with full
awareness of its significance and consequence. The evidence adduced by petitioner are
inadequate, nay meager, to prove that respondent contemplated renunciation of her Filipino
citizenship".1
In the 1995 local elections, respondent Rosalind Ybasco Lopez ran for re-election as governor
of Davao Oriental. Her opponent, Francisco Rabat, filed a petition for disqualification,
docketed as SPA No. 95-066 before the COMELEC, First Division, contesting her Filipino
citizenship but the said petition was likewise dismissed by the COMELEC, reiterating
substantially its decision in EPC 92-54.
The citizenship of private respondent was once again raised as an issue when she ran for reelection as governor of Davao Oriental in the May 11, 1998 elections. Her candidacy was
questioned by the herein petitioner, Cirilo Valles, in SPA No. 98-336.

On July 17, 1998, the COMELECs First Division came out with a Resolution dismissing the
petition, and disposing as follows:
"Assuming arguendo that res judicata does not apply and We are to dispose the instant case
on the merits trying it de novo, the above table definitely shows that petitioner herein has
presented no new evidence to disturb the Resolution of this Commission in SPA No. 95-066.
The present petition merely restates the same matters and incidents already passed upon by
this Commission not just in 1995 Resolution but likewise in the Resolution of EPC No. 92-54.
Not having put forth any new evidence and matter substantial in nature, persuasive in
character or sufficiently provocative to compel reversal of such Resolutions, the dismissal of
the present petition follows as a matter of course.
xxx
xxx
xxx
"WHEREFORE, premises considered and there being no new matters and issues tendered,
We find no convincing reason or impressive explanation to disturb and reverse the
Resolutions promulgated by this Commission in EPC 92-54 and SPA. 95-066. This
Commission RESOLVES as it hereby RESOLVES to DISMISS the present petition.
SO ORDERED."2
Petitioner interposed a motion for reconsideration of the aforesaid Resolution but to no avail.
The same was denied by the COMELEC in its en banc Resolution of January 15, 1999.
Undaunted, petitioner found his way to this Court via the present petition; questioning the
citizenship of private respondent Rosalind Ybasco Lopez.
The Commission on Elections ruled that private respondent Rosalind Ybasco Lopez is a
Filipino citizen and therefore, qualified to run for a public office because (1) her father,
Telesforo Ybasco, is a Filipino citizen, and by virtue of the principle of jus sanguinis she was a
Filipino citizen under the 1987 Philippine Constitution; (2) she was married to a Filipino,
thereby making her also a Filipino citizen ipso jure under Section 4 of Commonwealth Act 473;
(3) and that, she renounced her Australian citizenship on January 15, 1992 before the
Department of Immigration and Ethnic Affairs of Australia and her Australian passport was
accordingly cancelled as certified to by the Australian Embassy in Manila; and (4) furthermore,
there are the COMELEC Resolutions in EPC No. 92-54 and SPA Case No. 95-066, declaring
her a Filipino citizen duly qualified to run for the elective position of Davao Oriental governor.
Petitioner, on the other hand, maintains that the private respondent is an Australian citizen,
placing reliance on the admitted facts that:
a) In 1988, private respondent registered herself with the Bureau of Immigration as
an Australian national and was issued Alien Certificate of Registration No. 404695
dated September 19, 1988;
b) On even date, she applied for the issuance of an Immigrant Certificate of
Residence (ICR), and
c) She was issued Australian Passport No. H700888 on March 3, 1988.
Petitioner theorizes that under the aforestated facts and circumstances, the private
respondent had renounced her Filipino citizenship. He contends that in her application for
alien certificate of registration and immigrant certificate of residence, private respondent
expressly declared under oath that she was a citizen or subject of Australia; and said

94

declaration forfeited her Philippine citizenship, and operated to disqualify her to run for
elective office.
As regards the COMELECs finding that private respondent had renounced her Australian
citizenship on January 15, 1992 before the Department of Immigration and Ethnic Affairs of
Australia and had her Australian passport cancelled on February 11, 1992, as certified to by
the Australian Embassy here in Manila, petitioner argues that the said acts did not
automatically restore the status of private respondent as a Filipino citizen. According to
petitioner, for the private respondent to reacquire Philippine citizenship she must comply with
the mandatory requirements for repatriation under Republic Act 8171; and the election of
private respondent to public office did not mean the restoration of her Filipino citizenship since
the private respondent was not legally repatriated. Coupled with her alleged renunciation of
Australian citizenship, private respondent has effectively become a stateless person and as
such, is disqualified to run for a public office in the Philippines; petitioner concluded.
Petitioner theorizes further that the Commission on Elections erred in applying the principle of
res judicata to the case under consideration; citing the ruling in Moy Ya Lim Yao vs.
Commissioner of Immigration,3 that:
"xxx Everytime the citizenship of a person is material or indispensable in a judicial or
administrative case, whatever the corresponding court or administrative authority decides
therein as to such citizenship is generally not considered as res adjudicata, hence it has to be
threshed out again and again as the occasion may demand. xxx"
The petition is unmeritorious.
The Philippine law on citizenship adheres to the principle of jus sanguinis. Thereunder, a child
follows the nationality or citizenship of the parents regardless of the place of his/her birth, as
opposed to the doctrine of jus soli which determines nationality or citizenship on the basis of
place of birth.
Private respondent Rosalind Ybasco Lopez was born on May 16, 1934 in Napier Terrace,
Broome, Western Australia, to the spouses, Telesforo Ybasco, a Filipino citizen and native of
Daet, Camarines Norte, and Theresa Marquez, an Australian. Historically, this was a year
before the 1935 Constitution took into effect and at that time, what served as the Constitution
of the Philippines were the principal organic acts by which the United States governed the
country. These were the Philippine Bill of July 1, 1902 and the Philippine Autonomy Act of
August 29, 1916, also known as the Jones Law.
Among others, these laws defined who were deemed to be citizens of the Philippine islands.
The Philippine Bill of 1902 defined Philippine citizens as:
SEC. 4 xxx all inhabitants of the Philippine Islands continuing to reside therein who were
Spanish subjects on the eleventh day of April, eighteen hundred and ninety-nine, and then
resided in the Philippine Islands, and their children born subsequent thereto, shall be deemed
and held to be citizens of the Philippine Islands and as such entitled to the protection of the
United States, except such as shall have elected to preserve their allegiance to the Crown of
Spain in accordance with the provisions of the treaty of peace between the United States and
Spain signed at Paris December tenth, eighteen hundred and ninety-eight. (underscoring
ours)
The Jones Law, on the other hand, provides:

SEC. 2 That all inhabitants of the Philippine Islands who were Spanish subjects on the
eleventh day of April, eighteen hundred and ninety-nine, and then resided in said Islands, and
their children born subsequent thereto, shall be deemed and held to be citizens of the
Philippine Islands, except such as shall have elected to preserve their allegiance to the Crown
of Spain in accordance with the provisions of the treaty of peace between the United States
and Spain, signed at Paris December tenth, eighteen hundred and ninety-eight, and except
such others as have since become citizens of some other country: Provided, That the
Philippine Legislature, herein provided for, is hereby authorized to provide by law for the
acquisition of Philippine citizenship by those natives of the Philippine Islands who cannot
come within the foregoing provisions, the natives of the insular possessions of the United
States, and such other persons residing in the Philippine Islands who are citizens of the
United States, or who could become citizens of the United States under the laws of the United
States if residing therein. (underscoring ours)
Under both organic acts, all inhabitants of the Philippines who were Spanish subjects on April
11, 1899 and resided therein including their children are deemed to be Philippine citizens.
Private respondents father, Telesforo Ybasco, was born on January 5, 1879 in Daet,
Camarines Norte, a fact duly evidenced by a certified true copy of an entry in the Registry of
Births. Thus, under the Philippine Bill of 1902 and the Jones Law, Telesforo Ybasco was
deemed to be a Philippine citizen. By virtue of the same laws, which were the laws in force at
the time of her birth, Telesforos daughter, herein private respondent Rosalind Ybasco Lopez,
is likewise a citizen of the Philippines.
The signing into law of the 1935 Philippine Constitution has established the principle of jus
sanguinis as basis for the acquisition of Philippine citizenship, to wit:
(1) Those who are citizens of the Philippine Islands at the time of the adoption of
this Constitution.
(2) Those born in the Philippine Islands of foreign parents who, before the adoption
of this Constitution had been elected to public office in the Philippine Islands.
(3) Those whose fathers are citizens of the Philippines.
(4) Those whose mothers are citizens of the Philippines and, upon reaching the age
of majority, elect Philippine citizenship.
(5) Those who are naturalized in accordance with law.
So also, the principle of jus sanguinis, which confers citizenship by virtue of blood relationship,
was subsequently retained under the 19734 and 19875 Constitutions. Thus, the herein private
respondent, Rosalind Ybasco Lopez, is a Filipino citizen, having been born to a Filipino father.
The fact of her being born in Australia is not tantamount to her losing her Philippine
citizenship. If Australia follows the principle of jus soli, then at most, private respondent can
also claim Australian citizenship resulting to her possession of dual citizenship.
Petitioner also contends that even on the assumption that the private respondent is a Filipino
citizen, she has nonetheless renounced her Philippine citizenship. To buttress this contention,
petitioner cited private respondents application for an Alien Certificate of Registration (ACR)
and Immigrant Certificate of Residence (ICR), on September 19, 1988, and the issuance to
her of an Australian passport on March 3, 1988.
Under Commonwealth Act No. 63, a Filipino citizen may lose his citizenship:
(1) By naturalization in a foreign country;
95

(2) By express renunciation of citizenship;


(3) By subscribing to an oath of allegiance to support the constitution or laws of a
foreign country upon attaining twenty-one years of age or more;
(4) By accepting commission in the military, naval or air service of a foreign country;
(5) By cancellation of the certificate of naturalization;
(6) By having been declared by competent authority, a deserter of the Philippine
armed forces in time of war, unless subsequently, a plenary pardon or amnesty has
been granted: and
(7) In case of a woman, upon her marriage, to a foreigner if, by virtue of the laws in
force in her husbands country, she acquires his nationality.
In order that citizenship may be lost by renunciation, such renunciation must be express.
Petitioners contention that the application of private respondent for an alien certificate of
registration, and her Australian passport, is bereft of merit. This issue was put to rest in the
case of Aznar vs. COMELEC6 and in the more recent case of Mercado vs. Manzano and
COMELEC.7
In the case of Aznar, the Court ruled that the mere fact that respondent Osmena was a holder
of a certificate stating that he is an American did not mean that he is no longer a Filipino, and
that an application for an alien certificate of registration was not tantamount to renunciation of
his Philippine citizenship.
And, in Mercado vs. Manzano and COMELEC, it was held that the fact that respondent
Manzano was registered as an American citizen in the Bureau of Immigration and Deportation
and was holding an American passport on April 22, 1997, only a year before he filed a
certificate of candidacy for vice-mayor of Makati, were just assertions of his American
nationality before the termination of his American citizenship.
Thus, the mere fact that private respondent Rosalind Ybasco Lopez was a holder of an
Australian passport and had an alien certificate of registration are not acts constituting an
effective renunciation of citizenship and do not militate against her claim of Filipino citizenship.
For renunciation to effectively result in the loss of citizenship, the same must be express. 8 As
held by this court in the aforecited case of Aznar, an application for an alien certificate of
registration does not amount to an express renunciation or repudiation of ones citizenship.
The application of the herein private respondent for an alien certificate of registration, and her
holding of an Australian passport, as in the case of Mercado vs. Manzano, were mere acts of
assertion of her Australian citizenship before she effectively renounced the same. Thus, at the
most, private respondent had dual citizenship - she was an Australian and a Filipino, as well.
Moreover, under Commonwealth Act 63, the fact that a child of Filipino parent/s was born in
another country has not been included as a ground for losing ones Philippine citizenship.
Since private respondent did not lose or renounce her Philippine citizenship, petitioners claim
that respondent must go through the process of repatriation does not hold water.
Petitioner also maintains that even on the assumption that the private respondent had dual
citizenship, still, she is disqualified to run for governor of Davao Oriental; citing Section 40 of
Republic Act 7160 otherwise known as the Local Government Code of 1991, which states:
"SEC. 40. Disqualifications. The following persons are disqualified from running for any
elective local position:

xxx

xxx

xxx

(d) Those with dual citizenship;


xxx
xxx
xxx
Again, petitioners contention is untenable.
In the aforecited case of Mercado vs. Manzano, the Court clarified "dual citizenship" as used
in the Local Government Code and reconciled the same with Article IV, Section 5 of the 1987
Constitution on dual allegiance.9 Recognizing situations in which a Filipino citizen may, without
performing any act, and as an involuntary consequence of the conflicting laws of different
countries, be also a citizen of another state, the Court explained that dual citizenship as a
disqualification must refer to citizens with dual allegiance. The Court succinctly pronounced:
"xxx the phrase dual citizenship in R.A. No. 7160, xxx 40 (d) and in R.A. No. 7854, xxx 20
must be understood as referring to dual allegiance. Consequently, persons with mere dual
citizenship do not fall under this disqualification."
Thus, the fact that the private respondent had dual citizenship did not automatically disqualify
her from running for a public office. Furthermore, it was ruled that for candidates with dual
citizenship, it is enough that they elect Philippine citizenship upon the filing of their certificate
of candidacy, to terminate their status as persons with dual citizenship.10 The filing of a
certificate of candidacy sufficed to renounce foreign citizenship, effectively removing any
disqualification as a dual citizen.11 This is so because in the certificate of candidacy, one
declares that he/she is a Filipino citizen and that he/she will support and defend the
Constitution of the Philippines and will maintain true faith and allegiance thereto. Such
declaration, which is under oath, operates as an effective renunciation of foreign citizenship.
Therefore, when the herein private respondent filed her certificate of candidacy in 1992, such
fact alone terminated her Australian citizenship.
Then, too, it is significant to note that on January 15 1992, private respondent executed a
Declaration of Renunciation of Australian Citizenship, duly registered in the Department of
Immigration and Ethnic Affairs of Australia on May 12, 1992. And, as a result, on February 11,
1992, the Australian passport of private respondent was cancelled, as certified to by Second
Secretary Richard F. Munro of the Embassy of Australia in Manila. As aptly appreciated by the
COMELEC, the aforesaid acts were enough to settle the issue of the alleged dual citizenship
of Rosalind Ybasco Lopez. Since her renunciation was effective, petitioners claim that private
respondent must go through the whole process of repatriation holds no water.
Petitioner maintains further that when citizenship is raised as an issue in judicial or
administrative proceedings, the resolution or decision thereon is generally not considered res
judicata in any subsequent proceeding challenging the same; citing the case of Moy Ya Lim
Yao vs. Commissioner of Immigration.12 He insists that the same issue of citizenship may be
threshed out anew.
Petitioner is correct insofar as the general rule is concerned, i.e. the principle of res judicata
generally does not apply in cases hinging on the issue of citizenship. However, in the case of
Burca vs. Republic,13 an exception to this general rule was recognized. The Court ruled in that
case that in order that the doctrine of res judicata may be applied in cases of citizenship, the
following must be present:
1) a persons citizenship be raised as a material issue in a controversy where said
person is a party;
96

2) the Solicitor General or his authorized representative took active part in the
resolution thereof, and
3) the finding on citizenship is affirmed by this Court.
Although the general rule was set forth in the case of Moy Ya Lim Yao, the case did not
foreclose the weight of prior rulings on citizenship. It elucidated that reliance may somehow be
placed on these antecedent official findings, though not really binding, to make the effort
easier or simpler.14 Indeed, there appears sufficient basis to rely on the prior rulings of the
Commission on Elections in SPA. No. 95-066 and EPC 92-54 which resolved the issue of
citizenship in favor of the herein private respondent. The evidence adduced by petitioner is
substantially the same evidence presented in these two prior cases. Petitioner failed to show
any new evidence or supervening event to warrant a reversal of such prior resolutions.
However, the procedural issue notwithstanding, considered on the merits, the petition cannot
prosper.
WHEREFORE, the petition is hereby DISMISSED and the COMELEC Resolutions, dated July
17, 1998 and January 15, 1999, respectively, in SPA No. 98-336 AFFIRMED.
Private respondent Rosalind Ybasco Lopez is hereby adjudged qualified to run for governor of
Davao Oriental. No pronouncement as to costs.

97

G.R. No. 142840 May 7, 2001


ANTONIO BENGSON III, petitioner,
vs.
HOUSE OF REPRESENTATIVES ELECTORAL TRIBUNAL and TEODORO C. CRUZ,
respondents.
CONCURRING OPINION
DISSENTING OPINION
KAPUNAN, J.:
The citizenship of respondent Teodoro C. Cruz is at issue in this case, in view of the
constitutional requirement that "no person shall be a Member of the House of Representative
unless he is a natural-born citizen."1
Respondent Cruz was a natural-born citizen of the Philippines. He was born in San Clemente,
Tarlac, on April 27, 1960, of Filipino parents. The fundamental law then applicable was the
1935 Constitution.2
On November 5, 1985, however, respondent Cruz enlisted in the United States Marine Corps
and without the consent of the Republic of the Philippines, took an oath of allegiance to the
United States. As a Consequence, he lost his Filipino citizenship for under Commonwealth Act
No. 63, section 1(4), a Filipino citizen may lose his citizenship by, among other, "rendering
service to or accepting commission in the armed forces of a foreign country." Said provision of
law reads:
SECTION 1. How citizenship may be lost. A Filipino citizen may lose his
citizenship in any of the following ways and/or events:
xxx
(4) By rendering services to, or accepting commission in, the armed of a foreign
country: Provided, That the rendering of service to, or the acceptance of such
commission in, the armed forces of a foreign country, and the taking of an oath of
allegiance incident thereto, with the consent of the Republic of the Philippines, shall
not divest a Filipino of his Philippine citizenship if either of the following
circumstances is present:
(a) The Republic of the Philippines has a defensive and/or offensive pact of alliance
with said foreign country; or
(b) The said foreign country maintains armed forces on Philippine territory with the
consent of the Republic of the Philippines: Provided, That the Filipino citizen
concerned, at the time of rendering said service, or acceptance of said commission,
and taking the oath of allegiance incident thereto, states that he does so only in
connection with his service to said foreign country; And provided, finally, That any
Filipino citizen who is rendering service to, or is commissioned in, the armed forces
of a foreign country under any of the circumstances mentioned in paragraph (a) or
(b), shall not be Republic of the Philippines during the period of his service to, or
commission in, the armed forces of said country. Upon his discharge from the
service of the said foreign country, he shall be automatically entitled to the full
enjoyment of his civil and politically entitled to the full enjoyment of his civil political
rights as a Filipino citizen x x x.

Whatever doubt that remained regarding his loss of Philippine citizenship was erased by his
naturalization as a U.S. citizen on June 5, 1990, in connection with his service in the U.S.
Marine Corps.
On March 17, 1994, respondent Cruz reacquired his Philippine citizenship through repatriation
under Republic Act No. 2630.3 He ran for and was elected as the Representative of the
Second District of Pangasinan in the May 11, 1998 elections. He won by a convincing margin
of 26,671 votes over petitioner Antonio Bengson III, who was then running for
reelection.1wphi1.nt
Subsequently, petitioner filed a case for Quo Warranto Ad Cautelam with respondent House
of Representatives Electoral Tribunal (HRET) claiming that respondent Cruz was not qualified
to become a member of the House of Representatives since he is not a natural-born citizen as
required under Article VI, section 6 of the Constitution.4
On March 2, 2000, the HRET rendered its decision5 dismissing the petition for quo warranto
and declaring Cruz the duly elected Representative of the Second District of Pangasinan in
the May 1998 elections. The HRET likewise denied petitioner's motion for reconsideration of
the decision in its resolution dated April 27, 2000.6
Petitioner thus filed the present petition for certiorari assailing the HRET's decision on the
following grounds:
1. The HRET committed serious errors and grave abuse of discretion, amounting to
excess of jurisdiction, when it ruled that private respondent is a natural-born citizen
of the Philippines despite the fact that he had ceased being such in view of the loss
and renunciation of such citizenship on his part.
2. The HRET committed serious errors and grave abuse of discretion, amounting to
excess of jurisdiction, when it considered private respondent as a citizen of the
Philippines despite the fact he did not validly acquire his Philippine citizenship.
3. Assuming that private respondent's acquisition of Philippine citizenship was
invalid, the HRET committed serious errors and grave abuse of discretion,
amounting to excess of jurisdiction, when it dismissed the petition despite the fact
that such reacquisition could not legally and constitutionally restore his natural-born
status.7
The issue now before us is whether respondent Cruz, a natural-born Filipino who became an
American citizen, can still be considered a natural-born Filipino upon his reacquisition of
Philippine citizenship.
Petitioner asserts that respondent Cruz may no longer be considered a natural-born Filipino
since he lost h is Philippine citizenship when he swore allegiance to the United States in 1995,
and had to reacquire the same by repatriation. He insists that Article citizens are those who
are from birth with out having to perform any act to acquire or perfect such citizenship.
Respondent on the other hand contends that he reacquired his status as natural-born citizen
when he was repatriated since the phrase "from birth" in Article IV, Section 2 refers to the
innate, inherent and inborn characteristic of being a natural-born citizen.
The petition is without merit.
The 1987 Constitution enumerates who are Filipino citizens as follow:

98

(1) Those who are citizens of the Philippines at the time of the adoption of this
Constitution;
(2) Those whose fathers or mothers are citizens of the Philippines;
(3) Those born before January 17, 1973 of Filipino mother, who elect Philippine
citizenship upon reaching the age of majority, and
(4) Those who are naturalized in accordance with law.8
There are two ways of acquiring citizenship: (1) by birth, and (2) by naturalization. These ways
of acquiring citizenship correspond to the two kinds of citizens: the natural-born citizen, and
the naturalized citizen. A person who at the time of his birth is a citizen of a particular country,
is a natural-born citizen thereof.9
As defined in the same Constitution, natural-born citizens "are those citizens of the Philippines
from birth without having to perform any act to acquire or perfect his Philippine citezenship."10
On the other hand, naturalized citizens are those who have become Filipino citizens through
naturalization, generally under Commonwealth Act No. 473, otherwise known as the Revised
Naturalization Law, which repealed the former Naturalization Law (Act No. 2927), and by
Republic Act No. 530.11 To be naturalized, an applicant has to prove that he possesses all the
qualifications12 and none of the disqualification13 provided by law to become a Filipino citizen.
The decision granting Philippine citizenship becomes executory only after two (2) years from
its promulgation when the court is satisfied that during the intervening period, the applicant
has (1) not left the Philippines; (2) has dedicated himself to a lawful calling or profession; (3)
has not been convicted of any offense or violation of Government promulgated rules; or (4)
committed any act prejudicial to the interest of the nation or contrary to any Government
announced policies.14
Filipino citizens who have lost their citizenship may however reacquire the same in the
manner provided by law. Commonwealth Act. No. (C.A. No. 63), enumerates the three modes
by which Philippine citizenship may be reacquired by a former citizen: (1) by naturalization, (2)
by repatriation, and (3) by direct act of Congress.15
Naturalization is mode for both acquisition and reacquisition of Philippine citizenship. As a
mode of initially acquiring Philippine citizenship, naturalization is governed by Commonwealth
Act No. 473, as amended. On the other hand, naturalization as a mode for reacquiring
Philippine citizenship is governed by Commonwealth Act No. 63.16 Under this law, a former
Filipino citizen who wishes to reacquire Philippine citizenship must possess certain
qualifications17 and none of the disqualification mentioned in Section 4 of C.A. 473.18
Repatriation, on the other hand, may be had under various statutes by those who lost their
citizenship due to: (1) desertion of the armed forces;19 services in the armed forces of the
allied forces in World War II;20 (3) service in the Armed Forces of the United States at any
other time,21 (4) marriage of a Filipino woman to an alien;22 and (5) political economic
necessity.23
As distinguished from the lengthy process of naturalization, repatriation simply consists of the
taking of an oath of allegiance to the Republic of the Philippine and registering said oath in the
Local Civil Registry of the place where the person concerned resides or last resided.
In Angat v. Republic,24 we held:

xxx. Parenthetically, under these statutes [referring to RA Nos. 965 and 2630], the
person desiring to reacquire Philippine citizenship would not even be required to file
a petition in court, and all that he had to do was to take an oath of allegiance to the
Republic of the Philippines and to register that fact with the civil registry in the place
of his residence or where he had last resided in the Philippines. [Italics in the
original.25
Moreover, repatriation results in the recovery of the original nationality.26 This means that a
naturalized Filipino who lost his citizenship will be restored to his prior status as a naturalized
Filipino citizen. On the other hand, if he was originally a natural-born citizen before he lost his
Philippine citizenship, he will be restored to his former status as a natural-born Filipino.
In respondent Cruz's case, he lost his Filipino citizenship when he rendered service in the
Armed Forces of the United States. However, he subsequently reacquired Philippine
citizenship under R.A. No. 2630, which provides:
Section 1. Any person who had lost his Philippine citizenship by rendering service
to, or accepting commission in, the Armed Forces of the United States, or after
separation from the Armed Forces of the United States, acquired United States
citizenship, may reacquire Philippine citizenship by taking an oath of allegiance to
the Republic of the Philippines and registering the same with Local Civil Registry in
the place where he resides or last resided in the Philippines. The said oath of
allegiance shall contain a renunciation of any other citizenship.
Having thus taken the required oath of allegiance to the Republic and having registered the
same in the Civil Registry of Magantarem, Pangasinan in accordance with the aforecited
provision, respondent Cruz is deemed to have recovered his original status as a natural-born
citizen, a status which he acquired at birth as the son of a Filipino father.27 It bears stressing
that the act of repatriation allows him to recover, or return to, his original status before he
lost his Philippine citizenship.
Petitioner's contention that respondent Cruz is no longer a natural-born citizen since he had to
perform an act to regain his citizenship is untenable. As correctly explained by the HRET in its
decision, the term "natural-born citizen" was first defined in Article III, Section 4 of the 1973
Constitution as follows:
Sec. 4. A natural-born citizen is one who is a citizen of the Philippines from birth
without having to perform any act to acquire or perfect his Philippine citizenship.
Two requisites must concur for a person to be considered as such: (1) a person must be a
Filipino citizen birth and (2) he does not have to perform any act to obtain or perfect his
Philippine citizenship.
Under the 1973 Constitution definition, there were two categories of Filipino citizens which
were not considered natural-born: (1) those who were naturalized and (2) those born before
January 17, 1973,38 of Filipino mothers who, upon reaching the age of majority, elected
Philippine citizenship. Those "naturalized citizens" were not considered natural-born obviously
because they were not Filipino at birth and had to perform an act to acquire Philippine
citizenship. Those born of Filipino mothers before the effectively of the 1973 Constitution were
likewise not considered natural-born because they also had to perform an act to perfect their
Philippines citizenship.

99

The present Constitution, however, now consider those born of Filipino mothers before the
effectivity of the 1973 Constitution and who elected Philippine citizenship upon reaching the
majority age as natural-born. After defining who re natural-born citizens, Section 2 of Article IV
adds a sentence: "Those who elect Philippine citizenship in accordance with paragraph (3),
Section 1 hereof shall be deemed natural-born citizens." Consequently, only naturalized
Filipinos are considered not natural-born citizens. It is apparent from the enumeration of who
are citizens under the present Constitution that there are only two classes of citizens: (1)
those who are natural-born and (2) those who are naturalized in accordance with law. A
citizen who is not a naturalized Filipino, i.e., did not have to undergo the process of
naturalization to obtain Philippine citizenship, necessarily is natural-born Filipino. Noteworthy
is the absence in said enumeration of a separate category for persons who, after losing
Philippine citizenship, subsequently reacquire it. The reason therefor is clear: as to such
persons, they would either be natural-born or naturalized depending on the reasons for the
loss of their citizenship and the mode prescribed by the applicable law for the reacquisition
thereof. As respondent Cruz was not required by law to go through naturalization proceeding
in order to reacquire his citizenship, he is perforce a natural-born Filipino. As such, he
possessed all the necessary qualifications to be elected as member of the House of
Representatives.
A final point. The HRET has been empowered by the Constitution to be the "sole judge" of all
contests relating to the election, returns, and qualifications of the members of the House. 29
The Court's jurisdiction over the HRET is merely to check "whether or not there has been a
grave abuse of discretion amounting to lack or excess of jurisdiction" on the part of the latter. 30
In the absence thereof, there is no occasion for the Court to exercise its corrective power and
annul the decision of the HRET nor to substitute the Court's judgement for that of the latter for
the simple reason that it is not the office of a petition for certiorari to inquire into the
correctness of the assailed decision.31 There is no such showing of grave abuse of discretion
in this case.
WHEREFORE, the petition is hereby DISMISSED.
SO ORDERED.

100

G.R. No. 132244 September 14, 1999


GERARDO ANGAT, petitioner,
vs.
REPUBLIC OF THE PHILIPPINES, respondent.

Philippines, in his relations with the constituted government as well as with


the community in which he is living.
EIGHT. He is not opposed to an organized government or affiliated with
any association or group of persons who uphold and teach doctrines
opposing all organized government. He is not defending or teaching the
necessity or propriety of violence, personal assault or assassination for
the success and predominance of men's ideas. He is not a polygamist or
believer in the practice of polygamy. He has not been convicted of any
crime involving moral turpitude. He is not suffering from any mental
alienation or incurable contagious disease. The nation of which he is a
citizen or subject is not at war with the Philippines.
NINTH. It is his intention to reacquire Philippine citizenship and to
renounce absolutely and forever all allegiance and fidelity to any foreign
prince, potentate, state, or sovereignty, and particularly to the United State
of America to which at this time he is a citizen. 1
On 30 April 1996, the trial court, through the branch clerk of court, issued a notice setting the
case for initial hearing on 27 January 1997 2 which, along with the petition and its annexes,
was received by the Office of the Solicitor General ("OSG") on 10 May 1996.
On 13 June 1996, petitioner sought to be allowed to take his oath of allegiance to the Republic
of the Philippines pursuant to R.A. 8171. The motion was denied by the trial judge in his order
of 12 July 1996. Another motion filed by petitioner on 13 August 1996 to have the denial
reconsidered was found to be meritorious by the court a quo in an order, dated 20 September
1996, which stated, among other things, that
A close scrutiny of R.A. 8171 shows that petitioner is entitled to the
benefits of the said law considering that herein petitioner is a natural born
Filipino citizen who lost his citizenship by naturalization in a foreign
country. The petition and motion of the petitioner to take his oath of
allegiance to the Republic of the Philippines likewise show that the
petitioner possesses all the qualifications and none of the disqualifications
under R.A. 8171. 3
Concluding, the court ruled:
WHEREFORE, foregoing premises considered, the Order of the Court
dated July 12, 1996 is hereby set aside. The petitioner is ordered to take
his oath of allegiance to the Republic of the Philippines pursuant to R.A.
8171 before the undersigned on October 03, 1996 at 11:00 in the
morning.
SO ORDERED. 4
After taking his Oath of Allegiance on 03 October 1996, another order was issued by the trial
judge on 04 October 1996 to the following effect; viz:
After the oath of allegiance to the Republic of the Philippines had been
taken by the petitioner, Gerardo Angat y Legaspi before the undersigned,
the petitioner is hereby repatriated and declared as citizen of the Republic
of the Philippines pursuant to Republic Act No. 8171.

VITUG, J.:
The instant petition for review under Rule 45 assails the orders, dated 22 September 1997
and 29 December 1997, issued by the Regional Trial Court ("RTC") of Marikina City in Case
No. N-96-03-MK, entitled "in the Matter of the Petition of Gerardo Angat y Legaspi to be Readmitted as a Citizen of the Philippines under Commonwealth Act No. 63, as amended, and
Republic Act ("R.A.") No. 965 and 263[0]."
Petitioner Gerardo Angat was a natural born citizen of the Philippines until he lost his
citizenship by naturalization in the United States of America. Now residing at No. 69 New York
Street, Provident Village, Marikina City, Angat filed on 11 March 1996 before the RTC of
Marikina City, Branch 272, a petition to regain his Status as a citizen of the Philippines under
Commonwealth Act No. 63, Republic Act No. 965 and Republic Act No. 2630 (docketed as N96-03-MK). In his petition, "applying for naturalization," he averred that
FIRST. His full name is GERARDO LEGASPI ANGAT. Copy of his
latest picture is hereto attached and made an integral part of this petition.
SECOND. His present place of residence is #69 New York St.,
Provident Village, Marikina, Metro Manila and his former residence was in
Las Vegas, U.S.1wphi1.nt
THIRD. His trade or profession is in buy and sell and managing the
properties of his parents which he has been engaged since his arrival
here in the Philippines.
FOURTH. He was born on the 22nd day of June 1954 at Tondo,
Manila. He was formerly a citizen of the Philippines. He lost his Philippine
citizenship by naturalization in a foreign country. He is at present a citizen
or subject of the United States of America. Copy of his birth certificate is
hereto attached as Annex "A."
FIFTH. He is newly married to Zenaida Lim who was born in Tondo,
Manila and now resides at petitioner's residence at Marikina, Metro
Manila. Copy of their marriage contract is hereto attached as Annex "B."
SIXTH. He returned to the Philippines from the United States of
America in 1991. Copy of his alien registration is hereto attached as
Annex "C."
SEVENTH. He has the qualifications required by Commonwealth Act
No. 63 as amended, and Republic Act Nos. 965 and 2639 to reacquire
Philippine citizenship, and possesses none of the disqualification
prescribed in Commonwealth Act No. 473. He has resided in the
Philippines at least six months immediately preceding the date of this
petition, to wit: since 1991. He has conducted himself in a proper and
irreproachable manner during the entire period of his residence in the
101

The Bureau of Immigration is ordered to cancel the pertinent alien


certificate of registration and issue the certificate of identification as
Filipino citizen to the petitioner upon the finality of this order.
Likewise, let a copy of this Order be registered in the Local Civil Registry
of the Municipality of Marikina, Metro Manila and the General Civil
Registrar, Sta. Mesa, Manila, after its finality.
SO ORDERED. 5
On 19 March 1997, a Manifestation and Motion (virtually a motion for reconsideration) filed by
the OSG asserted that the petition itself should have been dismissed by the court a quo for
lack of jurisdiction because the proper forum for it was the Special Committee on
Naturalization consistently with Administrative Order No. 285 ("AO 285"), dated 22 August
1996, issued by President Fidel V. Ramos. AO 285 had tasked the Special Committee on
Naturalization to be the implementing agency of R.A 8171. The motion was found to be well
taken by the trial court; thus, in an order, dated 22 September 1997, it adjudged:
This resolves the Manifestation and Motion filed by the Office of the
Solicitor General on March 19, 1997.
The motion alleges that pursuant to Administrative Order No. 285 dated
August 22, 1996 issued by President Fidel V. Ramos, any person
desirous of repatriating or reacquiring Filipino citizenship pursuant to R.A.
8171 shall file a petition with the Special Committee on Naturalization,
which is composed of the Solicitor General as Chairman, the
Undersecretary of Foreign Affairs and the Director-General of the National
Intelligence Coordinating Agency, as members, which shall process the
application; that if their applications are approved they shall take the
necessary oath of allegiance to the Republic of the Philippines, affect
which they shall be deemed to have reacquired their Philippine citizenship
and the Commission of Immigration and Deportation shall thereupon
cancel their certificate of registration.
The motion prays that the herein petition be dismissed on the ground that
the same should be filed with the Special Committee on Naturalization.
The records show that on September 20, 1996, the Court granted the
herein petition and as a consequence thereof, the petitioner Gerardo
Angat y Legaspi took his oath of allegiance to the Republic of the
Philippines before the Presiding Judge of this Court on October 03, 1996
and on October 04, 1996, the petitioner was ordered repatriated and
declared as citizen of the Philippines.
On February 21, 1997, the Office of the Solicitor General entered its
appearance as counsel of the State in the subject petition and on March
19, 1997 filed the herein manifestation and motion.
The allegations in the manifestation and motion of the Office of the
Solicitor General clearly shows that this Court has no jurisdiction over the
herein petition as the same falls within the jurisdiction of the Special
Committee on Naturalization. Considering that this court has no

jurisdiction over this case, the order granting the same is therefore null
and void.
WHEREFORE, foregoing premises considered, the motion to dismiss filed
by the Office of the Solicitor General is hereby granted. The orders of this
Court dated September 20, 1996 and October 04, 1996 are hereby set
aside and the herein petition is ordered DISMISSED on the ground of lack
of jurisdiction without prejudice to its re-filing before the Special
Committee on Naturalization.
SO ORDERED. 6
A motion for reconsideration, filed by petitioner on 13 October 1997, questioned the
aforequoted order asservating that since his petition was filed on 14 March 1996, or months
before the Special Committee on Naturalization was constituted by the President under AO
285 on 22 August 1996, the court a quo had the authority to take cognizance of the case.
In the Order, dated 29 December 1997, the trial judge denied the motion for reconsideration.
The instant appeal by certiorari under Rule 45 of the 1997 Rules of Procedure submits the
lone assignment of error that
The Regional Trial Court (has) seriously erred in dismissing the petition by
giving retroactive effect to Administrative Order No. 285, absent a
provision on Retroactive Application.
Petitioner would insist that the trial court had jurisdiction over his petition for naturalization 7
filed on 11 March 1996, and that he had acquired a vested right as a repatriated citizen of the
Philippines when the court declared him repatriated following the order, dated 20 September
1996, allowing him to take an oath of allegiance to the Republic of the Philippines which was,
in fact, administered to him on 03 October 1996.
The contention is not meritorious.
R.A. No. 8171, which has lapsed into law on 23 October 1995, is an act providing for the
repatriation (a) of Filipino women who have lost their Philippine citizenship by marriage to
aliens and (b) of natural-born Filipinos who have lost their Philippine citizenship on account or
political or economic necessity. The pertinent provisions of the law read:
Sec. 1. Filipino women who have lost their Philippine citizenship by
marriage to aliens and natural-born Filipinos who have lost their Philippine
citizenship, including their minor children, on account of political or
economic necessity, may reacquire Philippine citizenship through
repatriation in the manner provided in Section 4 of Commonwealth Act No.
631, as amended: Provided, That the applicant is not a:
(1) Person opposed to organized government or affiliated with any
association or group of persons who uphold and teach doctrines opposing
organized government;
(2) Person defending or teaching the necessity or propriety of violence,
personal assault, or association for the predominance of their ideas;
(3) Person convicted of crimes involving moral turpitude: or
(4) Person suffering from mental alienation or incurable contagious
diseases.
102

Sec. 2. Repatriation shall be effected by taking the necessary oath of


allegiance to the Republic of the Philippines and registration in the proper
civil registry and in the Bureau of Immigration. The Bureau of Immigration
shall thereupon cancel the pertinent alien certificate of registration and
issue the certificate of identification as Filipino citizen to the repatriated
citizen.
Under Section 1 of Presidential Decree ("P.D.") No. 725, 8 dated 05 June 1975, amending
Commonwealth Act No. 63, an application for repatriation could be filed by Filipino women
who lost their Philippine citizenship by marriage to aliens, as well as by natural born Filipinos
who lost their Philippine citizenship, with the Special Committee on Naturalization. The
committee, chaired by the Solicitor General with the Undersecretary of Foreign Affairs and the
Director of the National Intelligence Coordinating Agency as the other members, was created
pursuant to Letter of Instruction ("LOI") No. 270, dated 11 April 1975, as amended by LOI No.
283 and LOI No. 491 issued, respectively, on 04 June 1975 and on 29 December 1976.
Although the agency was deactivated by virtue of President Corazon C. Aquino's
Memorandum of 27 March 1987, it was not however, abrogated. In Frivaldo vs. Commission
on Elections, 9 the Court observed that the aforedated memorandum of President Aquino had
merely directed the Special Committee on Naturalization "to cease and desist from
undertaking any and all proceedings . . . under Letter of Instruction ("LOI") 270." 10 The Court
elaborated:
This memorandum dated March 27, 1987 cannot by any stretch of legal
hermeneutics be construed as a law sanctioning or authorizing a repeal of
P.D. No. 725. Laws are repealed only by subsequent ones and a repeal
may be express or implied. It is obvious that no express repeal was made
because then President Aquino in her memorandum-based on the copy
furnished us by Lee-did not categorically and/or impliedly state that P.D.
725 was being repealed or was being rendered without any legal effect. In
fact, she did not even mention it specifically by its number or text. On the
other hand, it is a basic rule of statutory construction that repeals by
implication are not favored. An implied repeal will not be allowed "unless it
is convincingly and unambiguously demonstrated that the two laws are
clear repugnant and patently inconsistent that they cannot co-exist."
The memorandum of then President Aquino cannot even be regarded as
a legislative enactment, for not every pronouncement of the Chief
Executive even under the Transitory Provisions of the 1987 Constitution
can nor should be regarded as an exercise of her law-making powers. At
best, it could be treated as an executive policy addressed to the Special
Committee to halt the acceptance and processing of applications for
repatriation pending whatever "judgment the first Congress under the
1987 Constitution" might make. In other words, the former President did
not-repeal P.D. 725 but left it to the first Congress once created to
deal with the matter. If she had intended to repeal such law, she should
have unequivocally said so instead of referring the matter to Congress.
The fact is she carefully couched her presidential issuance in terms that
clearly indicated the intention of "the present government, in the exercise
of prudence and sound discretion" to leave the matter of repeal to the new

Congress. Any other interpretation of the said Presidential Memorandum,


such as is now being proffered to the Court by Lee, would visit
unmitigated violence not only upon statutory construction but on common
sense as well. 11
Indeed, the Committee was reactivated on 08 June 1995; 12 hence, when petitioner
filed his petition on 11 March 1996, the Special Committee on Naturalization
constituted pursuant to LOI No. 270 under P.D. No. 725 was in place. Administrative
Order 285, 13 promulgated on 22 August 1996 relative to R.A. No. 8171, in effect,
was merely then a confirmatory issuance.
The Office of the Solicitor General was right in maintaining that Angat's petition should have
been filed with the Committee, aforesaid, and not with the RTC which had no jurisdiction
thereover. The court's order of 04 October 1996 was thereby null and void, and it did not
acquire finality 14 nor could be a source of right on the part of petitioner. 15 It should also be
noteworthy that the petition in Case No. N-96-03-MK was one for repatriation, and it was thus
incorrect for petitioner to initially invoke Republic Act No. 965 16 and R.A. No. 2630 17 since
these laws could only apply to persons who had lost their citizenship by rendering service to,
or accepting commission in, the armed forces of an allied foreign country or the armed forces
of the United States of America, a factual matter not alleged in the petition, Parenthetically,
under these statutes, the person desiring to re-acquire Philippine citizenship would not even
be required to file a petition in court, and all that he had to do was to take an oath of
allegiance to the Republic of the Philippines and to register that fact with the civil registry in
the place of his residence or where he had last resided in the Philippines.
WHEREFORE, the petition for review is DENIED, and the Order, dated 22 September 1996,
issued by the court a quo, dismissing the petition of petitioner in Civil Case No. N-96-03-MK
for want of jurisdiction, is AFFIRMED. No costs.
SO ORDERED.

103

G.R. No. 163256


November 10, 2004
CICERON P. ALTAREJOS, petitioner,
vs.
COMMISSION ON ELECTIONS, JOSE ALMIE and VERNON VERSOZA, respondents.

The provisions of law governing the qualifications and disqualifications of elective


local officials are found in Sections 39 and 40 of Republic Act No. 7160 otherwise
known as the Local Government Code of 1991, which provide as follows:
SEC. 39. Qualifications. (a) An elective local official must be a citizen of the
Philippines; a registered voter in the barangay, municipality, city or province or, in
the case of member of the sangguniang panlalawigan, sangguniang panlungsod, or
sangguniang bayan, the district where he intends to be elected; a resident therein
for at least one (1) year immediately preceding the day of the election; and able to
read and write Filipino or any other local language or dialect.
xxx.
(c) Candidates for the position of mayor or vice-mayor of independent component
cities, component cities or municipalities must be at least twenty-one (21) years of
age on election day.
[SEC. 40. Disqualifications. The following persons are disqualified from running for
any elective position:]
xxx.
(d) Those with dual citizenship.
xxx.
(f) Permanent residents in a foreign country or those who have acquired the right to
reside abroad and continue to avail of the same right after the effectivity of this
Code; xxx
Under the terms of the above quoted statutory provisions, it is required that an
elective local official must be a citizen of the Philippines, and he must not have a
dual citizenship; must not be a permanent resident in a foreign country or must not
have acquired the right to reside abroad.
In the present case, it has been established by clear and convincing evidence that
respondent is a citizen of the United States of America. Such fact is proven by his
Alien Certificate of Registration (ACR) No. E139507 issued on 3 November 1997
and Immigration Certificate of Residence (ICR) with No. 320846 issued on 3
November 1997 by the Alien Registration Division, Bureau of Immigration and
Deportation. This was further confirmed in a letter dated 25 June 2001 of then
Commissioner ANDREA D. DOMINGO of the Bureau of Immigration and
Deportation.
Although respondent had petitioned for his repatriation as a Filipino citizen under
Republic Act No. 8171 on 17 December 1997, this did not restore to respondent his
Filipino citizenship, because Section 2 of the aforecited Republic Act No. 8171
specifically provides that "repatriation shall be effected by taking the necessary oath
of allegiance to the Republic of the Philippines and registration in the proper civil
registry and in the Bureau of Immigration."
It appears from the records of this case that respondent failed to prove that he has
fully complied with requirements of the above-quoted Section 2 of Republic Act
8171 to perfect his repatriation and reacquire his Filipino citizenship. Respondent
has not submitted any document to prove that he has taken his oath of allegiance to

DECISION
AZCUNA, J.:
This is a petition for certiorari, with prayer for the issuance of a temporary restraining order
and/or a writ of prohibitory and mandatory injunction, to set aside the Resolution promulgated
by the Commission on Elections (COMELEC), First Division, on March 22, 2004 disqualifying
petitioner Ciceron P. Altarejos from running as mayor of San Jacinto, Masbate, and another
resolution of the COMELEC en banc promulgated on May 7, 2004 denying petitioner's motion
for reconsideration.
The factual antecedents are as follows:
Petitioner Altarejos was a candidate for mayor in the Municipality of San Jacinto, Masbate in
the May 10, 2004 national and local elections.
On January 15, 2004, private respondents Jose Almie Altiche and Vernon Versoza,
registered voters of San Jacinto, Masbate, filed with the COMELEC, a petition to disqualify
and to deny due course or cancel the certificate of candidacy of petitioner on the ground that
he is not a Filipino citizen and that he made a false representation in his certificate of
candidacy that "[he] was not a permanent resident of or immigrant to a foreign country."
Private respondents alleged that based on a letter1 from the Bureau of Immigration dated June
25, 2001, petitioner was a holder of a permanent U.S. resident visa, an Alien Certificate of
Registration No. E139507 issued on November 3, 1997, and an Immigration Certificate of
Residence No. 320846 issued on November 3, 1997 by the Bureau of Immigration.2
On January 26, 2004, petitioner filed an Answer3 stating, among others, that he did not
commit false representation in his application for candidacy as mayor because as early as
December 17, 1997, he was already issued a Certificate of Repatriation by the Special
Committee on Naturalization, after he filed a petition for repatriation pursuant to Republic Act
No. 8171. Thus, petitioner claimed that his Filipino citizenship was already restored, and he
was qualified to run as mayor in the May 10, 2004 elections. Petitioner sought the dismissal of
the petition.
On the date of the hearing, the parties were required to submit their Memoranda within three
days. Private respondents filed their Memorandum, while petitioner did not file one within the
required period.4 Petitioner, however, filed a Reply Memorandum5 subsequently.
Atty. Zacarias C. Zaragoza, Jr., regional election director for Region V and hearing officer of
this case, recommended that petitioner Altarejos be disqualified from being a candidate for the
position of mayor of San Jacinto, Masbate in the May 10, 2004 national and local elections.
He found, thus:
xxx

104

the Republic of the Philippines and that he has registered his fact of repatriation in
the proper civil registry and in the Bureau of Immigration. In fact, in a letter date 25
June 2001, Commissioner ANDREA DOMINGO stated that RESPONDENT is still a
holder of visa under Section 13 (g) of the Philippine Immigration Act of 1940 as
amended, with an indefinite authorized stay in the Philippines, implying that
respondent did not register his supposed Certificate of Repatriation with the Bureau
of Immigration otherwise his Alien Visa would have already been cancelled. The rule
is that in case of doubt concerning the grant of citizenship, such doubt should be
resolved in favor of the State and against the applicant (Cheng vs. Republic, L16999, 22 June 1965).
xxx
Not having been able to prove that he has fully reacquired his Filipino citizenship
after being naturalized as a citizen of the United States, it is clear that respondent is
not qualified to be candidate for the position of Mayor of San Jacinto, Masbate, in
the 10 May 2004 National and Local Elections, pursuant to the aforequoted Sections
39 and 40 of the Local Government Code of 1991.
As a further consequence of his not being a Filipino citizen, respondent has also
committed false representation in his certificate of candidacy by stating therein that
he is a natural-born Filipino citizen, when in fact, he has not yet even perfected the
reacquisition of Filipino citizenship. Such false representation constitutes a material
misrepresentation as it relates to his qualification as a candidate for public office,
which could be a valid ground for the cancellation of his certificate of candidacy
under Section 78 of the Omnibus Election Code x x x. 6
In its Resolution promulgated on March 22, 2004, the COMELEC, First Division, adopted the
findings and recommendation of Director Zaragoza. The dispositive portion of said Resolution
stated, thus:
WHEREFORE, premises considered, respondent CICERON PEREZ ALTAREJOS
is hereby disqualified to run as Mayor of San Jacinto, Masbate. Accordingly, his
certificate of candidacy for the position of Municipal Mayor of San Jacinto, Masbate
is denied due course and cancelled and his name deleted from the certified list of
candidates for the May 10, 2004 elections.7
On March 25, 2004, petitioner filed a motion for reconsideration and attached the following
documents to prove that he had completed all the requirements for repatriation which thus
entitled him to run for an elective office, viz:
(1) Oath of Allegiance dated December 17, 1997;
(2) Identification Certificate No. 116543 issued by the Bureau of Immigration on March 1,
2004;
(3) Certification from the City Civil Registration Office, Makati City, that the Certificate of
Repatriation and Oath of Allegiance of petitioner was received by said office and registered,
with the corresponding fee paid, on February 18, 2004;
(4) A letter dated December 17, 1997 from the Special Committee on Naturalization to the
Bureau on Immigration and Deportation that it was furnishing said office with the Oath of
Allegiance and Certificate of Repatriation of petitioner for the cancellation of petitioner's

registration in said office as an alien, and the issuance to him of the corresponding
Identification Card as Filipino citizen;
(5) A letter dated December 17, 1997 from the Special Committee on Naturalization to the
Local Registrar of San Jacinto, Masbate that it was sending petitioner's Oath of Allegiance
and Certificate of Repatriation for registration in their records and for petitioner's reacquisition
of his former Philippine citizenship.
On May 7, 2004, the COMELEC en banc promulgated a resolution denying the motion for
reconsideration, the dispositive portion of which reads:
WHEREFORE, premises considered, the Commission (En Banc) RESOLVED as it
hereby RESOLVES to DENY the Motion for Reconsideration for UTTER LACK OF
MERIT and AFFIRMS the Resolution of the First Division.8
The Comelec en banc held, thus:
The Comelec Rules of Procedure provides that insufficiency of evidence to justify
the decision is a ground for a motion for reconsideration (Rule 19, Section 1). The
evidence referred to in the above provision and to be considered in the Motion for
Reconsideration are those which were submitted during the hearing and attached to
the respective Memoranda of the parties which are already part of the records of the
case. In this regard, the evidence of the respondent were not able to overcome the
evidence of the petitioners.
When the entire records of the case was forwarded to the Commission (First
Division) the respondent's only evidence was his Certificate of Repatriation dated 17
December 1977 and marked as Annex 1 of his answer. This piece of evidence was
not enough to controvert the evidence of the petitioners which consist of the letter of
the then Bureau of Immigration Commissioner Andrea Domingo dated 25 June 2001
which stated that as of the even date respondent is a holder of permanent resident
visa (page 15 of the records) and the certification of Josephine C. Camata dated 28
January 2004 certifying, that the name of the respondent could not be found in the
records of repatriation. (page 42 of the records) The questioned resolution, is
therefore, in order as the evidence submitted by the respondent were insufficient to
rebut the evidence of the petitioner.
Now, the respondent, in his Motion for Reconsideration, attempted to introduce to
the record new pieces of evidence, which introduction is not anymore allowed in a
Motion for Reconsideration. These are the following a) Annex "2" Oath of
Allegiance; b) Annex "3" Bureau of Immigration Identification Certificate; c) Annex
"4" Certification of the City Civil Registrar of Makati City; d) Annex "5" Letter
addressed to the Local Civil Registrar of San Jacinto, Masbate by Aurora P. Cortes
of Special Committee on Naturalization; and e) Annex "6" Letter addressed to the
Bureau of Immigration and Deportation by Aurora P. Cortes of Special Committee
on Naturalization.
Assuming that the new evidence of the respondent are admitted, with more reason
should we cancel his certificate of candidacy for his act of [misrepresenting] himself
as a Filipino citizen when at the time he filed his certificate of candidacy, he has not
yet perfected the process of repatriation. He failed to comply with the requirements
under Section 2 of [Republic Act No.] 8171 which provides that repatriation shall be
105

effected by taking the necessary oath of allegiance to the Republic of the Philippines
and registration in the proper civil registry and in the Bureau of Immigration.
The certification was issued by the same Ms. Josephine C. Camata, City Civil
Registrar, dated February 18, 2004. This time, she certifies that Ciceron Perez
Altarejos was registered under Registry No. 1, Page 19, Book No. 1, Series of 2004
and paid under OR nos. 88325/8833256 dated February 18, 2004. (page 65 of the
records). Obviously, he was able to register in the proper civil registry only on
February 18, 2004.
The respondent was able to register with the Bureau of Immigration only on March
1, 2004 as evidenced by the Bureau of Immigration Identification Certificate
attached to the Motion as Annex "3."
This fact confirms the finding of the Commission (First Division) that at the time
respondent filed his certificate of candidacy he is yet to complete the requirement
under section two (2) of RA 8171.
As a consequence of not being a Filipino citizen, he has committed false
representation in his certificate of candidacy. Such false representation constitutes a
material misrepresentation as it relates to his qualification as a candidate. As such
the certificate of candidacy may be cancelled on such ground. (Ycain vs. Caneja, 18
Phil. 778)9
On May 10, 2004, the election day itself, petitioner filed this petition praying that: (1) The
petition be given due course and a temporary restraining order and/or writ of preliminary
injunction be issued ex parte restraining the respondents and all persons acting on their
behalf, from fully implementing the questioned COMELEC Resolutions promulgated on March
22, 2004 and May 7, 2004; (2) a writ of preliminary mandatory injunction be issued ordering
the COMELEC and all persons acting on its behalf to allow petitioner to run as Mayor of San
Jacinto, Masbate in the May 10, 2004 elections, and to count and canvass the votes cast in
his favor and to proclaim him as the winning mayor of San Jacinto, Masbate; and (3) after
proper proceedings, judgment be rendered declaring null and void and setting aside the
COMELEC Resolutions promulgated on March 22, 2004 and May 7, 2004 and other related
Orders of the COMELEC or its representatives which have the effect of illegally preventing
petitioner from running as Mayor of San Jacinto, Masbate.
In its Comment,10 the Office of the Solicitor General stated that, based on the information
relayed to it by the COMELEC, petitioner's name, as a mayoralty candidate in San Jacinto,
Masbate, was retained in the list of candidates voted upon by the electorate in the said
municipality. Hence, the cancellation of petitioner's certificate of candidacy was never
implemented. The COMELEC also informed the Office of the Solicitor General that petitioner's
opponent, Dr. Emilio Aris V. Espinosa, was already proclaimed duly elected Mayor of San
Jacinto, Masbate.
The Office of the Solicitor General contends that said supervening event has rendered the
instant petition moot and academic, and it prayed for the dismissal of the petition.
In his Reply,11 petitioner opposed the dismissal of his petition. He claims that the COMELEC
resolutions disqualifying him from running as a mayoralty candidate adversely affected his
candidacy, since his supporters were made to believe that his votes would not be counted.

Moreover, he stated that said COMELEC resolutions cast a doubt on his Philippine
citizenship.
Petitioner points out that he took his Oath of Allegiance to the Republic of the Philippines on
December 17, 1997. In view thereof, he ran and was even elected as Mayor of San Jacinto,
Masbate during the 1998 elections. He argues that if there was delay in the registration of his
Certificate of Repatriation with the Bureau of Immigration and with the proper civil registry, the
same was brought about by the inaction on the part of said offices since the records of the
Special Committee on Naturalization show that his Certificate of Repatriation and Oath of
Allegiance have long been transmitted to said offices.
Petitioner also asserts that the subsequent registration of his Certificate of Repatriation with
the Bureau of Immigration and with the Civil Registry of Makati City prior to the May 10, 2004
elections has the effect of curing the defect, if any, in the reacquisition of his Filipino
citizenship as his repatriation retroacted to the date of his application for repatriation as held in
Frivaldo v. Comelec.
The pertinent issues raised are the following: (1) Is the registration of petitioner's repatriation
with the proper civil registry and with the Bureau of Immigration a prerequisite in effecting
repatriation; and (2) whether or not the COMELEC en banc committed grave abuse of
discretion amounting to excess or lack of jurisdiction in affirming the Resolution of the
COMELEC, First Division.
As stated by the Office of the Solicitor General, where the issues have become moot and
academic, there is no justiciable controversy, thereby rendering the resolution of the same of
no practical use or value.12 Nonetheless, courts will decide a question otherwise moot and
academic if it is capable of repetition, yet evading review.13
First Issue: Is the registration of petitioner's repatriation
with the proper civil registry and with the Bureau of
Immigration a prerequisite in effecting repatriation?
The provision of law applicable in this case is Section 2 of Republic Act No. 8171,14 thus:
SEC. 2. Repatriation shall be effected by taking the necessary oath of allegiance to
the Republic of the Philippines and registration in the proper civil registry and in the
Bureau of Immigration. The Bureau of Immigration shall thereupon cancel the
pertinent alien certificate of registration and issue the certificate of identification as
Filipino citizen to the repatriated citizen.
The law is clear that repatriation is effected "by taking the oath of allegiance to the Republic of
the Philippines and registration in the proper civil registry and in the Bureau of Immigration."
Hence, in addition to taking the Oath of Allegiance to the Republic of the Philippines, the
registration of the Certificate of Repatriation in the proper civil registry and the Bureau of
Immigration is a prerequisite in effecting the repatriation of a citizen.
In this case, petitioner took his Oath of Allegiance on December 17, 1997, but his Certificate
of Repatriation was registered with the Civil Registry of Makati City only after six years or on
February 18, 2004, and with the Bureau of Immigration on March 1, 2004. Petitioner,
therefore, completed all the requirements of repatriation only after he filed his certificate of
candidacy for a mayoralty position, but before the elections.
When does the citizenship qualification of a candidate for an elective office apply?
106

In Frivaldo v. Commission on Elections,15 the Court ruled that the citizenship qualification must
be construed as "applying to the time of proclamation of the elected official and at the start of
his term." The Court, through Justice Artemio V. Panganiban, discussed, thus:
Under Sec. 39 of the Local Government Code, "(a)n elective local official must be:
* a citizen of the Philippines;
* a registered voter in the barangay, municipality, city, or province x x x where he
intends to be elected;
* a resident therein for at least one (1) year immediately preceding the day of the
election;
* able to read and write Filipino or any other local language or dialect."
* In addition, "candidates for the position of governor x x x must be at least twentythree (23) years of age on election day."
From the above, it will be noted that the law does not specify any particular date or
time when the candidate must possess citizenship, unlike that for residence (which
must consist of at least one year's residency immediately preceding the day of
election) and age (at least twenty three years of age on election day).
Philippine citizenship is an indispensable requirement for holding an elective public
office, and the purpose of the citizenship qualification is none other than to ensure
that no alien, i.e., no person owing allegiance to another nation, shall govern our
people and our country or a unit of territory thereof. Now, an official begins to govern
or to discharge his functions only upon his proclamation and on the day the law
mandates his term of office to begin. Since Frivaldo re-assumed his citizenship on
June 30, 1995the very day the term of office of governor (and other elective
officials) beganhe was therefore already qualified to be proclaimed, to hold such
office and to discharge the functions and responsibilities thereof as of said date. In
short, at that time, he was already qualified to govern his native Sorsogon. This is
the liberal interpretation that should give spirit, life and meaning to our law on
qualifications consistent with the purpose for which such law was enacted. x x x
Paraphrasing this Court's ruling in Vasquez v. Giap and Li Seng Giap & Sons, if the
purpose of the citizenship requirement is to ensure that our people and country do
not end up being governed by aliens, i.e., persons owing allegiance to another
nation, that aim or purpose would not be thwarted but instead achieved by
construing the citizenship qualification as applying to the time of proclamation of the
elected official and at the start of his term.16 (Emphasis supplied.)
Moreover, in the case of Frivaldo v. Commission on Elections, the Court ruled that
"the repatriation of Frivaldo RETROACTED to the date of the filing of his
application." In said case, the repatriation of Frivaldo was by virtue of Presidential
Decree No. 725, which took effect on June 5, 1975. The Court therein declared that
Presidential Decree No. 725 was a curative statute, which is retroactive in nature.
The retroactivity of Frivaldo's repatriation to the date of filing of his application was
justified by the Court, thus:
xxx
The reason for this is simply that if, as in this case, it was the intent of the
legislative authority that the law should apply to past eventsi.e., situations and

transactions existing even before the law came into beingin order to benefit the
greatest number of former Filipinos possible thereby enabling them to enjoy and
exercise the constitutionally guaranteed right of citizenship, and such legislative
intention is to be given the fullest effect and expression, then there is all the more
reason to have the law apply in a retroactive or retrospective manner to situations,
events and transactions subsequent to the passage of such law. That is, the
repatriation granted to Frivaldo x x x can and should be made to take effect as of
date of his application. As earlier mentioned, there is nothing in the law that would
bar this or would show a contrary intention on the part of the legislative authority;
and there is no showing that damage or prejudice to anyone, or anything unjust or
injurious would result from giving retroactivity to his repatriation. Neither has Lee
shown that there will result the impairment of any contractual obligation, disturbance
of any vested right or breach of some constitutional guaranty.
xxx
Another argument for retroactivity to the date of filing is that it would prevent
prejudice to applicants. If P.D. 725 were not to be given retroactive effect, and the
Special Committee decides not to act, i.e., to delay the processing of applications
for any substantial length of time, then the former Filipinos who may be stateless, as
Frivaldohaving already renounced his American citizenshipwas, may be
prejudiced for causes outside their control. This should not be. In case of doubt in
the interpretation or application of laws, it is to be presumed that the law-making
body intended right and justice to prevail.17
Republic Act No. 817118 has impliedly repealed Presidential `Decree No. 725. They cover the
same subject matter: Providing for the repatriation of Filipino women who have lost their
Philippine citizenship by marriage to aliens and of natural-born Filipinos. The Court's ruling in
Frivaldo v. Commission on Elections that repatriation retroacts to the date of filing of one's
application for repatriation subsists for the same reasons quoted above.
Accordingly, petitioner's repatriation retroacted to the date he filed his application in 1997.
Petitioner was, therefore, qualified to run for a mayoralty position in the government in the
May 10, 2004 elections. Apparently, the COMELEC was cognizant of this fact since it did not
implement the assailed Resolutions disqualifying petitioner to run as mayor of San Jacinto,
Masbate.
Second Issue: Whether or not the COMELEC en banc
gravely abused its discretion in affirming the
Resolution of the COMELEC, First Division?
The Court cannot fault the COMELEC en banc for affirming the decision of the COMELEC,
First Division, considering that petitioner failed to prove before the COMELEC that he had
complied with the requirements of repatriation. Petitioner submitted the necessary documents
proving compliance with the requirements of repatriation only during his motion for
reconsideration, when the COMELEC en banc could no longer consider said evidence. As the
COMELEC en banc correctly stated:
The Comelec Rules of Procedure provides that insufficiency of evidence to justify
the decision is a ground for a motion for reconsideration (Rule 19, Section 1). The
evidence referred to in the above provision and to be considered in the Motion for
Reconsideration are those which were submitted during the hearing and attached to
107

the respective Memoranda of the parties which are already part of the records of the
case. In this regard, the evidence of the respondent were not able to overcome the
evidence of the petitioners.19
It is, therefore, incumbent upon candidates for an elective office, who are repatriated citizens,
to be ready with sufficient evidence of their repatriation in case their Filipino citizenship is
questioned to prevent a repetition of this case.
WHEREFORE, the petition seeking the nullification of the Resolution of the COMELEC en
banc of May 7, 2004, affirming the Resolution of its First Division dated March 22, 2004, is
hereby DENIED. No costs.
SO ORDERED.

108

G.R. No. 125793 August 29, 2006


JOEVANIE ARELLANO TABASA, Petitioner,
vs.
HON. COURT OF APPEALS, BUREAU OF IMMIGRATION and DEPORTATION and
WILSON SOLUREN, Respondents.
DECISION
VELASCO, JR., J.:
Citizenship is a priceless possession. Former U.S. Chief Justice Earl Warren fittingly
emphasized its crowning value when he wrote that "it is mans basic right for it is nothing less
than to have rights." 1 When a person loses citizenship, therefore, the State sees to it that its
reacquisition may only be granted if the former citizen fully satisfies all conditions and
complies with the applicable law. Without doubt, repatriation is not to be granted simply based
on the vagaries of the former Filipino citizen.
The Case
The instant petition for review 2 under Rule 45 of the 1997 Rules of Civil Procedure contests
the denial by the Court of Appeals (CA) of the Petition for Habeas Corpus interposed by
petitioner Joevanie Arellano Tabasa from the Order of Summary Deportation issued by the
Bureau of Immigration and Deportation (BID) for his return to the United States.
The Facts
The facts as culled by the CA from the records show that petitioner Joevanie Arellano Tabasa
was a natural-born citizen of the Philippines. In 1968, 3 when petitioner was seven years old, 4
his father, Rodolfo Tabasa, became a naturalized citizen 5 of the United States. By derivative
naturalization (citizenship derived from that of another as from a person who holds citizenship
by virtue of naturalization 6), petitioner also acquired American citizenship.
Petitioner arrived in the Philippines on August 3, 1995, and was admitted as a "balikbayan" for
one year. Thereafter, petitioner was arrested and detained by agent Wilson Soluren of the BID
on May 23, 1996, pursuant to BID Mission Order No. LIV-96-72 in Baybay, Malay, Aklan;
subsequently, he was brought to the BID Detention Center in Manila. 7
Petitioner was investigated by Special Prosecutor Atty. Edy D. Donato at the Law and
Investigation Division of the BID on May 28, 1996; and on the same day, Tabasa was accused
of violating Section 8, Chapter 3, Title 1, Book 3 of the 1987 Administrative Code, in a charge
sheet which alleged:
1. That on 3 August 1995, respondent (petitioner herein [Tabasa]) arrived in the Philippines
and was admitted as a balikbayan;
2. That in a letter dated 16 April 1996, Honorable Kevin Herbert, Consul General of [the] U.S.
Embassy, informed the Bureau that respondents Passport No. 053854189 issued on June 10,
1994 in San Francisco, California, U.S.A., had been revoked by the U.S. Department of State;
3. Hence, respondent [petitioner Tabasa] is now an undocumented and undesirable alien and
may be summarily deported pursuant to Law and Intelligence Instructions No. 53 issued by
then Commissioner Miriam Defensor Santiago to effect his deportation (Exhibit 3). 8
The pertinent portion of the Herbert letter is as follows:
The U.S. Department of State has revoked U.S. passport 053854189 issued on June 10, 1994
in San Francisco, California under the name of Joevanie Arellano Tabasa, born on February

21, 1959 in the Philippines. Mr. Tabasas passport has been revoked because he is the
subject of an outstanding federal warrant of arrest issued on January 25, 1996 by the U.S.
District Court for the Northern District of California, for violation of Section 1073, "Unlawful
Flight to Avoid Prosecution," of Title 18 of the United States Code. He is charged with one
count of a felon in possession of a firearm, in violation of California Penal Code, Section
12021(A)(1), and one count of sexual battery, in violation of California Penal Code, Section
243.4 (D). 9
The BID ordered petitioners deportation to his country of origin, the United States, on May 29,
1996, in the following summary deportation order:
Records show that on 16 April 1996, Mr. Kevin F. Herbert, Consul General of the U.S.
Embassy in Manila, filed a request with the Bureau to apprehend and deport the abovenamed
[sic] respondent [petitioner Tabasa] on the ground that a standing warrant for several federal
charges has been issued against him, and that the respondents Passport No. 053854189 has
been revoked.
By reason thereof, and on the strength of Mission Order No. LIV-96-72, Intelligence operatives
apprehended the respondent in Aklan on 23 May 1996.
In Schonemann vs. Commissioner Santiago, et al., (G.R. No. 81461 [sic, 81461 should be
86461], 30 May 1989), the Supreme Court ruled that if a foreign embassy cancels the
passport of an alien, or does not reissue a valid passport to him, the alien loses the privilege
to remain in the country. Further, under Office Memorandum Order No. 34 issued on 21
August 1989, summary deportation proceedings lie where the passport of the alien has
expired.
It is, thus, apparent that respondent has lost his privilege to remain in the country. 10
Petitioner filed before the CA a Petition for Habeas Corpus with Preliminary Injunction and/or
Temporary Restraining Order 11 on May 29, 1996, which was docketed as CA-G.R. SP No.
40771. Tabasa alleged that he was not afforded due process; that no warrant of arrest for
deportation may be issued by immigration authorities before a final order of deportation is
made; that no notice of the cancellation of his passport was made by the U.S. Embassy; that
he is entitled to admission or to a change of his immigration status as a non-quota immigrant
because he is married to a Filipino citizen as provided in Section 13, paragraph (a) of the
Philippine Immigration Act of 1940; and that he was a natural-born citizen of the Philippines
prior to his derivative naturalization when he was seven years old due to the naturalization of
his father, Rodolfo Tabasa, in 1968.
At the time Tabasa filed said petition, he was already 35 years old. 12
On May 30, 1996, the CA ordered the respondent Bureau to produce the person of the
petitioner on June 3, 1996 and show the cause of petitioners detention, and restrained the
Bureau from summarily deporting him. On June 3, 1996, the BID presented Tabasa before the
CA; and on June 6, 1996, the CA granted both parties ten (10) days within which to file their
memoranda, after which the case would be considered submitted for decision. 13 Meanwhile,
the Commissioner of Immigration granted the petitioners temporary release on bail on a PhP
20,000.00 cash bond. 14
However, on June 13, 1996, petitioner filed a Supplemental Petition alleging that he had
acquired Filipino citizenship by repatriation in accordance with Republic Act No. 8171 (RA

109

8171), and that because he is now a Filipino citizen, he cannot be deported or detained by the
respondent Bureau. 15
The Ruling of the Court of Appeals
The CA, in its August 7, 1996 Decision, 16 denied Tabasas petition on the ground that he had
not legally and successfully acquiredby repatriationhis Filipino citizenship as provided in
RA 8171. The court said that although he became an American citizen by derivative
naturalization when his father was naturalized in 1968, there is no evidence to show that he
lost his Philippine citizenship "on account of political or economic necessity," as explicitly
provided in Section 1, RA 8171the law governing the repatriation of natural-born Filipinos
who have lost their citizenship. The affidavit does not state that political or economic necessity
was the compelling reason for petitioners parents to give up their Filipino citizenship in 1968.
Moreover, the court a quo found that petitioner Tabasa did not dispute the truth of the April 16,
1996 letter of the United States Consul General Kevin F. Herbert or the various warrants
issued for his arrest by the United States court. The court a quo noted that after petitioner was
ordered deported by the BID on May 29, 1996, he successively executed an Affidavit of
Repatriation on June 6, 1996 and took an oath of allegiance to the Republic of the Philippines
on June 13, 1996more than ten months after his arrival in the country on August 3, 1995.
The appellate court considered petitioners "repatriation" as a last ditch effort to avoid
deportation and prosecution in the United States. The appellate court concluded that his only
reason to want to reacquire Filipino citizenship is to avoid criminal prosecution in the United
States of America. The court a quo, therefore, ruled against Tabasa, whose petition is now
before us.
The Issue
The only issue to be resolved is whether petitioner has validly reacquired Philippine
citizenship under RA 8171. If there is no valid repatriation, then he can be summarily deported
for his being an undocumented alien.
The Courts Ruling
The Court finds no merit in this petition.
RA 8171, "An Act Providing for the Repatriation of Filipino Women Who Have Lost Their
Philippine Citizenship by Marriage to Aliens and of Natural-Born Filipinos," was enacted on
October 23, 1995. It provides for the repatriation of only two (2) classes of persons, viz:
Filipino women who have lost their Philippine citizenship by marriage to aliens and naturalborn Filipinos who have lost their Philippine citizenship, including their minor children, on
account of political or economic necessity, may reacquire Philippine citizenship through
repatriation in the manner provided in Section 4 of Commonwealth Act No. 63, as amended:
Provided, That the applicant is not a:
(1) Person opposed to organized government or affiliated with any association or group of
persons who uphold and teach doctrines opposing organized government;
(2) Person defending or teaching the necessity or propriety of violence, personal assault, or
association for the predominance of their ideas;
(3) Person convicted of crimes involving moral turpitude; or
(4) Person suffering from mental alienation or incurable contagious diseases. 17 (Emphasis
supplied.)

Does petitioner Tabasa qualify as a natural-born Filipino who had lost his Philippine
citizenship by reason of political or economic necessity under RA 8171?
He does not.
Persons qualified for repatriation under RA 8171
To reiterate, the only persons entitled to repatriation under RA 8171 are the following:
a. Filipino women who lost their Philippine citizenship by marriage to aliens; and
b. Natural-born Filipinos including their minor children who lost their Philippine citizenship on
account of political or economic necessity.
Petitioner theorizes that he could be repatriated under RA 8171 because he is a child of a
natural-born Filipino, and that he lost his Philippine citizenship by derivative naturalization
when he was still a minor.
Petitioner overlooks the fact that the privilege of repatriation under RA 8171 is available only
to natural-born Filipinos who lost their citizenship on account of political or economic
necessity, and to the minor children of said natural-born Filipinos. This means that if a parent
who had renounced his Philippine citizenship due to political or economic reasons later
decides to repatriate under RA 8171, his repatriation will also benefit his minor children
according to the law. This includes a situation where a former Filipino subsequently had
children while he was a naturalized citizen of a foreign country. The repatriation of the former
Filipino will allow him to recover his natural-born citizenship and automatically vest Philippine
citizenship on his children of jus sanguinis or blood relationship: 18 the children acquire the
citizenship of their parent(s) who are natural-born Filipinos. To claim the benefit of RA 8171,
however, the children must be of minor age at the time the petition for repatriation is filed by
the parent. This is so because a child does not have the legal capacity for all acts of civil life
much less the capacity to undertake a political act like the election of citizenship. On their
own, the minor children cannot apply for repatriation or naturalization separately from their
parents.
In the case at bar, there is no dispute that petitioner was a Filipino at birth. In 1968, while he
was still a minor, his father was naturalized as an American citizen; and by derivative
naturalization, petitioner acquired U.S. citizenship. Petitioner now wants us to believe that he
is entitled to automatic repatriation as a child of natural-born Filipinos who left the country due
to political or economic necessity. This is absurd. Petitioner was no longer a minor at the time
of his "repatriation" on June 13, 1996. The privilege under RA 8171 belongs to children who
are of minor age at the time of the filing of the petition for repatriation.
Neither can petitioner be a natural-born Filipino who left the country due to political or
economic necessity. Clearly, he lost his Philippine citizenship by operation of law and not due
to political or economic exigencies. It was his father who could have been motivated by
economic or political reasons in deciding to apply for naturalization. The decision was his
parents and not his. The privilege of repatriation under RA 8171 is extended directly to the
natural-born Filipinos who could prove that they acquired citizenship of a foreign country due
to political and economic reasons, and extended indirectly to the minor children at the time of
repatriation.
In sum, petitioner is not qualified to avail himself of repatriation under RA 8171. However, he
can possibly reacquire Philippine citizenship by availing of the Citizenship Retention and Re110

acquisition Act of 2003 (Republic Act No. 9225) by simply taking an oath of allegiance to the
Republic of the Philippines.
Where to file a petition for repatriation pursuant to RA 8171
Even if we concede that petitioner Tabasa can avail of the benefit of RA 8171, still he failed to
follow the procedure for reacquisition of Philippine citizenship. He has to file his petition for
repatriation with the Special Committee on Naturalization (SCN), which was designated to
process petitions for repatriation pursuant to Administrative Order No. 285 (A.O. No. 285)
dated August 22, 1996, to wit:
Section 1. Composition.The composition of the Special Committee on Naturalization, with
the Solicitor General as Chairman, the Undersecretary of Foreign Affairs and the DirectorGeneral of the National Intelligence Coordinating Agency, as members, shall remain as
constituted.
Sec. 2. Procedure.Any person desirous of repatriating or reacquiring Filipino citizenship
pursuant to R.A. No. 8171 shall file a petition with the Special Committee on Naturalization
which shall process the same. If their applications are approved[,] they shall take the
necessary oath of allegiance to the Republic of the Philippines, after which they shall be
deemed to have reacquired Philippine citizenship. The Commission on Immigration and
Deportation shall thereupon cancel their certificate of registration (emphasis supplied).
Sec. 3. Implementing Rules.The Special Committee is hereby authorized to promulgate
rules and regulations and prescribe the appropriate forms and the required fees for the
processing of petitions.
Sec. 4. Effectivity.This Administrative Order shall take effect immediately.
In the Amended Rules and Regulations Implementing RA 8171 issued by the SCN on August
5, 1999, applicants for repatriation are required to submit documents in support of their
petition such as their birth certificate and other evidence proving their claim to Filipino
citizenship. 19 These requirements were imposed to enable the SCN to verify the qualifications
of the applicant particularly in light of the reasons for the renunciation of Philippine citizenship.
What petitioner simply did was that he took his oath of allegiance to the Republic of the
Philippines; then, executed an affidavit of repatriation, which he registered, together with the
certificate of live birth, with the Office of the Local Civil Registrar of Manila. The said office
subsequently issued him a certificate of such registration. 20 At that time, the SCN was already
in place and operational by virtue of the June 8, 1995 Memorandum issued by President Fidel
V. Ramos. 21 Although A.O. No. 285 designating the SCN to process petitions filed pursuant to
RA 8171 was issued only on August 22, 1996, it is merely a confirmatory issuance according
to the Court in Angat v. Republic. 22 Thus, petitioner should have instead filed a petition for
repatriation before the SCN.
Requirements for repatriation under RA 8171
Even if petitionernow of legal agecan still apply for repatriation under RA 8171, he
nevertheless failed to prove that his parents relinquished their Philippine citizenship on
account of political or economic necessity as provided for in the law. Nowhere in his affidavit
of repatriation did he mention that his parents lost their Philippine citizenship on account of
political or economic reasons. It is notable that under the Amended Rules and Regulations
Implementing RA 8171, the SCN requires a petitioner for repatriation to set forth, among
others, "the reason/s why petitioner lost his/her Filipino citizenship, whether by marriage in

case of Filipino woman, or whether by political or economic necessity in case of [a] naturalborn Filipino citizen who lost his/her Filipino citizenship. In case of the latter, such political or
economic necessity should be specified." 23
Petitioner Tabasa asserts, however, that the CA erred in ruling that the applicant for
repatriation must prove that he lost his Philippine citizenship on account of political or
economic necessity. He theorizes that the reference to political or economic reasons is
"merely descriptive, not restrictive, of the widely accepted reasons for naturalization in [a]
foreign country." 24
Petitioners argument has no leg to stand on.
A reading of Section 1 of RA 8171 shows the manifest intent of the legislature to limit the
benefit of repatriation only to natural-born Filipinos who lost their Philippine citizenship on
account of political or economic necessity, in addition to Filipino women who lost their
Philippine citizenship by marriage to aliens. The precursor of RA 8171, Presidential Decree
No. 725 (P.D. 725), 25 which was enacted on June 5, 1975 amending Commonwealth Act No.
63, also gives to the same groups of former Filipinos the opportunity to repatriate but without
the limiting phrase, "on account of political or economic necessity" in relation to natural-born
Filipinos. By adding the said phrase to RA 8171, the lawmakers clearly intended to limit the
application of the law only to political or economic migrants, aside from the Filipino women
who lost their citizenship by marriage to aliens. This intention is more evident in the following
sponsorship speech of Rep. Andrea B. Domingo on House Bill No. 1248, the origin of RA
8171, to wit:
Ms. Domingo: x x x
From my experience as the Commissioner of the Bureau of Immigration and Deportation, I
observed that there are only four types of Filipinos who leave the country.
The first is what we call the "economic refugees" who go abroad to work because there is no
work to be found in the country. Then we have the "political refugees" who leave the country
for fear of their lives because they are not in consonance with the prevailing policy of
government. The third type is those who have committed crimes and would like to escape
from the punishment of said crimes. Lastly, we have those Filipinos who feel that they are not
Filipinos, thereby seeking other citizenship elsewhere.
Of these four types of Filipinos, Mr. Speaker, the first two have to leave the country not of
choice, but rather out of sacrifice to look for a better life, as well as for a safer abode for
themselves and their families. It is for these two types of Filipinos that this measure is being
proposed for approval by this body. (Emphasis supplied.)
xxxx
x x x [I]f the body would recall, I mentioned in my short sponsorship speech the four types of
Filipinos who leave their country. And the two typesthe economic and political refugees
are the ones being addressed by this proposed law, and they are not really Filipino women
who lost their citizenship through marriage. We had a lot of problems with these people who
left the country because of political persecution or because of pressing economic reasons,
and after feeling that they should come back to the country and get back their citizenship and
participate as they should in the affairs of the country, they find that it is extremely difficult to
get their citizenship back because they are treated no different from any other class of alien. 26

111

From these two sources, namely, P.D. 725 and the sponsorship speech on House Bill No.
1248, it is incontrovertible that the intent of our legislators in crafting Section 1 of RA 8171, as
it is precisely worded out, is to exclude those Filipinos who have abandoned their country for
reasons other than political or economic necessity.
Petitioner contends it is not necessary to prove his political or economic reasons since the act
of renouncing allegiance to ones native country constitutes a "necessary and unavoidable
shifting of his political allegiance," and his fathers loss of Philippine citizenship through
naturalization "cannot therefore be said to be for any reason other than political or economic
necessity." 27
This argument has no merit.
While it is true that renunciation of allegiance to ones native country is necessarily a political
act, it does not follow that the act is inevitably politically or economically motivated as alleged
by petitioner. To reiterate, there are other reasons why Filipinos relinquish their Philippine
citizenship. The sponsorship speech of former Congresswoman Andrea B. Domingo illustrates
that aside from economic and political refugees, there are Filipinos who leave the country
because they have committed crimes and would like to escape from punishment, and those
who really feel that they are not Filipinos and that they deserve a better nationality, and
therefore seek citizenship elsewhere.
Thus, assuming petitioner Tabasa is qualified under RA 8171, it is incumbent upon him to
prove to the satisfaction of the SCN that the reason for his loss of citizenship was the decision
of his parents to forfeit their Philippine citizenship for political or economic exigencies. He
failed to undertake this crucial step, and thus, the sought relief is unsuccessful.
Repatriation is not a matter of right, but it is a privilege granted by the State. This is mandated
by the 1987 Constitution under Section 3, Article IV, which provides that citizenship may be
lost or reacquired in the manner provided by law. The State has the power to prescribe by law
the qualifications, procedure, and requirements for repatriation. It has the power to determine
if an applicant for repatriation meets the requirements of the law for it is an inherent power of
the State to choose who will be its citizens, and who can reacquire citizenship once it is lost. If
the applicant, like petitioner Tabasa, fails to comply with said requirements, the State is
justified in rejecting the petition for repatriation.
Petitioner: an undocumented alien subject to summary deportation
Petitioner claims that because of his repatriation, he has reacquired his Philippine citizenship;
therefore, he is not an undocumented alien subject to deportation.
This theory is incorrect.
As previously explained, petitioner is not entitled to repatriation under RA 8171 for he has not
shown that his case falls within the coverage of the law.
Office Memorandum No. 34 dated August 21, 1989 of the BID is enlightening on summary
deportation:
2. The Board of Special Inquiry and the Hearing Board IV shall observe summary deportation
proceedings in cases where the charge against the alien is overstaying, or the expiration or
cancellation by his government of his passport. In cases involving overstaying aliens, BSI and
the Hearing Board IV shall merely require the presentation of the aliens valid passport and
shall decide the case on the basis thereof.

3. If a foreign embassy cancels the passport of the alien, or does not reissue a valid passport
to him, the alien loses the privilege to remain in the country, under the Immigration Act,
Sections 10 and 15 (Schonemann v. Santiago, et al., G.R. No. 81461 [sic, should be 86461],
30 May 1989). The automatic loss of the privilege obviates deportation proceedings. In such
instance, the Board of Commissioners may issue summary judgment of deportation which
shall be immediately executory. 28
In addition, in the case of Schonemann v. Defensor Santiago, et al., this Court held:
It is elementary that if an alien wants to stay in the Philippines, he must possess the
necessary documents. One of these documents is a valid passport. There are, of course,
exceptions where in the exercise of its sovereign prerogatives the Philippines may grant
refugee status, refuse to extradite an alien, or otherwise allow him or her to stay here even if
he [the alien] has no valid passport or Philippine visa. "Boat people" seeking residence
elsewhere are examples. However, the grant of the privilege of staying in the Philippines is
discretionary on the part of the proper authorities. There is no showing of any grave abuse of
discretion, arbitrariness, or whimsicality in the questioned summary judgment. x x x 29
Petitioner Tabasa, whose passport was cancelled after his admission into the country,
became an undocumented alien who can be summarily deported. His subsequent
"repatriation" cannot bar such deportation especially considering that he has no legal and
valid reacquisition of Philippine citizenship.
WHEREFORE, this petition for review is DISMISSED, and the August 7, 1996 Decision of the
Court of Appeals is AFFIRMED. No costs to the petitioner.
SO ORDERED.

112

G.R. No. 135083 May 26, 1999


ERNESTO S. MERCADO, petitioner,
vs.
EDUARDO BARRIOS MANZANO and the COMMISSION ON ELECTIONS, respondents.

WHEREFORE, the Commission hereby declares the respondent Eduardo


Barrios Manzano DISQUALIFIED as candidate for Vice-Mayor of Makati
City.
On May 8, 1998, private respondent filed a motion for reconsideration. 3 The motion remained
pending even until after the election held on May 11, 1998.
Accordingly, pursuant to Omnibus Resolution No. 3044, dated May 10, 1998, of the
COMELEC, the board of canvassers tabulated the votes cast for vice mayor of Makati City but
suspended the proclamation of the winner.
On May 19, 1998, petitioner sought to intervene in the case for disqualification. 4 Petitioner's
motion was opposed by private respondent.
The motion was not resolved. Instead, on August 31, 1998, the COMELEC en banc rendered
its resolution. Voting 4 to 1, with one commissioner abstaining, the COMELEC en banc
reversed the ruling of its Second Division and declared private respondent qualified to run for
vice mayor of the City of Makati in the May 11, 1998 elections. 5 The pertinent portions of the
resolution of the COMELEC en banc read:
As aforesaid, respondent Eduardo Barrios Manzano was born in San
Francisco, California, U.S.A. He acquired US citizenship by operation of
the United States Constitution and laws under the principle of jus soli.
He was also a natural born Filipino citizen by operation of the 1935
Philippine Constitution, as his father and mother were Filipinos at the time
of his birth. At the age of six (6), his parents brought him to the Philippines
using an American passport as travel document. His parents also
registered him as an alien with the Philippine Bureau of Immigration. He
was issued an alien certificate of registration. This, however, did not result
in the loss of his Philippine citizenship, as he did not renounce Philippine
citizenship and did not take an oath of allegiance to the United States.
It is an undisputed fact that when respondent attained the age of majority,
he registered himself as a voter, and voted in the elections of 1992, 1995
and 1998, which effectively renounced his US citizenship under American
law. Under Philippine law, he no longer had U.S. citizenship.
At the time of the May 11, 1998 elections, the resolution of the Second
Division, adopted on May 7, 1998, was not yet final. Respondent Manzano
obtained the highest number of votes among the candidates for vicemayor of Makati City, garnering one hundred three thousand eight
hundred fifty three (103,853) votes over his closest rival, Ernesto S.
Mercado, who obtained one hundred thousand eight hundred ninety four
(100,894) votes, or a margin of two thousand nine hundred fifty nine
(2,959) votes. Gabriel Daza III obtained third place with fifty four thousand
two hundred seventy five (54,275) votes. In applying election laws, it
would be far better to err in favor of the popular choice than be embroiled
in complex legal issues involving private international law which may well
be settled before the highest court (Cf. Frivaldo vs. Commission on
Elections, 257 SCRA 727).

MENDOZA, J.:
Petitioner Ernesto S. Mercado and private respondent Eduardo B. Manzano were candidates
for vice mayor of the City of Makati in the May 11, 1998 elections. The other one was Gabriel
V. Daza III. The results of the election were as follows:
Eduardo B. Manzano 103,853
Ernesto S. Mercado 100,894
Gabriel V. Daza III 54,275 1
The proclamation of private respondent was suspended in view of a pending petition for
disqualification filed by a certain Ernesto Mamaril who alleged that private respondent was not
a citizen of the Philippines but of the United States.
In its resolution, dated May 7, 1998, 2 the Second Division of the COMELEC granted the
petition of Mamaril and ordered the cancellation of the certificate of candidacy of private
respondent on the ground that he is a dual citizen and, under 40(d) of the Local Government
Code, persons with dual citizenship are disqualified from running for any elective position. The
COMELEC's Second Division said:
What is presented before the Commission is a petition for disqualification
of Eduardo Barrios Manzano as candidate for the office of Vice-Mayor of
Makati City in the May 11, 1998 elections. The petition is based on the
ground that the respondent is an American citizen based on the record of
the Bureau of Immigration and misrepresented himself as a natural-born
Filipino citizen.
In his answer to the petition filed on April 27, 1998, the respondent
admitted that he is registered as a foreigner with the Bureau of
Immigration under Alien Certificate of Registration No. B-31632 and
alleged that he is a Filipino citizen because he was born in 1955 of a
Filipino father and a Filipino mother. He was born in the United States,
San Francisco, California, September 14, 1955, and is considered in
American citizen under US Laws. But notwithstanding his registration as
an American citizen, he did not lose his Filipino citizenship.
Judging from the foregoing facts, it would appear that respondent
Manzano is born a Filipino and a US citizen. In other words, he holds dual
citizenship.
The question presented is whether under our laws, he is disqualified from
the position for which he filed his certificate of candidacy. Is he eligible for
the office he seeks to be elected?
Under Section 40(d) of the Local Government Code, those holding dual
citizenship are disqualified from running for any elective local position.
113

WHEREFORE, the Commission en banc hereby REVERSES the


resolution of the Second Division, adopted on May 7, 1998, ordering the
cancellation of the respondent's certificate of candidacy.
We declare respondent Eduardo Luis Barrios Manzano to be QUALIFIED
as a candidate for the position of vice-mayor of Makati City in the May 11,
1998, elections.
ACCORDINGLY, the Commission directs the Makati City Board of
Canvassers, upon proper notice to the parties, to reconvene and proclaim
the respondent Eduardo Luis Barrios Manzano as the winning candidate
for vice-mayor of Makati City.
Pursuant to the resolution of the COMELEC en banc, the board of canvassers, on the evening
of August 31, 1998, proclaimed private respondent as vice mayor of the City of Makati.
This is a petition for certiorari seeking to set aside the aforesaid resolution of the COMELEC
en banc and to declare private respondent disqualified to hold the office of vice mayor of
Makati City. Petitioner contends that
[T]he COMELEC en banc ERRED in holding that:
A. Under Philippine law, Manzano was no longer a U.S. citizen when he:
1. He renounced his U.S. citizenship when he attained
the age of majority when he was already 37 years old;
and,
2. He renounced his U.S. citizenship when he (merely)
registered himself as a voter and voted in the
elections of 1992, 1995 and 1998.
B. Manzano is qualified to run for and or hold the elective office of ViceMayor of the City of Makati;
C. At the time of the May 11, 1998 elections, the resolution of the Second
Division adopted on 7 May 1998 was not yet final so that, effectively,
petitioner may not be declared the winner even assuming that Manzano is
disqualified to run for and hold the elective office of Vice-Mayor of the City
of Makati.
We first consider the threshold procedural issue raised by private respondent Manzano
whether petitioner Mercado his personality to bring this suit considering that he was not an
original party in the case for disqualification filed by Ernesto Mamaril nor was petitioner's
motion for leave to intervene granted.
I. PETITIONER'S RIGHT TO BRING THIS SUIT
Private respondent cites the following provisions of Rule 8 of the Rules of Procedure of the
COMELEC in support of his claim that petitioner has no right to intervene and, therefore,
cannot bring this suit to set aside the ruling denying his motion for intervention:
Sec. 1. When proper and when may be permitted to intervene. Any
person allowed to initiate an action or proceeding may, before or during
the trial of an action or proceeding, be permitted by the Commission, in its
discretion to intervene in such action or proceeding, if he has legal interest
in the matter in litigation, or in the success of either of the parties, or an

interest against both, or when he is so situated as to be adversely affected


by such action or proceeding.
xxx xxx xxx
Sec. 3. Discretion of Commission. In allowing or disallowing a motion
for intervention, the Commission or the Division, in the exercise of its
discretion, shall consider whether or not the intervention will unduly delay
or prejudice the adjudication of the rights of the original parties and
whether or not the intervenor's rights may be fully protected in a separate
action or proceeding.
Private respondent argues that petitioner has neither legal interest in the matter in
litigation nor an interest to protect because he is "a defeated candidate for the vicemayoralty post of Makati City [who] cannot be proclaimed as the Vice-Mayor of
Makati City if the private respondent be ultimately disqualified by final and executory
judgment."
The flaw in this argument is it assumes that, at the time petitioner sought to intervene in the
proceedings before the COMELEC, there had already been a proclamation of the results of
the election for the vice mayoralty contest for Makati City, on the basis of which petitioner
came out only second to private respondent. The fact, however, is that there had been no
proclamation at that time. Certainly, petitioner had, and still has, an interest in ousting private
respondent from the race at the time he sought to intervene. The rule in Labo v. COMELEC, 6
reiterated in several cases, 7 only applies to cases in which the election of the respondent is
contested, and the question is whether one who placed second to the disqualified candidate
may be declared the winner. In the present case, at the time petitioner filed a "Motion for
Leave to File Intervention" on May 20, 1998, there had been no proclamation of the winner,
and petitioner's purpose was precisely to have private respondent disqualified "from running
for [an] elective local position" under 40(d) of R.A. No. 7160. If Ernesto Mamaril (who
originally instituted the disqualification proceedings), a registered voter of Makati City, was
competent to bring the action, so was petitioner since the latter was a rival candidate for vice
mayor of Makati City.
Nor is petitioner's interest in the matter in litigation any less because he filed a motion for
intervention only on May 20, 1998, after private respondent had been shown to have garnered
the highest number of votes among the candidates for vice mayor. That petitioner had a right
to intervene at that stage of the proceedings for the disqualification against private respondent
is clear from 6 of R.A. No. 6646, otherwise known as the Electoral Reform Law of 1987,
which provides:
Any candidate who his been declared by final judgment to be disqualified
shall not be voted for, and the votes cast for him shall not be counted. If
for any reason a candidate is not declared by final judgment before an
election to be disqualified and he is voted for and receives the winning
number of votes in such election, the Court or Commission shall continue
with the trial and hearing of action, inquiry, or protest and, upon motion of
the complainant or any intervenor, may during the pendency thereof order
the suspension of the proclamation of such candidate whenever the
evidence of guilt is strong.
114

Under this provision, intervention may be allowed in proceedings for disqualification even after
election if there has yet been no final judgment rendered.
The failure of the COMELEC en banc to resolve petitioner's motion for intervention was
tantamount to a denial of the motion, justifying petitioner in filing the instant petition for
certiorari. As the COMELEC en banc instead decided the merits of the case, the present
petition properly deals not only with the denial of petitioner's motion for intervention but also
with the substantive issues respecting private respondent's alleged disqualification on the
ground of dual citizenship.
This brings us to the next question, namely, whether private respondent Manzano possesses
dual citizenship and, if so, whether he is disqualified from being a candidate for vice mayor of
Makati City.
II. DUAL CITIZENSHIP AS A GROUND FOR DISQUALIFICATION
The disqualification of private respondent Manzano is being sought under 40 of the Local
Government Code of 1991 (R.A. No. 7160), which declares as "disqualified from running for
any elective local position: . . . (d) Those with dual citizenship." This provision is incorporated
in the Charter of the City of Makati. 8
Invoking the maxim dura lex sed lex, petitioner, as well as the Solicitor General, who sides
with him in this case, contends that through 40(d) of the Local Government Code, Congress
has "command[ed] in explicit terms the ineligibility of persons possessing dual allegiance to
hold local elective office."
To begin with, dual citizenship is different from dual allegiance. The former arises when, as a
result of the concurrent application of the different laws of two or more states, a person is
simultaneously considered a national by the said states. 9 For instance, such a situation may
arise when a person whose parents are citizens of a state which adheres to the principle of jus
sanguinis is born in a state which follows the doctrine of jus soli. Such a person, ipso facto
and without any voluntary act on his part, is concurrently considered a citizen of both states.
Considering the citizenship clause (Art. IV) of our Constitution, it is possible for the following
classes of citizens of the Philippines to possess dual citizenship:
(1) Those born of Filipino fathers and/or mothers in foreign countries
which follow the principle of jus soli;
(2) Those born in the Philippines of Filipino mothers and alien fathers if by
the laws of their father's' country such children are citizens of that country;
(3) Those who marry aliens if by the laws of the latter's country the former
are considered citizens, unless by their act or omission they are deemed
to have renounced Philippine citizenship.
There may be other situations in which a citizen of the Philippines may, without performing
any act, be also a citizen of another state; but the above cases are clearly possible given the
constitutional provisions on citizenship.
Dual allegiance, on the other hand, refers to the situation in which a person simultaneously
owes, by some positive act, loyalty to two or more states. While dual citizenship is involuntary,
dual allegiance is the result of an individual's volition.
With respect to dual allegiance, Article IV, 5 of the Constitution provides: "Dual allegiance of
citizens is inimical to the national interest and shall be dealt with by law." This provision was

included in the 1987 Constitution at the instance of Commissioner Blas F. Ople who explained
its necessity as follows: 10
. . . I want to draw attention to the fact that dual allegiance is not dual
citizenship. I have circulated a memorandum to the Bernas Committee
according to which a dual allegiance and I reiterate a dual allegiance
is larger and more threatening than that of mere double citizenship which
is seldom intentional and, perhaps, never insidious. That is often a
function of the accident of mixed marriages or of birth on foreign soil. And
so, I do not question double citizenship at all.
What we would like the Committee to consider is to take constitutional
cognizance of the problem of dual allegiance. For example, we all know
what happens in the triennial elections of the Federation of FilipinoChinese Chambers of Commerce which consists of about 600 chapters all
over the country. There is a Peking ticket, as well as a Taipei ticket. Not
widely known is the fact chat the Filipino-Chinese community is
represented in the Legislative Yuan of the Republic of China in Taiwan.
And until recently, sponsor might recall, in Mainland China in the People's
Republic of China, they have the Associated Legislative Council for
overseas Chinese wherein all of Southeast Asia including some European
and Latin countries were represented, which was dissolved after several
years because of diplomatic friction. At that time, the Filipino-Chinese
were also represented in that Overseas Council.
When I speak of double allegiance, therefore, I speak of this unsettled
kind of allegiance of Filipinos, of citizens who are already Filipinos but
who, by their acts, may be said to be bound by a second allegiance, either
to Peking or Taiwan. I also took close note of the concern expressed by
some Commissioners yesterday, including Commissioner Villacorta, who
were concerned about the lack of guarantees of thorough assimilation,
and especially Commissioner Concepcion who has always been worried
about minority claims on our natural resources.
Dull allegiance can actually siphon scarce national capital to Taiwan,
Singapore, China or Malaysia, and this is already happening. Some of the
great commercial places in downtown Taipei are Filipino-owned, owned
by Filipino-Chinese it is of common knowledge in Manila. It can mean a
tragic capital outflow when we have to endure a capital famine which also
means economic stagnation, worsening unemployment and social unrest.
And so, this is exactly what we ask that the Committee kindly consider
incorporating a new section, probably Section 5, in the article on
Citizenship which will read as follows: DUAL ALLEGIANCE IS INIMICAL
TO CITIZENSHIP AND SHALL BE DEALT WITH ACCORDING TO LAW.
In another session of the Commission, Ople spoke on the problem of these citizens with dual
allegiance, thus: 11
. . . A significant number of Commissioners expressed their concern about
dual citizenship in the sense that it implies a double allegiance under a
double sovereignty which some of us who spoke then in a freewheeling
115

debate thought would be repugnant to the sovereignty which pervades the


Constitution and to citizenship itself which implies a uniqueness and which
elsewhere in the Constitution is defined in terms of rights and obligations
exclusive to that citizenship including, of course, the obligation to rise to
the defense of the State when it is threatened, and back of this,
Commissioner Bernas, is, of course, the concern for national security. In
the course of those debates, I think some noted the fact that as a result of
the wave of naturalizations since the decision to establish diplomatic
relations with the People's Republic of China was made in 1975, a good
number of these naturalized Filipinos still routinely go to Taipei every
October 10; and it is asserted that some of them do renew their oath of
allegiance to a foreign government maybe just to enter into the spirit of the
occasion when the anniversary of the Sun Yat-Sen Republic is
commemorated. And so, I have detected a genuine and deep concern
about double citizenship, with its attendant risk of double allegiance which
is repugnant to our sovereignty and national security. I appreciate what
the Committee said that this could be left to the determination of a future
legislature. But considering the scale of the problem, the real impact on
the security of this country, arising from, let us say, potentially great
numbers of double citizens professing double allegiance, will the
Committee entertain a proposed amendment at the proper time that will
prohibit, in effect, or regulate double citizenship?
Clearly, in including 5 in Article IV on citizenship, the concern of the Constitutional
Commission was not with dual citizens per se but with naturalized citizens who maintain their
allegiance to their countries of origin even after their naturalization. Hence, the phrase "dual
citizenship" in R.A. No. 7160, 40(d) and in R.A. No. 7854, 20 must be understood as
referring to "dual allegiance." Consequently, persons with mere dual citizenship do not fall
under this disqualification. Unlike those with dual allegiance, who must, therefore, be subject
to strict process with respect to the termination of their status, for candidates with dual
citizenship, it should suffice if, upon the filing of their certificates of candidacy, they elect
Philippine citizenship to terminate their status as persons with dual citizenship considering that
their condition is the unavoidable consequence of conflicting laws of different states. As
Joaquin G. Bernas, one of the most perceptive members of the Constitutional Commission,
pointed out: "[D]ual citizenship is just a reality imposed on us because we have no control of
the laws on citizenship of other countries. We recognize a child of a Filipino mother. But
whether she is considered a citizen of another country is something completely beyond our
control." 12
By electing Philippine citizenship, such candidates at the same time forswear allegiance to the
other country of which they are also citizens and thereby terminate their status as dual
citizens. It may be that, from the point of view of the foreign state and of its laws, such an
individual has not effectively renounced his foreign citizenship. That is of no moment as the
following discussion on 40(d) between Senators Enrile and Pimentel clearly shows: 13
SENATOR ENRILE. Mr. President, I would like to ask clarification of line
41, page 17: "Any person with dual citizenship" is disqualified to run for
any elective local position. Under the present Constitution, Mr. President,
someone whose mother is a citizen of the Philippines but his father is a

foreigner is a natural-born citizen of the Republic. There is no requirement


that such a natural born citizen, upon reaching the age of majority, must
elect or give up Philippine citizenship.
On the assumption that this person would carry two passports, one
belonging to the country of his or her father and one belonging to the
Republic of the Philippines, may such a situation disqualify the person to
run for a local government position?
SENATOR PIMENTEL. To my mind, Mr. President, it only means that at
the moment when he would want to run for public office, he has to
repudiate one of his citizenships.
SENATOR ENRILE. Suppose he carries only a Philippine passport but the
country of origin or the country of the father claims that person,
nevertheless, as a citizen? No one can renounce. There are such
countries in the world.
SENATOR PIMENTEL. Well, the very fact that he is running for public
office would, in effect, be an election for him of his desire to be considered
as a Filipino citizen.
SENATOR ENRILE. But, precisely, Mr. President, the Constitution does
not require an election. Under the Constitution, a person whose mother is
a citizen of the Philippines is, at birth, a citizen without any overt act to
claim the citizenship.
SENATOR PIMENTEL. Yes. What we are saying, Mr. President, is: Under
the Gentleman's example, if he does not renounce his other citizenship,
then he is opening himself to question. So, if he is really interested to run,
the first thing he should do is to say in the Certificate of Candidacy that: "I
am a Filipino citizen, and I have only one citizenship."
SENATOR ENRILE. But we are talking from the viewpoint of Philippine
law, Mr. President. He will always have one citizenship, and that is the
citizenship invested upon him or her in the Constitution of the Republic.
SENATOR PIMENTEL. That is true, Mr. President. But if he exercises
acts that will prove that he also acknowledges other citizenships, then he
will probably fall under this disqualification.
This is similar to the requirement that an applicant for naturalization must renounce "all
allegiance and fidelity to any foreign prince, potentate, state, or sovereignty" 14 of which at the
time he is a subject or citizen before he can be issued a certificate of naturalization as a
citizen of the Philippines. In Parado v. Republic, 15 it was held:
[W]hen a person applying for citizenship by naturalization takes an oath
that he renounce, his loyalty to any other country or government and
solemnly declares that he owes his allegiance to the Republic of the
Philippines, the condition imposed by law is satisfied and compiled with.
The determination whether such renunciation is valid or fully complies with
the provisions of our Naturalization Law lies within the province and is an
exclusive prerogative of our courts. The latter should apply the law duly
enacted by the legislative department of the Republic. No foreign law may
116

or should interfere with its operation and application. If the requirement of


the Chinese Law of Nationality were to be read into our Naturalization
Law, we would be applying not what our legislative department has
deemed it wise to require, but what a foreign government has thought or
intended to exact. That, of course, is absurd. It must be resisted by all
means and at all cost. It would be a brazen encroachment upon the
sovereign will and power of the people of this Republic.
III. PETITIONER'S ELECTION OF PHILIPPINE CITIZENSHIP
The record shows that private respondent was born in San Francisco, California on
September 4, 1955, of Filipino parents. Since the Philippines adheres to the principle of jus
sanguinis, while the United States follows the doctrine of jus soli, the parties agree that, at
birth at least, he was a national both of the Philippines and of the United States. However, the
COMELEC en banc held that, by participating in Philippine elections in 1992, 1995, and 1998,
private respondent "effectively renounced his U.S. citizenship under American law," so that
now he is solely a Philippine national.
Petitioner challenges this ruling. He argues that merely taking part in Philippine elections is
not sufficient evidence of renunciation and that, in any event, as the alleged renunciation was
made when private respondent was already 37 years old, it was ineffective as it should have
been made when he reached the age of majority.
In holding that by voting in Philippine elections private respondent renounced his American
citizenship, the COMELEC must have in mind 349 of the Immigration and Nationality Act of
the United States, which provided that "A person who is a national of the United States,
whether by birth or naturalization, shall lose his nationality by: . . . (e) Voting in a political
election in a foreign state or participating in an election or plebiscite to determine the
sovereignty over foreign territory." To be sure this provision was declared unconstitutional by
the U.S. Supreme Court in Afroyim v. Rusk 16 as beyond the power given to the U.S.
Congress to regulate foreign relations. However, by filing a certificate of candidacy when he
ran for his present post, private respondent elected Philippine citizenship and in effect
renounced his American citizenship. Private respondent's certificate of candidacy, filed on
March 27, 1998, contained the following statements made under oath:
6. I AM A FILIPINO CITIZEN (STATE IF "NATURALBORN" OR "NATURALIZED") NATURAL-BORN
xxx xxx xxx
10. I AM A REGISTERED VOTER OF PRECINCT NO. 747-A,
BARANGAY SAN LORENZO, CITY/MUNICIPALITY OF MAKATI,
PROVINCE OF NCR.
11. I AM NOT A PERMANENT RESIDENT OF, OR IMMIGRANT TO, A
FOREIGN COUNTRY.
12. I AM ELIGIBLE FOR THE OFFICE I SEEK TO BE ELECTED. I WILL
SUPPORT AND DEFEND THE CONSTITUTION OF THE PHILIPPINES
AND WILL MAINTAIN TRUE FAITH AND ALLEGIANCE THERETO;
THAT I WILL OBEY THE LAWS, LEGAL ORDERS AND DECREES
PROMULGATED BY THE DULY CONSTITUTED AUTHORITIES OF THE
REPUBLIC OF THE PHILIPPINES; AND THAT I IMPOSE THIS

OBLIGATION UPON MYSELF VOLUNTARILY, WITHOUT MENTAL


RESERVATION OR PURPOSE OF EVASION. I HEREBY CERTIFY
THAT THE FACTS STATED HEREIN ARE TRUE AND CORRECT OF
MY OWN PERSONAL KNOWLEDGE.
The filing of such certificate of candidacy sufficed to renounce his American citizenship,
effectively removing any disqualification he might have as a dual citizen. Thus, in Frivaldo v.
COMELEC it was held: 17
It is not disputed that on January 20, 1983 Frivaldo became an American.
Would the retroactivity of his repatriation not effectively give him dual
citizenship, which under Sec. 40 of the Local Government Code would
disqualify him "from running for any elective local position?" We answer
this question in the negative, as there is cogent reason to hold that
Frivaldo was really STATELESS at the time he took said oath of
allegiance and even before that, when he ran for governor in 1988. In his
Comment, Frivaldo wrote that he "had long renounced and had long
abandoned his American citizenship long before May 8, 1995. At best,
Frivaldo was stateless in the interim when he abandoned and
renounced his US citizenship but before he was repatriated to his Filipino
citizenship."
On this point, we quote from the assailed Resolution dated December 19,
1995:
By the laws of the United States, petitioner Frivaldo
lost his American citizenship when he took his oath of
allegiance to the Philippine Government when he ran
for Governor in 1988, in 1992, and in 1995. Every
certificate of candidacy contains an oath of allegiance
to the Philippine Government.
These factual findings that Frivaldo has lost his foreign nationality long
before the elections of 1995 have not been effectively rebutted by Lee.
Furthermore, it is basic that such findings of the Commission are
conclusive upon this Court, absent any showing of capriciousness or
arbitrariness or abuse.
There is, therefore, no merit in petitioner's contention that the oath of allegiance contained in
private respondent's certificate of candidacy is insufficient to constitute renunciation that, to be
effective, such renunciation should have been made upon private respondent reaching the
age of majority since no law requires the election of Philippine citizenship to be made upon
majority age.
Finally, much is made of the fact that private respondent admitted that he is registered as an
American citizen in the Bureau of Immigration and Deportation and that he holds an American
passport which he used in his last travel to the United States on April 22, 1997. There is no
merit in this. Until the filing of his certificate of candidacy on March 21, 1998, he had dual
citizenship. The acts attributed to him can be considered simply as the assertion of his
American nationality before the termination of his American citizenship. What this Court said
in Aznar v. COMELEC 18 applies mutatis mundatis to private respondent in the case at bar:
117

. . . Considering the fact that admittedly Osmea was both a Filipino and
an American, the mere fact that he has a Certificate staring he is an
American does not mean that he is not still a Filipino. . . . [T]he
Certification that he is an American does not mean that he is not still a
Filipino, possessed as he is, of both nationalities or citizenships. Indeed,
there is no express renunciation here of Philippine citizenship; truth to tell,
there is even no implied renunciation of said citizenship. When We
consider that the renunciation needed to lose Philippine citizenship must
be "express," it stands to reason that there can be no such loss of
Philippine citizenship when there is no renunciation, either "express" or
"implied."
To recapitulate, by declaring in his certificate of candidacy that he is a Filipino citizen; that he
is not a permanent resident or immigrant of another country; that he will defend and support
the Constitution of the Philippines and bear true faith and allegiance thereto and that he does
so without mental reservation, private respondent has, as far as the laws of this country are
concerned, effectively repudiated his American citizenship and anything which he may have
said before as a dual citizen.
On the other hand, private respondent's oath of allegiance to the Philippines, when
considered with the fact that he has spent his youth and adulthood, received his education,
practiced his profession as an artist, and taken part in past elections in this country, leaves no
doubt of his election of Philippine citizenship.
His declarations will be taken upon the faith that he will fulfill his undertaking made under oath.
Should he betray that trust, there are enough sanctions for declaring the loss of his Philippine
citizenship through expatriation in appropriate proceedings. In Yu v. Defensor-Santiago, 19 we
sustained the denial of entry into the country of petitioner on the ground that, after taking his
oath as a naturalized citizen, he applied for the renewal of his Portuguese passport and
declared in commercial documents executed abroad that he was a Portuguese national. A
similar sanction can be taken against any one who, in electing Philippine citizenship,
renounces his foreign nationality, but subsequently does some act constituting renunciation of
his Philippine citizenship.
WHEREFORE, the petition for certiorari is DISMISSED for lack of merit.1wphi1.nt
SO ORDERED.

118

Das könnte Ihnen auch gefallen